You are on page 1of 164

1. Republic v. "G" Holdings, Inc., G.R. No. 141241, November 22, 2005, 475 SCRA 608.

2. Tolentino v. Leviste, G.R. No. 156118, November 19, 2004, 443 SCRA 274.
3. Vette Industrial Sales Co., Inc. v. Cheng, G.R. Nos. 170232 & 170301, December 5, 2006, 509 SCRA 532, 543.
4. Cuenca vs PCGG, G.R. Nos. 159104-05, October 5, 2007 535 SCRA 102
5. Arranza v. B.F. Homes, G.R. No. 131683, June 19, 2000, 333 SCRA 799.
6. Guinhawa v. People, G.R. No. 162822, August 25, 2005, 468 SCRA 278, 299.
7. Dela Cruz v. CA, G.R. No. 139442, December 6, 2006, 510 SCRA 103, 115.
8. Borra vs CA, G.R. No. 167484, September 9, 2013
9. BF Homes vs Manila Electric Company 636 SCRA 495, G.R. No. 171624, December 6, 2010.
10. Republic v. Estipular, 336 SCRA 333 (2000)
11. Tolentino v. Social Security Commission, No. L-28870, September 6, 1985, 138 SCRA 428
12. Gustilo vs Gustilo 659 SCRA 428, G.R. No. 175497, October 19, 2011.
13. Delos Reyes v. Odones, G.R. No. 178096, March 23, 2011, 646 SCRA 328
14. Tomas Claudio Memorial College, Inc. v. Court of Appeals, 316 SCRA 502, 509 (1999)
15. Javellana v. Presiding Judge, RTC, Branch 30, Manila, G.R. No. 139067, 23 November 2004, 443 SCRA 497,
506.
16. Hilado v. Chavez, 438 SCRA 623, 641 (2004)
17. Echegaray v. Secretary of Justice, G.R. No. 132601, January 19, 1999, 301 SCRA 96, 106.
18. Victory Liner vs. Bellosillo, A.M. MTJ-00-1321, 10 March 2004, 425 SCRA 79.
19. Calimlim, et al. v. Ramirez, et al., G.R. No. 34362, November 12, 1982, 118 SCRA 399.
20. Davao Light & Power., Inc. vs. Court of Appeals, et al., G.R. No. 93262, 204 SCRA 343 (1991)
21. Macasaet v. Co, Jr., G.R. No. 156759, June 5, 2013, 697 SCRA 187.
22. Navale v. Court of Appeals, 253 SCRA 705 (1996).
23. French Oil Mill Machinery Co., Inc. v. Court of Appeals, 295 SCRA 462 (1998)
24. Asiavest Limited v. Court of Appeals, 296 SCRA 539, 552 (1998).
25. Alba v. Court of Appeals, G.R. No. 164041. July 29, 2005
26. Boston Equity Resources, Inc. v. Court of Appeals, G.R. No. 173946, June 19, 2013

1. Republic v. "G" Holdings, Inc., G.R. No. 141241, November 22, 2005, 475 SCRA 608.

THIRD DIVISION

[G.R. No. 141241. November 22, 2005.]

REPUBLIC OF THE PHILIPPINES, through its trustee, the ASSET PRIVATIZATION TRUST, petitioner, vs. "G"
HOLDINGS, INC., respondent.

Raul E. Villanueva and Rhoel Z. Mabazza for petitioner.

Juan G. Ranola, Jr. and Janis B. Alonzo for petitioner PMO.

Siguion Reyna Montecillo & Ongsiako for private respondent.

SYLLABUS

1. REMEDIAL LAW; CIVIL PROCEDURE; ANNULMENT OF JUDGMENTS; INSTANT PETITION LACKS


REQUISITE IMPRIMATUR FROM THE OFFICE OF THE SOLICITOR GENERAL. — We note that the instant petition
suffers from a basic infirmity for lack of the requisite imprimatur from the Office of the Solicitor General, hence, it is
dismissible on that ground. The general rule is that only the Solicitor General can bring or defend actions on behalf of the
Republic of the Philippines and that actions filed in the name of the Republic, or its agencies and instrumentalities for that
matter, if not initiated by the Solicitor General, should be summarily dismissed. As an exception to the general rule, the
Solicitor General is empowered to "deputize legal officers of government departments, bureaus, agencies and offices to
assist the Solicitor General and appear or represent the Government in cases involving their respective offices, brought
before the courts and exercise supervision and control over such legal officers with respect to such cases." Here, the
petition was signed and filed on behalf of the Republic by Atty. Raul B. Villanueva, the executive officer of the legal
department of the APT, and Atty. Rhoel L. Mabazza. However, they did not present any proof that they had been duly
deputized by the Solicitor General to initiate and litigate this action. Thus, this petition can be dismissed on that ground. In
the interest of justice, however, we shall proceed to discuss the issues propounded by the Republic. THcaDA
1
2. ID.; ID.; ID.; LACK OF JURISDICTION AS A GROUND FOR ANNULMENT OF JUDGMENT REFERS TO EITHER
LACK OF JURISDICTION OVER THE PERSON OF THE DEFENDING PARTY OR OVER SUBJECT MATTER OF THE
CLAIM. — A petition for annulment of judgment is an extraordinary action. By virtue of its exceptional character, the action
is restricted exclusively to the grounds specified in the rules, namely, (1) extrinsic fraud and (2) lack of jurisdiction. The
rationale for the restriction is to prevent the extraordinary action from being used by a losing party to make a complete farce
of a duly promulgated decision that has long become final and executory. The remedy may not be invoked where the party
has availed himself of the remedy of new trial, appeal, petition for relief or other appropriate remedy and lost, or where he
has failed to avail himself of those remedies through his own fault or negligence. Lack of jurisdiction as a ground for
annulment of judgment refers to either lack of jurisdiction over the person of the defending party or over the subject matter
of the claim. Where the court has jurisdiction over the defendant and over the subject matter of the case, its decision will
not be voided on the ground of absence of jurisdiction.

3. ID.; ID.; ID.; INTERPRETATION OF THE REPUBLIC CONTRAVENES THE VERY RATIONALE OF THE
RESTRICTIVE APPLICATION OF ANNULMENT OF JUDGMENTS; INCLUDING ACTS COMMITTED WITH GRAVE
ABUSE OF DISCRETION WILL TEND TO ENLARGE THE CONCEPT OF LACK OF JURISDICTION AS A GROUND FOR
AVAILMENT OF THE REMEDY. — The interpretation of the Republic contravenes the very rationale of the restrictive
application of annulment of judgment. By seeking to include acts committed with grave abuse of discretion, it tends to
enlarge the concept of lack of jurisdiction as a ground for the availment of the remedy. In a petition for annulment of
judgment based on lack of jurisdiction, the petitioner must show not merely an abuse of jurisdictional discretion but an
absolute lack of jurisdiction. Thus, the concept of lack of jurisdiction as a ground to annul a judgment does not embrace
abuse of discretion. HSTAcI

4. ID.; ID.; ID.; BY CLAIMING GRAVE ABUSE OF DISCRETION ON THE PART OF THE TRIAL COURT, THE
REPUBLIC ACTUALLY CONCEDES AND PRESUPPOSES THE JURISDICTION OF THE COURT TO TAKE
COGNIZANCE OF THE CASE. — By claiming grave abuse of discretion on the part of the trial court, the Republic actually
concedes and presupposes the jurisdiction of the court to take cognizance of the case. Hence, the Republic effectively
admits that the two grounds for which lack of jurisdiction may be validly invoked to seek the annulment of a judgment —
want of jurisdiction over the parties and want of jurisdiction over the subject matter — do not exist. It only assails the manner
in which the trial court formulated its judgment in the exercise of its jurisdiction. Jurisdiction is distinct from the exercise
thereof. We amply explained the distinction between the two in Tolentino v. Leviste, thus: Jurisdiction is not the same as
the exercise of jurisdiction. As distinguished from the exercise of jurisdiction, jurisdiction is the authority to decide a cause,
and not the decision rendered therein. Where there is jurisdiction over the person and the subject matter, the decision on
all other questions arising in the case is but an exercise of the jurisdiction. And the errors which the court may commit in
the exercise of jurisdiction are merely errors of judgment which are the proper subject of an appeal.

5. ID.; ID.; ID.; NO GRAVE ABUSE OF DISCRETION CAN BE IMPUTED TO THE TRIAL COURT IN RENDERING
THE ASSAILED DECISION. — No grave abuse of discretion can be imputed to the trial court when it rendered the decision.
The pieces of evidence considered by the court a quo to arrive at its decision were documents attached as annexes to the
various pleadings filed by the parties. It is well-settled that documents attached to the pleadings form part thereof and may
be considered as evidence even if not formally introduced as evidence. The court may and should consider as evidence
documents attached to the pleadings filed by the parties and made a part thereof, without necessity of introducing them
expressly as evidence when their authenticity and due execution have not been denied under oath. Moreover, the minutes
of the pre-trial conference on May 27, 1996 show that the exhibits presented by both parties were "marked, offered and
admitted" during the pre-trial. This fact coupled with the manifestation of the parties during the pre-trial that the sole issue
to be resolved was one of law — the interpretation of the provisions of the purchase and sale agreement which was adopted
by the parties as their common exhibit — show that the trial court did not commit an abuse of discretion. The conclusion
that there was no abuse of discretion on the part of the trial court would be the same even if it were to be assumed that a
procedural mistake was committed when it decided the case before the parties could formally offer their evidence. We have
held that where the court has jurisdiction and, having all the facts necessary for a judgment, it renders a decision without
holding any trial or hearing (where the parties are allowed to present their respective evidence in support of their cause of
action and defense), such judgment cannot be assailed as having been rendered without or in excess of jurisdiction nor
rendered with grave abuse of discretion. IHcTDA

6. ID.; ID.; ID.; EXTRINSIC FRAUD; NOT ESTABLISHED. — In the matter of extrinsic fraud, the circumstances of
this case do not establish its existence. Extrinsic fraud refers to any fraudulent act of the prevailing party in the litigation
which is committed outside of the trial of the case, whereby the unsuccessful party is prevented from fully proving his case,
by fraud or deception practiced on him by his opponent. Fraud is regarded as extrinsic where it prevents a party from having
a trial or from presenting his entire case to the court, or where it operates upon matters pertaining not to the judgment itself
but to the manner in which it is procured. The overriding consideration when extrinsic fraud is alleged is that the fraudulent
scheme of the prevailing litigant prevented a party from having his day in court. The Republic has not proven, or even

2
alleged, that "G" Holdings practiced deceit or employed subterfuge on it, precluding it from fully and completely presenting
its case to the court. Since the prevailing party did not commit or participate in the commission of fraud which prevented
the other party from having his day in court, there was no reason for the appellate court to annul the decision of the trial
court.

7. ID.; ID.; ID.; THE SOLICITOR GENERAL MAY NOT BE EXCUSED FROM ITS SHORTCOMINGS BY INVOKING
THE DOCTRINE THAT THE REPUBLIC OR THE GOVERNMENT IS NOT ESTOPPED BY THE MISTAKE OR ERROR
ON THE PART OF ITS OFFICIALS OR AGENTS AS IF IT WERE SOME MAGIC INCANTATION THAT COULD
BENIGNLY, IF ARBITRARILY, CONDONE, AND ERASE ITS ERRORS. — The unfortunate predicament of the Republic
was caused by the Solicitor General, its own counsel. We have consistently ruled that, to render a judgment void, the fraud
must be committed by the adverse party and not by one's own counsel. While the Republic or the government is usually
not estopped by the mistake or error on the part of its officials or agents, the Republic cannot now take refuge in the rule
as it does not afford a blanket or absolute immunity. Our pronouncement in Republic v. Court of Appeals is instructive: the
Solicitor General may not be excused from its shortcomings by invoking the doctrine as if it were some magic incantation
that could benignly, if arbitrarily, condone and erase its errors. Here, no fault had been ascribed to "G" Holdings and the
proceedings in the trial court were proper. The judgment has already attained finality as a result of the fault and inaction of
the Solicitor General. This was aggravated by the fact that this petition was filed by those who had no authority to do so.
DEcSaI

DECISION

CORONA, J p:

This petition for review on certiorari under Rule 45 of the Rules of Court assails the December 21, 1999 resolution 1 of the
Court of Appeals (CA) dismissing the petition for annulment of judgment in CA-G.R. SP No. 53517.

On May 21, 1992, the Committee on Privatization approved the proposal of the Asset Privatization Trust (APT) for the
negotiated sale of 90% of the shares of stock of the government-owned Maricalum Mining Corporation (MMC). Learning of
the government's intention to sell MMC, the respondent "G" Holdings, Inc. signified its interest to purchase MMC and
submitted the best bid. CAcEaS

The series of negotiations between the petitioner Republic of the Philippines, through the APT as its trustee, 2 and "G"
Holdings culminated in the execution of a purchase and sale agreement on October 2, 1992. Under the agreement, the
Republic undertook to sell and deliver 90% of the entire issued and outstanding shares of MMC, as well as its company
notes, to "G" Holdings in consideration of the purchase price of P673,161,280. It also provided for a down payment of
P98,704,000 with the balance divided into four tranches payable in installment over a period of ten years.

Subsequently, a disagreement on the matter of when the installment payments should commence arose between the
parties. The Republic claimed that it should be on the seventh month from the signing of the agreement while "G" Holdings
insisted that it should begin seven months after the fulfillment of the closing conditions.

Unable to settle the issue, "G" Holdings filed a complaint for specific performance and damages with the Regional Trial
Court of Manila, Branch 49, against the Republic to compel it to close the sale in accordance with the purchase and sale
agreement. The complaint was docketed as Civil Case No. 95-76132.

During the pre-trial, the respective counsels of the parties manifested that the issue involved in the case was one of law
and submitted the case for decision. On June 11, 1996, the trial court rendered its decision. It ruled in favor of "G" Holdings
and held:

In line with the foregoing, this Court having been convinced that the Purchase and Sale Agreement is indeed subject to the
final closing conditions prescribed by Stipulation No. 5.02 and conformably to Rule 39, Section 10 of the Rules of Court,
accordingly orders that the Asset Privatization Trust execute the corresponding Document of Transfer of the subject shares
and financial notes and cause the actual delivery of subject shares and notes to "G" Holdings, Inc., within a period of thirty
(30) days from receipt of this Decision, and after the "G" Holdings, Inc. shall have paid in full the entire balance, at its
present value of P241,702,122.86, computed pursuant to the prepayment provisions of the Agreement. Plaintiff shall pay
the balance simultaneously with the delivery of the Deed of Transfer and actual delivery of the shares and notes. CDISAc

SO ORDERED. 3

The Solicitor General filed a notice of appeal on behalf of the Republic on June 28, 1996. Contrary to the rules of procedure,
however, the notice of appeal was filed with the Court of Appeals (CA), not with the trial court which rendered the judgment
appealed from.

3
No other judicial remedy was resorted to until July 2, 1999 when the Republic, through the APT, filed a petition for annulment
of judgment with the CA. It claimed that the decision should be annulled on the ground of abuse of discretion amounting to
lack of jurisdiction on the part of the trial court. It characterized the fashion by which the trial court handled the case as
highly aberrant and peculiar because the court a quo promulgated its decision prior to the submission of the Republic's
formal offer of evidence and without ruling on the admissibility of the evidence offered by "G" Holdings. The Republic also
asserted that the failure of the Solicitor General to file the notice of appeal with the proper forum amounted to extrinsic fraud
which prevented it from appealing the case.

Finding that the grounds necessary for the annulment of judgment were inexistent, the appellate court dismissed the
petition. It ruled that there was no extrinsic fraud because "G" Holdings had no participation in the failure of the Solicitor
General to properly appeal the decision of the trial court. Neither was there any connivance between "G" Holdings' and the
Republic's counsels in the commission of the error.

The appellate court also held that the trial court had jurisdiction over the subject matter of the case, as well as over the
person of the parties. Hence, whatever error the trial court committed in the exercise of its jurisdiction was merely an error
of judgment, not an error of jurisdiction. As an error of judgment, it was correctable by appeal. Unfortunately, appeal could
no longer be availed of by the Republic. AHDTIE

The appellate court further declared that there was no grave abuse of discretion on the part of the court a quo when it
decided the case before its receipt of the Republic's formal offer of evidence. The evidence of both parties was already in
the possession of the court and painstakingly considered before the decision was arrived at. Thus, if at all, the trial court
perpetrated an "irregularity" which should have been the subject of an appeal. But no appeal was perfected and the decision
of the trial court thus attained finality.

The Republic now assails the resolution of the appellate court on the following grounds:

THE COURT OF APPEALS ERRED IN NOT HOLDING THAT THE TRIAL COURT COMMITTED GRAVE ABUSE OF
DISCRETION AMOUNTING TO LACK OF JURISDICTION WHICH RESULTED IN THE NULLITY OF THE TRIAL
COURT'S DECISION

THE TRIAL COURT RENDERED ITS DECISION EVEN PRIOR TO THE SUBMISSION OF PETITIONER'S FORMAL
OFFER OF EVIDENCE AND EVEN BEFORE PETITIONER COULD FILE ITS COMMENT TO RESPONDENT'S FORMAL
OFFER OF EVIDENCE

THE TRIAL COURT RENDERED ITS DECISION WITHOUT RULING ON THE ADMISSION OF THE EVIDENCE
OFFERED BY RESPONDENT

II

THE FAILURE OF THE [SOLICITOR GENERAL] TO FILE THE NOTICE OF APPEAL WITH THE PROPER FORUM
AMOUNTED TO EXTRINSIC FRAUD WHICH PREVENTED THE PETITIONER FROM APPEALING THE CASE WITH
THE COURT OF APPEALS. 4

Before anything else, we note that the instant petition suffers from a basic infirmity for lack of the requisite imprimatur from
the Office of the Solicitor General, hence, it is dismissible on that ground. 5 The general rule is that only the Solicitor General
can bring or defend actions on behalf of the Republic of the Philippines and that actions filed in the name of the Republic,
or its agencies and instrumentalities for that matter, if not initiated by the Solicitor General, should be summarily dismissed.
6 As an exception to the general rule, the Solicitor General is empowered to "deputize legal officers of government
departments, bureaus, agencies and offices to assist the Solicitor General and appear or represent the Government in
cases involving their respective offices, brought before the courts and exercise supervision and control over such legal
officers with respect to such cases." 7

Here, the petition was signed and filed on behalf of the Republic by Atty. Raul B. Villanueva, the executive officer of the
legal department of the APT, and Atty. Rhoel Z. Mabazza. 8 However, they did not present any proof that they had been
duly deputized by the Solicitor General to initiate and litigate this action. Thus, this petition can be dismissed on that ground.
THEcAS

In the interest of justice, however, we shall proceed to discuss the issues propounded by the Republic.
4
A petition for annulment of judgment is an extraordinary action. 9 By virtue of its exceptional character, the action is
restricted exclusively to the grounds specified in the rules, 10 namely, (1) extrinsic fraud and (2) lack of jurisdiction. 11 The
rationale for the restriction is to prevent the extraordinary action from being used by a losing party to make a complete farce
of a duly promulgated decision that has long become final and executory. 12 The remedy may not be invoked where the
party has availed himself of the remedy of new trial, appeal, petition for relief or other appropriate remedy and lost, or where
he has failed to avail himself of those remedies through his own fault or negligence. 13

Lack of jurisdiction as a ground for annulment of judgment refers to either lack of jurisdiction over the person of the
defending party or over the subject matter of the claim. 14 Where the court has jurisdiction over the defendant and over the
subject matter of the case, its decision will not be voided on the ground of absence of jurisdiction.

The Republic does not deny that the trial court had jurisdiction over it as well as over the subject matter of the case. What
the Republic questions is the grave abuse of discretion allegedly committed by the court a quo in rendering the decision.
IECcaA

We cannot agree with the Republic.

First, the interpretation of the Republic contravenes the very rationale of the restrictive application of annulment of judgment.
By seeking to include acts committed with grave abuse of discretion, it tends to enlarge the concept of lack of jurisdiction
as a ground for the availment of the remedy.

In a petition for annulment of judgment based on lack of jurisdiction, the petitioner must show not merely an abuse of
jurisdictional discretion but an absolute lack of jurisdiction. 15 Thus, the concept of lack of jurisdiction as a ground to annul
a judgment does not embrace abuse of discretion.

Second, by claiming grave abuse of discretion on the part of the trial court, the Republic actually concedes and presupposes
the jurisdiction of the court to take cognizance of the case. Hence, the Republic effectively admits that the two grounds for
which lack of jurisdiction may be validly invoked to seek the annulment of a judgment — want of jurisdiction over the parties
and want of jurisdiction over the subject matter — do not exist. It only assails the manner in which the trial court formulated
its judgment in the exercise of its jurisdiction.

Jurisdiction is distinct from the exercise thereof. We amply explained the distinction between the two in Tolentino v. Leviste,
16 thus:

Jurisdiction is not the same as the exercise of jurisdiction. As distinguished from the exercise of jurisdiction, jurisdiction is
the authority to decide a cause, and not the decision rendered therein. Where there is jurisdiction over the person and the
subject matter, the decision on all other questions arising in the case is but an exercise of the jurisdiction. And the errors
which the court may commit in the exercise of jurisdiction are merely errors of judgment which are the proper subject of an
appeal. EcIaTA

Finally, no grave abuse of discretion can be imputed to the trial court when it rendered the decision. The pieces of evidence
considered by the court a quo to arrive at its decision were documents attached as annexes to the various pleadings filed
by the parties. It is well-settled that documents attached to the pleadings form part thereof and may be considered as
evidence even if not formally introduced as evidence. 17 The court may and should consider as evidence documents
attached to the pleadings filed by the parties and made a part thereof, without necessity of introducing them expressly as
evidence when their authenticity and due execution have not been denied under oath. 18

Moreover, the minutes of the pre-trial conference 19 on May 27, 1996 show that the exhibits presented by both parties
were "marked, offered and admitted" during the pre-trial. This fact coupled with the manifestation of the parties during the
pre-trial that the sole issue to be resolved was one of law — the interpretation of the provisions of the purchase and sale
agreement which was adopted by the parties as their common exhibit — show that the trial court did not commit an abuse
of discretion.

The conclusion that there was no abuse of discretion on the part of the trial court would be the same even if it were to be
assumed that a procedural mistake was committed when it decided the case before the parties could formally offer their
evidence. We have held that where the court has jurisdiction and, having all the facts necessary for a judgment, it renders
a decision without holding any trial or hearing (where the parties are allowed to present their respective evidence in support
of their cause of action and defense), such judgment cannot be assailed as having been rendered without or in excess of
jurisdiction nor rendered with grave abuse of discretion. 20

In the matter of extrinsic fraud, the circumstances of this case do not establish its existence.

5
Extrinsic fraud refers to any fraudulent act of the prevailing party in the litigation which is committed outside of the trial of
the case, whereby the unsuccessful party is prevented from fully proving his case, by fraud or deception practiced on him
by his opponent. 21 Fraud is regarded as extrinsic where it prevents a party from having a trial or from presenting his entire
case to the court, or where it operates upon matters pertaining not to the judgment itself but to the manner in which it is
procured. 22 The overriding consideration when extrinsic fraud is alleged is that the fraudulent scheme of the prevailing
litigant prevented a party from having his day in court. 23

The Republic has not proven, or even alleged, that "G" Holdings practiced deceit or employed subterfuge on it, precluding
it from fully and completely presenting its case to the court. Since the prevailing party did not commit or participate in the
commission of fraud which prevented the other party from having his day in court, there was no reason for the appellate
court to annul the decision of the trial court.

The unfortunate predicament of the Republic was caused by the Solicitor General, its own counsel. We have consistently
ruled that, to render a judgment void, the fraud must be committed by the adverse party and not by one's own counsel. 24

While the Republic or the government is usually not estopped by the mistake or error on the part of its officials or agents,
25 the Republic cannot now take refuge in the rule as it does not afford a blanket or absolute immunity. Our pronouncement
in Republic v. Court of Appeals 26 is instructive: the Solicitor General may not be excused from its shortcomings by invoking
the doctrine as if it were some magic incantation that could benignly, if arbitrarily, condone and erase its errors. AHCcET

Here, no fault had been ascribed to "G" Holdings and the proceedings in the trial court were proper. The judgment has
already attained finality as a result of the fault and inaction of the Solicitor General. This was aggravated by the fact that
this petition was filed by those who had no authority to do so.

Litigation should end and terminate sometime and somewhere. 27 It is essential to an effective and efficient administration
of justice that, once a judgment has become final, the winning party should not be deprived of the fruits of the verdict. 28
Courts must therefore guard against any scheme calculated to bring about that undesirable result. 29 Thus, it is only proper
for this Court to now write finis to this decade-old controversy.

WHEREFORE, the petition is hereby DENIED. The December 21, 1999 resolution of the Court of Appeals in CA-G.R. SP
No. 53517 is AFFIRMED.

Costs against petitioner.

SO ORDERED.

2. Tolentino v. Leviste, G.R. No. 156118, November 19, 2004, 443 SCRA 274.

SECOND DIVISION

[G.R. No. 156118. November 19, 2004.]

PABLO T. TOLENTINO and TEMPUS PLACE REALTY MANAGEMENT CORPORATION, petitioners, vs. HON. OSCAR
LEVISTE, Presiding Judge, RTC, Quezon City, Br. 97 and SPOUSES GERARDO CINCO, JR. and PAMELA H. CINCO,
respondents.

Camacho & Associates for petitioners.

Epifanio Cua for respondents.

SYNOPSIS

Respondents Spouses Gerardo Cinco, Jr. and Pamela Cinco filed a complaint for specific performance with damages
against petitioners Tempus Place Realty Management Corporation and Pablo T. Tolentino. As petitioners failed to file their
answer to the complaint, the trial court issued an order granting respondents' motion to declare petitioners in default. After
reception of evidence, the trial court rendered a decision in favor of respondents. Petitioners thereafter filed a motion for
new trial. They contended that their right to fair and impartial trial had been impaired by reason of accident, mistake or
excusable negligence of their former counsel. The trial court denied the motion for new trial for lack of merit. Petitioners
appealed to the Court of Appeals which was, however, dismissed on the ground of abandonment as petitioners failed to
submit the required appeal brief. The decision became final and executory and was recorded in the Book of Entries of
Judgment. Petitioners then filed with the Court of Appeals an action for annulment of judgment. The Court of Appeals found
that the trial court's decision may not be annulled on the ground of extrinsic fraud. It stated that the failure of petitioners'

6
counsel to file an appellant's brief in the Court of Appeals did not amount to extrinsic fraud as to justify annulment of
judgment, as it was not shown that their former counsel's omission was tainted with fraud and/or deception tantamount to
extrinsic or collateral fraud. Neither may it be annulled on the ground of lack of jurisdiction as the action for specific
performance and damages was within the jurisdiction of the RTC. Hence, the present petition for review. IEaHSD

The Supreme Court denied the petition. According to the Court, the overriding consideration when extrinsic fraud is alleged
is that the fraudulent scheme of the prevailing litigant prevented a party from having his day in court. Petitioners did not
allege nor present evidence of fraud or deception employed on them by the respondents to deprive them of opportunity to
present their case to the court. Petitioners only assert that the negligence of their former counsel in failing to file the appeal
brief amounts to extrinsic fraud which would serve as basis for their petition for annulment of judgment. The Court reiterated
the settled rule that a client is bound by the mistakes of his counsel except when the negligence of his counsel is so gross,
reckless and inexcusable that the client is deprived of his day in court. Only when the application of the general rule would
result in serious injustice should the exception apply. The Court found no reason to apply the exception in the present case.
The Court further held that Section 2 of Rule 47 of the Rules of Court provides that extrinsic fraud shall not be a valid ground
if it was availed of, or could have been availed of, in a motion for new trial or petition for relief. It is effectively barred if it
could have been raised as a ground in an available remedial measure. The records show that after petitioners learned of
the judgment by default, they filed a motion for new trial on the ground of extrinsic fraud. It was, however, denied by the
trial court. They filed a notice of appeal thereafter. Hence, they are now precluded from alleging extrinsic fraud as a ground
for their petition for annulment of the trial court's decision.

SYLLABUS

1. REMEDIAL LAW; CIVIL PROCEDURE; ANNULMENT OF JUDGMENTS; EXTRINSIC FRAUD; CONSTRUED. —


Under the Rule, an action for annulment of judgments may only be availed of on the following grounds: (1) extrinsic fraud
and (2) lack of jurisdiction. Extrinsic fraud refers to any fraudulent act of the prevailing party in the litigation which is
committed outside of the trial of the case, whereby the unsuccessful party has been prevented from exhibiting fully his
case, by fraud or deception practiced on him by his opponent. Fraud is regarded as extrinsic where it prevents a party from
having a trial or from presenting his entire case to the court, or where it operates upon matters pertaining not to the judgment
itself but to the manner in which it is procured. The overriding consideration when extrinsic fraud is alleged is that the
fraudulent scheme of the prevailing litigant prevented a party from having his day in court.

2. ID.; ID.; ID.; A PARTY IS BOUND BY HIS COUNSEL'S ACTIONS AND DECISIONS REGARDING THE
CONDUCT OF THE CASE ESPECIALLY WHERE HE DOES NOT COMPLAIN AGAINST THE MANNER HIS COUNSEL
HANDLES THE SUIT; CASE AT BAR. — Petitioners in this case did not allege nor present evidence of fraud or deception
employed on them by the respondents to deprive them of opportunity to present their case to the court. They, however,
assert that the negligence of their former counsel in failing to file the appeal brief amounts to extrinsic fraud which would
serve as basis for their petition for annulment of judgment. We disagree. The Court has held that when a party retains the
services of a lawyer, he is bound by his counsel's actions and decisions regarding the conduct of the case. This is true
especially where he does not complain against the manner his counsel handles the suit. Such is the case here. When the
complaint was filed before the trial court, summons was served upon the petitioners. They allegedly referred the matter to
Atty. Villamor who was holding office at the building owned and managed by respondent Tempus Place Realty Management
Corporation. However, after they have endorsed the summons to said lawyer, they did not exert any effort to follow up the
developments of the suit. Hence, they were declared in default and judgment was rendered against them. Even in the
course of the appeal, they never bothered to check with their counsel, Atty. Ricardo Santos, the status of the appeal. The
notice of appeal was filed on November 3, 1997 and petitioners learned of the dismissal of the appeal in October 1999,
after petitioner Tolentino received notice of garnishment of his insurance benefits in connection with the judgment in Civil
Case No. Q-96-29207. It was only at that time that they learned that Atty. Santos had migrated to Australia. This only shows
that petitioners, as what happened during the pendency of the case before the trial court, never bothered to confer with
their counsel regarding the conduct and status of their appeal. We reiterate the rule that a client is bound by the mistakes
of his counsel except when the negligence of his counsel is so gross, reckless and inexcusable that the client is deprived
of his day in court. Only when the application of the general rule would result in serious injustice should the exception apply.
We find no reason to apply the exception in this case. aCTcDS

3. ID.; ID.; ID.; EXTRINSIC FRAUD IS EFFECTIVELY BARRED IF IT COULD HAVE BEEN RAISED AS GROUND
IN AN AVAILABLE REMEDIAL MEASURE. — It is provided in Section 2 of Rule 47 that extrinsic fraud shall not be a valid
ground if it was availed of, or could have been availed of, in a motion for new trial or petition for relief. In other words, it is
effectively barred if it could have been raised as a ground in an available remedial measure. The records show that after
petitioners learned of the judgment of default, they filed a motion for new trial on the ground of extrinsic fraud. It was
however denied by the trial court. They filed a notice of appeal thereafter. Hence, they are now precluded from alleging
extrinsic fraud as a ground for their petition for annulment of the trial court decision.

7
4. ID.; ID.; ID.; ISSUE OF LACK OF JURISDICTION OF TRIAL COURT FOUND BASELESS. — We are also not
persuaded by petitioners' assertion that the trial court judge lacked jurisdiction so as to justify the annulment of his decision
in Civil Case No. Q-96-29207. Lack of jurisdiction as a ground for annulment of judgment refers to either lack of jurisdiction
over the person of the defending party or over the subject matter of the claim. Jurisdiction over the person of the defendant
or respondent is acquired by voluntary appearance or submission by the defendant or respondent to the court, or by
coercive process issued by the court to him, generally by the service of summons. The trial court clearly had jurisdiction
over the person of the defending party, the petitioners herein, when the latter received the summons from the court. On the
other hand, jurisdiction over the subject matter of the claim is conferred by law and is determined from the allegations in
the complaint. Under the law, the action for specific performance and damages is within the jurisdiction of the RTC.
Petitioners' submission, therefore, that the trial court lacked jurisdiction does not hold water.

5. ID.; ID.; ID.; PETITION FOR ANNULMENT ASSAILED THE TRIAL COURT'S EXERCISE OF ITS JURISDICTION
BUT PETITIONERS FAILED TO SHOW THAT THE TRIAL COURT DID NOT HAVE THE AUTHORITY TO DECIDE THE
CASE. — We note that petitioners' arguments to support their stand that the trial court did not have jurisdiction actually
pertain to the substance of the decision. Jurisdiction is not the same as the exercise of jurisdiction. As distinguished from
the exercise of jurisdiction, jurisdiction is the authority to decide a cause, and not the decision rendered therein. Where
there is jurisdiction over the person and the subject matter, the decision on all other questions arising in the case is but an
exercise of the jurisdiction. And the errors which the court may commit in the exercise of jurisdiction are merely errors of
judgment which are the proper subject of an appeal. The errors raised by petitioners in their petition for annulment assail
the content of the decision of the trial court and not the court's authority to decide the suit. In other words, they relate to the
court's exercise of its jurisdiction, but petitioners failed to show that the trial court did not have the authority to decide the
case. Based on the foregoing discussion, it is clear that petitioners' petition for annulment of judgment had no basis and
was rightly dismissed by the Court of Appeals. ADEacC

DECISION

PUNO, J p:

Petitioners Pablo T. Tolentino and Tempus Place Realty Management Corporation seek the review and reversal of the
decision and amended decision of the Court of Appeals in CA-G.R. SP No. 59506 entitled "Tempus Place Realty
Management Corporation and Pablo T. Tolentino vs. Hon. Oscar Leviste, Presiding Judge, RTC — Quezon City, Branch
97 and Sps. Gerardo Cinco, Jr., and Pamela H. Cinco." The Court of Appeals denied petitioners' petition for annulment of
the decision of the Regional Trial Court (RTC) of Quezon City, Branch 97, on the action for specific performance with
damages filed by respondents Spouses Gerardo and Pamela Cinco against them. HaIATC

The antecedent facts are as follows:

On October 18, 1996, respondents Spouses Gerardo Cinco, Jr. and Pamela Cinco filed a complaint for specific performance
with damages against petitioners Tempus Place Realty Management Corporation and Pablo T. Tolentino. The complaint
alleged that respondents purchased from petitioners a condominium unit in Tempus Place Condominium II at Katarungan
St., Diliman, Quezon City. Despite, however, the execution of the Deed of Absolute Sale and the delivery of the owner's
copy of the condominium certificate of title, petitioners failed to deliver possession of the unit because they have allegedly
leased it to a third party. The complaint further alleged that petitioners refused to pay the corresponding capital gains tax
and documentary stamp tax on the transaction, and execute the necessary board resolution for the transfer of the property,
thus preventing respondents from registering the Deed of Absolute Sale and transferring the title to the unit in their names.
The respondents claimed that because petitioners refused to deliver possession of the unit and instead leased it to a third
party, they are entitled to a reasonable rental value in the amount of P20,000.00 a month from May 1994 until the time the
possession of the unit is delivered to them. They also claimed moral damages in the amount of P1,000,000.00 and
exemplary damages in the amount of P1,000,000.00 plus attorney's fees in the amount of P1,000,000.00. 1

As petitioners failed to file their answer to the complaint, Hon. Oscar Leviste, Presiding Judge, RTC, Branch 97, Quezon
City, issued an order on January 17, 1997 granting respondents' motion to declare petitioners in default. He also appointed
the Branch Clerk of Court to act as commissioner to receive respondents' evidence ex parte. 2 After reception of evidence,
the trial court, on April 15, 1997, issued a decision for the respondents. It stated:

This Court after considering the oral and documentary evidences presented by the plaintiff finds that the allegation
contained in their pleadings are all true facts and are entitled to the relief as prayed for, to wit:

1) To deliver to the plaintiffs the possession of the condominium unit covered by CCT No. 5002 of the Register of
Deeds of Quezon City;

8
2) To pay the corresponding capital gains tax and documentary stamps tax on the transaction, and deliver the receipts
thereof to the plaintiffs;

3) To execute and deliver to the plaintiffs the necessary Board Resolution;

4) Jointly and severally, to pay plaintiffs the following:

a. Actual damages in the amount of P20,000.00 a month from May 1994, up to the time possession of the
condominium units (sic) is delivered to the plaintiffs representing the reasonable rental value of the unit;

b. Moral damages in the amount of P1,000,000.00;

c. Exemplary damages in the amount of P1,000,000.00; ASEIDH

d. Attorney's fees in the amount of P1,000,000.00. 3

Petitioners thereafter filed a motion for new trial. They contended that their right to fair and impartial trial had been impaired
by reason of accident, mistake or excusable negligence of their former counsel, a certain Atty. Villamor. 4 The trial court
denied the motion for new trial for lack of merit. 5

On November 3, 1997, petitioners, through their new counsel, Atty. Ricardo A. Santos, filed a notice of appeal of the April
15 decision of the trial court. 6 The Court of Appeals, however, dismissed the appeal on February 26, 1999 on the ground
of abandonment as petitioners failed to submit the required appeal brief. 7 The decision became final and executory on
March 26, 1999 and was recorded in the Book of Entries of Judgment. 8

On July 4, 2000, petitioners filed with the Court of Appeals an action for annulment of judgment based on the following
grounds:

1. The judgment in default granted reliefs in excess of what is prayed for in the complaint in gross violation of the
clear provisions of the 1997 Rules of Civil Procedure.

2. The judgment in default awarded unliquidated damages in palpable violation of the mandatory provision of Section
3[,] Rule 9, 1997 Rules of Civil Procedure.

3. The judgment in default is in gross violation of Section 14, Article VIII, 1987 Constitution and Section 1, Rule 36,
1997 Rules of Civil Procedure.

4. The judgment in default was rendered in violation of the rights of the petitioner to substantive and procedural due
process.

5. Corollarily, the gargantuan award for damages by the court a quo in patent and blatant violation of the law and
settled jurisprudence [is] unconscionable and clearly violative of substantial justice and equities of the case.

6. Petitioners have good and substantial defenses in respect of private respondents' claims.

7. A fortiori, the court has no jurisdiction and/or authority and has committed a grave abuse of discretion in awarding
amounts in excess of what is prayed for in the complaint nor proved by the evidence as well as in palpable violation of the
mandatory provisions of the Civil Code and the Rules of Court and applicable decisions of the Supreme Court.
Consequently, the challenged judgment in default is an absolute nullity. 9

On April 23, 2002, the appellate court issued a decision modifying the trial court decision. It explained that the annulment
of judgment may be based on the grounds of extrinsic fraud and lack of jurisdiction, and it is important that petitioner failed
to move for new trial, or appeal, or file a petition for relief, or take other appropriate remedies assailing the questioned
judgment, final order or resolution through no fault attributable to him. The Court of Appeals found that the trial court decision
may not be annulled on the ground of extrinsic fraud. It stated that the failure of petitioners' counsel to file an appellant's
brief in the Court of Appeals did not amount to extrinsic fraud as to justify annulment of judgment, as it was not shown that
their former counsel's omission was tainted with fraud and/or deception tantamount to extrinsic or collateral fraud. Neither
may it be annulled on the ground of lack of jurisdiction as the action for specific performance and damages was within the
jurisdiction of the RTC. Nonetheless, the appellate court, in the interest of justice and in the exercise of its sound discretion
in determining the amount of damages that may be awarded, held that the moral damages in the amount of one million
pesos (P1,000,000.00) was excessive. It lowered the moral damages to P100,000.00. It also reduced the exemplary
damages to P100,000.00, and the attorney's fees to P100,000.00. 10

Respondents filed a motion for reconsideration of the Decision of the Court of Appeals. On November 18, 2002, the Court
of Appeals issued an Amended Decision, the dispositive portion of which reads:
9
WHEREFORE, the Motion for Reconsideration is partly GRANTED in that the dispositive portion of the assailed decision
is modified as follows:

a) Actual damages in the amount of P10,000.00 a month from May 1994, up to the time possession of the
condominium units [sic] is delivered to the plaintiffs (private respondents herein) representing the reasonable rental value
of the unit.

b) Moral damages in the amount of One Hundred Thousand Pesos (P100,000.00);

c) Exemplary damages in the amount of One Hundred Thousand Pesos (P100,000.00); and, cDCaTS

d) Attorney's fees in the amount of One [H]undred Thousand Pesos (P100,000.00).

SO ORDERED. 11

Petitioners filed the instant petition for review of the decision and amended decision of the Court of Appeals. They raise the
following arguments:

1. The petitioners can avail of the remedy of annulment of judgment to annul the decision of the RTC in Civil Case
No. 96-29707 as Hon. Judge Leviste had no jurisdiction and/or acted without jurisdiction in issuing the April 15, 1997
Decision because:

a. The judgment in default granted reliefs in excess of what is prayed for in the complaint in gross violation of the
clear provisions of the 1997 Rules of Civil Procedure.

b. The judgment in default awarded unliquidated damages in palpable violation of the mandatory provision of Section
3[,] Rule 9, 1997 Rules of Civil Procedure.

c. The judgment in default is in gross violation of Sec. 14, Art. VIII, 1987 Constitution and Sec. 1, Rule 36, 1997 Rules
of Civil Procedure.

d. The judgment in default was rendered in violation of the rights of the petitioner to substantive and procedural due
process.

2. The petitioners were prevented from having their day in court because of the gross negligence of their former
counsel, which gross negligence amounts to extrinsic fraud.

3. The remedies of appeal, petition for relief or other remedies are no longer available through no fault of petitioners.

4. The petitioners have valid and substantial defenses to respondents' cause of action. 12

The petition is without merit.

The issue that needs to be resolved in this petition for review is whether the Court of Appeals erred in dismissing the petition
for annulment of judgment filed by petitioners.

The governing rule is Rule 47 of the 1997 Rules of Civil Procedure on Annulment of Judgments or Final Orders and
Resolutions. Sections 1 and 2 of the Rule provide for its coverage and the grounds therefor, thus:

Sec. 1. Coverage. — This Rule shall govern the annulment by the Court of Appeals of judgments or final orders and
resolutions in civil actions of Regional Trial Courts for which the ordinary remedies of new trial, appeal, petition for relief or
other appropriate remedies are no longer available through no fault of the petitioner.

Sec. 2. Grounds for annulment. — The annulment may be based only on the grounds of extrinsic fraud and lack of
jurisdiction.

Extrinsic fraud shall not be a valid ground if it was availed of, or could have been availed of, in a motion for new trial or
petition for relief.

Under the Rule, an action for annulment of judgments may only be availed of on the following grounds: (1) extrinsic fraud
and (2) lack of jurisdiction.

Extrinsic fraud refers to any fraudulent act of the prevailing party in the litigation which is committed outside of the trial of
the case, whereby the unsuccessful party has been prevented from exhibiting fully his case, by fraud or deception practiced
on him by his opponent. Fraud is regarded as extrinsic where it prevents a party from having a trial or from presenting his
entire case to the court, or where it operates upon matters pertaining not to the judgment itself but to the manner in which
10
it is procured. The overriding consideration when extrinsic fraud is alleged is that the fraudulent scheme of the prevailing
litigant prevented a party from having his day in court. 13

Petitioners in this case did not allege nor present evidence of fraud or deception employed on them by the respondents to
deprive them of opportunity to present their case to the court. They, however, assert that the negligence of their former
counsel in failing to file the appeal brief amounts to extrinsic fraud which would serve as basis for their petition for annulment
of judgment. We disagree. The Court has held that when a party retains the services of a lawyer, he is bound by his
counsel's actions and decisions regarding the conduct of the case. This is true especially where he does not complain
against the manner his counsel handles the suit. 14 Such is the case here. When the complaint was filed before the trial
court, summons was served upon the petitioners. 15 They allegedly referred the matter to Atty. Villamor who was holding
office at the building owned and managed by respondent Tempus Place Realty Management Corporation. 16 However,
after they have endorsed the summons to said lawyer, they did not exert any effort to follow up the developments of the
suit. Hence, they were declared in default and judgment was rendered against them. Even in the course of the appeal, they
never bothered to check with their counsel, Atty. Ricardo Santos, the status of the appeal. The notice of appeal was filed
on November 3, 1997 and petitioners learned of the dismissal of the appeal in October 1999, after petitioner Tolentino
received notice of garnishment of his insurance benefits in connection with the judgment in Civil Case No. Q-96-29207. It
was only at that time that they learned that Atty. Santos had migrated to Australia. This only shows that petitioners, as what
happened during the pendency of the case before the trial court, never bothered to confer with their counsel regarding the
conduct and status of their appeal. The Court stated in Villaruel, Jr. vs. Fernando: 17

. . . Litigants represented by counsel should not expect that all they need to do is sit back, relax and await the outcome of
their case. To agree with petitioner's stance would enable every party to render inutile any adverse order or decision through
the simple expedient of alleging negligence on the part of his counsel. The Court will not countenance such ill-founded
argument which contradicts long-settled doctrines of trial and procedure. 18

We reiterate the rule that a client is bound by the mistakes of his counsel except when the negligence of his counsel is so
gross, reckless and inexcusable that the client is deprived of his day in court. 19 Only when the application of the general
rule would result in serious injustice should the exception apply. 20 We find no reason to apply the exception in this case.
aAHSEC

In addition, it is provided in Section 2 of Rule 47 that extrinsic fraud shall not be a valid ground if it was availed of, or could
have been availed of, in a motion for new trial or petition for relief. In other words, it is effectively barred if it could have
been raised as a ground in an available remedial measure. 21 The records show that after petitioners learned of the
judgment of default, they filed a motion for new trial on the ground of extrinsic fraud. It was however denied by the trial
court. They filed a notice of appeal thereafter. Hence, they are now precluded from alleging extrinsic fraud as a ground for
their petition for annulment of the trial court decision.

We are also not persuaded by petitioners' assertion that the trial court judge lacked jurisdiction so as to justify the annulment
of his decision in Civil Case No. Q-96-29207. Lack of jurisdiction as a ground for annulment of judgment refers to either
lack of jurisdiction over the person of the defending party or over the subject matter of the claim. 22 Jurisdiction over the
person of the defendant or respondent is acquired by voluntary appearance or submission by the defendant or respondent
to the court, or by coercive process issued by the court to him, generally by the service of summons. The trial court clearly
had jurisdiction over the person of the defending party, the petitioners herein, when the latter received the summons from
the court. On the other hand, jurisdiction over the subject matter of the claim is conferred by law and is determined from
the allegations in the complaint. Under the law, the action for specific performance and damages is within the jurisdiction
of the RTC. Petitioners' submission, therefore, that the trial court lacked jurisdiction does not hold water.

We note that petitioners' arguments to support their stand that the trial court did not have jurisdiction actually pertain to the
substance of the decision. Jurisdiction is not the same as the exercise of jurisdiction. As distinguished from the exercise of
jurisdiction, jurisdiction is the authority to decide a cause, and not the decision rendered therein. Where there is jurisdiction
over the person and the subject matter, the decision on all other questions arising in the case is but an exercise of the
jurisdiction. And the errors which the court may commit in the exercise of jurisdiction are merely errors of judgment which
are the proper subject of an appeal. 23 The errors raised by petitioners in their petition for annulment assail the content of
the decision of the trial court and not the court's authority to decide the suit. In other words, they relate to the court's exercise
of its jurisdiction, but petitioners failed to show that the trial court did not have the authority to decide the case.

Based on the foregoing discussion, it is clear that petitioners' petition for annulment of judgment had no basis and was
rightly dismissed by the Court of Appeals. LLjur

IN VIEW WHEREOF, the petition at bar is DENIED.

SO ORDERED.
11
Austria-Martinez, Callejo, Sr., Tinga and Chico-Nazario, JJ ., concur.

3. ` Industrial Sales Co., Inc. v. Cheng, G.R. Nos. 170232 & 170301, December 5, 2006, 509 SCRA 532, 543.

FIRST DIVISION

[G.R. No. 170232. December 5, 2006.]

VETTE INDUSTRIAL SALES CO., INC., KENNETH TAN, ESTRELLA CHENG, LUISITO RAMOS, YVETTE TAN,
KESSENTH CHENG, VEVETTE CHENG and FELESAVETTE CHENG, petitioners, vs. SUI SOAN S. CHENG a.k.a.
CHENG SUI SOAN, respondent.

[G.R. No. 170301. December 5, 2006.]

SUI SOAN S. CHENG a.k.a. CHENG SUI SOAN, petitioner, vs. VETTE INDUSTRIAL SALES CO., INC., KENNETH TAN,
ESTRELLA CHENG, LUISITO RAMOS, YVETTE TAN, KESSENTH CHENG, VEVETTE CHENG and FELESAVETTE
CHENG, respondents.

DECISION

YNARES-SANTIAGO, J p:

These consolidated Petitions for Review on Certiorari 1 assail the Decision 2 dated September 22, 2005 of the Court of
Appeals in CA-G.R. SP No. 88863 entitled, "Vette Industrial Sales, Company, Inc., Kenneth Tan, Estrella Cheng, Luisito
Ramos, Yvette Tan, Kessenth Cheng, Vevette Cheng, and Felesavette Cheng, Petitioners versus Hon. Regional Trial Court
of Manila, Branch 173, and Sui Soan S. Cheng a.k.a. Cheng Sui Soan, Respondents." Also assailed is the Resolution 3
dated October 27, 2005 denying petitioners' motion for partial reconsideration and respondent Sui's motion for
reconsideration. AacCIT

In his Complaint 4 for specific performance and damages filed against Vette Industrial Sales Company, Inc., Kenneth Tan,
Estrella Cheng, Luisito Ramos, Yvette Tan, Kessenth Cheng, Vevette Cheng, and Felesavette Cheng (petitioners) and
docketed as Civil Case No. 03-105691, Sui Soan S. Cheng a.k.a. Cheng Sui Soan (Sui) alleged that on October 24, 2001,
he executed a Deed of Assignment, 5 where he transferred his 40,000 shares in the company in favor of Kenneth Tan,
Vevette Cheng, Felesavette Cheng, and Yvette Tan (Petitioners-Assignees). To implement the Deed of Assignment, the
company acknowledged in a Memorandum of Agreement (MOA), 6 that it owed him P6.8 million pesos, plus insurance
proceeds amounting to P760,000.00 and a signing bonus of P300,000.00. Thereafter, he was issued 48 postdated checks
but after the 11th check, the remaining checks were dishonored by the bank. Sui also claimed that petitioners did not remit
to him the insurance proceeds, thus breaching their obligation under the MOA which entitled him to moral and exemplary
damages, and attorney's fees.

In their Answer With Compulsory Counterclaim, 7 petitioners alleged that Sui sold his shares for only P1.00 per share which
they already paid; that the MOA was unenforceable because it was executed without authorization from the board of
directors; that the MOA was void for want of consideration; and that petitioner Kenneth Tan executed the MOA after Sui
issued threats and refused to sign the waiver and quitclaim.

After the issues were joined, pre-trial was set on July 3, 2003. 8 However, the case was first submitted for mediation but it
was referred back to the court for continuation of the proceedings when no settlement was arrived at during mediation.
SCIacA

Sui thereafter filed a Motion to Set Pre-trial 9 on December 16, 2003. Petitioners received the motion but they did not attend
because there was no notice from the Court setting the pre-trial date. On December 29, 2003, petitioners received two
orders from the trial court. The first Order 10 allowed Sui to present evidence ex-parte, while the second Order 11 revoked
the first order after the trial court noted that "what was set for consideration on December 16, 2003 was merely a motion to
set pre-trial." Thus, the trial court reset the pre-trial on January 15, 2004 but it was postponed and moved to May 21, 2004.
On said date, Sui and his counsel, Atty. Pedro M. Ferrer (Atty. Ferrer), failed to appear. Consequently, the trial court ordered
the dismissal of the case without prejudice on the part of petitioners to present and prove their counterclaim and set the
hearing for reception of evidence on June 22, 2004. 12

Atty. Ferrer filed a Manifestation and Motion for Reconsideration 13 of the order of dismissal, explaining that he arrived late
for the hearing because he had to drop by his office to get the case folder because he had just arrived from South Cotabato
where he served as Chief Counsel in the Provincial Board of Canvassers for Governor Datu Pax Mangudadatu and
Congressman Suharto Mangudadatu.
12
The trial court required petitioners to file their Comment on the Manifestation and Motion for Reconsideration. In their
Opposition, 14 petitioners asserted that the motion for reconsideration be denied outright because (1) Sui did not comply
with the three-day notice rule which is mandatory under Section 4, Rule 15 of the Rules of Court considering that petitioners
received the manifestation and motion for reconsideration only one day prior to the date of hearing of the motion for
resolution, thus the same must be treated as a mere scrap of paper; (2) the trial court did not comply with Section 6 of Rule
15 of the Rules 15 when it acted on the manifestation and motion of Sui despite the latter's failure to submit proof of receipt
by petitioners of the manifestation and motion; (3) the negligence of counsel binds the client, thus, when Atty. Ferrer arrived
late for the hearing, the trial court correctly dismissed the complaint; and (4) the explanation of Atty. Ferrer is unacceptable
because traffic gridlocks are daily events in the metropolis, thus, Atty. Ferrer should have left his place early. EacHSA

In his Reply, 16 Sui averred that the motion complied with Section 5 of Rule 15 of the Rules 17 and that the setting of the
hearing of the motion on May 28, 2004 was within the three day period for it was filed on May 25, 2004. He added that the
same was not heard because the trial court allowed petitioners to file a comment on the manifestation and motion for
reconsideration, which was received by the latter prior to the said setting.

In an Order dated December 16, 2004, 18 the trial court granted Sui's motion for reconsideration and set aside the dismissal
of the complaint, the dispositive portion of which provides:

WHEREFORE, prescinding with such ruling and in the interest of substantial justice, plaintiff's motion is GRANTED and the
order dated May 21, 2004 is hereby lifted and set aside with the warning that any delay in this proceedings will not be
countenanced by the Court.

Set pre-trial anew on February 15, 2005.

Notify the parties.

SO ORDERED. 19

The trial court cited Ace Navigation Co., Inc. v. Court of Appeals, 20 which held that since rules of procedure are mere tools
designed to facilitate the attainment of justice, their strict and rigid application which would result in technicalities that tend
to frustrate rather than promote substantial justice must always be avoided — the dismissal of an appeal on purely technical
ground is frowned upon especially if it will result to unfairness.

The Motion for Reconsideration 21 filed by petitioners was denied by the trial court 22 hence they filed a Petition for
Certiorari 23 with the Court of Appeals which granted the petition, thus:

UPON THE VIEW WE TAKE OF THIS CASE, THUS, the writ applied for is partly GRANTED. The assailed orders must
be, as they hereby are, VACATED and SET ASIDE, and another hereby issued dismissing the instant complaint, but
"without prejudice." This means that the complaint can be REINSTATED. On the other hand, petitioners are hereby given
leave to present before the Trial Court evidence of their counterclaim. Without costs in this instance. AECcTS

SO ORDERED. 24

The Court of Appeals noted that both Atty. Ferrer and Sui were not in attendance at the pre-trial conference; that Section
5 of Rule 18 mentions only the effect of the failure to appear on the part of "the plaintiff" but is silent on the effect of failure
of the party's counsel to appear at the pre-trial; that the Manifestation and Motion for Reconsideration 25 mentioned only
the reasons why Atty. Ferrer was absent without stating that he was fully authorized in writing to enter into an amicable
settlement, or to submit to alternative modes of dispute resolution, or to enter into stipulations or admissions of facts and
of documents; and that there was no explanation for Sui's nonappearance. Thus, based on these circumstances, the Court
of Appeals held that dismissal of the case is proper but without prejudice to the filing of a new action. 26

Both parties moved for reconsideration but the same were jointly denied in a Resolution dated October 27, 2005.

Hence, these consolidated Petitions.

In G.R. No. 170232, petitioners raise the following errors:

I.

THE COURT OF APPEALS ERRED IN NOT DISMISSING THE COMPLAINT OF RESPONDENT CHENG IN CIVIL CASE
NO. 03-105691 WITH PREJUDICE.

II.

13
THE COURT OF APPEALS ERRED IN CONCLUDING THAT RESPONDENT'S COUNSEL FAILED TO APPRECIATE
THE BASIC RULES ON PRE-TRIAL. DcTSHa

III.

THE COURT OF APPEALS ERRED IN NOT CONSIDERING THE MISTAKE OR NEGLIGENCE OF RESPONDENT'S
COUNSEL AS BINDING ON THE RESPONDENT HIMSELF.

IV.

THE COURT OF APPEALS ERRED IN APPLYING THE RULINGS OF THE HONORABLE COURT IN THE DE LOS
REYES VS. CAPULE (102 PHIL. 464) AND SUAREZ VS. COURT OF APPEALS (220 SCRA 274) CASES.

V.

THE COURT OF APPEALS ERRED IN NOT CONSIDERING RESPONDENT'S MANIFESTATION AND MOTION FOR
RECONSIDERATION DATED MAY 21, 2004 FILED BEFORE THE TRIAL COURT AS A MERE SCRAP, AND A USELESS
PIECE, OF PAPER AND IN NOT CONSIDERING THE ORDER DATED MAY 21, 2004 OF THE TRIAL COURT AS
ALREADY FINAL IN VIEW OF THE PROCEDURAL INVALIDITY/DEFECTIVENESS (I.E. IT FAILED TO COMPLY WITH
SECTIONS 4 AND 6 OF THE RULES) OF RESPONDENT'S MANIFESTATION AND MOTION FOR RECONSIDERATION
DATED MAY 21, 2004.

In G.R. No. 170301, Sui raises the following issues, thus:

I. THE COURT OF APPEALS ERRED IN NOT RULING THAT THE NON-APPEARANCE OF PETITIONER IN THE
PRE-TRIAL MAY BE EXCUSED FOR A VALID CAUSE.

II. THE COURT OF APPEALS ERRED IN NOT RULING THAT THE CASE OF ACE NAVIGATION CO. INC. VS.
COURT OF APPEALS IS SQUARELY APPLICABLE TO THE INSTANT CASE. DcCEHI

The core issue for resolution is whether the Court of Appeals erred in dismissing without prejudice Civil Case No. 03-
105691 and in ruling that the trial court committed grave abuse of discretion when it granted Sui's motion for reconsideration
to set aside the order of dismissal of the complaint.

The judge has the discretion whether or not to declare a party non-suited. 27 It is, likewise, settled that the determination
of whether or not an order of dismissal issued under such conditions should be maintained or reconsidered rests upon the
sound discretion of the trial judge. 28 The next question to be resolved is whether there was grave abuse of discretion of
the trial judge. We hold that there was none.

The case of Estate of Salud Jimenez v. Philippine Export Processing Zone 29 discussed the propriety of filing a Petition for
Certiorari under Section 1 of Rule 65 of the Rules of Court, thus:

A petition for certiorari is the proper remedy when any tribunal, board, or officer exercising judicial or quasi-judicial functions
has acted without or in excess of its jurisdiction, or with grave abuse of discretion amounting to lack or excess of jurisdiction
and there is no appeal, nor any plain, speedy, and adequate remedy at law. Grave abuse of discretion is defined as the
capricious and whimsical exercise of judgment as is equivalent to lack of jurisdiction. An error of judgment committed in the
exercise of its legitimate jurisdiction is not the same as "grave abuse of discretion." An abuse of discretion is not sufficient
by itself to justify the issuance of a writ of certiorari. The abuse must be grave and patent, and it must be shown that the
discretion was exercised arbitrarily and despotically.

As a general rule, a petition for certiorari will not lie if an appeal is the proper remedy thereto such as when an error of
judgment as well as of procedure are involved. As long as a court acts within its jurisdiction and does not gravely abuse its
discretion in the exercise thereof, any supposed error committed by it will amount to nothing more than an error of judgment
reviewable by a timely appeal and not assailable by a special civil action of certiorari. However, in certain exceptional cases,
where the rigid application of such rule will result in a manifest failure or miscarriage of justice, the provisions of the Rules
of Court which are technical rules may be relaxed. Certiorari has been deemed to be justified, for instance, in order to
prevent irreparable damage and injury to a party where the trial judge has capriciously and whimsically exercised his
judgment, or where there may be danger of clear failure of justice, or where an ordinary appeal would simply be inadequate
to relieve a party from the injurious effects of the judgment complained of. 30 (Emphasis supplied) cEHSIC

Lack of jurisdiction and excess of jurisdiction are distinguished thus: the respondent acts without jurisdiction if he does not
have the legal power to determine the case; where the respondent, being clothed with the power to determine the case,
oversteps his authority as determined by law, he is performing a function in excess of his jurisdiction. 31 Thus, we now
discuss whether the trial court granted the motion for reconsideration of Sui and reinstated the complaint without basis in
14
law. Citing the case of Ace Navigation Co., Inc. v. Court of Appeals, 32 the trial court held that rules of procedures are mere
tools designed to facilitate the attainment of justice and must be relaxed if its strict and rigid application would frustrate
rather than promote substantial justice. Thus, it lifted and set aside its order of dismissal in the interest of substantial justice,
which is the legal basis for the trial court to grant the motion for reconsideration of Sui.

We have repeatedly warned against the injudicious and often impetuous issuance of default orders. 33 While it is desirable
that the Rules of Court be faithfully observed, courts should not be so strict about procedural lapses that do not really impair
the proper administration of justice. If the rules are intended to ensure the proper and orderly conduct of litigation, it is
because of the higher objective they seek which is the attainment of justice and the protection of substantive rights of the
parties. Thus, the relaxation of procedural rules, or saving a particular case from the operation of technicalities when
substantial justice requires it, as in the instant case, should no longer be subject to cavil. 34

When the Court of Appeals held that the case is dismissible because Sui did not attend the pre-trial conference, it failed to
consider the explanation of Atty. Ferrer that Sui executed a "Special Power of Attorney" in his behalf and that he was not
absent on the scheduled pre-trial but was only late. CHDAEc

Under Section 4 of Rule 18 of the Rules, 35 the non-appearance of a party at the pre-trial may be excused when there is a
valid cause shown or when a representative shall appear in his behalf, and is fully authorized in writing to enter into an
amicable settlement, to submit to alternative modes of dispute resolution, and to enter into stipulations or admissions of
facts and of documents. Although Sui was absent during the pre-trial, Atty. Ferrer alleged that he was fully authorized to
represent Sui. Moreover, it is not entirely accurate to state that Atty. Ferrer was absent during the pre-trial because he was
only late, the reasons for which he explained in his Manifestation and Motion for Reconsideration. The circumstances
attendant in the instant case compel this Court to relax the rules of procedure in the interest of substantial justice.

Petitioners claim that the motion for reconsideration of Sui was procedurally defective because it was not served three days
before the date of the hearing and no proof of service was given to the court, in violation of Sections 4 and 6 of Rule 15.
Petitioners also aver that they received the Manifestation and Motion for Reconsideration of Sui on May 27, 2004 but the
hearing was scheduled on May 28, 2004. Thus, it is nothing but a scrap of paper because it violated the three-day notice
rule.

We are not persuaded.

In the instant case, we find that the purpose of a notice of hearing had been served. In Vlason Enterprises Corporation v.
Court of Appeals, 36 we enumerated the exceptions to the rule on notice of hearing, to wit:

The Court has consistently held that a motion which does not meet the requirements of Sections 4 and 5 of Rule 15 of the
Rules of Court is considered a worthless piece of paper, which the clerk of court has no right to receive and the trial court
has no authority to act upon. Service of a copy of a motion containing a notice of the time and the place of hearing of that
motion is a mandatory requirement, and the failure of movants to comply with these requirements renders their motions
fatally defective. However, there are exceptions to the strict application of this rule. These exceptions are as follows:

". . . Liberal construction of this rule has been allowed by this Court in cases (1) where a rigid application will result in a
manifest failure or miscarriage of justice; especially if a party successfully shows that the alleged defect in the questioned
final and executory judgment is not apparent on its face or from the recitals contained therein; (2) where the interest of
substantial justice will be served; (3) where the resolution of the motion is addressed solely to the sound and judicious
discretion of the court; and (4) where the injustice to the adverse party is not commensurate [to] the degree of his
thoughtlessness in not complying with the procedure prescribed."

The present case falls under the first exception. Petitioner was not informed of any cause of action or claim against it. All
of a sudden, the vessels which petitioner used in its salvaging business were levied upon and sold in execution to satisfy
a supposed judgment against it. To allow this to happen simply because of a lapse in fulfilling the notice requirement —
which, as already said, was satisfactorily explained — would be a manifest failure or miscarriage of justice. cSDHEC

A notice of hearing is conceptualized as an integral component of procedural due process intended to afford the adverse
parties a chance to be heard before a motion is resolved by the court. Through such notice, the adverse party is permitted
time to study and answer the arguments in the motion.

Circumstances in the case at bar show that private respondent was not denied procedural due process, and that the very
purpose of a notice of hearing had been served. On the day of the hearing, Atty. Desierto did not object to the said Motion
for lack of notice to him; in fact, he was furnished in open court with a copy of the motion and was granted by the trial court
thirty days to file his opposition to it. These circumstances clearly justify a departure from the literal application of the notice
of hearing rule. In other cases, after the trial court learns that a motion lacks such notice, the prompt resetting of the hearing
with due notice to all the parties is held to have cured the defect.
15
Verily, the notice requirement is not a ritual to be followed blindly. Procedural due process is not based solely on a
mechanistic and literal application that renders any deviation inexorably fatal. Instead, procedural rules are liberally
construed to promote their objective and to assist in obtaining a just, speedy and inexpensive determination of any action
and proceeding. For the foregoing reasons, we believe that Respondent Court committed reversible error in holding that
the Motion for Reconsideration was a mere scrap of paper. 37 (Emphasis supplied)

When the trial court received Sui's Manifestation and Motion for Reconsideration, it did not immediately resolve the motion.
Instead, it allowed petitioners to file their comment and also leave to file a rejoinder if Sui files a reply. 38 These
circumstances justify a departure from the literal application of the rule because petitioners were given the opportunity to
study and answer the arguments in the motion. cDTaSH

Petitioners' claim that Sui failed to attach proof of service in violation of Section 6, Rule 15 of the Rule, must fail. In Republic
of the Philippines v. Court of Appeals, 39 we held, thus:

Nonetheless, considering the question raised in the appeal of the government and the amount involved in this case, we
think the Court of Appeals should have considered the subsequent service of the motion for reconsideration to be a
substantial compliance with the requirement in Rule 15, §6. In De Rapisura v. Nicolas, the movant also failed to attach to
his motion for reconsideration proof of service of a copy thereof to the other party. Nonetheless, this Court held the failure
not fatal as the adverse party had actually received a copy of the motion and was in fact present in court when the motion
was heard. It was held that the demands of substantial justice were satisfied by the actual receipt of said motion under
those conditions. 40

Petitioners admitted that they received a copy of Sui's Manifestation and Motion for Reconsideration. In fact, they had the
opportunity to oppose the same. Under these circumstances, we find that the demands of substantial justice and due
process were satisfied.

It is the policy of the Court to afford party-litigants the amplest opportunity to enable them to have their cases justly
determined, free from the constraints of technicalities. 41 It should be remembered that rules of procedure are but tools
designed to facilitate the attainment of justice, such that when rigid application of the rules tend to frustrate rather than
promote substantial justice, this Court is empowered to suspend their operation. 42

WHEREFORE, in view of the foregoing, the Decision dated September 22, 2005 and the Resolution dated October 27,
2005 of the Court of Appeals in CA-G.R. SP No. 88863 is REVERSED and SET ASIDE. The Order of the Regional Trial
Court in Civil Case No. 03-105691, lifting its previous order of dismissal is REINSTATED and AFFIRMED. DHcSIT

SO ORDERED.

Panganiban, C.J., Austria-Martinez, Callejo, Sr. and Chico-Nazario, JJ., concur.

4. Cuenca vs PCGG, G.R. Nos. 159104-05, October 5, 2007 535 SCRA 102

SECOND DIVISION

[G.R. Nos. 159104-05. October 5, 2007.]

RODOLFO M. CUENCA and CUENCA INVESTMENT CORP., petitioners, vs. THE PRESIDENTIAL COMMISSION ON
GOOD GOVERNMENT, INDEPENDENT REALTY CORP., and UNIVERSAL HOLDINGS CORP., respondents.

DECISION

VELASCO, JR., J p:

The Case

In this Petition for Review on Certiorari under Rule 45, petitioners assail the January 6, 2003 Decision 1 of the Court of
Appeals (CA) in consolidated cases CA-G.R. CV No. 60338 2 and CA-G.R. SP No. 49686 3 which upheld the jurisdiction
of Sandiganbayan over a dispute involving the transfer of stocks and subscription rights of respondent Universal Holdings
Corporation (UHC), a sequestered company, in favor of petitioners Rodolfo M. Cuenca and Cuenca Investment Corporation
(CIC); and its July 15, 2003 Resolution 4 denying petitioners' Motion for Reconsideration. 5 The consolidated cases
originated from Civil Case No. 91-2721 entitled Rodolfo M. Cuenca, et al. v. Independent Realty Corp., et al. filed before
the Makati City Regional Trial Court (RTC), Branch 61 — CA-G.R. CV No. 60338 being an appeal from the April 23, 1998
Decision rendered by the Makati City RTC, and CA-G.R. SP No. 49686 being a special civil action formerly filed as a petition

16
for certiorari before the Supreme Court, but was remanded to the CA for a review of the denial of the motion for intervention
filed by respondent Presidential Commission on Good Government (PCGG). aDSAEI

The Facts

Respondent UHC is a wholly owned subsidiary of Independent Realty Corporation (IRC). UHC had an authorized capital
stock of PhP200,000,000 of which 401,995 shares worth PhP40,199,500 were subscribed and PhP10,050,000 was paid
up by IRC. Five stockholders of IRC held qualifying shares in UHC and served in its Board of Directors. UHC became an
inactive holding company until the later months of 1978.

In 1978, petitioner Rodolfo M. Cuenca and his family's holding company, petitioner CIC, negotiated and reached an
agreement with respondents IRC and UHC, whereby petitioners Cuenca and CIC would purchase all the shares of stock
and subscription rights of IRC in UHC for PhP10,000,000 and assume IRC's unpaid subscription of PhP30,000,000.
Petitioners Cuenca and CIC were then the controlling stockholders of the Construction and Development Corporation of
the Philippines (CDCP), now the Philippine National Construction Corporation (PNCC), Sta. Ines Melale Forest Products
Corporation (Sta. Ines), and Resort Hotels Corporation (Resort Hotels). In order to build up UHC as his flagship company,
petitioner Cuenca transferred to UHC the shares of stocks in CDCP, Sta. Ines, and Resort Hotels worth PhP67,233,405,
with UHC assuming Cuenca's various bank obligations, some or all of which were secured by pledges or liens on the
stocks. DCIAST

On October 21, 1978, petitioner Cuenca was elected Chairperson and President of UHC at a special stockholders' meeting
in accordance with the acquisition plan, and through UHC, Cuenca continued to control and manage CDCP, Sta. Ines, and
Resort Hotels. Pursuant to the acquisition plan and agreement with IRC, Cuenca and CIC transferred their shares of stock
in CDCP, Sta. Ines, and Resort Hotels to UHC, which in turn paid PhP10,000,000 to IRC. In addition, petitioners assumed
IRC's unpaid subscription of PhP30,000,000 in UHC. The only remaining matter to be accomplished was the transfer of the
stocks and subscription rights of IRC in UHC to petitioners, but despite demand, IRC did not comply.

In 1986, the instant controversy between petitioners and respondent IRC was overtaken by dramatic political events.
President Marcos was ousted in a bloodless revolution and left behind an unbelievably large amount of funds and assets
that were sequestered by the new government of President Aquino through PCGG. In July 1987, because of Marcos
nominee Jose Yao Campos' sworn statement, respondent PCGG directed Santos Luis Diego, President of IRC, to dissolve
all the boards of directors of IRC's fully-owned subsidiaries. A year later, it turned over IRC and its subsidiary, UHC, to the
Asset Privatization Trust (APT) for rehabilitation, conservation, or disposition, enabling APT to assign one share of stock in
IRC and in each of its 25 subsidiaries, including UHC, to Paterno Bacani, Jr. AIHTEa

Amidst this state of affairs, petitioners filed the October 2, 1991 Complaint 6 against IRC, UHC, APT, and Bacani before
the Makati City RTC, which was docketed as Civil Case No. 91-2721, to compel IRC to transfer all its stock and subscription
rights in UHC to them or order IRC and UHC to return and re-convey to them all the assets and shares of stock in CDCP,
Sta. Ines, and Resort Hotels that they had transferred to UHC.

The Ruling of the Regional Trial Court

On November 29, 1991, respondents IRC and UHC filed a Joint Motion to Dismiss 7 on the ground of lack of jurisdiction,
claiming that the exclusive jurisdiction was lodged in the Sandiganbayan and not in the RTC. Meanwhile, on December 9,
1991, respondents IRC and UHC, represented by respondent PCGG, filed another Motion to Dismiss 8 on the ground of
litis pendentia as petitioner Cuenca had a pending case filed by respondent PCGG before the Sandiganbayan and docketed
as Civil Case No. 0016 entitled Republic of the Philippines v. Rodolfo M. Cuenca, et al., which involved respondent UHC
and several other corporations beneficially owned or controlled by petitioner Cuenca for and in behalf of the Marcoses.
Meanwhile, in the May 14, 1992 Order, the trial court dismissed the Complaint against APT and Bacani, and dropped them
as defendants on October 16, 1992. 9 On March 25, 1993, the trial court, however, denied both motions to dismiss on the
ground that respondent PCGG was not impleaded in the instant case and that the transaction involved specific performance
of a contract entered into in 1978 before the PCGG came into existence. aHCSTD

Consequently, on August 19, 1993, respondents IRC and UHC filed their Answer with Counterclaim. 10 Before pre-trial,
petitioners sent their Interrogatories 11 to IRC and UHC, which were answered by IRC on July 25, 1994. 12 After
considerable time had elapsed without UHC filing its answer to the interrogatories, and unsatisfied with IRC's answer not
accomplished, duly signed, and sworn to by a competent and responsible IRC officer as only IRC's counsel signed it,
petitioners filed on August 30, 1994 a Motion to Compel UHC to Answer Interrogatories 13 to which the trial court issued
two related Orders, the first dated January 17, 1995 directing IRC to submit proper and complete answers and UHC to
answer the interrogatories, 14 and the second dated February 10, 1995 granting respondents IRC and UHC an extension
of 15 days to file their answers to the interrogatories. 15

17
On September 29, 1995, petitioners filed a Motion to Declare Defendants in Default 16 for non-compliance with Section 5
of Rule 29, 17 Revised Rules of Civil Procedure. Respondents IRC and UHC filed their respective Answers to
Interrogatories 18 on October 17, 1995 or only after the motion to declare them in default was filed and served.
Consequently, the trial court issued its February 7, 1996 Order of default, which also granted petitioners the right to adduce
their evidence ex-parte. 19 On September 9, 1996, the trial court likewise denied 20 the Motion for Reconsideration and/or
Lift Order of Default 21 filed by respondents IRC and UHC. AHDTIE

Subsequently, respondent PCGG filed its Motion for Leave to Intervene with Motion to Dismiss on December 18, 1996,
which was denied by the trial court only on April 20, 1998. 22

Parenthetically, on October 22, 1996, petitioners filed an Urgent Ex-Parte Application for Receivership which was granted
through an October 28, 1996 Order, appointing Jaime C. Laya as UHC's receiver. After posting the requisite bond, the trial
court issued on November 5, 1996 an Order approving the bond, and receiver Laya submitted his November 13, 1996 Oath
of Office.

Petitioners adduced their evidence and presented the testimonies of petitioner Rodolfo Cuenca and Lourdes G. Labao, a
supervisor of Caval Securities Registry, Inc., who testified on the transfers of shares of stock of CDCP, Sta. Ines, and
Resort Hotels from Cuenca and CIC to UHC. On March 20, 1998, petitioners filed their Formal Offer of Exhibits. 23

On April 23, 1998, the trial court rendered a Decision in favor of petitioners. The fallo reads: HIAEcT

Accordingly, JUDGMENT is hereby rendered in favor of plaintiffs and as against defendants IRC and UHC, who are hereby
ordered to immediately return and reconvey to plaintiffs all of the shares of stocks and stock subscriptions in Philippine
National Construction Corporation (formerly known as Construction and Development [Corporation] of the Philippines),
Resort Hotels Corporation and Sta. Ines Melale Forest Products Corporation, including those transferred by plaintiffs to
UHC such as the 24,780,746 shares in CDCP/PNCC, the 468,062 shares in Resort Hotels Corporation and the 23,748,932
shares in Sta. Ines Melale Forest Products Corporation plus all fruits thereof such as stock and cash dividends and stock
splits.

The plaintiffs' prayer for damages and attorney's fees are hereby DENIED.

The counterclaim of defendants UHC and IRC for damages and attorney's fees is hereby DENIED for lack of evidence.

The appointment of JAIME C. LAYA as Receiver of defendant UHC is hereby MAINTAINED until finality of this Decision
and full execution of this Decision or full compliance herewith by defendants. 24

From the adverse Decision, respondents IRC and UHC appealed to the CA, which was docketed as CA-G.R. CV No.
60338. On the other hand, after the trial court denied respondent PCGG's Motion for Reconsideration 25 through its July
22, 1998 Order, 26 PCGG brought the instant case before this Court in G.R. No. 13516. Said PCGG special civil action
was remanded to the CA and docketed as CA-G.R. SP No. 49686 entitled Presidential Commission on Good Government
(PCGG) v. Hon. Fernando V. Gorospe, as Presiding Judge RTC of Makati City, Branch 61, et al. In the petition before the
CA, PCGG also assailed the April 20, 1998 Order of the trial court denying its motion for intervention in Civil Case No. 91-
2721. Thus, the petition for certiorari (CA-G.R. SP No. 49686) and the appeal (CA-G.R. CV No. 60338) were consolidated.
HaIATC

The Ruling of the Court of Appeals

Through its assailed Decision, the appellate court reversed the Makati City RTC's Decision, granted the petition filed by
PCGG, and dismissed the instant case for lack of jurisdiction. The appellate court ratiocinated that the Sandiganbayan had
exclusive jurisdiction to hear the instant case involving petitioners and the sequestered respondents corporations. It held
that the recourse of parties, petitioners in the instant case, who wish to challenge respondent PCGG's acts or orders, would
be to the Sandiganbayan pursuant to Executive Order No. (EO) 14 issued on May, 7, 1986, 27 which ordained that this
body alone had the original jurisdiction over all of respondent PCGG's cases, civil or criminal, citing PCGG v. Peña 28 as
authority. The appellate court applied Republic v. Sandiganbayan 29 on the issue of sequestration by respondent PCGG
of UHC, CIC, and CDCP (now PNCC) against petitioner Cuenca, the Marcos spouses, their relatives, friends, and
colleagues.

The CA applied the doctrine of conclusiveness of judgment that any rule which had already been authoritatively established
in a previous litigation should be deemed the law of the case between the same parties. As such, the appellate court
adopted the ruling in Republic on the continuing force of the order of sequestration and concluded that, indeed, respondent
UHC is a sequestered company. The CA did not find merit in petitioners' contention that sequestration did not affect their
transaction with respondents as it arose before PCGG was created. aEcTDI

18
Even if petitioners had initially a cause of action, the CA ruled that the complaint was certainly affected by the passage of
the law charging respondent PCGG with the performance of certain tasks over the subject matter of the action; and that
the same subject matter had become subject to the new exclusive jurisdiction vested in the Sandiganbayan at the time
petitioners filed the instant case.

Aggrieved, petitioners filed their Motion for Reconsideration 30 which was denied by the assailed July 15, 2003 CA
Resolution. 31 Hence, they filed this petition for review.

The Issues

Petitioners raise the following grounds for our consideration:

THE COURT OF APPEALS COMMITTED REVERSIBLE ERROR IN DISMISSING CIVIL CASE NO. 91-2721 BELOW ON
THE GROUND THAT THE SANDIGANBAYAN HAS EXCLUSIVE JURISDICTION OVER THE SUBJECT MATTER OF
THE CASE.

A.

THE FACT ALONE THAT RESPONDENT UHC MAY HAVE BEEN SEQUESTERED DID NOT DIVEST THE REGIONAL
TRIAL COURT OF ITS JURISDICTION OVER THE SUBJECT MATTER OF PETITIONERS' COMPLAINT IN CIVIL CASE
NO. 91-2721 BELOW. IDaEHS

B.

THE COURT OF APPEALS' RELIANCE ON THE CASE OF REPUBLIC VS. SANDIGANBAYAN, 240 SCRA 376 (1995),
IS MISPLACED.

C.

THE COURT OF APPEALS' APPLICATION OF THE DOCTRINE OF CONCLUSIVENESS OF JUDGMENT IS


ERRONEOUS. 32

The Court's Ruling

The petition must fail.

The core issue before us is that of jurisdiction. In gist, petitioners argue that UHC was not sequestered, and even if it was
sequestered, the trial court still has the jurisdiction to hear the case for rescission of contract or specific performance, and
conclude that the doctrine of conclusiveness of judgment does not apply in the instant case. TAcDHS

Issue of Jurisdiction

Jurisdiction is defined as the power and authority of a court to hear, try, and decide a case. 33 Jurisdiction over the subject
matter is conferred by the Constitution or by law while jurisdiction over the person is acquired by his/her voluntary
submission to the authority of the court or through the exercise of its coercive processes. Jurisdiction over the res is obtained
by actual or constructive seizure placing the property under the orders of the court. 34

We are primarily concerned here with the first kind of jurisdiction, that is, jurisdiction over the subject matter.

Petitioners contend that even if UHC was indeed sequestered, jurisdiction over the subject matter of petitioners' Complaint
for enforcement or rescission of contract between petitioners and respondents belonged to the RTC and not the
Sandiganbayan. Petitioners cited Philippine Amusement and Gaming Corporation v. Court of Appeals, 35 involving
Philippine Casino Operators Corporation (PCOC) which was sequestered on March 19, 1986. In said case, this Court held
that the fact of sequestration alone did not automatically oust the RTC of jurisdiction to decide upon the question of
ownership of the disputed gaming and office equipment as PCGG must be a party to the suit in order that the
Sandiganbayan's exclusive jurisdiction may be correctly invoked, and as Section 2 36 of EO 14 was duly applied in PCGG
v. Peña 37 and PCGG v. Nepomuceno, 38 which ineluctably spoke of respondent PCGG as a party-litigant. ESacHC

Likewise, petitioners cited Holiday Inn (Phils.), Inc. v. Sandiganbayan, 39 which also involved a sequestered company,
New Riviera Hotel and Development Co., Inc. (NRHDCI), where this Court held that there is a distinction between an action
for the recovery of ill-gotten wealth, as well as all incidents arising from, incidental to, or related to such cases, and cases
filed by those who wish to question or challenge respondent PCGG's acts or orders in such cases vis-à-vis ordinary civil
cases that do not pertain to the Sandiganbayan. As such, petitioners contend that the instant ordinary civil case for the
enforcement or rescission of the 1978 contract between petitioners and respondents UHC and IRC is distinct from and has
absolutely no bearing with the unrelated issue of the sequestration of respondents UHC and IRC. Thus, petitioners strongly
19
contend that the trial court indeed had jurisdiction over the instant case. Besides, petitioners point out that PCGG was not
impleaded as a defendant in Civil Case No. 91-2721, and that the Complaint "does not question the PCGG's alleged
sequestration of respondent UHC . . . or any other act or order of the PCGG." 40

Sandiganbayan has exclusive jurisdiction over the instant case

A rigorous examination of the antecedent facts and existing records at hand shows that Sandiganbayan has exclusive
jurisdiction over the instant case.

Thus, the petition must fail for the following reasons: CcAIDa

First, it is a fact that the shares of stock of UHC and CDCP, the subject matter of Civil Case No. 91-2721 before the Makati
City RTC, were also the subject matter of an ill-gotten wealth case, specifically Civil Case No. 0016 before the
Sandiganbayan. In Civil Case No. 91-2721 of the Makati City RTC, petitioners prayed for a judgment either transferring the
UHC shares or restoring and reconveying the PNCC shares to them. In the event a final judgment is rendered in said Makati
City RTC case in favor of petitioners, then such adjudication tends to render moot and academic the judgment to be
rendered in Sandiganbayan Civil Case No. 0016 considering that the legal ownership of either the UHC or PNCC shares
would now be transferred to petitioners Rodolfo Cuenca and CIC. Such adverse judgment would run counter to the rights
of ownership of the government over the UHC and PNCC shares in question. It must be remembered that on March 21,
1986, a Sworn Statement 41 executed by Mr. Jose Y. Campos in Vancouver, Canada, whereby Mr. Campos, a crony and
close business associate of the deposed President Marcos, named and identified IRC and UHC (a wholly-owned subsidiary
of IRC) as among the several corporations organized, established, and managed by him and other business associates for
and in behalf of the former President Marcos. Subsequently, the UHC and IRC shares were surrendered and turned over
by Mr. Campos to PCGG, transferring, in effect, the ownership of the shares to the Government. DHcESI

Moreover, inasmuch as UHC was impleaded in Civil Case No. 0016 as a defendant and was listed among the corporations
beneficially owned or controlled by petitioner Cuenca, the issue of the latter's right to acquire ownership of UHC shares is
inexorably intertwined with the right of the Republic of the Philippines, through PCGG, to retain ownership of said UHC
shares.

It must be borne in mind that the Sandiganbayan was created in 1978 pursuant to Presidential Decree No. (PD) 1606. 42
Said law has been amended during the interim period after the Edsa Revolution of 1986 and before the 1987 Constitution
was drafted, passed, and ratified. Thus, the executive issuances during such period before the ratification of the 1987
Constitution had the force and effect of laws. Specifically, then President Corazon C. Aquino issued the following Executive
Orders which amended PD 1606 in so far as the jurisdiction of the Sandiganbayan over civil and criminal cases instituted
and prosecuted by the PCGG is concerned, viz:

a) EO 1, entitled "Creating the Presidential Commission on Good Government," dated February 28, 1986;

b) EO 2, entitled "Regarding the Funds, Moneys, Assets, and Properties Illegally Acquired or Misappropriated by
Former President Ferdinand E. Marcos, Mrs. Imelda Romualdez Marcos, Their Close Relatives, Subordinates, Business
Associates, Dummies, Agents, or Nominees," dated March 12, 1986; IaEHSD

c) EO 14, entitled "Defining the Jurisdiction over Cases Involving the Ill-gotten Wealth of Former President Ferdinand
E. Marcos, Mrs. Imelda R. Marcos, Members of their Immediate Family, Close Relatives, Subordinates, Close and/or
Business Associates, Dummies, Agents and Nominees," dated May 7, 1986; and

d) EO 14-A, entitled "Amending Executive Order No. 14," dated August 18, 1986.

Bearing on the jurisdiction of the Sandiganbayan over cases of ill-gotten wealth, EO 14, Secs. 1 and 2 provide:

SECTION 1. Any provision of the law to the contrary notwithstanding, the Presidential Commission on Good
Government with the assistance of the Office of the Solicitor General and other government agencies, is hereby empowered
to file and prosecute all cases investigated by it under Executive Order No. 1, dated February 28, 1986 and Executive
Order No. 2, dated March 12, 1986, as may be warranted by its findings.

SECTION 2. The Presidential Commission on Good Government shall file all such cases, whether civil or criminal, with
the Sandiganbayan, which shall have exclusive and original jurisdiction thereof. (Emphasis supplied.)

Notably, these amendments had been duly recognized and reflected in subsequent amendments to PD 1606, specifically
Republic Act Nos. 7975 43 and 8249. 44 aCcEHS

In the light of the foregoing provisions, it is clear that it is the Sandiganbayan and not the Makati City RTC that has
jurisdiction over the disputed UHC and PNCC shares, being the alleged "ill-gotten wealth" of former President Ferdinand
20
E. Marcos and petitioner Cuenca. The fact that the Makati City RTC civil case involved the performance of contractual
obligations relative to the UHC shares is of no importance. The benchmark is whether said UHC shares are alleged to be
ill-gotten wealth of the Marcoses and their perceived cronies. More importantly, the interests of orderly administration of
justice dictate that all incidents affecting the UHC shares and PCGG's right of supervision or control over the UHC must be
addressed to and resolved by the Sandiganbayan. Indeed, the law and courts frown upon split jurisdiction and the resultant
multiplicity of suits, which result in much lost time, wasted effort, more expenses, and irreparable injury to the public interest.

Second, the UHC shares in dispute were sequestered by respondent PCGG. Sequestration is a provisional remedy or
freeze order issued by the PCGG designed to prevent the disposal and dissipation of ill-gotten wealth. 45 The power to
sequester property means to

place or cause to be placed under [PCGG's] possession or control said property, or any building or office wherein any such
property or any records pertaining thereto may be found, including business enterprises and entities, for the purpose of
preventing the destruction of, and otherwise conserving and preserving the same, until it can be determined, through
appropriate judicial proceedings, whether the property was in truth ill-gotten. (Silverio v. PCGG, 155 SCRA 60 [1987]). 46
IcHSCT

Considering that the UHC shares were already sequestered, enabling the PCGG to exercise the power of supervision,
possession, and control over said shares, then such power would collide with the legal custody of the Makati City RTC over
the UHC shares subject of Civil Case No. 91-2721. Whatever the outcome of Civil Case No. 91-2721, whether from
enforcement or rescission of the contract, would directly militate on PCGG's control and management of IRC and UHC,
and consequently hamper or interfere with its mandate to recover ill-gotten wealth. As aptly pointed out by respondents,
petitioners' action is inexorably entwined with the Government's action for the recovery of ill-gotten wealth — the subject of
the pending case before the Sandiganbayan. Verily, the transfer of shares of stock of UHC to petitioners or the return of
the shares of stock of CDCP (now PNCC) will wreak havoc on the sequestration case as both UHC and CDCP are subject
of sequestration by PCGG.

Third, Philippine Amusement and Gaming Corporation and Holiday Inn (Phils.), Inc. 47 are not analogous to the case at
bar. The first dealt with ownership of gaming and office equipment, which is distinct from and will not impact on the
sequestration issue of PCOC. The second dealt with an ordinary civil case for performance of a contractual obligation which
did not in any way affect the sequestration proceeding of NRHDCI; thus, the complaint-in-intervention of Holiday Inn (Phils.),
Inc. was properly denied for lack of jurisdiction over the subject matter. IDScTE

In both cases cited by petitioners, there was a substantial distinction between the sequestration proceedings and the subject
matter of the actions. This does not prevail in the instant case, as the ownership of the shares of stock of the sequestered
companies, UHC and CDCP, is the subject matter of a pending case and thus addressed to the exclusive jurisdiction of the
Sandiganbayan.

Sec. 2 of EO 14 pertinently provides: "The Presidential Commission on Good Government shall file all such cases, whether
civil or criminal, with the Sandiganbayan, which shall have exclusive and original jurisdiction thereof."

The above proviso has been squarely applied in Peña, 48 where this Court held that the exclusive jurisdiction conferred on
the Sandiganbayan would evidently extend not only to the principal causes of action, that is, recovery of alleged ill-gotten
wealth, but also to all incidents arising from, incidental to, or related to such cases, including a dispute over the sale of the
shares, the propriety of the issuance of ancillary writs of relative provisional remedies, and the sequestration of the shares,
which may not be made the subject of separate actions or proceedings in another forum. Indeed, the issue of the ownership
of the sequestered companies, UHC and PNCC, as well as IRC's ownership of them, is undeniably related to the recovery
of the alleged ill-gotten wealth and can be squarely addressed via the exclusive jurisdiction of the Sandiganbayan. aHSAIT

Fourth, while it is clear that the exclusive jurisdiction of the Sandiganbayan only encompasses cases where PCGG is
impleaded, such requirement is satisfied in the instant case. The appellate court clearly granted PCGG's petition for
certiorari in CA-G.R. SP No. 49686, assailing the trial court's denial of its Motion for Leave to Intervene with Motion to
Dismiss. Thus, the trial court's April 20, 1998 Order was reversed and set aside by the appellate court through its assailed
Decision. Consequently, PCGG was granted the right to intervene and thus became properly impleaded in the instant case.
Without doubt, the trial court has no jurisdiction to hear and decide Civil Case No. 91-2721. DCASIT

Respondent UHC duly sequestered by PCGG

The trial court ruled that respondent PCGG could not stop the transfer of the shares of respondent UHC in CDCP to
petitioners as there was no proof of sequestration except a writ of sequestration of Cuenca's stocks in CDCP. On the other
hand, petitioners contend that the appellate court's reliance on Republic 49 is misplaced. They point out that neither PCGG
nor respondent corporations relied on said case. Besides, petitioners contend that the Court's statements in said case did

21
not constitute a ruling but mere references to unproven allegations by PCGG in its complaint against Cuenca in
Sandiganbayan Civil Case No. 0016; and as such, it cannot be relied upon to hold that UHC was a sequestered corporation.
As it is, petitioners conclude that it was a mere obiter dictum which was not essential to the disposition of the aforecited
case and thus, it is not binding upon the parties for purposes of res judicata or conclusiveness of judgment.

We are not moved by petitioners' submission.

While it may be true that in Republic, our statement on Civil Case No. 0016, as cited by PCGG, refers to the allegations in
the complaint filed by PCGG against petitioner Cuenca, 50 we nonetheless stated in said case the fact of the sequestration
of the assets and records of Rodolfo Cuenca, UHC, CIC, CDCP, San Mariano Mining Corp., etc. on May 23, 1986 and July
23, 1987. We took factual notice of the sequestration of various companies and properties in said case, thus: TSIaAc

III. Orders of Sequestration issued by PCGG

During 1986 and 1987 numerous orders of sequestration, freezing or provisional takeover of companies or properties, real
or personal, were issued and implemented. Among those were the orders handed out against the firms or assets hereunder
listed, with the dates of sequestration, freezing or take-over, to wit:

SUBJECTS/OBJECTS OF SEQUESTRATION DATE

xxx xxx xxx

i. Assets and records of Rodolfo Cuenca, May 23, 1986,

Universal Holdings Corp., Cuenca July 23, 1987

Investment Corporation, Philippine

National Construction Corp. (formerly

CDCP), San Mariano Mining Corp., etc. 51

From the foregoing account, we concluded that UHC had indeed been sequestered by the PCGG in 1986 and 1987.
Consequently, the appellate court properly applied Republic as basis for its finding that UHC was a sequestered company.
Since the issue of sequestration has been resolved, we see no need to delve into the issue of conclusiveness of judgment.
Suffice it to say that with the unequivocal finding that UHC was indeed sequestered, then it is the Sandiganbayan, not the
Makati City RTC, that has exclusive jurisdiction over the subject matter of Civil Case No. 91-2721. DISHEA

WHEREFORE, the instant petition is DISMISSED for lack of merit. The January 6, 2003 Decision and July 15, 2003
Resolution of the CA in CA-G.R. CV No. 60338 and CA-G.R. SP No. 49686 are AFFIRMED IN TOTO. No costs.

SO ORDERED.

Puno, C.J., * Carpio-Morales, Tinga and Ynares-Santiago, * JJ., concur.

5. Arranza v. B.F. Homes, G.R. No. 131683, June 19, 2000, 333 SCRA 799.

FIRST DIVISION

[G.R. No. 131683. June 19, 2000.]

JESUS LIM ARRANZA; LORENZO CINCO; QUINTIN TAN; JOSE ESCOBAR; ELBERT FRIEND; CLASSIC HOMES
VILLAGE ASSOCIATION, INC.; BF NORTHWEST HOMEOWNERS' ASSOCIATION, INC.; and UNITED BF
HOMEOWNERS' ASSOCIATIONS, INC., petitioners, vs. B.F. HOMES, INC. AND THE HONORABLE COURT OF
APPEALS, respondent.

Robles Ricafrente Aguirre San Vicente and Cacho Law Firm and Veneranda Acaylar-Cruz for petitioners.

Benjamin B. Bernardino for respondents.

Antonio R. Bautista & Partners for BF Homes, Inc.

Ata & Habawel for Intervenors.

SYNOPSIS
22
The issue involved in this petition is whether it is the SEC or the HLURB that has jurisdiction over a complaint for specific
performance filed by subdivision homeowners against a subdivision developer to enforce the latter's obligations to provide
them their basic needs as purchasers of the lots: water, security and open spaces. Respondent BF Homes, Inc. (BFHI)
claimed that inasmuch as BFHI is under receivership, the case is exclusively within the jurisdiction of the SEC.

The Supreme Court upheld HLURB's jurisdiction over the specific performance case as the SEC proceeds with the
rehabilitation of BFHI. Applicable laws confer upon the HLURB jurisdiction over matters relating to the observance of laws
governing corporations engaged in developing subdivisions and condominiums.

As BFHI undergoes rehabilitation receivership, BFHI continues to exist as a corporation, hence, should continue to perform
its obligations to petitioners as homeowners. Petitioners' complaint against BFHI should not be considered as "claims" of
creditors which should be suspended as provided in Sec. 6 (c) of P.D. No. 902-A. Such claims referred solely to monetary
claims, which are but incidental to petitioners' complaint against BFHI. Once monetary awards are determined in the
HLURB, however, the same should be submitted to the SEC as established claims. cdrep

SYLLABUS

1. ADMINISTRATIVE LAW; ADMINISTRATIVE AGENCIES; HOUSING AND LAND USE REGULATORY BOARD
(HLURB); JURISDICTION OF HLURB IS DETERMINED BY APPLICABLE LAWS AND THE ALLEGATIONS OF THE
COMPLAINT. — Jurisdiction is the authority to hear and determine a cause — the right to act in a case. It is conferred by
law and not by mere administrative policy of any court or tribunal. It is determined by the averments of the complaint and
not by the defense contained in the answer. Hence, the jurisdictional issue involved here shall be determined upon an
examination of the applicable laws and the allegations of petitioners' complaint before the HLURB.

2. ID.; ID.; ID.; ID.; THE HLURB HAS JURISDICTION OVER CASES ARISING FROM CONTRACTS BETWEEN
SUBDIVISION DEVELOPER AND LOT OWNERS; CASE AT BAR. — In the cases that reached this Court, the ruling has
consistently been that the NHA or the HLURB has jurisdiction over complaints arising from contracts between the
subdivision developer and the lot buyer or those aimed at compelling the subdivision developer to comply with its
contractual and statutory obligations to make the subdivision a better place to live in. . . . In the case at bar, petitioners'
complaint is for specific performance to enforce their rights as purchasers of subdivision lots as regards rights of way, water,
open spaces, road and perimeter wall repairs, and security. Indisputably then, the HLURB has jurisdiction over the
complaint. . . . The fact that respondent is under receivership does not divest the HLURB of that jurisdiction.

3. CORPORATION LAW; RECEIVER; DEFINITION AND DUTIES; EFFECT OF THE APPOINTMENT OF A


RECEIVER ON THE CORPORATION'S EXISTENCE AND ON ITS OBLIGATIONS; PURPOSE OF RECEIVERSHIP. — A
receiver is a person appointed by the court, or in this instance, by a quasi-judicial administrative agency, in behalf of all the
parties for the purpose of preserving and conserving the property and preventing its possible destruction or dissipation, if it
were left in the possession of any of the parties. It is the duty of the receiver to administer the assets of the receivership
estate; and in the management and disposition of the property committed to his possession, he acts in a fiduciary capacity
and with impartiality towards all interested persons. The appointment of a receiver does not dissolve a corporation, nor
does it interfere with the exercise of its corporate rights. In this case where there appears to be no restraints imposed upon
respondent as it undergoes rehabilitation receivership, respondent continues to exist as a corporation and hence, continues
or should continue to perform its contractual and statutory responsibilities to petitioners as homeowners. Receivership is
aimed at the preservation of, and at making more secure, existing rights; it cannot be used as an instrument for the
destruction of those rights.

4. ID.; RECEIVERSHIP; SUSPENSION OF CLAIMS IN PAYMENTS; ACTION FOR SPECIFIC PERFORMANCE


BEFORE HLURB IS NOT PECUNIARY IN NATURE AS TO WARRANT THE SUSPENSION OF PROCEEDINGS
PROVIDED IN SEC. 6(C) OF P.D. NO. 902-A, AS AMENDED; CASE AT BAR. — Neither may petitioners be considered
as having "claims" against respondent within the context of the following proviso of Section 6 (c) of P.D. No. 902-A, as
amended by P.D. Nos. 1653, 1758 and 1799, to warrant suspension of the HLURB proceedings. . . . No violation of the
SEC order suspending payments to creditors would result as far as petitioners' complaint before the HLURB is concerned.
To reiterate, what petitioners seek to enforce are respondent's obligations as a subdivision developer. Such claims are
basically not pecuniary in nature although it could incidentally involve monetary considerations. All that petitioners' claims
entail is the exercise of proper subdivision management on the part of the SEC-appointed Board of Receivers towards the
end that homeowners shall enjoy the ideal community living that respondent portrayed they would have when they bought
real estate from it. . . . Whatever monetary awards the HLURB may impose upon respondent are incidental matters that
should be addressed to the sound discretion of the Board of Receivers charged with maintaining the viability of respondent
as a corporation. Any controversy that may arise in that regard should then be addressed to the SEC. It is worth noting that
the parties agreed at the 1 July 1998 hearing that should the HLURB establish and grant petitioners' claims, the same

23
should be referred to the SEC. Thus, the proceedings at the HLURB should not be suspended notwithstanding that
respondent is still under receivership.

5. ID.; P.D. NO. 902-A, AS AMENDED; JURISDICTION OF THE SECURITIES AND EXCHANGE COMMISSION
(SEC), ELEMENTS; THE SEC HAS NO JURISDICTION OVER COMPLAINTS FOR SPECIFIC PERFORMANCE FILED
BY LOT BUYERS AGAINST SUBDIVISION DEVELOPERS. — On the other hand the jurisdiction of the SEC is defined by
P.D. No. 902-A, as amended. . . . For the SEC to acquire jurisdiction over any controversy under these provisions, two
elements must be considered: (1) the status or relationship of the parties; and (2) the nature of the question that is the
subject of their controversy. The first element requires that the controversy must arise "out of intra-corporate or partnership
relations between and among stockholders, members or associates; between any or all of them and the corporation,
partnership or association of which they are stockholders, members or associates, respectively; and between such
corporation, partnership or association and the State in so far as it concerns their individual franchises." Petitioners are not
stockholders, members or associates of respondent. They are lot buyers and now homeowners in the subdivision
developed by the respondent. The second element requires that the dispute among the parties be intrinsically connected
with the regulation or the internal affairs of the corporation, partnership or association. The controversy in this case is
remotely related to the "regulation" of respondent corporation or to respondent's "internal affairs." . . . Their claim for
reimbursement should be viewed in the light of respondent's alleged failure to observe its statutory and contractual
obligations to provide petitioners a "decent human settlement" and "ample opportunities for improving their quality of life."
The HLURB, not the SEC, is equipped with the expertise to deal with that matter. cETCID

6. ID.; ID.; ID.; ID.; PROVISIONS OF P.D. NO. 902-A AND P.D. NO. 957, ARE LAWS IN PARI MATERIA. — It should
be stressed that the main concern in this case is the issue of jurisdiction over petitioners' complaint against respondent for
specific performance. P.D. No. 902-A, as amended, defines the jurisdiction of the SEC; while P.D. No. 957, as amended,
delineates that of the HLURB. These two quasi-judicial agencies exercise functions that are distinct from each other. The
SEC has authority over the operation of all kinds of corporations, partnerships or associations with the end in view of
protecting the interests of the investing public and creditors. On the other hand, the HLURB has jurisdiction over matters
relating to observance of laws governing corporations engaged in the specific business of development of subdivisions and
condominiums. The HLURB and the SEC being bestowed with distinct powers and functions, the exercise of those functions
by one shall not abate the performance by the other of its own functions. As respondent puts it, "there is no contradiction
between P.D. No. 902-A and P.D. No. 957." . . . P.D. Nos. 902-A and 957, as far as both are concerned with corporations,
are laws in pari materia. P.D. No. 902-A relates to all corporations, while P.D. No. 957 pertains to corporations engaged in
the particular business of developing subdivisions and condominiums. Although the provisions of these decrees on the
issue of jurisdiction appear to collide when a corporation engaged in developing subdivisions and condominiums is under
receivership, the same decrees should be construed as far as reasonably possible to be in harmony with each other to
attain the purpose of an expressed national policy.

DECISION

DAVIDE, JR., C.J p:

For resolution in this petition is the issue of whether it is the Securities and Exchange Commission (SEC) or the Housing
and Land Use Regulatory Board (HLURB) that has jurisdiction over a complaint filed by subdivision homeowners against
a subdivision developer that is under receivership for specific performance regarding basic homeowners' needs such as
water, security and open spaces. cdrep

Respondent BF Homes, Inc. (BFHI), is a domestic corporation engaged in developing subdivisions and selling residential
lots. One of the subdivisions that respondent developed was the BF Homes Parañaque Subdivision, which now sprawls
across not only a portion of the City of Parañaque but also those of the adjoining cities of Las Piñas and Muntinlupa.

When the Central Bank ordered the closure of Banco Filipino, which had substantial investments in respondent BFHI,
respondent filed with the SEC a petition for rehabilitation and a declaration that it was in a state of suspension of payments.
On 18 March 1985, the SEC placed respondent under a management committee. Upon that committee's dissolution on 2
February 1988, the SEC appointed Atty. Florencio B. Orendain as a Receiver, and approved a Revised Rehabilitation Plan.

As a Receiver, Orendain instituted a central security system and unified the sixty-five homeowners' associations into an
umbrella homeowners' association called United BF Homeowners' Associations, Inc. (UBFHAI), which was thereafter
incorporated with the Home Insurance and Guaranty Corporation (HIGC). 1

In 1989, respondent, through Orendain, turned over to UBFHAI control and administration of security in the subdivision,
the Clubhouse and the open spaces along Concha Cruz Drive. Through the Philippine Waterworks and Construction
Corporation (PWCC), respondent's managing company for waterworks in the various BF Homes subdivisions, respondent

24
entered into an agreement with UBFHAI for the annual collection of community assessment fund and for the purchase of
eight new pumps to replace the over-capacitated pumps in the old wells.

On 7 November 1994, Orendain was relieved by the SEC of his duties as a Receiver, and a new Board of Receivers
consisting of eleven members of respondent's Board of Directors was appointed for the implementation of Phases II and
III of respondent's rehabilitation. 2 The new Board, through its Chairman, Albert C. Aguirre, revoked the authority given by
Orendain to use the open spaces at Concha Cruz Drive and to collect community assessment funds; deferred the purchase
of new pumps; recognized BF Parañaque Homeowners' Association, Inc., (BFPHAI) as the representative of all
homeowners in the subdivision; took over the management of the Clubhouse; and deployed its own security guards in the
subdivision.

Consequently, on 5 July 1995, herein petitioners filed with the HLURB a class suit "for and in behalf of the more than 7,000
homeowners in the subdivision" against respondent BFHI, BF Citiland Corporation, PWCC and A.C. Aguirre Management
Corporation "to enforce the rights of purchasers of lots" in BF Homes Parañaque. 3 They alleged that:

1. The forty (40) wells, mostly located at different elevations in Phases 3 and 4 of the subdivision and with only twenty-
seven (27) productive, are the sources of the inter-connected water system in the 765-hectare subdivision;

2. There is only one drainage and sewer system;

3. There is one network of roads;

4. There are eight (8) entry and exit points to the subdivision and from three (3) municipalities (now cities), a situation
obtaining in this subdivision only and nowhere else;

5. There was no security force for the entire subdivision until 1988;

6. There are not enough open spaces in the subdivision in relation to the total land area developed; and whatever
open spaces are available have been left unkempt, undeveloped and neglected;

7. There are no zoning guidelines which resulted in unregulated constructions of structures and the proliferation of
business establishments in residential areas; and

8. The BFPHAI became "moribund" sometime in 1980 on account of its failure to cope with the delivery of basic
services except for garbage collection.

Petitioners raised "issues" on the following basic needs of the homeowners: rights-of-way; water; open spaces; road and
perimeter wall repairs; security; and the interlocking corporations that allegedly made it convenient for respondent "to
compartmentalize its obligations as general developer, even if all of these are hooked into the water, roads, drainage and
sewer systems of the subdivision." 4 Thus, petitioners prayed that:

A. A cease-and-desist order from selling any of the properties within the subdivision be issued against respondent
BFHI, BF Citi, ACAMC, and/or any and all corporations acting as surrogates/alter-egos, sister companies of BFHI and/or
its stockholders until the warranties, facilities and infrastructures shall have been complied with or put up (and) the advances
of UBFHAI reimbursed, otherwise, to cease and desist from rescinding valid agreements or contracts for the benefit of
complainants, or committing acts diminishing, diluting or otherwise depriving complainants of their rights under the law as
homeowners;

B. After proper proceedings the bond or deposit put up by respondent BF Homes, Inc. be forfeited in favor of
petitioners;

C. Respondent BFHI be ordered to immediately turnover the roads, open spaces, and other facilities built or put up
for the benefit of lot buyers/homeowners in the subdivision to complainant UBFHAI as representative of all homeowners in
BF Homes Parañaque, free from all liens, encumbrances, and taxes in arrears;

D. If the open spaces in the subdivision are not sufficient as required by law, to impose said penalties/sanctions
against BFHI or the persons responsible therefor;

E. Order the reimbursement of advances made by UBFHAI;

F. Turn over all amounts which may have been collected from users' fees of the strip of open space at Concha Cruz
Drive;

G. Order PWCC to effect and restore 24-hour water supply to all residents by adding new wells replacing over-
capacitated pumps and otherwise improving water distribution facilities;
25
H. Order PWCC to continue collecting the Community Development Fund and remit all amounts collected to UBFHAI;

I. Order BFHI to immediately withdraw the guards at the clubhouse and the 8 entry and exit points to the subdivision,
this being an act of usurpation and blatant display of brute force;

J. The appropriate penalties/sanctions be imposed against BF Citi, ACAMC or any other interlocking corporation of
BFHI or any of its principal stockholders in respect of the diminution/encroaching/violation on the rights of the residents of
the subdivision to enjoy/avail of the facilities/services due them; and

K. Respondents be made to pay attorney's fees and the costs of this suit. 5

In its answer, respondent claimed that (a) it had complied with its contractual obligations relative to the subdivision's
development; (b) respondent could not be compelled to abide by agreements resulting from Orendain's ultra vires acts; and
(c) petitioners were precluded from instituting the instant action on account of Section 6(c) of P.D. No. 902-A providing for
the suspension of all actions for claims against a corporation under receivership. Respondent interposed counterclaims
and prayed for the dismissal of the complaint. 6

Petitioners thereafter filed an urgent motion for a cease-and-desist/status quo order. Acting on this motion, HLURB Arbiter
Charito M. Bunagan issued a 20-day temporary restraining order to avoid rendering nugatory and ineffectual any judgment
that could be issued in the case; 7 and subsequently, an Order granting petitioners' prayer for preliminary injunction was
issued

enjoining and restraining respondent BF Homes, Incorporated, its agents and all persons acting for and in its behalf from
taking over/administering the Concha Garden Row, from issuing stickers to residents and non-residents alike for free or
with fees, from preventing necessary improvements and repairs of infrastructures within the authority and administration of
complainant UBFHAI, and from directly and indirectly taking over security in the eight (8) exit points of the subdivision or in
any manner interfering with the processing and vehicle control in subject gates and otherwise to remove its guards from
the gates upon posting of a bond of One Hundred Thousand Pesos (P100,000.00) which bond shall answer for whatever
damages respondents may sustain by reason of the issuance of the writ of preliminary injunction if it turns out that
complainant is not entitled thereto. 8

Respondent thus filed with the Court of Appeals a petition for certiorari and prohibition docketed as CA-G.R. SP No. 39685.
It contended in the main that the HLURB acted "completely without jurisdiction" in issuing the Order granting the writ of
preliminary injunction considering that inasmuch as respondent is under receivership, the "subject matter of the case is one
exclusively within the jurisdiction of the SEC." 9

On 28 November 1997, the Court of Appeals rendered a decision 10 annulling and setting aside the writ of preliminary
injunction issued by the HLURB. It ruled that private respondents' action may properly be regarded as a "claim" within the
contemplation of PD No. 902-A which should be placed on equal footing with those of petitioners' other creditor or creditors
and which should be filed with the Committee of Receivers. In any event, pursuant to Section 6(c) of P.D. No. 902-A and
SEC's Order of 18 March 1985, petitioners' action against respondent, which is under receivership, should be suspended.

Hence, petitioners filed the instant petition for review on certiorari. On 26 January 1998, the Court issued a temporary
restraining order (TRO) enjoining respondent, its officers, representatives and persons acting upon its orders from.

(a) taking over/administering the Concha Garden Row; (b) issuing stickers to residents and non-residents alike for free or
with fees; (c) preventing necessary improvements and repairs of infrastructures within the authority and administration of
complainant United BF Homeowners' Association, Inc. (UBFHAI); (d) directly and indirectly taking over security in the eight
(8) exit points of all of BF Homes Parañaque Subdivision or in any-manner interfering with the processing and vehicle
control in the subject gates; and (e) otherwise to remove its guards from the gates . . . 11

Respondent's motion to lift the TRO was denied.

At the hearing on 1 July 1998, the primary issue in this case was defined as "which body has jurisdiction over petitioners'
claims, the Housing and Land Use Regulatory Board (HLURB) or the Securities and Exchange Commission (SEC)?" The
collateral issue to be addressed is "assuming that the HLURB has jurisdiction, may the proceedings therein be suspended
pending the outcome of the receivership before the SEC?"

For their part, petitioners argue that the complaint referring to rights of way, water, open spaces, road and perimeter wall
repairs, security and respondent's interlocking corporations that facilitated circumvention of its obligation involves unsound
real estate practices. The action is for specific performance of a real estate developers' obligations under P.D. No. 957,
and the relief sought is revocation of the subdivision project's registration certificate and license to sell. These issues are
within the jurisdiction of the HLURB. Even if respondent is under receivership, its obligations as a real estate developer

26
under P.D. No. 957 are not suspended. Section 6(c) of P.D. No. 902-A, as amended by P.D. No. 957, on "suspension of
all actions for claims against corporations" refers solely to monetary claims which are but incidental to petitioner's
complaints against BFHI, and if filed elsewhere than the HLURB, it would result to splitting causes of action. Once
determined in the HLURB, however, the monetary awards should be submitted to the SEC as established claims. Lastly,
the acts enjoined by the HLURB are not related to the disposition of BFHI's assets as a corporation undergoing its final
phase of rehabilitation.

On the other hand, respondent asserts that the SEC, not the HLURB, has jurisdiction over petitioners' complaint based on
the contracts entered into by the former receiver. The SEC, being the appointing authority, should be the one to take
cognizance of controversies arising from the performance of the receiver's duties. Since respondent's properties are under
the SEC's custodia legis, they are exempt from any court process. cdll

Jurisdiction is the authority to hear and determine a cause — the right to act in a case. 12 It is conferred by law and not by
mere administrative policy of any court or tribunal. 13 It is determined by the averments of the complaint and not by the
defense contained in the answer. 14 Hence, the jurisdictional issue involved here shall be determined upon an examination
of the applicable laws and the allegations of petitioners' complaint before the HLURB.

Presidential Decree No. 957 (The Subdivision and Condominium Buyers' Protective Decree) was issued on 12 July 1976
in answer to the popular call for correction of pernicious practices of subdivision owners and/or developers that adversely
affected the interests of subdivision lot buyers. Thus, one of the "whereas clauses" of P.D. No. 957 states:

WHEREAS, numerous reports reveal that many real estate subdivision owners, developers, operators, and/or sellers have
reneged on their representations and obligations to provide and maintain properly subdivision roads, drainage, sewerage,
water systems, lighting systems, and other similar basic requirements, thus endangering the health and safety of home and
lot buyers . . .

Section 3 of P.D. No. 957 empowered the National Housing Authority (NHA) with the "exclusive jurisdiction to regulate the
real estate trade and business." On 2 April 1978, P.D. No. 1344 was issued to expand the jurisdiction of the NHA to include
the following:

SECTION 1. In the exercise of its functions to regulate the real estate trade and business and in addition to its powers
provided for in Presidential Decree No. 957, the National Housing Authority shall have exclusive jurisdiction to hear and
decide cases of the following nature:

A. Unsound real estate business practices;

B. Claims involving refund and any other claims filed by subdivision lot or condominium unit buyer against the project
owner, developer, dealer, broker or salesman; and

C. Cases involving specific performance of contractual and statutory obligations filed by buyers of subdivision lot or
condominium unit against the owner, developer, dealer, broker or salesman. (Italics supplied.)

Thereafter, the regulatory and quasi-judicial functions of the NHA were transferred to the Human Settlements Regulatory
Commission (HSRC) by virtue of Executive Order No. 648 dated 7 February 1981. Section 8 thereof specifies the functions
of the NHA that were transferred to the HSRC including the authority to hear and decide "cases on unsound real estate
business practices; claims involving refund filed against project owners, developers, dealers, brokers or salesmen and
cases of specific performance." Executive Order No. 90 dated 17 December 1986 renamed the HSRC as the Housing and
Land Use Regulatory Board (HLURB). 15

The boom in the real estate business all over the country resulted in more litigation between subdivision owners/developers
and lot buyers with the issue of the jurisdiction of the NHA or the HLURB over such controversies as against that of regular
courts. In the cases 16 that reached this Court, the ruling has consistently been that the NHA or the HLURB has jurisdiction
over complaints arising from contracts between the subdivision developer and the lot buyer or those aimed at compelling
the subdivision developer to comply with its contractual and statutory obligations to make the subdivision a better place to
live in.

Notably, in Antipolo Really Corporation v. National Housing Authority 17 one of the issues raised by the homeowners was
the failure of Antipolo Realty to develop the subdivision in accordance with its undertakings under the contract to sell. Such
undertakings include providing the subdivision with concrete curbs and gutters, underground drainage system, asphalt
paved roads, independent water system, electrical installation with concrete posts, landscaping and concrete sidewalks,
developed park or amphitheater and 24-hour security guard service. The Court held that the complaint filed by the
homeowners was within the jurisdiction of the NHA.

27
Similarly, in Alcasid v. Court of Appeals, 18 the Court ruled that the HLURB, not the RTC, has jurisdiction over the complaint
of lot buyers for specific performance of alleged contractual and statutory obligations of the defendants, to wit, the execution
of contracts of sale in favor of the plaintiffs and the introduction in the disputed property of the necessary facilities such as
asphalting and street lights.

In the case at bar, petitioners' complaint is for specific performance to enforce their rights as purchasers of subdivision lots
as regards rights of way, water, open spaces, road and perimeter wall repairs, and security. Indisputably then, the HLURB
has jurisdiction over the complaint.

The fact that respondent is under receivership does not divest the HLURB of that jurisdiction. A receiver is a person
appointed by the court, or in this instance, by a quasi-judicial administrative agency, in behalf of all the parties for the
purpose of preserving and conserving the property and preventing its possible destruction or dissipation, if it were left in
the possession of any of the parties. 19 It is the duty of the receiver to administer the assets of the receivership estate; and
in the management and disposition of the property committed to his possession, he acts in a fiduciary capacity and with
impartiality towards all interested persons. 20 The appointment of a receiver does not dissolve a corporation, nor does it
interfere with the exercise of its corporate rights. 21 In this case where there appears to be no restraints imposed upon
respondent as it undergoes rehabilitation receivership, 22 respondent continues to exist as a corporation and hence,
continues or should continue to perform its contractual and statutory responsibilities to petitioners as homeowners.

Receivership is aimed at the preservation of, and at making more secure, existing rights; it cannot be used as an instrument
for the destruction of those rights. 23

No violation of the SEC order suspending payments to creditors would result as far as petitioners' complaint before the
HLURB is concerned. To reiterate, what petitioners seek to enforce are respondent's obligations as a subdivision developer.
Such claims are basically not pecuniary in nature although it could incidentally involve monetary considerations. All that
petitioners' claims entail is the exercise of proper subdivision management on the part of the SEC-appointed Board of
Receivers towards the end that homeowners shall enjoy the ideal community living that respondent portrayed they would
have when they bought real estate from it.

Neither may petitioners be considered as having "claims" against respondent within the context of the following proviso of
Section 6 (c) of P.D. No. 902-A, as amended by P.D. Nos. 1653, 1758 and 1799, to warrant suspension of the HLURB
proceedings:

[U]pon appointment of a management committee, rehabilitation receiver, board or body, pursuant to this Decree, all actions
for claims against corporations, partnerships or associations under management or receivership pending before any court,
tribunal, board or body shall be suspended accordingly. (Italics supplied.)

In Finasia Investments and Finance Corporation v. Court of Appeals, 24 this Court defined and explained the term "claim"
in Section 6 (c) of P.D. No. 902-A, as amended, as follows:

We agree with the public respondent that the word "claim" as used in Sec. 6 (c) of P.D. 902-A, as amended, refers to debts
or demands of a pecuniary nature. It means "the assertion of a right to have money paid. It is used in special proceedings
like those before administrative court, on insolvency." (Italics supplied.)

Hence, in Finasia Investments, the Court held that a civil case to nullify a special power of attorney because the principal's
signature was forged should not be suspended upon the appointment of a receiver of the mortgagee to whom a person
mortgaged the property owned by such principal. The Court ruled that the cause of action in that civil case "does not consist
of demand for payment of debt or enforcement of pecuniary liability." It added:

It has nothing to do with the purpose of Section 6 (c) of P.D. 902-A, as amended, which is to prevent a creditor from
obtaining an advantage or preference over another with respect to action against corporation, partnership, association
under management or receivership and to protect and preserve the rights of party litigants as well as the interest of the
investing public or creditors. Moreover, a final verdict on the question of whether the special power of attorney in question
is a forgery or not will not amount to any preference or advantage to Castro who was not shown to be a creditor of FINASIA.
25

In this case, under the complaint for specific performance before the HLURB, petitioners do not aim to enforce a pecuniary
demand. Their claim for reimbursement should be viewed in the light of respondent's alleged failure to observe its statutory
and contractual obligations to provide petitioners a "decent human settlement" and "ample opportunities for improving their
quality of life." 26 The HLURB, not the SEC, is equipped with the expertise to deal with that matter.

On the other hand, the jurisdiction of the SEC is defined by P.D. No. 902-A, as amended, as follows:

28
SEC. 5. In addition to the regulatory and adjudicative functions of the Securities and Exchange Commission over
corporations, partnerships and other forms of associations registered with it as expressly granted under existing laws and
decrees, it shall have original and exclusive jurisdiction to hear and decide cases involving:

a) Devices or schemes employed by or any act of the board of directors, business associates, its officers or partners,
amounting to fraud and misrepresentation which may be detrimental to the interest of the public and/or of the stockholders,
partners, members of associations or organizations registered with the Commission;

b) Controversies arising out of intra-corporate or partnership relations, between and among stockholders, members
of associates; between any or all of them and the corporation, partnership or association of which they are stockholders,
members, or associates, respectively; and between such corporation, partnership or association and the State insofar as it
concerns their individual franchise or right to exist as such entity; [and]

c) Controversies in the election or appointments of directors, trustees, officers, or managers of such corporation,
partnerships or associations.

For the SEC to acquire jurisdiction over any controversy under these provisions, two elements must be considered: (1) the
status or relationship of the parties; and (2) the nature of the question that is the subject of their controversy. 27 The first
element requires that the controversy must arise "out of intra-corporate or partnership relations between and among
stockholders, members or associates; between any or all of them and the corporation, partnership or association of which
they are stockholders, members or associates, respectively; and between such corporation, partnership or association and
the State in so far as it concerns their individual franchises." 28 Petitioners are not stockholders, members or associates of
respondent. They are lot buyers and now homeowners in the subdivision developed by the respondent.

The second element requires that the dispute among the parties be intrinsically connected with the regulation or the internal
affairs of the corporation, partnership or association. 29 The controversy in this case is remotely related to the "regulation"
of respondent corporation or to respondent's "internal affairs."

It should be stressed that the main concern in this case is the issue of jurisdiction over petitioners' complaint against
respondent for specific performance. P.D. No. 902-A, as amended, defines the jurisdiction of the SEC; while P.D. No. 957,
as amended, delineates that of the HLURB. These two quasi-judicial agencies exercise functions that are distinct from each
other. The SEC has authority over the operation of all kinds of corporations, partnerships or associations with the end in
view of protecting the interests of the investing public and creditors. On the other hand, the HLURB has jurisdiction over
matters relating to observance of laws governing corporations engaged in the specific business of development of
subdivisions and condominiums. The HLURB and the SEC being bestowed with distinct powers and functions, the exercise
of those functions by one shall not abate the performance by the other of its own functions. As respondent puts it, "there is
no contradiction between P.D. No. 902-A and P.D. No. 957." 30

What complicated the jurisdictional issue in this case is the fact that petitioners are primarily praying for the retention of
respondent's obligations under the Memorandum of Agreement that Receiver Orendain had entered into with them but
which the present Board of Receivers had revoked.

In Figueroa v. SEC, 31 this Court has declared that the power to overrule or revoke the previous acts of the management
or Board of Directors of the entity under receivership is within a receiver's authority, as provided for by Section 6 (d) (2) of
P.D. No. 902-A. Indeed, when the acts of a previous receiver or management committee prove disadvantageous or inimical
to the rehabilitation of a distressed corporation, the succeeding receiver or management committee may abrogate or cast
aside such acts. However, that prerogative is not absolute. It should be exercised upon due consideration of all pertinent
and relevant laws when public interest and welfare are involved. The business of developing subdivisions and corporations
being imbued with public interest and welfare, any question arising from the exercise of that prerogative should be brought
to the proper agency that has technical know-how on the matter.

P.D. No. 957 was promulgated to encompass all questions regarding subdivisions and condominiums. It is aimed at
providing for an appropriate government agency, the HLURB, to which all parties aggrieved in the implementation of its
provisions and the enforcement of contractual rights with respect to said category of real estate may take recourse.
Nonetheless, the powers of the HLURB may not in any way be deemed as in derogation of the SEC's authority. P.D. Nos.
902-A and 957, as far as both are concerned with corporations, are laws in pari materia. P.D. No. 902-A relates to all
corporations, while P.D. No. 957 pertains to corporations engaged in the particular business of developing subdivisions
and condominiums. Although the provisions of these decrees on the issue of jurisdiction appear to collide when a
corporation engaged in developing subdivisions and condominiums is under receivership. the same decrees should be
construed as far as reasonably possible to be in harmony with each other to attain the purpose of an expressed national
policy. 32

29
Hence, the HLURB should take jurisdiction over petitioners' complaint because it pertains to matters within the HLURB's
competence and expertise. The HLURB should view the issue of whether the Board of Receivers correctly revoked the
agreements entered into between the previous receiver and the petitioners from the perspective of the homeowners'
interests, which P.D. No. 957 aims to protect. Whatever monetary awards the HLURB may impose upon respondent are
incidental matters that should be addressed to the sound discretion of the Board of Receivers charged with maintaining the
viability of respondent as a corporation. Any controversy that may arise in that regard should then be addressed to the
SEC. Cdpr

It is worth noting that the parties agreed at the 1 July 1998 hearing that should the HLURB establish and grant petitioners'
claims, the same should be referred to the SEC. Thus, the proceedings at the HLURB should not be suspended
notwithstanding that respondent is still under receivership. The TRO that this Court has issued should accordingly continue
until such time as the HLURB shall have resolved the controversy. The present members of the Board of Receivers should
be reminded of their duties and responsibilities as an impartial Board that should serve the interests of both the homeowners
and respondent's creditors. Their interests, financial or otherwise, as members of respondent's Board of Directors should
be circumscribed by judicious and unbiased performance of their duties and responsibilities as members of the Board of
Receivers. Otherwise, respondent's full rehabilitation may face a bleak future. Both parties should never give full rein to
acts that could prove detrimental to the interests of the homeowners and eventually jeopardize respondent's rehabilitation.

WHEREFORE, the questioned Decision of the Court of Appeals is hereby REVERSED and SET ASIDE. The case is
REMANDED to the Housing and Land Use Regulatory Board for continuation of proceedings with dispatch as the Securities
and Exchange Commission proceeds with the rehabilitation of respondent BF Homes, Inc., through the Board of Receivers.
Thereafter, any and all monetary claims duly established before the HLURB shall be referred to the Board of Receivers for
proper disposition and thereafter, to the SEC, if necessary. No costs. prLL

SO ORDERED.

Puno, Kapunan, Pardo and Ynares-Santiago, JJ., concur.

6. Guinhawa v. People, G.R. No. 162822, August 25, 2005, 468 SCRA 278, 299.

SECOND DIVISION

[G.R. No. 162822. August 25, 2005.]

JAIME GUINHAWA, petitioner, vs. PEOPLE OF THE PHILIPPINES, respondent.

Benjamin E. Bulalacao for petitioner.

Luis M. Contreras, Jr. for Bolante Imperial & Associates.

SYLLABUS

1. REMEDIAL LAW; CRIMINAL PROCEDURE; INFORMATION; MUST ALLEGE CLEARLY AND ACCURATELY
THE ELEMENTS OF THE CRIME CHARGED. — Section 6, Rule 110 of the Rules of Criminal Procedure requires that the
Information must allege the acts or omissions complained of as constituting the offense: SEC. 6. Sufficiency of complaint
or information. — A complaint or information is sufficient if it states the name of the accused; the designation of the offense
given by the statute; the acts or omissions complained of as constituting the offense; the name of the offended party; the
approximate date of the commission of the offense; and the place where the offense was committed. When an offense is
committed by more than one person, all of them shall be included in the complaint or information. The real nature of the
offense charged is to be ascertained by the facts alleged in the body of the Information and the punishment provided by
law, not by the designation or title or caption given by the Prosecutor in the Information. The Information must allege clearly
and accurately the elements of the crime charged. AaCTID

2. CRIMINAL LAW; OTHER DECEITS; ELEMENTS. — For one to be liable for "other deceits" under the law, it is
required that the prosecution must prove the following essential elements: (a) false pretense, fraudulent act or pretense
other than those in the preceding articles; (b) such false pretense, fraudulent act or pretense must be made or executed
prior to or simultaneously with the commission of the fraud; and (c) as a result, the offended party suffered damage or
prejudice. It is essential that such false statement or fraudulent representation constitutes the very cause or the only motive
for the private complainant to part with her property. The provision includes any kind of conceivable deceit other than those
enumerated in Articles 315 to 317 of the Revised Penal Code. It is intended as the catchall provision for that purpose with
its broad scope and intendment.

30
3. ID.; ID.; ID.; CONCEALMENT; WHEN SUPPRESSION OF MATERIAL FACT WHICH A PARTY IS BOUND IN
GOOD FAITH TO DISCLOSE IS EQUIVALENT TO FALSE REPRESENTATION; RATIONALE. — It is true that mere
silence is not in itself concealment. Concealment which the law denounces as fraudulent implies a purpose or design to
hide facts which the other party sought to know. Failure to reveal a fact which the seller is, in good faith, bound to disclose
may generally be classified as a deceptive act due to its inherent capacity to deceive. Suppression of a material fact which
a party is bound in good faith to disclose is equivalent to a false representation. Moreover, a representation is not confined
to words or positive assertions; it may consist as well of deeds, acts or artifacts of a nature calculated to mislead another
and thus allow the fraud-feasor to obtain an undue advantage. Fraudulent nondisclosure and fraudulent concealment are
of the same genre. Fraudulent concealment presupposes a duty to disclose the truth and that disclosure was not made
when opportunity to speak and inform was presented, and that the party to whom the duty of disclosure, as to a material
fact was due, was induced thereby to act to his injury. Article 1389 of the New Civil Code provides that failure to disclose
facts when there is a duty to reveal them constitutes fraud. In a contract of sale, a buyer and seller do not deal from equal
bargaining positions when the latter has knowledge, a material fact which, if communicated to the buyer, would render the
grounds unacceptable or, at least, substantially less desirable. If, in a contract of sale, the vendor knowingly allowed the
vendee to be deceived as to the thing sold in a material matter by failing to disclose an intrinsic circumstance that is vital to
the contract, knowing that the vendee is acting upon the presumption that no such fact exists, deceit is accomplished by
the suppression of the truth. IEaHSD

4. ID.; ID.; ID.; ID.; VISUAL INSPECTION OF THE THING SOLD WILL NOT RELIEVE THE SELLER OF HIS
CRIMINAL LIABILITY; APPLICATION IN CASE AT BAR. — The petitioner is not relieved of his criminal liability for deceitful
concealment of material facts, even if the private complainant made a visual inspection of the van's interior and exterior
before she agreed to buy it and failed to inspect its under chassis. Case law has it that where the vendee made only a
partial investigation and relies, in part, upon the representation of the vendee, and is deceived by such representation to
his injury, he may maintain an action for such deceit. The seller cannot be heard to say that the vendee should not have
relied upon the fraudulent concealment; that negligence, on the part of the vendee, should not be a defense in order to
prevent the vendor from unjustifiably escaping with the fruits of the fraud. In one case, the defendant who repainted an
automobile, worked it over to resemble a new one and delivered it to the plaintiff was found to have warranted and
represented that the automobile being sold was new. This was found to be "a false representation of an existing fact; and,
if it was material and induced the plaintiff to accept something entirely different from that which he had contracted for, it
clearly was a fraud which, upon its discovery and a tender of the property back to the seller, [it] entitled the plaintiff to
rescind the trade and recover the purchase money."

5. ID.; ID.; CAVEAT EMPTOR IS NOT A DEFENSE. — On the petitioner's insistence that the private complainant
was proscribed from charging him with estafa based on the principle of caveat emptor, case law has it that this rule only
requires the purchaser to exercise such care and attention as is usually exercised by ordinarily prudent men in like business
affairs, and only applies to defects which are open and patent to the service of one exercising such care. In an avuncular
case, it was held that: . . . The rule of caveat emptor, like the rule of sweet charity, has often been invoked to cover a
multitude of sins; but we think its protecting mantle has never been stretched to this extent. It can only be applied where it
is shown or conceded that the parties to the contract stand on equal footing and have equal knowledge or equal means of
knowledge and there is no relation of trust or confidence between them. But, where one party undertakes to sell to another
property situated at a distance and of which he has or claims to have personal knowledge and of which the buyer knows
nothing except as he is informed by the seller, the buyer may rightfully rely on the truth of the seller's representations as to
its kind, quality, and value made in the course of negotiation for the purpose of inducing the purchase. If, in such case, the
representations prove to be false, neither law nor equity will permit the seller to escape responsibility by the plea that the
buyer ought not to have believed him or ought to have applied to other sources to ascertain the facts. . . . It bears stressing
that Azotea and the petitioner had every opportunity to reveal to the private complainant that the van was defective. They
resolved to maintain their silence, to the prejudice of the private complainant, who was a garment merchant and who had
no special knowledge of parts of motor vehicles. Based on the surrounding circumstances, she relied on her belief that the
van was brand new. In fine, she was the innocent victim of the petitioner's fraudulent nondisclosure or concealment. The
petitioner cannot pin criminal liability for his fraudulent omission on his general manager, Azotea. The two are equally liable
for their collective fraudulent silence. Case law has it that wherever the doing of a certain act or the transaction of a given
affair, or the performance of certain business is confided to an agent, the authority to so act will, in accordance with a
general rule often referred to, carry with it by implication the authority to do all of the collateral acts which are the natural
and ordinary incidents of the main act or business authorized. CScaDH

6. ID.; ID.; IMPOSABLE PENALTY. — An indeterminate penalty may be imposed if the minimum of the penalty is
one year or less, and the maximum exceeds one year. For example, the trial court may impose an indeterminate penalty
of six months of arresto mayor, as minimum, to two years and four months of prision correccional, as maximum, since the
maximum term of imprisonment it imposed exceeds one year. If the trial court opts to impose a penalty of imprisonment of
one year or less, it should not impose an indeterminate penalty, but a straight penalty of one year or less instead. Thus,
31
the petitioner may be sentenced to a straight penalty of one year, or a straight penalty of less than one year, i.e., ten months
or eleven months. We believe that considering the attendant circumstances, a straight penalty of imprisonment of six
months is reasonable. Conformably with Article 39 in relation to paragraph 3, Article 38 of the Revised Penal Code, the
petitioner shall suffer subsidiary imprisonment if he has no property with which to pay the penalty of fine.

7. STATUTORY CONSTRUCTION; EJUSDEM GENERIS; DEFINED. — Under the principle of ejusdem generis,
where a statement ascribes things of a particular class or kind accompanied by words of a generic character, the generic
words will usually be limited to things of a similar nature with those particularly enumerated unless there be something in
the context to the contrary.

8. REMEDIAL LAW; CRIMINAL PROCEDURE; JURISDICTION; CANNOT BE CONFERRED BY THE WILL OF THE
PARTIES NOR DIMINISHED NOR WAIVED BY THEM. — Jurisdiction is conferred by the Constitution or by law. It cannot
be conferred by the will of the parties, nor diminished or waived by them. The jurisdiction of the court is determined by the
averments of the complaint or Information, in relation to the law prevailing at the time of the filing of the criminal complaint
or Information, and the penalty provided by law for the crime charged at the time of its commission. ECcTaH

DECISION

CALLEJO, SR., J p:

Jaime Guinhawa was engaged in the business of selling brand new motor vehicles, including Mitsubishi vans, under the
business name of Guinrox Motor Sales. His office and display room for cars were located along Panganiban Avenue, Naga
City. He employed Gil Azotea as his sales manager.

On March 17, 1995, Guinhawa purchased a brand new Mitsubishi L-300 Versa Van with Motor No. 4D56A-C8929 and
Serial No. L069WQZJL-07970 from the Union Motors Corporation (UMC) in Paco, Manila. The van bore Plate No. DLK
406. Guinhawa's driver, Leopoldo Olayan, drove the van from Manila to Naga City. However, while the van was traveling
along the highway in Labo, Daet, Camarines Norte, Olayan suffered a heart attack. The van went out of control, traversed
the highway onto the opposite lane, and was ditched into the canal parallel to the highway. 1 The van was damaged, and
the left front tire had to be replaced. TCcSDE

The incident was reported to the local police authorities and was recorded in the police blotter. 2 The van was repaired and
later offered for sale in Guinhawa's showroom. 3

Sometime in October 1995, the spouses Ralph and Josephine Silo wanted to buy a new van for their garment business;
they purchased items in Manila and sold them in Naga City. 4 They went to Guinhawa's office, and were shown the L-300
Versa Van which was on display. The couple inspected its interior portion and found it beautiful. They no longer inspected
the under chassis since they presumed that the vehicle was brand new. 5 Unaware that the van had been damaged and
repaired on account of the accident in Daet, the couple decided to purchase the van for P591,000.00. Azotea suggested
that the couple make a downpayment of P118,200.00, and pay the balance of the purchase price by installments via a loan
from the United Coconut Planters Bank (UCPB), Naga Branch, with the L-300 Versa Van as collateral. Azotea offered to
make the necessary arrangements with the UCPB for the consummation of the loan transaction. The couple agreed. On
November 10, 1995, the spouses executed a Promissory Note 6 for the amount of P692,676.00 as payment of the balance
on the purchase price, and as evidence of the chattel mortgage over the van in favor of UCPB.

On October 11, 1995, the couple arrived in Guinhawa's office to take delivery of the van. Guinhawa executed the deed of
sale, and the couple paid the P161,470.00 downpayment, for which they were issued Receipt No. 0309. 7 They were
furnished a Service Manual 8 which contained the warranty terms and conditions. Azotea instructed the couple on how to
start the van and to operate its radio. Ralph Silo no longer conducted a test drive; he and his wife assumed that there were
no defects in the van as it was brand new. 9

On October 12, 1995, Josephine Silo, accompanied by Glenda Pingol, went to Manila on board the L-300 Versa Van, with
Glenda's husband, Bayani Pingol III, as the driver. Their trip to Manila was uneventful. However, on the return trip to Naga
from Manila on October 15 or 16, 1995, Bayani Pingol heard a squeaking sound which seemed to be coming from
underneath the van. They were in Calauag, Quezon, where there were no humps along the road. 10 Pingol stopped the
van in Daet, Camarines Norte, and examined the van underneath, but found no abnormalities or defects. 11 But as he
drove the van to Naga City, the squeaking sound persisted. Believing that the van merely needed grease, Pingol stopped
at a Shell gasoline station where it was examined. The mechanic discovered that some parts underneath the van had been
welded. When Pingol complained to Guinhawa, the latter told him that the defects were mere factory defects. As the defects
persisted, the spouses Silo requested that Guinhawa change the van with two Charade-Daihatsu vehicles within a week or
two, with the additional costs to be taken from their downpayment. Meanwhile, the couple stopped paying the monthly
amortization on their loan, pending the replacement of the van. Guinhawa initially agreed to the couple's proposal, but later

32
changed his mind and told them that he had to sell the van first. The spouses then brought the vehicle to the Rx Auto Clinic
in Naga City for examination. Jesus Rex Raquitico, Jr., the mechanic, examined the van and discovered that it was the left
front stabilizer that was producing the annoying sound, and that it had been repaired. 12 Raquitico prepared a Job Order
containing the following notations and recommendations:

1. CHECK UP SUSPENSION (FRONT)

2. REPLACE THE ROD END

3. REPLACE BUSHING

NOTE: FRONT STEP BOARD HAS BEEN ALREADY DAMAGED AND REPAIRED.

NOTE: FRONT LEFT SUSPENSION MOUNTING IS NOT ON SPECIFIED ALIGNMENT/MEASUREMENT 13

Josephine Silo filed a complaint for the rescission of the sale and the refund of their money before the Department of Trade
and Industry (DTI). During the confrontation between her and Guinhawa, Josephine learned that Guinhawa had bought the
van from UMC before it was sold to them, and after it was damaged in Daet. Subsequently, the spouses Silo withdrew their
complaint from the DTI.

On February 14, 1996, Josephine Silo filed a criminal complaint for violation of paragraph 1, Article 318 of the Revised
Penal Code against Guinhawa in the Office of the City Prosecutor of Naga City. After the requisite investigation, an
Information was filed against Guinhawa in the Municipal Trial Court (MTC) of Naga City. The inculpatory portion reads:

The undersigned Assistant Prosecutor of Naga City accuses Jaime Guinhawa of the crime of OTHER DECEITS defined
and penalized under Art. 318, par. 1 of the Revised Penal Code, committed as follows:

"That on or about October 11, 1995, in the City of Naga, Philippines, and within the jurisdiction of this Honorable Court, the
said accused, being a motor vehicle dealer using the trade name of Guinhawa Motor Sales at Panganiban Avenue, Naga
City, and a dealer of brand new cars, by means of false pretenses and fraudulent acts, did then and there willfully, unlawfully
and feloniously defraud private complainant, JOSEPHINE P. SILO, as follows: said accused by means of false
manifestations and fraudulent representations, sold to said private complainant, as brand new, an automobile with trade
name L-300 Versa Van colored beige and the latter paid for the same in the amount of P591,000.00, when, in truth and in
fact, the same was not brand new because it was discovered less than a month after it was sold to said Josephine P. Silo
that said L-300 Versa Van had defects in the underchassis and stepboard and repairs had already been done thereat even
before said sale, as was found upon check-up by an auto mechanic; that private complainant returned said L-300 Versa
Van to the accused and demanded its replacement with a new one or the return of its purchase price from said accused
but despite follow-up demands no replacement was made nor was the purchase price returned to private complainant up
to the present to her damage and prejudice in the amount of P591,000.00, Philippine Currency, plus other damages that
may be proven in court." 14

Guinhawa testified that he was a dealer of brand new Toyota, Mazda, Honda and Mitsubishi cars, under the business name
Guinrox Motor Sales. He purchased Toyota cars from Toyota Philippines, and Mitsubishi cars from UMC in Paco, Manila.
15 He bought the van from the UMC in March 1995, but did not use it; he merely had it displayed in his showroom in Naga
City. 16 He insisted that the van was a brand new unit when he sold it to the couple. 17 The spouses Silo bought the van
and took delivery only after inspecting and taking it for a road tests. 18 His sales manager, Azotea, informed him sometime
in November 1995 that the spouses Silo had complained about the defects under the left front portion of the van. By then,
the van had a kilometer reading of 4,000 kilometers. 19 He insisted that he did not make any false statement or fraudulent
misrepresentation to the couple about the van, either before or simultaneous with its purchase. He posited that the defects
noticed by the couple were not major ones, and could be repaired. However, the couple refused to have the van repaired
and insisted on a refund of their payment for the van which he could not allow. He then had the defects repaired by the
UMC. 20 He claimed that the van was never involved in any accident, and denied that his driver, Olayan, met an accident
and sustained physical injuries when he drove the van from Manila to Naga City. 21 He even denied meeting Bayani Pingol.

The accused claimed that the couple filed a Complaint 22 against him with the DTI on January 25, 1996, only to withdraw
it later. 23 The couple then failed to pay the amortizations for the van, which caused the UCPB to file a petition for the
foreclosure of the chattel mortgage and the sale of the van at public auction. 24

Azotea testified that he had been a car salesman for 16 years and that he sold brand new vans. 25 Before the couple took
delivery of the vehicle, Pingol inspected its exterior, interior, and underside, and even drove it for the couple. 26 He was
present when the van was brought to the Rx Auto Clinic, where he noticed the dent on its front side. 27 He claimed that the
van never figured in any vehicular accident in Labo, Daet, Camarines Norte on March 17, 1995. 28 In fact, he declared, he

33
found no police record of a vehicular accident involving the van on the said date. 29 He admitted that Olayan was their
driver, and was in charge of taking delivery of cars purchased from the manufacturer in Manila. 30

On November 6, 2001, the trial court rendered judgment convicting Guinhawa. The fallo of the decision reads:

WHEREFORE, premises considered, judgment is hereby rendered declaring the accused, JAIME GUINHAWA, guilty of
the crime of Other Deceits defined and penalized under Art. 318(1) of the Revised Penal Code, the prosecution having
proven the guilt of the accused beyond reasonable doubt and hereby imposes upon him the penalty of imprisonment from
2 months and 1 day to 4 months of Arresto Mayor and a fine of One Hundred Eighty Thousand Seven Hundred and Eleven
Pesos (P180,711.00) the total amount of the actual damages caused to private complainant. ITScAE

As to the civil aspect of this case which have been deemed instituted with this criminal case, Articles 2201 and 2202 of the
Civil Code provides:

"Art. 2201. In contracts and quasi-contracts, the damages for which the obligor who acted in good faith is liable shall
be those that are the natural and probable consequences of the breach of the obligation, and which the parties have
foreseen or could have reasonably foreseen at the time the obligation was constituted.

"In case of fraud, malice or wanton attitude, the obligor shall be responsible for all damages which may be reasonably
attributed to the non-performance of the obligation."

"Art. 2202. In crimes and quasi-delicts, the defendant shall be liable for all damages which are the natural and
probable consequences of the act or omission complained of. It is not necessary that such damages have been foreseen
or could have reasonably been foreseen by the defendant."

Thus, accused is condemned to pay actual damages in the amount of One Hundred Eighty Thousand Seven Hundred and
Eleven Pesos (Php180,711.00), which represents the 20% downpayment and other miscellaneous expenses paid by the
complainant plus the amount of Nineteen Thousand Two Hundred Forty-One (Php19,241.00) Pesos, representing the 1st
installment payment made by the private complainant to the bank. Accused is, likewise, ordered to pay moral damages in
the amount of One Hundred Thousand Pesos (Php100,000.00) in view of the moral pain suffered by the complainant; for
exemplary damages in the amount of Two Hundred Thousand Pesos (Php200,000.00) to serve as deterrent for those
businessmen similarly inclined to take undue advantage over the public's innocence. As for attorney's fees, the reasonable
amount of One Hundred Thousand Pesos (Php100,000.00) is hereby awarded.

SO ORDERED. 31

The trial court declared that the accused made false pretenses or misrepresentations that the van was a brand new one
when, in fact, it had figured in an accident in Labo, Daet, Camarines Norte, and sustained serious damages before it was
sold to the private complainant.

Guinhawa appealed the decision to the Regional Trial Court (RTC) of Naga City, Branch 19, in which he alleged that:

1. The lower court erred in its finding that the repair works on the left front portion and underchassis of the van was
the result of the accident in Labo, Camarines Norte, where its driver suffered an attack of hypertension.

2. The lower court erred in its four (4) findings of fact that accused-appellant made misrepresentation or false
pretenses "that the van was a brand new car," which constituted deceit as defined in Article 318, paragraph 1 of the Revised
Penal Code.

3. The lower court erred in finding accused-appellant civilly liable to complainant Josephine Silo. But, even if there
be such liability, the action therefor has already prescribed and the amount awarded was exorbitant, excessive and
unconscionable. 32

Guinhawa insisted that he never talked to the couple about the sale of the van; hence, could not have made any false
pretense or misrepresentation.

On August 1, 2002, the RTC affirmed the appealed judgment. 33

Guinhawa filed a petition for review with the Court of Appeals (CA), where he averred that:

THE COURT A QUO ERRED IN CONVICTING PETITIONER OF THE CRIME OF OTHER DECEITS AND SENTENCING
HIM TO SUFFER IMPRISONMENT OF TWO MONTHS AND ONE DAY TO FOUR MONTHS OF ARRESTO MAYOR AND
TO PAY FINE IN THE AMOUNT OF P180,711.00.
34
II

THE COURT A QUO ERRED IN ORDERING PETITIONER TO PAY PRIVATE COMPLAINANT P180,711.00 AS
DOWNPAYMENT, P19,241.00 AS FIRST INSTALLMENT WITH UCPB NAGA, P100,000.00 AS MORAL DAMAGES,
P200,000.00 AS EXEMPLARY DAMAGES AND P100,000.00 AS ATTORNEY'S FEES. 34

On January 5, 2004, the CA rendered judgment affirming with modification the decision of the RTC. The fallo of the decision
reads:

WHEREFORE, premises considered, the instant petition is hereby partially granted insofar as the following are concerned:
a) the award of moral damages is hereby REDUCED to P10,000.00 and b) the award of attorney's fees and exemplary
damages are hereby DELETED for lack of factual basis. In all other respects, We affirm the decision under review. IDAEHT

Costs against petitioner.

SO ORDERED. 35

The CA ruled that the private complainant had the right to assume that the van was brand new because Guinhawa held
himself out as a dealer of brand new vans. According to the appellate court, the act of displaying the van in the showroom
without notice to any would-be buyer that it was not a brand new unit was tantamount to deceit. Thus, in concealing the
van's true condition from the buyer, Guinhawa committed deceit.

The appellate court denied Guinhawa's motion for reconsideration, prompting him to file the present petition for review on
certiorari, where he contends:

THE COURT A QUO ERRED IN NOT HOLDING THAT THE INFORMATION CHARGED AGAINST PETITIONER DID
NOT INFORM HIM OF A CHARGE OF OTHER DECEITS.

II

THE COURT A QUO ERRED IN HOLDING THAT PETITIONER EMPLOYED FRAUD OR DECEIT AS DEFINED UNDER
ARTICLE 318, REVISED PENAL CODE.

III

THE COURT A QUO ERRED IN NOT CONSIDERING THE CIRCUMSTANCES POINTING TO THE INNOCENCE OF
THE PETITIONER. 36

The issues for resolution are (1) whether, under the Information, the petitioner was charged of other deceits under
paragraph 1, Article 318 of the Revised Penal Code; and (2) whether the respondent adduced proof beyond reasonable
doubt of the petitioner's guilt for the crime charged.

The petitioner asserts that based on the allegations in the Information, he was charged with estafa through false pretenses
under paragraph 2, Article 315 of the Revised Penal Code. Considering the allegation that the private complainant was
defrauded of P591,000.00, it is the RTC, not the MTC, which has exclusive jurisdiction over the case. The petitioner
maintains that he is not estopped from assailing this matter because the trial court's lack of jurisdiction can be assailed at
any time, even on appeal, which defect cannot even be cured by the evidence adduced during the trial. The petitioner
further avers that he was convicted of other deceits under paragraph 1, Article 318 of the Revised Penal Code, a crime for
which he was not charged; hence, he was deprived of his constitutional right to be informed of the nature of the charge
against him. And in any case, even if he had been charged of other deceits under paragraph 1 of Article 318, the CA erred
in finding him guilty. He insists that the private complainant merely assumed that the van was brand new, and that he did
not make any misrepresentation to that effect. He avers that deceit cannot be committed by concealment, the absence of
any notice to the public that the van was not brand new does not amount to deceit. He posits that based on the principle of
caveat emptor, if the private complainant purchased the van without first inspecting it, she must suffer the consequences.
Moreover, he did not attend to the private complainant when they examined the van; thus, he could not have deceived
them.

The petitioner maintains that, absent evidence of conspiracy, he is not criminally liable for any representation Azotea may
have made to the private complainant, that the van was brand new. He insists that the respondent was estopped from
adducing evidence that the vehicle was involved in an accident in Daet, Camarines Norte on March 17, 1995, because
such fact was not alleged in the Information. STcEIC

35
In its comment on the petition, the Office of the Solicitor General avers that, as gleaned from the material averments of the
Information, the petitioner was charged with other deceits under paragraph 1, Article 318 of the Revised Penal Code, a
felony within the exclusive jurisdiction of the MTC. The petitioner was correctly charged and convicted, since he falsely
claimed that the vehicle was brand new when he sold the same to the private complainant. The petitioner's concealment
of the fact that the van sustained serious damages as an aftermath of the accident in Daet, Camarines Norte constituted
deceit within the meaning of paragraph 1 of Article 318.

The Information filed against the petitioner reads:

That on or about October 11, 1995, in the City of Naga, Philippines, and within the jurisdiction of this Honorable Court, the
said accused, being a motor vehicle dealer using the trade name of Guinhawa Motor Sales at Panganiban Avenue, Naga
City, and dealer of brand new cars, by means of false pretenses and fraudulent acts, did then and there, willfully, unlawfully
and feloniously defraud private complainant, JOSEPHINE P. SILO, as follows: said accused by means of false
manifestations and fraudulent representations, sold to said private complainant, as brand new, an automobile with trade
name L-300 Versa Van colored beige and the latter paid for the same in the amount of P591,000.00, when, in truth and in
fact, the same was not brand new because it was discovered less than a month after it was sold to said Josephine P. Silo
that said L-300 Versa Van had defects in the underchassis and stepboard and repairs have already been done thereat
even before said sale, as was found upon check-up by an auto mechanic; that private complainant returned said L-300
Versa Van to the accused and demanded its replacement with a new one or the return of its purchase price from said
accused but despite follow-up demands no replacement was made nor was the purchase price returned to private
complainant up to the present to her damage and prejudice in the amount of P591,000.00, Philippine Currency, plus other
damages that may be proven in court.

CONTRARY TO LAW. 37

Section 6, Rule 110 of the Rules of Criminal Procedure requires that the Information must allege the acts or omissions
complained of as constituting the offense:

SEC. 6. Sufficiency of complaint or information. — A complaint or information is sufficient if it states the name of the
accused; the designation of the offense given by the statute; the acts or omissions complained of as constituting the offense;
the name of the offended party; the approximate date of the commission of the offense; and the place where the offense
was committed.

When an offense is committed by more than one person, all of them shall be included in the complaint or information.

The real nature of the offense charged is to be ascertained by the facts alleged in the body of the Information and the
punishment provided by law, not by the designation or title or caption given by the Prosecutor in the Information. 38 The
Information must allege clearly and accurately the elements of the crime charged. 39

As can be gleaned from its averments, the Information alleged the essential elements of the crime under paragraph 1,
Article 318 of the Revised Penal Code.

The false or fraudulent representation by a seller that what he offers for sale is brand new (when, in fact, it is not) is one of
those deceitful acts envisaged in paragraph 1, Article 318 of the Revised Penal Code. The provision reads:

Art. 318.Other deceits. — The penalty of arresto mayor and a fine of not less than the amount of the damage caused and
not more than twice such amount shall be imposed upon any person who shall defraud or damage another by any other
deceit not mentioned in the preceding articles of this chapter.

This provision was taken from Article 554 of the Spanish Penal Code which provides:

El que defraudare o perjudicare a otro, usando de cualquier engaño que no se halle expresado en los artículos anteriores
de esta sección, será castigado con una multa del tanto al duplo del perjuicio que irrogare; y en caso de reincidencia, con
la del duplo y arresto mayor en su grado medio al máximo.

For one to be liable for "other deceits" under the law, it is required that the prosecution must prove the following essential
elements: (a) false pretense, fraudulent act or pretense other than those in the preceding articles; (b) such false pretense,
fraudulent act or pretense must be made or executed prior to or simultaneously with the commission of the fraud; and (c)
as a result, the offended party suffered damage or prejudice. 40 It is essential that such false statement or fraudulent
representation constitutes the very cause or the only motive for the private complainant to part with her property.

The provision includes any kind of conceivable deceit other than those enumerated in Articles 315 to 317 of the Revised
Penal Code. 41 It is intended as the catchall provision for that purpose with its broad scope and intendment. 42

36
Thus, the petitioner's reliance on paragraph 2(a), Article 315 of the Revised Penal Code is misplaced. The said provision
reads:

2. By means of any of the following false pretenses or fraudulent acts executed prior to or simultaneously with the
commission of the fraud:

(a) By using fictitious name, or falsely pretending to possess power, influence, qualifications, property, credit, agency,
business or imaginary transactions; or by means of other similar deceits.

The fraudulent representation of the seller, in this case, that the van to be sold is brand new, is not the deceit contemplated
in the law. Under the principle of ejusdem generis, where a statement ascribes things of a particular class or kind
accompanied by words of a generic character, the generic words will usually be limited to things of a similar nature with
those particularly enumerated unless there be something in the context to the contrary. 43

Jurisdiction is conferred by the Constitution or by law. It cannot be conferred by the will of the parties, nor diminished or
waived by them. The jurisdiction of the court is determined by the averments of the complaint or Information, in relation to
the law prevailing at the time of the filing of the criminal complaint or Information, and the penalty provided by law for the
crime charged at the time of its commission.

Section 32 of Batas Pambansa Blg. 129, as amended by Republic Act No. 7691, provides that the MTC has exclusive
jurisdiction over offenses punishable with imprisonment not exceeding six years, irrespective of the amount of the fine:

Sec. 32. Jurisdiction of Metropolitan Trial Courts, Municipal Trial Courts and Municipal Circuit Trial Courts in Criminal Cases.
— Except in cases falling within the exclusive original jurisdiction of Regional Trial Courts and of the Sandiganbayan, the
Metropolitan Trial Courts, Municipal Trial Courts, and Municipal Circuit Trial Courts shall exercise:

(1) Exclusive original jurisdiction over all violations of city or municipal ordinances committed within their respective
territorial jurisdiction; and

(2) Exclusive original jurisdiction over all offenses punishable with imprisonment not exceeding six (6) years
irrespective of the amount of fine, and regardless of other imposable accessory or other penalties, including the civil liability
arising from such offenses or predicated thereon, irrespective of kind, nature, value or amount thereof: Provided, however,
That in offenses involving damage to property through criminal negligence, they shall have exclusive original jurisdiction
thereof.

Since the felony of other deceits is punishable by arresto mayor, the MTC had exclusive jurisdiction over the offense lodged
against the petitioner.

On the merits of the petition, the Court agrees with the petitioner's contention that there is no evidence on record that he
made direct and positive representations or assertions to the private complainant that the van was brand new. The record
shows that the private complainant and her husband Ralph Silo were, in fact, attended to by Azotea. However, it bears
stressing that the representation may be in the form of words, or conduct resorted to by an individual to serve as an
advantage over another. Indeed, as declared by the CA based on the evidence on record:

Petitioner cannot barefacedly claim that he made no personal representation that the herein subject van was brand new
for the simple reason that nowhere in the records did he ever refute the allegation in the complaint, which held him out as
a dealer of brand new cars. It has thus become admitted that the petitioner was dealing with brand new vehicles — a fact
which, up to now, petitioner has not categorically denied. Therefore, when private complainant went to petitioner's
showroom, the former had every right to assume that she was being sold brand new vehicles there being nothing to indicate
otherwise. But as it turned out, not only did private complainant get a defective and used van, the vehicle had also earlier
figured in a road accident when driven by no less than petitioner's own driver. 44

Indeed, the petitioner and Azotea obdurately insisted in the trial court that the van was brand new, and that it had never
figured in vehicular accident. This representation was accentuated by the fact that the petitioner gave the Service Manual
to the private complainant, which manual contained the warranty terms and conditions, signifying that the van was "brand
new." Believing this good faith, the private complainant decided to purchase the van for her buy-and-sell and garment
business, and even made a downpayment of the purchase price.

As supported by the evidence on record, the van was defective when the petitioner sold it to the private complainant. It had
ditched onto the shoulder of the highway in Daet, Camarines Norte on its way from Manila to Naga City. The van was
damaged and had to be repaired; the rod end and bushing had to be replaced, while the left front stabilizer which gave out
a persistent annoying sound was repaired. Some parts underneath the van were even welded together. Azotea and the

37
petitioner deliberately concealed these facts from the private complainant when she bought the van, obviously so as not to
derail the sale and the profit from the transaction.

The CA is correct in ruling that fraud or deceit may be committed by omission. As the Court held in People v. Balasa: 45

Fraud, in its general sense, is deemed to comprise anything calculated to deceive, including all acts, omissions, and
concealment involving a breach of legal or equitable duty, trust, or confidence justly reposed, resulting in damage to
another, or by which an undue and unconscientious advantage is taken of another. It is a generic term embracing all
multifarious means which human ingenuity can device, and which are resorted to by one individual to secure an advantage
over another by false suggestions or by suppression of truth and includes all surprise, trick, cunning, dissembling and any
unfair way by which another is cheated. On the other hand, deceit is the false representation of a matter of fact whether by
words or conduct, by false or misleading allegations, or by concealment of that which should have been disclosed which
deceives or is intended to deceive another so that he shall act upon it to his legal injury. 46

It is true that mere silence is not in itself concealment. Concealment which the law denounces as fraudulent implies a
purpose or design to hide facts which the other party sought to know. 47 Failure to reveal a fact which the seller is, in good
faith, bound to disclose may generally be classified as a deceptive act due to its inherent capacity to deceive. 48
Suppression of a material fact which a party is bound in good faith to disclose is equivalent to a false representation. 49
Moreover, a representation is not confined to words or positive assertions; it may consist as well of deeds, acts or artifacts
of a nature calculated to mislead another and thus allow the fraud-feasor to obtain an undue advantage. 50

Fraudulent nondisclosure and fraudulent concealment are of the same genre. Fraudulent concealment presupposes a duty
to disclose the truth and that disclosure was not made when opportunity to speak and inform was presented, and that the
party to whom the duty of disclosure, as to a material fact was due, was induced thereby to act to his injury. 51

Article 1389 of the New Civil Code provides that failure to disclose facts when there is a duty to reveal them constitutes
fraud. In a contract of sale, a buyer and seller do not deal from equal bargaining positions when the latter has knowledge,
a material fact which, if communicated to the buyer, would render the grounds unacceptable or, at least, substantially less
desirable. 52 If, in a contract of sale, the vendor knowingly allowed the vendee to be deceived as to the thing sold in a
material matter by failing to disclose an intrinsic circumstance that is vital to the contract, knowing that the vendee is acting
upon the presumption that no such fact exists, deceit is accomplished by the suppression of the truth. 53

In the present case, the petitioner and Azotea knew that the van had figured in an accident, was damaged and had to be
repaired. Nevertheless, the van was placed in the showroom, thus making it appear to the public that it was a brand new
unit. The petitioner was mandated to reveal the foregoing facts to the private complainant. But the petitioner and Azotea
even obdurately declared when they testified in the court a quo that the vehicle did not figure in an accident, nor had it been
repaired; they maintained that the van was brand new, knowing that the private complainant was going to use it for her
garment business. Thus, the private complainant bought the van, believing it was brand new.

Significantly, even when the petitioner was apprised that the private complainant had discovered the van's defects, the
petitioner agreed to replace the van, but changed his mind and insisted that it must be first sold. ACTISE

The petitioner is not relieved of his criminal liability for deceitful concealment of material facts, even if the private complainant
made a visual inspection of the van's interior and exterior before she agreed to buy it and failed to inspect its under chassis.
Case law has it that where the vendee made only a partial investigation and relies, in part, upon the representation of the
vendee, and is deceived by such representation to his injury, he may maintain an action for such deceit. 54 The seller
cannot be heard to say that the vendee should not have relied upon the fraudulent concealment; that negligence, on the
part of the vendee, should not be a defense in order to prevent the vendor from unjustifiably escaping with the fruits of the
fraud.

In one case, 55 the defendant who repainted an automobile, worked it over to resemble a new one and delivered it to the
plaintiff was found to have warranted and represented that the automobile being sold was new. This was found to be "a
false representation of an existing fact; and, if it was material and induced the plaintiff to accept something entirely different
from that which he had contracted for, it clearly was a fraud which, upon its discovery and a tender of the property back to
the seller, [it] entitled the plaintiff to rescind the trade and recover the purchase money." 56

On the petitioner's insistence that the private complainant was proscribed from charging him with estafa based on the
principle of caveat emptor, case law has it that this rule only requires the purchaser to exercise such care and attention as
is usually exercised by ordinarily prudent men in like business affairs, and only applies to defects which are open and patent
to the service of one exercising such care. 57 In an avuncular case, it was held that:

. . . The rule of caveat emptor, like the rule of sweet charity, has often been invoked to cover a multitude of sins; but we
think its protecting mantle has never been stretched to this extent. It can only be applied where it is shown or conceded
38
that the parties to the contract stand on equal footing and have equal knowledge or equal means of knowledge and there
is no relation of trust or confidence between them. But, where one party undertakes to sell to another property situated at
a distance and of which he has or claims to have personal knowledge and of which the buyer knows nothing except as he
is informed by the seller, the buyer may rightfully rely on the truth of the seller's representations as to its kind, quality, and
value made in the course of negotiation for the purpose of inducing the purchase. If, in such case, the representations
prove to be false, neither law nor equity will permit the seller to escape responsibility by the plea that the buyer ought not
to have believed him or ought to have applied to other sources to ascertain the facts. . . . 58

It bears stressing that Azotea and the petitioner had every opportunity to reveal to the private complainant that the van was
defective. They resolved to maintain their silence, to the prejudice of the private complainant, who was a garment merchant
and who had no special knowledge of parts of motor vehicles. Based on the surrounding circumstances, she relied on her
belief that the van was brand new. In fine, she was the innocent victim of the petitioner's fraudulent nondisclosure or
concealment.

The petitioner cannot pin criminal liability for his fraudulent omission on his general manager, Azotea. The two are equally
liable for their collective fraudulent silence. Case law has it that wherever the doing of a certain act or the transaction of a
given affair, or the performance of certain business is confided to an agent, the authority to so act will, in accordance with
a general rule often referred to, carry with it by implication the authority to do all of the collateral acts which are the natural
and ordinary incidents of the main act or business authorized. 59

The MTC sentenced the petitioner to suffer imprisonment of from two months and one day, as minimum, to four months of
arresto mayor, as maximum. The CA affirmed the penalty imposed by the trial court. This is erroneous. Section 2 of Act
4103, as amended, otherwise known as the Indeterminate Sentence Law, provides that the law will not apply if the maximum
term of imprisonment does not exceed one year:

SEC. 2. This Act shall not apply to persons convicted of offenses punished with death penalty or life-imprisonment; to those
convicted of treason, conspiracy or proposal to commit treason; to those convicted of misprision of treason, rebellion,
sedition or espionage; to those convicted of piracy; to those who are habitual delinquents; to those who shall have escaped
from confinement or evaded sentence; to those who having been granted conditional pardon by the Chief Executive shall
have violated the terms thereof; to those whose maximum term of imprisonment does not exceed one year, not to those
already sentenced by final judgment at the time of approval of this Act, except as provided in Section 5 hereof. (As amended
by Act No. 4225.)

In this case, the maximum term of imprisonment imposed on the petitioner was four months and one day of arresto mayor.
Hence, the MTC was proscribed from imposing an indeterminate penalty on the petitioner. An indeterminate penalty may
be imposed if the minimum of the penalty is one year or less, and the maximum exceeds one year. For example, the trial
court may impose an indeterminate penalty of six months of arresto mayor, as minimum, to two years and four months of
prision correccional, as maximum, since the maximum term of imprisonment it imposed exceeds one year. If the trial court
opts to impose a penalty of imprisonment of one year or less, it should not impose an indeterminate penalty, but a straight
penalty of one year or less instead. Thus, the petitioner may be sentenced to a straight penalty of one year, or a straight
penalty of less than one year, i.e., ten months or eleven months. We believe that considering the attendant circumstances,
a straight penalty of imprisonment of six months is reasonable. CcADHI

Conformably with Article 39 in relation to paragraph 3, Article 38 of the Revised Penal Code, the petitioner shall suffer
subsidiary imprisonment if he has no property with which to pay the penalty of fine.

IN LIGHT OF ALL THE FOREGOING, the petition is DENIED. The assailed Decision and Resolution are AFFIRMED WITH
MODIFICATION. Considering the surrounding circumstances of the case, the petitioner is hereby sentenced to suffer a
straight penalty of six (6) months imprisonment. The petitioner shall suffer subsidiary imprisonment in case of insolvency.

Costs against the petitioner.

SO ORDERED.

Puno, Austria-Martinez, Tinga and Chico-Nazario, JJ., concur.

7. Dela Cruz v. CA, G.R. No. 139442, December 6, 2006, 510 SCRA 103, 115.

THIRD DIVISION

[G.R. No. 139442. December 6, 2006.]

39
LOURDES DELA CRUZ, petitioner, vs. HON. COURT OF APPEALS and MELBA TAN TE, respondents.

DECISION

VELASCO, JR., J p:

For unto every one that hath shall be given, and he shall have abundance: but from him that hath not shall be taken away
even that which he hath.

— Holy Bible, Matthew 25:29

The Case

This petition for review seeks to nullify the April 30, 1999 Decision and the July 16, 1999 Resolution of the Court of Appeals
in CA-G.R. SP No. 49097, which reversed the Decision of the Manila Regional Trial Court (RTC), Branch 35, in Civil Case
No. 98-89174, and reinstated the Decision of the Manila Metropolitan Trial Court (MeTC), Branch 20, which ordered
petitioner Dela Cruz to vacate the subject lot in favor of respondent Tan Te. 1 DCcAIS

The Facts

The Reyes family, represented by Mr. Lino Reyes, owned the lot located at No. 1332 Lacson Street (formerly Gov. Forbes
Street), Sampaloc, Manila. Petitioner Lourdes Dela Cruz was one of their lessees, and she religiously paid rent over a
portion of the lot for well over 40 years. Sometime in 1989, a fire struck the premises and destroyed, among others,
petitioner's dwelling. After the fire, petitioner and some tenants returned to the said lot and rebuilt their respective houses;
simultaneously, the Reyes family made several verbal demands on the remaining lessees, including petitioner, to vacate
the lot but the latter did not comply. On February 21, 1994, petitioner was served a written demand to vacate said lot but
refused to leave. Despite the setback, the Reyes family did not initiate court proceedings against any of the lessees.

On November 26, 1996, the disputed lot was sold by the Reyeses to respondent Melba Tan Te by virtue of the November
26, 1996 Deed of Absolute Sale. Respondent bought the lot in question for residential purposes. Despite the sale, petitioner
Dela Cruz did not give up the lot. HSIaAT

On January 14, 1997, petitioner was sent a written demand to relinquish the premises which she ignored, prompting
respondent Tan Te to initiate conciliation proceedings at the barangay level. While respondent attempted to settle the
dispute by offering financial assistance, petitioner countered by asking PhP 500,000.00 for her house. Respondent rejected
the counter offer which she considered unconscionable. As a result, a certificate to file action was issued to Tan Te.

On September 8, 1997, respondent Tan Te filed an ejectment complaint with damages before the Manila MeTC, entitled
Melba Tan Te v. Lourdes Dela Cruz and docketed as Civil Case No. 156730-CV. The complaint averred that: (1) the
previous owners, the Reyeses were in possession and control of the contested lot; (2) on November 26, 1996, the lot was
sold to Tan Te; (3) prior to the sale, Dela Cruz forcibly entered the property with strategy and/or stealth; (4) the petitioner
unlawfully deprived the respondent of physical possession of the property and continues to do so; and, (5) the respondent
sent several written demands to petitioner to vacate the premises but refused to do so. HSEIAT

On October 24, 1997, petitioner filed her answer and alleged that: (1) the MeTC had no jurisdiction over the case because
it falls within the jurisdiction of the RTC as more than one year had elapsed from petitioner's forcible entry; (2) she was a
rent-paying tenant protected by PD 20; 2 (3) her lease constituted a legal encumbrance upon the property; and (4) the lot
was subject of expropriation.

The Ruling of the Manila MeTC

On April 3, 1998, the MeTC decided as follows:

WHEREFORE, judgment is hereby rendered in favor of the plaintiff as follows:

1. Ordering the defendant and all persons claiming right under her to vacate the premises situated at 1332 Lacson
Street (formerly Gov. Forbes Street), Sampaloc, Manila and peacefully return possession thereof to plaintiff; THaDAE

2. Ordering the defendant to pay the plaintiff the amount of P360.00 a month from December 1996 to November
1997; P432.00 a month from December 1997 to November 1998, plus 20% for each subsequent year until the premises
shall have been vacated and turned over to the plaintiff;

3. Ordering the defendant to pay the plaintiff the amount of P10,000.00 as attorney's fees; and, the costs of the suit.

SO ORDERED. 3
40
The Ruling of the Regional Trial Court

Unconvinced, petitioner Dela Cruz appealed the Decision of the MeTC in the Manila RTC and the appeal was docketed as
Civil Case No. 98-89174. On September 1, 1998, the RTC rendered its judgment setting aside the April 3, 1998 Decision
of the Manila MeTC and dismissed respondent Tan Te's Complaint on the ground that it was the RTC and not the MeTC
which had jurisdiction over the subject matter of the case. The RTC believed that since Tan Te's predecessor-in-interest
learned of petitioner's intrusion into the lot as early as February 21, 1994, the ejectment suit should have been filed within
the one-year prescriptive period which expired on February 21, 1995. Since the Reyes did not file the ejectment suit and
respondent Tan Te filed the action only on September 8, 1997, then the suit had become an accion publiciana cognizable
by the RTC. aDcHIS

The Ruling of the Court of Appeals

Disappointed at the turn of events, respondent Tan Te appealed the adverse Decision to the Court of Appeals (CA) which
was docketed as CA-G.R. SP No. 49097. This time, the CA rendered a Decision in favor of respondent Tan Te reversing
the Manila RTC September 1, 1998 Decision and reinstated the Manila MeTC April 3, 1998 Decision.

Petitioner tried to have the CA reconsider its Decision but was rebutted in its July 16, 1999 Resolution.

Unyielding to the CA Decision and the denial of her request for reconsideration, petitioner Dela Cruz now seeks legal
remedy through the instant Petition for Review on Certiorari before the Court.

The Issues

Petitioner Dela Cruz claims two (2) reversible errors on the part of the appellate court, to wit: EAISDH

THE HON. COURT OF APPEALS, WITH DUE RESPECT, WENT BEYOND THE ISSUES OF THE CASE AND
CONTRARY TO THOSE OF THE TRIAL COURT.

THE HON. COURT OF APPEALS, WITH DUE RESPECT, ERRED IN REVERSING THE DECISION OF THE RTC AND
IN EFFECT, REINSTATING THE DECISION OF THE [MeTC] WHICH IS CONTRADICTED BY THE EVIDENCE ON
RECORD. 4

The Court's Ruling

Discussion on Rule 45

Before we dwell on the principal issues, a few procedural matters must first be resolved. HDTSCc

Petitioner Dela Cruz asks the Court to review the findings of facts of the CA, a course of action proscribed by Section 1,
Rule 45. Firm is the rule that findings of fact of the CA are final and conclusive and cannot be reviewed on appeal to this
Court provided they are supported by evidence on record or substantial evidence. Fortunately for petitioner, we will be
liberal with her petition considering that the CA's factual findings contradict those of the RTC, and there was an asseveration
that the court a quo went beyond the issues of the case. Indeed, these grounds were considered exceptions to the factual
issue bar rule.

Secondly, the petition unnecessarily impleaded the CA in violation of Section 4, Rule 45. We will let this breach pass only
because there is a need to entertain the petition due to the conflicting rulings between the lower courts; however, a repetition
may result to sanctions.

The actual threshold issue is which court, the Manila RTC or the Manila MeTC, has jurisdiction over the Tan Te ejectment
suit. Once the jurisdictional issue is settled, the heart of the dispute is whether or not respondent is entitled to the ejectment
of petitioner Dela Cruz from the premises. ECaITc

However, the petition is bereft of merit.

On the Issue of Jurisdiction

Jurisdiction is the power or capacity given by the law to a court or tribunal to entertain, hear and determine certain
controversies. 5 Jurisdiction over the subject matter is conferred by law.

41
Section 33 of Chapter III — on Metropolitan Trial Courts, Municipal Trial Courts, and Municipal Circuit Trial Courts of B. P.
No. 129 6 provides:

Section 33. Jurisdiction of Metropolitan Trial Courts, Municipal Trial Courts and Municipal Circuit Trial Courts in civil
cases. — Metropolitan Trial Courts, Municipal Trial Courts, and Municipal Circuit Trial Courts shall exercise:

xxx xxx xxx

(2) Exclusive original jurisdiction over cases of forcible entry and unlawful detainer: Provided, That when, in such
cases, the defendant raises the question of ownership in his pleadings and the question of possession cannot be resolved
without deciding the issue of ownership, the issue of ownership shall be resolved only to determine the issue of possession.
aTHCSE

Thus exclusive, original jurisdiction over ejectment proceedings (accion interdictal) is lodged with the first level courts. This
is clarified in Section 1, Rule 70 of the 1997 Rules of Civil Procedure that embraces an action for forcible entry (detentacion),
where one is deprived of physical possession of any land or building by means of force, intimidation, threat, strategy, or
stealth. In actions for forcible entry, three (3) requisites have to be met for the municipal trial court to acquire jurisdiction.
First, the plaintiffs must allege their prior physical possession of the property. Second, they must also assert that they were
deprived of possession either by force, intimidation, threat, strategy, or stealth. Third, the action must be filed within one
(1) year from the time the owners or legal possessors learned of their deprivation of physical possession of the land or
building. TAIESD

The other kind of ejectment proceeding is unlawful detainer (desahucio), where one unlawfully withholds possession of the
subject property after the expiration or termination of the right to possess. Here, the issue of rightful possession is the one
decisive; for in such action, the defendant is the party in actual possession and the plaintiff's cause of action is the
termination of the defendant's right to continue in possession. 7 The essential requisites of unlawful detainer are: (1) the
fact of lease by virtue of a contract express or implied; (2) the expiration or termination of the possessor's right to hold
possession; (3) withholding by the lessee of the possession of the land or building after expiration or termination of the right
to possession; (4) letter of demand upon lessee to pay the rental or comply with the terms of the lease and vacate the
premises; and (5) the action must be filed within one (1) year from date of last demand received by the defendant.

A person who wants to recover physical possession of his real property will prefer an ejectment suit because it is governed
by the Rule on Summary Procedure which allows immediate execution of the judgment under Section 19, Rule 70 unless
the defendant perfects an appeal in the RTC and complies with the requirements to stay execution; all of which are
nevertheless beneficial to the interests of the lot owner or the holder of the right of possession. aITECA

On the other hand, Section 19, of Chapter II of B.P. No. 129 on Regional Trial Courts provides:

Section 19. Jurisdiction in civil cases. — Regional Trial Courts shall exercise exclusive original jurisdiction:

xxx xxx xxx

(2) In all civil actions which involve the title to, or possession of, real property, or any interest therein, except actions
for forcible entry into and unlawful detainer of lands or buildings, original jurisdiction over which is conferred upon
Metropolitan Trial Courts, Municipal Trial Courts and Municipal Circuit Trial Courts.

Two (2) kinds of action to recover possession of real property which fall under the jurisdiction of the RTC are: (1) the plenary
action for the recovery of the real right of possession (accion publiciana) when the dispossession has lasted for more than
one year or when the action was filed more than one (1) year from date of the last demand received by the lessee or
defendant; and (2) an action for the recovery of ownership (accion reivindicatoria) which includes the recovery of
possession. TCaADS

These actions are governed by the regular rules of procedure and adjudication takes a longer period than the summary
ejectment suit.

To determine whether a complaint for recovery of possession falls under the jurisdiction of the MeTC (first level court) or
the RTC (second level court), we are compelled to go over the allegations of the complaint. The general rule is that what
determines the nature of the action and the court that has jurisdiction over the case are the allegations in the complaint.
These cannot be made to depend upon the defenses set up in the answer or pleadings filed by the defendant. 8

This general rule however admits exceptions. In Ignacio v. CFI of Bulacan, it was held "that while the allegations in the
complaint make out a case for forcible entry, where tenancy is averred by way of defense and is proved to be the real issue,
the case should be dismissed for lack of jurisdiction as the case should properly be filed with the then Court of Agrarian
Relations." 9 SHIcDT
42
The cause of action in a complaint is not what the designation of the complaint states, but what the allegations in the body
of the complaint define and describe. The designation or caption is not controlling, more than the allegations in the complaint
themselves are, for it is not even an indispensable part of the complaint. 10

Let us refer to the allegations of the complaint filed in the Manila MeTC in Civil Case No. 98-89174, which we quote
verbatim:

3. That plaintiff is the absolute and registered owner of a parcel of land located at No. 1332, Lacson Street, Sampaloc,
Manila now being occupied by defendant;

4. That plaintiff purchased the above-said parcel of land together with its improvements from the legal heirs of the
late EMERLINDA DIMAYUGA REYES on November 26, 1996, under and by virtue of a Deed of Absolute Sale . . .; DTIaHE

5. That pursuant to the said deed of sale, the title to the land and all its improvements was transferred in plaintiff's
name as evidenced by Transfer Certificate of Title No. 233273 issued by the Register of Deeds of Manila on April 22, 1997
. . .;

6. That prior to said sale, the previous owners, represented by Mr. Lino Reyes, husband of the said deceased
Emerlinda D. Reyes and the administrator of her estate, was in possession and control of the property subject of this
complaint;

7. That also prior to said sale, defendant, without the knowledge and consent of Mr. Lino Reyes, surreptitiously and
by means of stealth and strategy entered, used and occupied the said premises thus depriving the former of rightful
possession thereof;

8. That on February 21, 1994, Mr. Lino Reyes, through Atty. Alejo Sedico, his lawyer, furnished the defendants a
letter formally demanding that defendant vacate the premises . . .;

9. That, however, defendant failed and refused to vacate despite just and legal demand by Mr. Lino Reyes; cHDaEI

10. That after the sale to plaintiff of said premises, plaintiff has several times demanded of defendants to vacate the
premises, the last demand having been made on them personally and in writing on January 14, 1997 . . .;

11. That defendant failed and refused and still fails and refuses to vacate the premises without legal cause or justifiable
reason whatsoever; 11

The answer of petitioner averred:

4. The Court has no jurisdiction over the case, having been filed by plaintiff more than the reglementary one year
period to commence forcible entry case, which is reckoned from the date of the alleged unlawful entry of defendant by the
use of stealth and strategy into the premises;

5. For more than four decades now, defendant has been and still is a rent-paying tenant of the subject land occupied
by their residential house, dating back to the original owner-lessor, the Dimayuga family. Her lease with no definite duration,
commenced with a rent at P60.00 per month until it was gradually increased in the ensuing years. As of November 1996,
it stood at P300.00 a month; ESAHca

6. In this circumstances [sic], defendant enjoys the protective mantle of P.D. 20 and the subsequent rental control
status against dispossession. She cannot be ejected other than for causes prescribed under B.P. Blg. 25. Further, in case
of sale of the land, she has the right of first refusal under the express provision of P.D. 1571;

7. Throughout the years of her tenancy, defendant has been updated in her rental payment until the collector of the
original owner-lessor no longer came around as she has done theretofore;

7.1. As a result, she was compelled to file a petition for consignation of rent before the Metropolitan Trial Court of
Manila;

8. A bona fide tenant within the ambit if [sic] P.D. 20 and the subsequent rental control status, including B.P. Blg. 25,
under its terms, cannot be ousted on a plea of expiration of her monthly lease;

9. Her lease constitutes a legal encumbrance upon the property of the lessor/owner and binds the latter's successor-
in-interest who is under obligation to respect it; aAcHCT

10. The land at bench is the subject of a pending expropriation proceedings;

43
11. Plaintiff being a married woman cannot sue or be sued without being joined by her husband; 12

Undeniably, the aforequoted allegations of the complaint are vague and iffy in revealing the nature of the action for
ejectment.

The allegations in the complaint show that prior to the sale by Lino Reyes, representing the estate of his wife Emerlinda
Reyes, he was in possession and control of the subject lot but were deprived of said possession when petitioner, by means
of stealth and strategy, entered and occupied the same lot. These circumstances imply that he had prior physical
possession of the subject lot and can make up a forcible entry complaint. ECcTaH

On the other hand, the allegation that petitioner Dela Cruz was served several demands to leave the premises but refused
to do so would seem to indicate an action for unlawful detainer since a written demand is not necessary in an action for
forcible entry. It is a fact that the MeTC complaint was filed on September 8, 1997 within one (1) year from the date of the
last written demand upon petitioner Dela Cruz on January 14, 1997.

As previously discussed, the settled rule is jurisdiction is based on the allegations in the initiatory pleading and the defenses
in the answer are deemed irrelevant and immaterial in its determination. However, we relax the rule and consider the
complaint at bar as an exception in view of the special and unique circumstances present. First, as in Ignacio v. CFI of
Bulacan, 13 the defense of lack of jurisdiction was raised in the answer wherein there was an admission that petitioner
Dela Cruz was a lessee of the former owners of the lot, the Reyeses, prior to the sale to respondent Tan Te. The fact that
petitioner was a tenant of the predecessors-in-interest of respondent Tan Te is material to the determination of jurisdiction.
Since this is a judicial admission against the interest of petitioner, such admission can be considered in determining
jurisdiction. Second, the ejectment suit was filed with the Manila MeTC on September 8, 1997 or more than nine (9) years
ago. To dismiss the complaint would be a serious blow to the effective dispensation of justice as the parties will start anew
and incur additional legal expenses after having litigated for a long time. Equitable justice dictates that allegations in the
answer should be considered to aid in arriving at the real nature of the action. Lastly, Section 6, Rule 1 of the Rules of Court
clearly empowers the Court to construe Rule 70 and other pertinent procedural issuances "in a liberal manner to promote
just, speedy, and inexpensive disposition of every action and proceeding." DcICEa

Based on the complaint and the answer, it is apparent that the Tan Te ejectment complaint is after all a complaint for
unlawful detainer. It was admitted that petitioner Dela Cruz was a lessee of the Reyeses for around four (4) decades. Thus,
initially petitioner as lessee is the legal possessor of the subject lot by virtue of a contract of lease. When fire destroyed her
house, the Reyeses considered the lease terminated; but petitioner Dela Cruz persisted in returning to the lot and occupied
it by strategy and stealth without the consent of the owners. The Reyeses however tolerated the continued occupancy of
the lot by petitioner. Thus, when the lot was sold to respondent Tan Te, the rights of the Reyeses, with respect to the lot,
were transferred to their subrogee, respondent Tan Te, who for a time also tolerated the stay of petitioner until she decided
to eject the latter by sending several demands, the last being the January 14, 1997 letter of demand. Since the action was
filed with the MeTC on September 8, 1997, the action was instituted well within the one (1) year period reckoned from
January 14, 1997. Hence, the nature of the complaint is one of unlawful detainer and the Manila MeTC had jurisdiction over
the complaint. IHEAcC

Thus, an ejectment complaint based on possession by tolerance of the owner, like the Tan Te complaint, is a specie of
unlawful detainer cases.

As early as 1913, case law introduced the concept of possession by tolerance in ejectment cases as follows:

It is true that the landlord might, upon the failure of the tenant to pay the stipulated rents, consider the contract broken and
demand immediate possession of the rented property, thus converting a legal possession into illegal possession. Upon the
other hand, however, the landlord might conclude to give the tenant credit for the payment of the rents and allow him to
continue indefinitely in the possession of the property. In other words, the landlord might choose to give the tenant credit
from month to month or from year to year for the payment of their rent, relying upon his honesty of his financial ability to
pay the same. During such period the tenant would not be in illegal possession of the property and the landlord could not
maintain an action of desahucio until after he had taken steps to convert the legal possession into illegal possession. A
mere failure to pay the rent in accordance with the contract would justify the landlord, after the legal notice, in bringing an
action of desahucio. The landlord might, however, elect to recognize the contract as still in force and sue for the sums due
under it. It would seem to be clear that the landlord might sue for the rents due and [unpaid, without electing to terminate
the contract of tenancy;] [w]hether he can declare the contract of tenancy broken and sue in an action desahucio for the
possession of the property and in a separate actions for the rents due and damages, etc. 14

The concept of possession by tolerance in unlawful detainer cases was further refined and applied in pertinent cases
submitted for decision by 1966. The rule was articulated as follows: TIEHSA

44
Where despite the lessee's failure to pay rent after the first demand, the lessor did not choose to bring an action in court
but suffered the lessee to continue occupying the land for nearly two years, after which the lessor made a second demand,
the one-year period for bringing the detainer case in the justice of the peace court should be counted not from the day the
lessee refused the first demand for payment of rent but from the time the second demand for rents and surrender of
possession was not complied with. 15

In Calubayan v. Pascual, a case usually cited in subsequent decisions on ejectment, the concept of possession by tolerance
was further elucidated as follows:

In allowing several years to pass without requiring the occupant to vacate the premises nor filing an action to eject him,
plaintiffs have acquiesced to defendant's possession and use of the premises. It has been held that a person who occupies
the land of another at the latter's tolerance or permission, without any contract between them, is necessarily bound by an
implied promise that he will vacate upon demand, failing which a summary action for ejectment is the proper remedy against
them. The status of the defendant is analogous to that of a lessee or tenant whose term of lease has expired but whose
occupancy continued by tolerance of the owner. In such a case, the unlawful deprivation or withholding of possession is to
be counted from the date of the demand to vacate. 16 (Emphasis supplied.) HDacIT

From the foregoing jurisprudence, it is unequivocal that petitioner's possession after she intruded into the lot after the fire
— was by tolerance or leniency of the Reyeses and hence, the action is properly an unlawful detainer case falling under
the jurisdiction of the Manila MeTC.

Even if we concede that it is the RTC and not the MeTC that has jurisdiction over the Tan Te complaint, following the
reasoning that neither respondent nor her predecessor-in-interest filed an ejectment suit within one (1) year from February
21, 1994 when the Reyeses knew of the unlawful entry of petitioner, and hence, the complaint is transformed into an accion
publiciana, the Court deems it fair and just to suspend its rules in order to render efficient, effective, and expeditious justice
considering the nine (9) year pendency of the ejectment suit. More importantly, if there was uncertainty on the issue of
jurisdiction that arose from the averments of the complaint, the same cannot be attributed to respondent Tan Te but to her
counsel who could have been confused as to the actual nature of the ejectment suit. The lawyer's apparent imprecise
language used in the preparation of the complaint without any participation on the part of Tan Te is sufficient special or
compelling reason for the grant of relief. HSDaTC

The case of Barnes v. Padilla 17 elucidates the rationale behind the exercise by this Court of the power to relax, or even
suspend, the application of the rules of procedure:

Let it be emphasized that the rules of procedure should be viewed as mere tools designed to facilitate the attainment of
justice. Their strict and rigid application, which would result in technicalities that tend to frustrate rather than promote
substantial justice, must always be eschewed. Even the Rules of Court reflect this principle. The power to suspend or even
disregard rules can be so pervasive and compelling as to alter even that which this Court itself has already declared to be
final . . . .

The emerging trend in the rulings of this Court is to afford every party litigant the amplest opportunity for the proper and
just determination of his cause, free from the constraints of technicalities. Time and again, this Court has consistently held
that rules must not be applied rigidly so as not to override substantial justice. 18

Moreover, Section 8, Rule 40 authorizes the RTC — in case of affirmance of an order of the municipal trial court dismissing
a case without trial on the merits and the ground of dismissal is lack of jurisdiction over the subject matter — to try the case
on the merits as if the case was originally filed with it if the RTC has jurisdiction over the case. In the same vein, this Court,
in the exercise of its rule-making power, can suspend its rules with respect to this particular case (pro hac vice), even if
initially, the MeTC did not have jurisdiction over the ejectment suit, and decide to assume jurisdiction over it in order to
promptly resolve the dispute. cEHSTC

The issue of jurisdiction settled, we now scrutinize the main issue.

At the heart of every ejectment suit is the issue of who is entitled to physical possession of the lot or possession de facto.

We rule in favor of respondent Tan Te for the following reasons:

1. Petitioner admitted in her Answer that she was a rent-paying tenant of the Reyeses, predecessors-in-interest of
respondent Tan Te. As such, she recognized the ownership of the lot by respondent, which includes the right of possession.

2. After the fire raged over the structures on the subject lot in late 1989 the contracts of lease expired, as a result of
which Lino Reyes demanded that all occupants, including petitioner, vacate the lot but the latter refused to abandon the

45
premises. During the duration of the lease, petitioner's possession was legal but it became unlawful after the fire when the
lease contracts were deemed terminated and demands were made for the tenants to return possession of the lot. EACIaT

3. Petitioner's possession is one by the Reyeses' tolerance and generosity and later by respondent Tan Te's.

Petitioner fully knows that her stay in the subject lot is at the leniency and magnanimity of Mr. Lino Reyes and later of
respondent Tan Te; and her acquiescence to such use of the lot carries with it an implicit and assumed commitment that
she would leave the premises the moment it is needed by the owner. When respondent Tan Te made a last, written demand
on January 14, 1997 and petitioner breached her promise to leave upon demand, she lost her right to the physical
possession of the lot. Thus, respondent Tan Te should now be allowed to occupy her lot for residential purposes, a dream
that will finally be realized after nine (9) years of litigation.

Petitioner raises the ancillary issue that on March 15, 1998, the Manila City Council passed and approved Ordinance No.
7951: DHaECI

[a]uthorizing the Manila City Mayor to acquire either by negotiation or expropriation certain parcels of land covered by
Transfer Certificates of Title Nos. 233273, 175106 and 140471, containing an area of One Thousand Four Hundred Twenty
Five (1,425) square meters, located at Maria Clara and Governor Forbes Streets, Sta. Cruz, Manila, for low cost housing
and award to actual bonafide residents thereat and further authorizing the City Mayor to avail for that purpose any available
funds of the city and other existing funding facilities from other government agencies . . . . 19

It readily appears that this issue was not presented before the Court of Appeals in CA-G.R. SP No. 49097 despite the fact
that the respondent's petition was filed on September 25, 1998, six months after the ordinance was passed. Thus, this
issue is proscribed as are all issues raised for the first time before the Court are proscribed.

Even granting for the sake of argument that we entertain the issue, we rule that the intended expropriation of respondent's
lot (TCT No. 233273) by the city government of Manila will not affect the resolution of this petition. For one thing, the issue
can be raised by petitioner in the appropriate legal proceeding. Secondly, the intended expropriation might not even be
implemented since it is clear from the ordinance that the City Mayor will still locate available funds for project, meaning the
said expense is not a regular item in the budget. IaDSEA

WHEREFORE, this petition is DENIED for lack of merit. The April 30, 1999 Decision of the Court of Appeals reinstating the
April 3, 1998 MeTC Decision in Civil Case No. 156730-CV and the July 16, 1999 Resolution in CA-G.R. SP No. 49097 are
hereby AFFIRMED IN TOTO.

No costs.

SO ORDERED.

Quisumbing, Carpio, Carpio Morales and Tinga, JJ., concur.

8. Borra vs CA

THIRD DIVISION

[G.R. No. 167484. September 9, 2013.]

HERNANDO BORRA, JOHN PACHEO, DANILO PEREZ, FELIZARDO SIMON, RAMON BUENACOSA, JR., FELIX
BELADOR, WILFREDO LUPO, RONALD VILLARIAS, ARSENIO MINDANAO, MAX NONALA, SIMPLICIO DE ERIT,
NOEL DONGUINES, JULIO BORRA, MELCHOR JAVIER, JOHNNY ENRICO VARGAS, PAQUITO SONDIA, JOSE
SALAJOG, ELMER LUPO, RAZUL ARANEZ, NELSON PEREZ, BALBINO ABLAY, FERNANDO SIMON, JIMMY
VILLARTA, ROMEO CAINDOC, SALVADOR SANTILLAN, ROMONEL JANEO, ERNESTO GONZALUDO, JOSE PAJES,
ROY TAN, FERNANDO SANTILLAN JR., DEMETRIO SEMILLA, RENE CORDERO, EDUARDO MOLENO, ROMY
DINAGA, HERNANDO GUMBAN, FEDERICO ALVARICO, ELMER CATO, ROGELIO CORDERO, RODNEY PAJES,
ERNIE BAYER, ARMANDO TABARES, NOLI AMADOR, MARIO SANTILLAN, ALANIL TRASMONTE, VICTOR ORTEGA,
JOEVING ROQUERO, CYRUS PINAS, DANILO PERALES, and ALFONSO COSAS, JR., petitioners, vs. COURT OF
APPEALS SECOND AND NINETEENTH DIVISIONS and HAWAIIAN PHILIPPINE COMPANY, respondents.

DECISION

PERALTA, J p:

46
Before the Court is a special civil action for certiorari under Rule 65 of the Rules of Court seeking the nullification of the
November 14, 2003 Resolution, 1 as well as the subsequent Decision 2 and Resolution, 3 dated June 22, 2004 and January
14, 2005, respectively, of the Court of Appeals (CA) in CA-G.R. SP No. 78729. The November 14, 2003 Resolution granted
private respondent's motion for the issuance of a preliminary mandatory injunction. The assailed CA Decision, on the other
hand, set aside the Order of the Labor Arbiter, dated August 12, 2003, and dismissed RAB Case No. 09-10698-97, while
the January 14, 2005 CA Resolution denied petitioners' motion for reconsideration. HTCaAD

The factual and procedural antecedents of the case are as follows:

On September 12, 1997, herein petitioners filed with the National Labor Relations Commission (NLRC) Regional Arbitration
Branch No. VI in Bacolod City two separate complaints which were docketed as RAB Case No. 06-09-10698-97 and RAB
Case No. 06-09-10699-97. RAB Case No. 06-09-10698-97 was filed against herein private respondent alone, while RAB
Case No. 06-09-10699-97 impleaded herein private respondent and a certain Fela Contractor as respondents. In RAB
Case No. 06-09-10698-97, herein petitioners asked that they be recognized and confirmed as regular employees of herein
private respondent and further prayed that they be awarded various benefits received by regular employees for three (3)
years prior to the filing of the complaint, while in RAB Case No. 06-09-10699-97, herein petitioners sought for payment of
unpaid wages, holiday pay, allowances, 13th month pay, service incentive leave pay, moral and exemplary damages also
during the three (3) years preceding the filing of the complaint. CEHcSI

On October 16, 1997, private respondent filed a Motion to Consolidate 4 the abovementioned cases, but the Labor Arbiter
in charge of the case denied the said Motion in its Order 5 dated October 20, 1997.

On January 9, 1998, private respondent filed a Motion to Dismiss 6 RAB Case No. 06-09-10698-97 on the ground of res
judicata. Private respondent cited an earlier decided case entitled "Humphrey Perez, et al. v. Hawaiian Philippine Co., et
al." (Perez case) and docketed as RAB Case No. 06-04-10169-95, which was an action for recovery of 13th month pay
and service incentive leave pay, and it includes herein petitioners among the complainants and herein private respondent
and one Jose Castillon (Castillon) as respondents. Private respondent contended that the Perez case, which has already
become final and executory, as no appeal was taken therefrom, serves as a bar to the litigation of RAB Case No. 06-09-
10698-97, because it was ruled therein that petitioners are not employees of private respondent but of Castillon. DEAaIS

In an Order 7 dated July 9, 1998, the Labor Arbiter granted private respondent's Motion to Dismiss.

Petitioners appealed to the NLRC which set aside the Order of the Labor Arbiter, reinstated the complaint in RAB Case No.
06-09-10698-97 and remanded the same for further proceedings. 8

Private respondent appealed to the CA. On January 12, 2001, the CA rendered judgment, affirming the Decision of the
NLRC and denied the subsequent motion for reconsideration. aITECA

Aggrieved, private respondent filed a petition for review on certiorari before this Court. The case was entitled as "Hawaiian
Philippine Company v. Borra" and docketed as G.R. No. 151801. On November 12, 2002, this Court rendered its Decision
denying the petition and affirming the Decision of the CA. Quoting with approval, the assailed Decision of the CA, this Court
held, thus:

The Court of Appeals committed no reversible error. The two cases in question indeed involved different causes of action.
The previous case of "Humphrey Perez vs. Hawaiian Philippine Company" concerned a money claim and pertained to the
years 1987 up until 1995. During that period, private respondents were engaged by contractor Jose Castillon to work for
petitioner at its warehouse. It would appear that the finding of the Labor Arbiter, to the effect that no employer-employee
relationship existed between petitioner and private respondents, was largely predicated on the absence of privity between
them. The complaint for confirmation of employment, however, was filed by private respondents on 12 September 1997,
by which time, Jose Castillon was no longer the contractor. The Court of Appeals came out with these findings; viz.: ATcaID

At first glance, it would appear that the case at bench is indeed barred by Labor Arbiter Drilon's findings since both petitioner
and private respondents are parties in Perez and the issue of employer-employee relationship was finally resolved therein.

However, the factual milieu of the Perez case covered the period November 1987 to April 6, 1995 (date of filing of the
complaint), during which time private respondents, by their own admission, were engaged by Castillon to work at petitioner's
warehouse.

In contrast, the instant case was filed on September 12, 1997, by which time, the contractor involved was Fela Contractor;
and private respondents' prayer is for confirmation of their status as regular employees of petitioner.

Stated differently, Perez pertains to private respondents' employment from 1987 to 1995, while the instant case covers a
different (subsequent) period. Moreover, in Perez, the finding that no employer-employee relationship existed between

47
petitioner and private respondents was premised on absence of privity between Castillon and petitioner. Consequently,
Perez and the instant case involve different subject matters and causes of action. aTHASC

On the other hand, resolution of the case at bench would hinge on the nature of the relationship between petitioner and
Fela Contractor. In other words, private respondents' action for declaration as regular employees of petitioner will not
succeed unless it is established that Fela Contractor is merely a "labor-only" contractor and that petitioner is their real
employer.

Indeed, it is pure conjecture to conclude that the circumstances obtaining in Perez subsisted until the filing of the case at
bench as there is no evidence supporting such conclusion. There is, as yet, no showing that Fela Contractor merely stepped
into the shoes of Castillon. Neither has Fela Contractor's real principal been shown: petitioner or the sugar traders/planters?
cdll

Consequently, factual issues must first be ventilated in appropriate proceedings before the issue of employer-employee
relationship between petitioner and private respondents [herein private respondent and petitioners] can be determined.

It is premature to conclude that the evidence in Perez would determine the outcome of the case at bench because as earlier
pointed out, there is still no showing that the contractor (Fela contractor) in this case can be considered as on the same
footing as the previous contractor (Castillon). Such factual issue is crucial in determining whether petitioner is the real
employer of private respondents. 9 aTcHIC

In the meantime, on December 21, 1998, the Labor Arbiter rendered a Decision 10 in RAB Case No. 06-09-10699-97
holding that there is no employer-employee relations between private respondent and petitioners. The Labor Arbiter held
as follows:

. . . Fela Contractor as may be noted happened to replace Jose Castillon, as Contractor of the traders or sugar planters,
who absorbed the workers of the erstwhile contractor Castillon. The complainants herein, who were the workers of Castillon,
formally applied for employment with respondent Jose Castillon, the owner of Fela Contractor, the new handler and hauler
of the sugar planters and traders. Thus, on February 15, 1996, respondent Jardinico, representative of respondent Fela
Contractor, wrote a letter to the Administrative Manager of respondent Hawaiian informing the latter that as of March 1,
1996, the former workers of Castillon the previous contractor, who undertook the handling and withdrawal of the sugar of
the traders and planters[,] have been absorbed and employed by Fela, with a request to allow them to enter the premises
of the company. aHICDc

In this suit, the same complainants now seek monetary benefits arising from the employment and they again impleaded
respondent Hawaiian.

We, thus resolve to dismiss the complaint against respondent Hawaiian, who as we have found in an earlier pronouncement
has no employer-employee relations with the complainant, let alone, any privity of relationship, except for the fact that it is
the depository of sugar where the sugar of the planters and traders are hauled by the workers of the contractor, like
respondent herein Fela Contractor/Jardinico. 11 ISCaDH

No appeal was taken from the abovequoted Decision. Thus, the same became final and executory. 12

As a consequence of the finality of the Decision in RAB Case No. 06-09-10699-97, herein private respondent again filed a
Motion to Dismiss 13 RAB Case No. 06-09-10698-97 on the ground, among others, of res judicata. Private respondent
contended that the final and executory Decision of the Labor Arbiter in RAB Case No. 06-09-10699-97, which found no
employer-employee relations between private respondent and petitioners, serves as a bar to the further litigation of RAB
Case No. 06-09-10698-97. CSDTac

On August 12, 2003, the Labor Arbiter handling RAB Case No. 06-09-10698-97 issued an Order 14 denying private
respondent's Motion to Dismiss.

Private respondent then filed a petition for certiorari and prohibition with the CA assailing the August 12, 2003 Order of the
Labor Arbiter.

On June 22, 2004, the CA rendered its questioned Decision, the dispositive portion of which reads, thus:

WHEREFORE, foregoing premises considered, the petition is GRANTED. Accordingly, the Order dated August 12, 2003
of public respondent is hereby ANNULLED and SET ASIDE. RAB Case No. 09-10698-97 is ordered DISMISSED.
CHDAaS

SO ORDERED. 15

48
Petitioners filed a Motion for Reconsideration, but the CA denied it in its Resolution 16 dated January 14, 2005.

Hence, the present petition for certiorari based on the following grounds:

I. THE COURT OF APPEALS ACTED ABSOLUTELY WITHOUT ANY JURISDICTION WHEN IT TOOK
COGNIZANCE OF THE 2nd PETITION OF HPCO DESPITE THE ABSOLUTE LACK OF ANY INTERVENING OR
SUPERVENING EVENT THAT WOULD RENDER THE ORDERS OF THE SUPREME COURT AND COURT OF
APPEALS INAPPLICABLE AND THE CLEAR AND ESTABLISHED DECISION LAID DOWN BY THE FIRST DIVISION OF
THE SUPREME COURT UNDER CHIEF JUSTICE HILARIO G. DAVIDE, JR., ASSOCIATE JUSTICES JOSE C. VITUG,
CONSUELO YNARES-SANTIAGO, ANTONIO T. CARPIO, AND ADOLFO S. AZCUNA AND BY THE COURT OF
APPEALS UNDER JUSTICES EDGARDO P. CRUZ, RAMON MABUTAS, JR., ROBERTO A. BARRIOS, MA. ALICIA
AUSTRIA-MARTINEZ AND HILARION L. AQUINO, RULING THAT FURTHER HEARINGS AND TRIAL MUST BE
CONDUCTED BY THE LABOR ARBITER WHICH SIGNIFICANTLY FOUND THE EXISTENCE OF EMPLOYER-
EMPLOYEE RELATIONSHIP IN HIS DECISION DATED SEPTEMBER 25, 2003. EICSDT

II. THE COURT OF APPEALS HAD SERIOUSLY ERRED, IF NOT GRAVELY ABUSED ITS DISCRETION WHEN
IT CHOSE TO DELIBERATELY IGNORE AND/OR ENTIRELY DISREGARD THE CLEAR AND ESTABLISHED FACTS
ON RECORD AS TO THE EXISTENCE OF THE IDENTITY OF SUBJECT MATTER AND CAUSE OF ACTION BETWEEN
HPCO VS. BORRA & 48 OTHERS/NLRC, ET AL., C.A. G.R. NO. 59132 AND HPCO VS. NLRC, BORRA, ET AL., G.R.
NO. 151801 ON ONE HAND AND HPCO VS. HON. PHEBUN PURA/BORRA & 48 OTHERS, C.A. G.R. NO. 78729 ON
THE OTHER HAND.

III. THE COURT OF APPEALS SERIOUSLY ERRED IN TAKING COGNIZANCE OF THE SECOND PETITION OF
HPCO DESPITE THE CLEAR AND ESTABLISHED FACT ON RECORD THAT HPCO HAD SIMULTANEOUSLY AND
SUCCESSIVELY FILED AN (sic) IDENTICAL THREE (3) MOTIONS TO DISMISS IN THE SALA OF LABOR ARBITERS
AND TWO (2) PETITIONS FOR CERTIORARI IN THE COURT OF APPEALS WHICH IS A FLAGRANT VIOLATION ON
THE LAW OF FORUM SHOPPING. 17

The petition lacks merit. HTASIa

This Court is not persuaded by petitioners' argument that the CA has no jurisdiction over private respondent's petition for
certiorari because this Court, in G.R. No. 151801, lodged jurisdiction in the Labor Arbiter by directing the remand of RAB
Case No. 06-09-10698-97 thereto for further proceedings.

It is settled that jurisdiction over the subject matter is conferred by law and it is not within the courts, let alone the parties,
to themselves determine or conveniently set aside. 18

In this regard, it should be reiterated that what has been filed by private respondent with the CA is a special civil action for
certiorari assailing the Labor Arbiter's Order which denied its motion to dismiss.

Section 3, Rule V of the NLRC Rules of Procedure, which was then prevailing at the time of the filing of private respondent's
petition for certiorari with the CA, clearly provides: IESDCH

SECTION 3. MOTION TO DISMISS. — On or before the date set for the conference, the respondent may file a motion
to dismiss. Any motion to dismiss on the ground of lack of jurisdiction, improper venue, or that the cause of action is barred
by prior judgment, prescription or forum shopping, shall be immediately resolved by the Labor Arbiter by a written order.
An order denying the motion to dismiss or suspending its resolution until the final determination of the case is not
appealable. 19 DASEac

In the case of Metro Drug Distribution, Inc. v. Metro Drug Corporation Employees Association-Federation of Free Workers,
20 this Court held that:

. . . The NLRC rule proscribing appeal from a denial of a motion to dismiss is similar to the general rule observed in civil
procedure that an order denying a motion to dismiss is interlocutory and, hence, not appealable until final judgment or order
is rendered. The remedy of the aggrieved party in case of denial of the motion to dismiss is to file an answer and interpose,
as a defense or defenses, the ground or grounds relied upon in the motion to dismiss, proceed to trial and, in case of
adverse judgment, to elevate the entire case by appeal in due course. In order to avail of the extraordinary writ of certiorari,
it is incumbent upon petitioner to establish that the denial of the motion to dismiss was tainted with grave abuse of discretion.
21 HTCIcE

In this regard, Rule 41 of the Rules of Court, which is applied in a suppletory character to cases covered by the NLRC
Rules, provides that in all the instances enumerated under the said Rule, where the judgment or final order is not
appealable, the aggrieved party may file an appropriate special civil action under Rule 65. 22 Thus, this Court has held that

49
when the denial of a motion to dismiss is tainted with grave abuse of discretion, the grant of the extraordinary remedy of
certiorari may be justified. 23 On the basis of the foregoing, it is clear that the CA has jurisdiction over the special civil action
for certiorari filed by private respondent as the latter was able to allege and establish that the denial of its motion to dismiss
was tainted with grave abuse of discretion. Petitioners are wrong to argue that this Court's directive in G.R. No. 151801 to
remand RAB Case No. 06-09-10698-97 to the Labor Arbiter for further proceedings deprives the CA of its jurisdiction over
private respondent's petition for certiorari. The essence of this Court's ruling in G.R. No. 151801 is simply to require
resolution of the factual issue of whether or not Fela Contractor has stepped into the shoes of Castillon and, thus, has taken
petitioners in its employ. In other words, this Court called for a prior determination as to who is the real employer of
petitioners. This issue, however, was already settled as will be discussed below. SHaATC

At the outset, the underlying question which has to be resolved in both RAB Case Nos. 06-09-10698-97 and 06-09-10699-
97, before any other issue in these cases could be determined, is the matter of determining petitioners' real employer. Is it
Fela Contractor, or is it private respondent? Indeed, the tribunals and courts cannot proceed to decide whether or not
petitioners should be considered regular employees, and are thus entitled to the benefits they claim, if there is a prior finding
that they are, in the first place, not employees of private respondent. Stated differently, and as correctly held by the CA,
petitioners' prayer for regularization in RAB Case No. 06-09-10698-97 is essentially dependent on the existence of
employer-employee relations between them and private respondent, because one cannot be made a regular employee of
one who is not his employer. In the same vein, petitioners' prayer in RAB Case No. 06-09-10699-97 for the recovery of
backwages, 13th month pay, holiday pay and service incentive leave pay from private respondent likewise rests on the
determination of whether or not the former are, indeed, employees of the latter. aCHDST

As earlier mentioned, this issue has already been settled. In the already final and executory decision of the Labor Arbiter
in RAB Case No. 06-09-10699-97, it was ruled therein that no employer-employee relationship exists between private
respondent and petitioners because the latter's real employer is Fela Contractor. Thus, insofar as the question of employer
and employee relations between private respondent and petitioners is concerned, the final judgment in RAB Case No. 06-
09-10699-97 has the effect and authority of res judicata by conclusiveness of judgment.

Discussing the concept of res judicata, this Court held in Antonio v. Sayman Vda. de Monje 24 that: EScAID

. . . [R]es judicata is defined as "a matter adjudged; a thing judicially acted upon or decided; a thing or matter settled by
judgment." According to the doctrine of res judicata, an existing final judgment or decree rendered on the merits, and without
fraud or collusion, by a court of competent jurisdiction, upon any matter within its jurisdiction, is conclusive of the rights of
the parties or their privies, in all other actions or suits in the same or any other judicial tribunal of concurrent jurisdiction on
the points and matters in issue in the first suit. To state simply, a final judgment or decree on the merits by a court of
competent jurisdiction is conclusive of the rights of the parties or their privies in all later suits on all points and matters
determined in the former suit. cDAITS

The principle of res judicata is applicable by way of (1) "bar by prior judgment" and (2) "conclusiveness of judgment." This
Court had occasion to explain the difference between these two aspects of res judicata as follows:

There is "bar by prior judgment" when, as between the first case where the judgment was rendered and the second case
that is sought to be barred, there is identity of parties, subject matter, and causes of action. In this instance, the judgment
in the first case constitutes an absolute bar to the second action. Otherwise put, the judgment or decree of the court of
competent jurisdiction on the merits concludes the litigation between the parties, as well as their privies, and constitutes a
bar to a new action or suit involving the same cause of action before the same or other tribunal. DTEHIA

But where there is identity of parties in the first and second cases, but no identity of causes of action, the first judgment is
conclusive only as to those matters actually and directly controverted and determined and not as to matters merely involved
therein. This is the concept of res judicata known as "conclusiveness of judgment." Stated differently, any right, fact or
matter in issue directly adjudicated or necessarily involved in the determination of an action before a competent court in
which judgment is rendered on the merits is conclusively settled by the judgment therein and cannot again be litigated
between the parties and their privies whether or not the claim, demand, purpose, or subject matter of the two actions is the
same. TSIEAD

Stated differently, conclusiveness of judgment finds application when a fact or question has been squarely put in issue,
judicially passed upon, and adjudged in a former suit by a court of competent jurisdiction. The fact or question settled by
final judgment or order binds the parties to that action (and persons in privity with them or their successors-in-interest), and
continues to bind them while the judgment or order remains standing and unreversed by proper authority on a timely motion
or petition; the conclusively-settled fact or question cannot again be litigated in any future or other action between the same
parties or their privies and successors-in-interest, in the same or in any other court of concurrent jurisdiction, either for the
same or for a different cause of action. Thus, only the identities of parties and issues are required for the operation of the
principle of conclusiveness of judgment. 25 HDATCc
50
Hence, there is no point in determining the main issue raised in RAB Case No. 06-09-10698-97, i.e., whether petitioners
may be considered regular employees of private respondent, because, in the first place, they are not even employees of
the latter. As such, the CA correctly held that the Labor Arbiter committed grave abuse of discretion in denying private
respondent's motion to dismiss RAB Case No. 06-09-10698-97.

The question that follows is whether private respondent is guilty of forum shopping, considering that it already filed a motion
to dismiss RAB Case No. 06-09-10698-97 in 1998? The Court answers in the negative.

In Pentacapital Investment Corporation v. Mahinay, 26 this Court's discussion on forum shopping is instructive, to wit:
SCHcaT

Forum-shopping is the act of a litigant who repetitively availed of several judicial remedies in different courts, simultaneously
or successively, all substantially founded on the same transactions and the same essential facts and circumstances, and
all raising substantially the same issues, either pending in or already resolved adversely by some other court, to increase
his chances of obtaining a favorable decision if not in one court, then in another.

What is important in determining whether forum-shopping exists is the vexation caused the courts and parties-litigants by
a party who asks different courts and/or administrative agencies to rule on the same or related causes and/or grant the
same or substantially the same reliefs, in the process creating the possibility of conflicting decisions being rendered by the
different fora upon the same issues. DHSaCA

Forum-shopping can be committed in three ways: (1) by filing multiple cases based on the same cause of action and with
the same prayer, the previous case not having been resolved yet (where the ground for dismissal is litis pendentia); (2) by
filing multiple cases based on the same cause of action and with the same prayer, the previous case having been finally
resolved (where the ground for dismissal is res judicata); and (3) by filing multiple cases based on the same cause of action
but with different prayers (splitting of causes of action, where the ground for dismissal is also either litis pendentia or res
judicata). HScaCT

More particularly, the elements of forum-shopping are: (a) identity of parties or at least such parties that represent the same
interests in both actions; (b) identity of rights asserted and reliefs prayed for, the relief being founded on the same facts; (c)
identity of the two preceding particulars, such that any judgment rendered in the other action will, regardless of which party
is successful, amount to res judicata in the action under consideration. 27

In the instant case, there can be no forum shopping, because the grounds cited by private respondent in its motions to
dismiss filed in 1998 and in the present case are different. In 1998, the motion to dismiss is based on the argument that the
final and executory decision in the Perez case serves as res judicata and, thus, bars the re-litigation of the issue of
employer-employee relations between private respondent and petitioners. In the instant case, private respondent again
cites res judicata as a ground for its motion to dismiss. This time, however, the basis for such ground is not Perez but the
final and executory decision in RAB Case No. 06-09-10699-97. Thus, the relief prayed for in private respondent's motion
to dismiss subject of the instant case is founded on totally different facts and issues. aDHCcE

As a final note, this Court cannot help but call the attention of the Labor Arbiter regarding Our observation that the resolution
of RAB Case No. 06-09-10698-97 has been unnecessarily pending for almost sixteen (16) years now. The resulting delay
in the resolution of the instant case could have been avoided had the Labor Arbiter granted private respondent's Motion to
Consolidate RAB Case Nos. 06-09-10698-97 and 06-09-10699-97. This Court quotes with approval the contention of
private respondent in its Motion, to wit: cAaDHT

3. That in light of the fact that the question as to whether or not there exists employer-employee relations as between
complainants [herein petitioners] and herein respondent HPCO will indispensably have to be resolved in light of the
presence of an independent contractor (FELA Contractors) in RAB Case No. 06-09-10699-97 — which should otherwise
be determinative of the issue involved in the present suit — it should only be logical and proper that for purposes of abating
separate and inconsistent verdicts by two distinct arbitration salas of this Commission that the present suit be accordingly
consolidated for joint hearing and resolution with said RAB Case No. 06-09-10699-97 . . . . 28 LLpr

Under Section 3, Rule IV of the then prevailing, as well as in the presently existing, NLRC Rules of Procedure, it is clearly
provided that:

Section 3. Consolidation of Cases. — Where there are two or more cases pending before different Labor Arbiters in
the same Regional Arbitration Branch involving the same employer and issues, or the same parties and different issues,
whenever practicable, the subsequent case/s shall be consolidated with the first to avoid unnecessary costs or delay. . . .

In the same manner, Section 1, Rule 31 of the 1997 Rules of Civil Procedure, allows consolidation, thus:

51
SECTION 1. Consolidation. — When actions involving a common question of law or fact are pending before the court,
it may order a joint hearing or trial of any or all the matters in issue in the actions; it may order all the actions consolidated;
and it may make such orders concerning proceedings therein as may tend to avoid unnecessary costs or delay. AIDTHC

Considering that the abovementioned cases involved essentially the same parties and the basic issue of employer-
employee relations between private respondent and petitioners, the Labor Arbiter should have been more circumspect and
should have allowed the cases to be consolidated. This would be in consonance with the parties' constitutional right to a
speedy disposition of cases as well as in keeping with the orderly and efficient disposition of cases.

WHEREFORE, the petition is DISMISSED. The assailed Decision and Resolutions of the Court of Appeals in CA-G.R. SP
No. 78729 are AFFIRMED.

SO ORDERED. EDaHAT

Velasco, Jr., Abad, Mendoza and Leonen, JJ., concur.

9. BF Homes vs Manila Electric Company 636 SCRA 495

FIRST DIVISION

[G.R. No. 171624. December 6, 2010.]

BF HOMES, INC. and THE PHILIPPINE WATERWORKS AND CONSTRUCTION CORP., petitioners, vs. MANILA
ELECTRIC COMPANY, respondent.

DECISION

LEONARDO-DE CASTRO, J p:

This Petition for Review on Certiorari under Rule 45 of the Rules of Court assails the Decision 1 dated October 27, 2005 of
the Court of Appeals in CA-G.R. SP No. 82826, nullifying and setting aside (1) the Order 2 dated November 21, 2003 of
the Regional Trial Court (RTC), Branch 202 of Las Piñas City, in Civil Case No. 03-0151, thereby dissolving the writ of
injunction against respondent Manila Electric Company (MERALCO); and (2) the Resolution 3 dated February 7, 2006 of
the Court of Appeals denying the Motion for Reconsideration of petitioners BF Homes, Inc. (BF Homes) and Philippine
Waterworks and Construction Corporation (PWCC). TIHCcA

MERALCO is a corporation duly organized and existing under Philippine laws engaged in the distribution and sale of electric
power in Metro Manila. On the other hand, BF Homes and PWCC are owners and operators of waterworks systems
delivering water to over 12,000 households and commercial buildings in BF Homes subdivisions in Parañaque City, Las
Piñas City, Caloocan City, and Quezon City. The water distributed in the waterworks systems owned and operated by BF
Homes and PWCC is drawn from deep wells using pumps run by electricity supplied by MERALCO.

On June 23, 2003, BF Homes and PWCC filed a Petition [With Prayer for the Issuance of Writ of Preliminary Injunction and
for the Immediate Issuance of Restraining Order] against MERALCO before the RTC, docketed as Civil Case No. 03-0151.

In their Petition before the RTC, BF Homes and PWCC invoked their right to refund based on the ruling of this Court in
Republic v. Manila Electric Company: 4

7. It is of judicial notice that on November 15, 2002, in G.R. No. 141314, entitled Republic of the Philippines vs. Manila
Electric Company, and G.R. No. 141369, entitled Lawyers Against Monopoly and Poverty (LAMP) et al. vs. Manila Electric
Company (MERALCO), (both cases shall hereafter be referred to as "MERALCO Refund cases," for brevity), the Supreme
Court ordered MERALCO to refund its customers, which shall be credited against the customer's future consumption, the
excess average amount of P0.167 per kilowatt hour starting with the customer's billing cycles beginning February 1998.
The dispositive portion of the Supreme Court Decision in the MERALCO Refund cases reads: TaDAIS

WHEREFORE, in view of the foregoing, the instant petitions are GRANTED and the decision of the Court of Appeals in
C.A. G.R. SP No. 46888 is REVERSED. Respondent MERALCO is authorized to adopt a rate adjustment in the amount of
P0.017 kilowatthour, effective with respect to MERALCO's billing cycles beginning February 1994. Further, in accordance
with the decision of the ERB dated February 16, 1998, the excess average amount of P0.167 per kilowatt hour starting with
the applicant's billing cycles beginning February 1998 is ordered to be refunded to MERALCO's customers or
correspondingly credited in their favor for future consumption.

xxx xxx xxx.

52
8. The Motion for Reconsideration filed by MERALCO in the MERALCO Refund cases was DENIED WITH FINALITY
(the uppercase letters were used by the Supreme Court) in the Resolution of the Supreme Court dated April 9, 2003.

9. The amount that MERALCO was mandated to refund to [BF Homes and PWCC] pursuant to the MERALCO Refund
cases is in the amount of P11,834,570.91. 5

BF Homes and PWCC then alleged in their RTC Petition that:

10. On May 20, 2003, without giving any notice whatsoever, MERALCO disconnected electric supply to [BF Homes
and PWCC's] sixteen (16) water pumps located in BF Homes in Parañaque, Caloocan, and Quezon City, which thus
disrupted water supply in those areas. IDTcHa

11. On June 4, 2003, [BF Homes and PWCC] received by facsimile transmission a letter from MERALCO, . . ., in which
MERALCO demanded to [BF Homes and PWCC] the payment of electric bills amounting to P4,717,768.15.

12. [MERALCO] replied in a letter dated June 11, 2003, . . ., requesting MERALCO to apply the P4,717,768.15 electric
bill against the P11,834,570.91 that MERALCO was ordered to refund to [BF Homes and PWCC] pursuant to the
MERALCO Refund cases. . . .

13. Displaying the arrogance that has become its distinction, MERALCO, in its letter dated June 16, 2003, . . ., denied
[BF Homes and PWCC's] request alleging that it has not yet come up with the schedule for the refund of large amounts,
such as those of [BF Homes and PWCC].

14. Even while MERALCO was serving its reply-letter to [BF Homes and PWCC], MERALCO, again, without giving
any notice, cut off power supply to [BF Homes and PWCC's] five (5) water pumps located in BF Homes Parañaque and BF
Resort Village, in Pamplona, Las Piñas City.

15. In its letter dated June 4, 2003 (Annex A), MERALCO threatened to cut off electric power connections to all of [BF
Homes and PWCC's] water pumps if [BF Homes and PWCC] failed to pay their bills demanded by MERALCO by June 20,
2003. 6

BF Homes and PWCC thus cited the following causes of action for their RTC Petition: EHaDIC

16. In refusing to apply [MERALCO's] electric bills against the amounts that it was ordered to refund to [BF Homes
and PWCC] pursuant to the MERALCO Refund cases and in making the implementation of the refund ordered by the
Supreme Court dependent upon its own will and caprice, MERALCO acted with utmost bad faith.

17. [BF Homes and PWCC] are clearly entitled to the remedies under the law to compel MERALCO to consider [BF
Homes and PWCC's] electric bills fully paid by the amounts which MERALCO was ordered to refund to [BF Homes and
PWCC] pursuant to the MERALCO Refund cases, to enjoin MERALCO to reconnect electric power to all of [BF Homes and
PWCC's] water pumps, and to order MERALCO to desist from further cutting off power connection to [BF Homes and
PWCC's] water pumps.

18. MERALCO's unjust and oppressive acts have cast dishonor upon [BF Homes and PWCC's] good name and
besmirched their reputation for which [BF Homes and PWCC] should be indemnified by way of moral damages in the
amount of not less than P1,000,000.00.

19. As an example for the public good, to dissuade others from emulating MERALCO's unjust, oppressive and
mercenary conduct, MERALCO should be directed to pay [BF Homes and PWCC] exemplary damages of at least
P1,000,000.00.

20. MERALCO's oppressive and inequitable conduct forced [BF Homes and PWCC] to engage the services of counsel
to defend their rights and thereby incur litigation expenses in the amount of at least P500,000.00 for which [BF Homes and
PWCC] should be indemnified. 7 DHITcS

BF Homes and PWCC additionally prayed that the RTC issue a writ of preliminary injunction and restraining order
considering that:

21. As indicated in its letter dated June 4, 2003 (Annex A), unless seasonably restrained, MERALCO will cut off electric
power connections to all of [BF Homes and PWCC's] water pumps on June 20, 2003.

22. Part of the reliefs herein prayed for is to restrain MERALCO from cutting off electric power connections to [BF
Homes and PWCC's] water pumps.

53
23. Unless MERALCO'S announced intention to cut off electric power connections to [BF Homes and PWCC's] water
pumps is restrained, [BF Homes and PWCC] will suffer great and irreparable injury because they would not [be] able to
supply water to their customers.

24. [BF Homes and PWCC] therefore pray that a writ for preliminary injunction be issued upon posting of a bond in an
amount as will be determined by this Honorable Court.

25. [BF Homes and PWCC] further pray that, in the meantime and immediately upon the filing of the above captioned
Petition, a restraining order be issued before the matter of preliminary injunction can be heard. 8

On August 15, 2003, MERALCO filed before the RTC its Answer with Counterclaims and Opposition to the Application for
Writ of Preliminary Injunction 9 of BF Homes and PWCC. SCaDAE

According to MERALCO:

2.2. Both petitioners BF Homes, Incorporated and Philippine Waterworks Corporation are admittedly the registered
customers of [MERALCO] by virtue of the service contracts executed between them under which the latter undertook to
supply electric energy to the former for a fee. The following twenty-three (23) Service Identification Nos. (SINs) are
registered under the name of BF Homes, Incorporated: . . . . While the following twenty-one (21) Service Identification Nos.
(SINs) are registered under the name of Philippine Waterworks Construction Corporation: . . .

xxx xxx xxx

2.4. The service contracts as well as the terms and conditions of [MERALCO's] service as approved by BOE [Board of
Energy], now ERC [Energy Regulatory Commission], provide in relevant parts, that [BF Homes and PWCC] agree as
follows:

DISCONTINUANCE OF SERVICE:

The Company reserves the right to discontinue service in case the customer is in arrears in the payment of bills or for failure
to pay the adjusted bills in those cases where the meter stopped or failed to register the correct amount of energy
consumed, or for failure to comply with any of these terms and conditions, or in case of or to prevent fraud upon the
Company. Before disconnection is made in the case of, or to prevent fraud, the Company may adjust the bill of said
customer accordingly and if the adjusted bill is not paid, the Company may disconnect the same." (Emphasis supplied)
SITCEA

2.5. This contractual right of [MERALCO] to discontinue electric service for default in the payment of its regular bills is
sanctioned and approved by the rules and regulations of ERB (now the ERC). This right is necessary and reasonable
means to properly protect and enable [MERALCO] to perform and discharge its legal and contractual obligation under its
legislative franchise and the law. Cutting off service for non-payment by the customers of the regular monthly electric bills
is the only practical way a public utility, such as [MERALCO], can ensure and maintain efficient service in accordance with
the terms and conditions of its legislative franchise and the law.

xxx xxx xxx

2.14. Instead of paying their unpaid electric bills and before [MERALCO] could effect its legal and contractual right to
disconnect [BF Homes and PWCC's] electric services, [BF Homes and PWCC] filed the instant petition to avoid payment
of [MERALCO's] valid and legal claim for regular monthly electric bills.

2.15. [BF Homes and PWCC's] unpaid regular bills totaled P6,551,969.55 covering the May and June 2003 electric bills.
...

xxx xxx xxx

2.17. [BF Homes and PWCC] knew that [MERALCO] is already in the process of implementing the decision of the
Supreme Court as to the refund case. But this refund has to be implemented in accordance with the guidelines and schedule
to be approved by the ERC. Thus [BF Homes and PWCC's] filing of the instant petition is merely to evade payment of their
unpaid electric bills to [MERALCO]. 10 caSDCA

Hence, MERALCO sought the dismissal of the RTC Petition of BF Homes and PWCC on the following grounds:

3.1. The Honorable Court has no jurisdiction to award the relief prayed for by [BF Homes and PWCC] because:

a) The petition is in effect preempting or defeating the power of the ERC to implement the decision of the Supreme
Court.
54
b) [MERALCO] is a utility company whose business activity is wholly regulated by the ERC. The latter, being the
regulatory agency of the government having the authority over the respondent, is the one tasked to approve the guidelines,
schedules and details of the refund.

c) The decision of the Supreme Court, dated November 15, 2002, clearly states that respondent is directed to make
the refund to its customers in accordance with the decision of the ERC (formerly ERB) dated February 16, 1998. Hence,
[MERALCO] has to wait for the schedule and details of the refund to be approved by the ERC before it can comply with the
Supreme Court decision.

3.2. [MERALCO] has the right to disconnect the electric service to [BF Homes and PWCC] in that: DSAEIT

a) The service contracts between [MERALCO] and [BF Homes and PWCC] expressly authorize the former to
discontinue and disconnect electric services of the latter for their failure to pay the regular electric bills rendered.

b) It is [MERALCO's] legal duty as a public utility to furnish its service to the general public without arbitrary
discrimination and, consequently, [MERALCO] is obligated to discontinue and disconnect electric services to [BF Homes
and PWCC] for their refusal or failure to pay the electric energy actually used by them. 11

For its compulsory counterclaims, MERALCO prayed that the RTC orders BF Homes and PWCC to pay MERALCO
P6,551,969.55 as actual damages (representing the unpaid electric bills of BF Homes and PWCC for May and June 2003),
P1,500,000.00 as exemplary damages, P1,500,000.00 as moral damages, and P1,000,000.00 as attorney's fees.

Lastly, MERALCO opposed the application for writ of preliminary injunction of BF Homes and PWCC because:

[MERALCO] HAS THE LEGAL AND CONTRACTUAL RIGHT TO DEMAND PAYMENT OF THE ELECTRIC BILLS AND,
IN CASE OF NON-PAYMENT, TO DISCONTINUE THE ELECTRIC SERVICES OF [BF HOMES and PWCC]

II

[BF HOMES and PWCC] HAVE NO CLEAR RIGHT WHICH WARRANTS PROTECTION BY INJUNCTIVE PROCESS
HEDSIc

After hearing, 12 the RTC issued an Order on November 21, 2003 granting the application of BF Homes and PWCC for
the issuance of a writ of preliminary injunction. The RTC found that the records showed that all requisites for the issuance
of said writ were sufficiently satisfied by BF Homes and PWCC. The RTC stated in its Order:

Albeit, this Court respects the right of a public utility company like MERALCO, being a grantee of a legislative franchise
under Republic Act No. 9029, to collect overdue payments from its subscribers or customers for their respective
consumption of electric energy, such right must, however, succumb to the paramount substantial and constitutional rights
of the public to the usage and enjoyment of waters in their community. Thus, there is an urgent need for the issuance of a
writ of preliminary injunction in order to prevent social unrest in the community for having been deprived of the use and
enjoyment of waters flowing through [BF Homes and PWCC's] water pumps. 13

The RTC decreed in the end:

WHEREFORE, in the light of the foregoing, [BF Homes and PWCC's] prayer for the issuance of a writ of preliminary
injunction is hereby GRANTED. Respondent Manila Electric Company is permanently restrained from proceeding with its
announced intention to cut-off electric power connection to [BF Homes and PWCC's] water pumps unless otherwise ordered
by this Court. Further, [BF Homes and PWCC] are hereby ordered to post a bond in the amount of P500,000 to answer for
whatever injury or damage that may be caused by reason of the preliminary injunction. 14 DcCIAa

The Motion for Reconsideration of MERALCO of the aforementioned Order was denied by the RTC in another Order issued
on January 9, 2004. 15 The RTC reiterated its earlier finding that all the requisites for the proper issuance of an injunction
had been fully complied with by BF Homes and PWCC, thus:

Records indubitably show that all the requisites for the proper issuance of an injunction have been fully complied with in
the instant case.

It should be noted that a disconnection of power supply would obviously cause irreparable injury because the pumps that
supply water to the BF community will be without electricity, thereby rendering said community without water. Water is a
basic and endemic necessity of life. This is why its enjoyment and use has been constitutionally safeguarded and protected.
Likewise, a community without water might create social unrest, which situation this Court has the mandate to prevent.

55
There is an urgent and paramount necessity for the issuance of the injunctive writ to prevent serious damage to the
guaranteed rights of [BF Homes and PWCC] and the residents of the community to use and enjoy water. 16

The RTC resolved the issue on jurisdiction raised by MERALCO, as follows: ACTIcS

As to the jurisdictional issue raised by respondent MERALCO, it can be gleaned from a re-evaluation and re-assessment
of the records that this Court has jurisdiction to delve into the case. This Court gave both parties the opportunity to be heard
as they introduced evidence on the propriety of the issuance of the injunctive writ. It is well-settled that no grave abuse of
discretion could be attributed to its issuance where a party was not deprived of its day in court as it was heard and had
exhaustively presented all its arguments and defenses. (National Mines and Allied Workers Union vs. Valero, 132 SCRA
578, 1984.) 17

Aggrieved, MERALCO filed with the Court of Appeals a Petition for Certiorari under Rule 65 of the Rules of Court, docketed
as CA-G.R. SP No. 82826. MERALCO sought the reversal of the RTC Orders dated November 21, 2003 and January 9,
2004 granting a writ of preliminary injunction in favor of BF Homes and PWCC. MERALCO asserted that the RTC had no
jurisdiction over the application of BF Homes and PWCC for issuance of such a writ.

In its Decision dated October 27, 2005, the Court of Appeals agreed with MERALCO that the RTC had no jurisdiction to
issue a writ of preliminary injunction in Civil Case No. 03-0151, as said trial court had no jurisdiction over the subject matter
of the case to begin with. It ratiocinated in this wise:

For one, it cannot be gainsaid that the ERC has original and exclusive jurisdiction over the case. Explicitly, Section 43(u)
of Republic Act No. 9136, otherwise known as the "Electric Power Industry Reform Act," (RA 9136), states that the ERC
shall have the original and exclusive jurisdiction over all cases contesting rates, fees, fines and penalties imposed by the
ERC in the exercise of its powers, functions and responsibilities and over all cases involving disputes between and among
participants or players in the energy sector. Section 4(o) of Rule 3 of the Implementing Rules and Regulations of RA 9136
likewise provides that the ERC shall also be empowered to issue such other rules that are essential in the discharge of its
functions as an independent quasi-judicial body. AcISTE

For another, the respondent judge, instead of presiding over the case, should have dismissed the same and yielded
jurisdiction to the ERC pursuant to the doctrine of primary jurisdiction. It is plain error on the part of the respondent judge
to determine, preliminary or otherwise, a controversy involving a question which is within the jurisdiction of an administrative
tribunal, especially so where the question demands the exercise of sound administrative discretion.

Needless to state, the doctrine of primary jurisdiction applies where the administrative agency, as in the case of ERC,
exercises its quasi-judicial and adjudicatory function. Thus, in cases involving specialized disputes, the practice has been
to refer the same to an administrative agency of special competence pursuant to the doctrine of primary jurisdiction. The
courts will not determine a controversy involving a question which is within the jurisdiction of the administrative tribunal prior
to the resolution of that question by the administrative tribunal, where the question demands the exercise of sound
administrative discretion requiring the special knowledge, experience and services of the administrative tribunal to
determine technical and intricate matters of fact, and a uniformity of ruling is essential to comply with the premises of the
regulatory statute administered.

Verily, the cause of action of [BF Homes and PWCC] against [MERALCO] originates from the Meralco Refund Decision as
it involves the perceived right of the former to compel the latter to set-off or apply their refund to their present electric bill.
The issue delves into the right of the private respondents to collect their refund without submitting to the approved schedule
of the ERC, and in effect give unto themselves preferential right over other equally situated consumers of [MERALCO].
Perforce, the ERC, as can be gleaned from the afore-stated legal provisions, has primary, original and exclusive jurisdiction
over the said controversy. SacTCA

Indeed, the respondent judge glaringly erred in enjoining the right of [MERALCO] to disconnect its services to [BF Homes
and PWCC] on the premise that the court has jurisdiction to apply the provisions on compensation or set-off in this case.
Although [MERALCO] recognizes the right of [BF Homes and PWCC] to the refund as provided in the Meralco Refund
Decision, it is the ERC which has the authority to implement the same according to its approved schedule, it being a dispute
arising from the exercise of its jurisdiction.

Moreover, it bears to stress that the Meralco Refund Decision was brought into fore by the Decision dated 16 February
1998 of the ERC (then Energy Regulatory Board) granting refund to [MERALCO's] consumers. Being the agency of origin,
the ERC has the jurisdiction to execute the same. Besides, as stated, it is empowered to promulgate rules that are essential
in the discharge of its functions as an independent quasi-judicial body. 18

The dispositive portion of the judgment of the appellate court reads:

56
WHEREFORE, the foregoing considered, the instant petition is hereby GRANTED and the assailed Orders REVERSED
and SET ASIDE. Accordingly, the writ of injunction against [MERALCO] is hereby DISSOLVED. No costs. 19

In a Resolution dated February 7, 2006, the Court of Appeals denied the Motion for Reconsideration of BF Homes and
PWCC for failing to raise new and persuasive and meritorious arguments.

Now, BF Homes and PWCC come before this Court via the instant Petition, raising the following assignment of errors:
CTSHDI

1. The Court of Appeals ERRED in saying that the respondent judge committed grave abuse of discretion by issuing
the disputed writ of injunction pending the merits of the case including the issue of subject matter jurisdiction.

2. The Court of Appeals ERRED in saying that the ERC under the doctrine of primary jurisdiction has the original and
EXCLUSIVE jurisdiction to take cognizance of a petition for injunction to prevent electrical disconnection to a customer
entitled to a refund.

3. The Court of Appeals ERRED in NOT SAYING that the ERC as a quasi-judicial body under RA 9136 has no power
to issue any injunctive relief or remedy to prevent disconnection.

4. The Court of Appeals ERRED in not resolving the issue as to the violation of MERALCO of a standing injunction
order while the case remains undecided. 20

At the core of the Petition is the issue of whether jurisdiction over the subject matter of Civil Case No. 03-0151 lies with the
RTC or the Energy Regulatory Commission (ERC). If it is with the RTC, then the said trial court also has jurisdiction to issue
the writ of preliminary injunction against MERALCO. If it is with the ERC, then the RTC also has no jurisdiction to act on
any incidents in Civil Case No. 03-0151, including the application for issuance of a writ of preliminary injunction of BF
Homes and PWCC therein.

BF Homes and PWCC argued that due to the threat of MERALCO to disconnect electric services, BF Homes and PWCC
had no other recourse but to seek an injunctive remedy from the RTC under its general jurisdiction. The merits of Civil Case
No. 03-0151 was not yet in issue, only the propriety of issuing a writ of preliminary injunction to prevent an irreparable
injury. Even granting that the RTC has no jurisdiction over the subject matter of Civil Case No. 03-0151, the ERC by
enabling law has no injunctive power to prevent the disconnection by MERALCO of electric services to BF Homes and
PWCC.

The Petition has no merit. EAcHCI

Settled is the rule that jurisdiction is conferred only by the Constitution or the law. 21 Republic v. Court of Appeals 22 also
enunciated that only a statute can confer jurisdiction on courts and administrative agencies.

Related to the foregoing and equally well-settled is the rule that the nature of an action and the subject matter thereof, as
well as which court or agency of the government has jurisdiction over the same, are determined by the material allegations
of the complaint in relation to the law involved and the character of the reliefs prayed for, whether or not the
complainant/plaintiff is entitled to any or all of such reliefs. A prayer or demand for relief is not part of the petition of the
cause of action; nor does it enlarge the cause of action stated or change the legal effect of what is alleged. In determining
which body has jurisdiction over a case, the better policy is to consider not only the status or relationship of the parties but
also the nature of the action that is the subject of their controversy. 23

In Manila Electric Company v. Energy Regulatory Board, 24 the Court traced the legislative history of the regulatory
agencies which preceded the ERC, presenting a summary of these agencies, the statutes or issuances that created them,
and the extent of the jurisdiction conferred upon them, viz.:

1. The first regulatory body, the Board of Rate Regulation (BRR), was created by virtue of Act No. 1779. Its regulatory
mandate under Section 5 of the law was limited to fixing or regulating rates of every public service corporation. cTEICD

2. In 1913, Act No. 2307 created the Board of Public Utility Commissioners (BPUC) to take over the functions of the
BRR. By express provision of Act No. 2307, the BPUC was vested with jurisdiction, supervision and control over all public
utilities and their properties and franchises.

3. On November 7, 1936, Commonwealth Act (C.A.) No. 146, or the Public Service Act (PSA), was passed creating
the Public Service Commission (PSC) to replace the BPUC. Like the BPUC, the PSC was expressly granted jurisdiction,
supervision and control over public services, with the concomitant authority of calling on the public force to exercise its
power, to wit:

57
"SEC. 13. Except as otherwise provided herein, the Commission shall have general supervision and regulation of,
jurisdiction and control over, all public utilities, and also over their property, property rights, equipment, facilities and
franchises so far as may be necessary for the purpose of carrying out the provisions of this Act, and in the exercise of its
authority it shall have the necessary powers and the aid of the public force . . . ."

Section 14 of C.A. No. 146 defines the term "public service" or "public utility" as including "every individual, copartnership,
association, corporation or joint-stock company, . . . that now or hereafter may own, operate, manage or control within the
Philippines, for hire or compensation, any common carrier, . . ., electric light, heat, power, . . ., when owned, operated and
managed for public use or service within the Philippines . . . ." Under the succeeding Section 17(a), the PSC has the power
even without prior hearing — HIACac

(a) To investigate, upon its own initiative, or upon complaint in writing, any matter concerning any public service as
regards matters under its jurisdiction; to require any public service to furnish safe, adequate and proper service as the
public interest may require and warrant, to enforce compliance with any standard, rule, regulation, order or other
requirement of this Act or of the Commission, . . . .

4. Then came Presidential Decree (P.D.) No. 1, reorganizing the national government and implementing the
Integrated Reorganization Plan. Under the reorganization plan, jurisdiction, supervision and control over public services
related to electric light, and power heretofore vested in the PSC were transferred to the Board of Power and Waterworks
(BOPW).

Later, P.D. No. 1206 abolished the BOPW. Its powers and function relative to power utilities, including its authority to grant
provisional relief, were transferred to the newly-created Board of Energy (BOE).

5. On May 8, 1987, then President Corazon C. Aquino issued E.O. No. 172 reconstituting the BOE into the ERB,
transferring the former's functions and powers under P.D. No. 1206 to the latter and consolidating in and entrusting on the
ERB "all the regulatory and adjudicatory functions covering the energy sector." Section 14 of E.O. No. 172 states that "(T)he
applicable provisions of [C.A.] No. 146, as amended, otherwise known as the 'Public Service Act'; . . . and [P.D.] No. 1206,
as amended, creating the Department of Energy, shall continue to have full force and effect, except insofar as inconsistent
with this Order." 25 aAEIHC

Thereafter, on June 8, 2001, Republic Act No. 9136, known as the Electric Power Industry Reform Act of 2001 (EPIRA),
was enacted, providing a framework for restructuring the electric power industry. One of the avowed purposes of the EPIRA
is to establish a strong and purely independent regulatory body. The Energy Regulatory Board (ERB) was abolished and
its powers and functions not inconsistent with the provision of the EPIRA were expressly transferred to the ERC. 26

The powers and functions of the ERB not inconsistent with the EPIRA were transferred to the ERC by virtue of Sections 44
and 80 of the EPIRA, which read:

Sec. 44. Transfer of Powers and Functions. — The powers and functions of the Energy Regulatory Board not inconsistent
with the provisions of this Act are hereby transferred to the ERC. The foregoing transfer of powers and functions shall
include all applicable funds and appropriations, records, equipment, property and personnel as may be necessary.

Sec. 80. Applicability and Repealing Clause. — The applicability provisions of Commonwealth Act No. 146, as amended,
otherwise known as the "Public Service Act." Republic Act 6395, as amended, revising the charter of NPC; Presidential
Decree 269, as amended, referred to as the National Electrification Decree; Republic Act 7638, otherwise known as the
"Department of Energy Act of 1992"; Executive Order 172, as amended, creating the ERB; Republic Act 7832 otherwise
known as the "Anti-Electricity and Electric Transmission Lines/Materials Pilferage Act of 1994"; shall continue to have full
force and effect except insofar as they are inconsistent with this Act. THaDAE

The provisions with respect to electric power of Section 11(c) of Republic Act 7916, as amended, and Section 5(f) of
Republic Act 7227, are hereby repealed or modified accordingly.

Presidential Decree No. 40 and all laws, decrees, rules and regulations, or portions thereof, inconsistent with this Act are
hereby repealed or modified accordingly.

In addition to the foregoing, the EPIRA also conferred new powers upon the ERC under Section 43, among which are:

SEC. 43. Functions of the ERC. — The ERC shall promote competition, encourage market development, ensure
customer choice and penalize abuse of market power in the restructured electricity industry. In appropriate cases, the ERC
is authorized to issue cease and desist order after due notice and hearing. Towards this end, it shall be responsible for the
following key functions in the restructured industry:

xxx xxx xxx


58
(f) In the public interest, establish and enforce a methodology for setting transmission and distribution wheeling rates
and retail rates for the captive market of a distribution utility, taking into account all relevant considerations, including the
efficiency or inefficiency of the regulated entities. The rates must be such as to allow the recovery of just and reasonable
costs and a reasonable return on rate base (RORB) to enable the entity to operate viably. The ERC may adopt alternative
forms of internationally-accepted rate-setting methodology as it may deem appropriate. The rate-setting methodology so
adopted and applied must ensure a reasonable price of electricity. The rates prescribed shall be non-discriminatory. To
achieve this objective and to ensure the complete removal of cross subsidies, the cap on the recoverable rate of system
losses prescribed in Section 10 of Republic Act No. 7832, is hereby amended and shall be replaced by caps which shall
be determined by the ERC based on load density, sales mix, cost of service, delivery voltage and other technical
considerations it may promulgate. The ERC shall determine such form of rate-setting methodology, which shall promote
efficiency. . . . . TCHcAE

xxx xxx xxx

(u) The ERC shall have the original and exclusive jurisdiction over all cases contesting rates, fees, fines and penalties
imposed by the ERC in the exercise of the abovementioned powers, functions and responsibilities and over all cases
involving disputes between and among participants or players in the energy sector.

All notices of hearings to be conducted by the ERC for the purpose of fixing rates or fees shall be published at least twice
for two successive weeks in two (2) newspapers of nationwide circulation.

A careful review of the material allegations of BF Homes and PWCC in their Petition before the RTC reveals that the very
subject matter thereof is the off-setting of the amount of refund they are supposed to receive from MERALCO against the
electric bills they are to pay to the same company. This is squarely within the primary jurisdiction of the ERC.

The right of BF Homes and PWCC to refund, on which their claim for off-setting depends, originated from the MERALCO
Refund cases. In said cases, the Court (1) authorized MERALCO to adopt a rate adjustment in the amount of P0.017 per
kilowatthour, effective with respect to its billing cycles beginning February 1994; and (2) ordered MERALCO to refund to its
customers or credit in said customers' favor for future consumption P0.167 per kilowatthour, starting with the customers'
billing cycles that begin February 1998, in accordance with the ERB Decision dated February 16, 1998.

It bears to stress that in the MERALCO Refund cases, this Court only affirmed the February 16, 1998 Decision of the ERB
(predecessor of the ERC) fixing the just and reasonable rate for the electric services of MERALCO and granting refund to
MERALCO consumers of the amount they overpaid. Said Decision was rendered by the ERB in the exercise of its
jurisdiction to determine and fix the just and reasonable rate of power utilities such as MERALCO. ACcEHI

Presently, the ERC has original and exclusive jurisdiction under Rule 43 (u) of the EPIRA over all cases contesting rates,
fees, fines, and penalties imposed by the ERC in the exercise of its powers, functions and responsibilities, and over all
cases involving disputes between and among participants or players in the energy sector. Section 4 (o) of the EPIRA
Implementing Rules and Regulations provides that the ERC "shall also be empowered to issue such other rules that are
essential in the discharge of its functions as in independent quasi-judicial body."

Indubitably, the ERC is the regulatory agency of the government having the authority and supervision over MERALCO.
Thus, the task to approve the guidelines, schedules, and details of the refund by MERALCO to its consumers, to implement
the judgment of this Court in the MERALCO Refund cases, also falls upon the ERC. By filing their Petition before the RTC,
BF Homes and PWCC intend to collect their refund without submitting to the approved schedule of the ERC, and in effect,
enjoy preferential right over the other equally situated MERALCO consumers.

Administrative agencies, like the ERC, are tribunals of limited jurisdiction and, as such, could wield only such as are
specifically granted to them by the enabling statutes. In relation thereto is the doctrine of primary jurisdiction involving
matters that demand the special competence of administrative agencies even if the question involved is also judicial in
nature. Courts cannot and will not resolve a controversy involving a question within the jurisdiction of an administrative
tribunal, especially when the question demands the sound exercise of administrative discretion requiring special knowledge,
experience and services of the administrative tribunal to determine technical and intricate matters of fact. The court cannot
arrogate into itself the authority to resolve a controversy, the jurisdiction of which is initially lodged with the administrative
body of special competence. 27 IEaHSD

Since the RTC had no jurisdiction over the Petition of BF Homes and PWCC in Civil Case No. 03-0151, then it was also
devoid of any authority to act on the application of BF Homes and PWCC for the issuance of a writ of preliminary injunction
contained in the same Petition. The ancillary and provisional remedy of preliminary injunction cannot exist except only as
an incident of an independent action or proceeding. 28

59
Incidentally, BF Homes and PWCC seemed to have lost sight of Section 8 of Executive Order No. 172 which explicitly
vested on the ERB, as an incident of its principal function, the authority to grant provisional relief, thus:

Section 8. Authority to Grant Provisional Relief. — The Board may, upon the filing of an application, petition or
complaint or at any stage thereafter and without prior hearing, on the basis of supporting papers duly verified or
authenticated, grant provisional relief on motion of a party in the case or on its own initiative, without prejudice to a final
decision after hearing, should the Board find that the pleadings, together with such affidavits, documents and other evidence
which may be submitted in support of the motion, substantially support the provisional order: Provided, That the Board shall
immediately schedule and conduct a hearing thereon within thirty (30) days thereafter, upon publication and notice to all
affected parties.

The aforequoted provision is still applicable to the ERC as it succeeded the ERB, by virtue of Section 80 of the EPIRA. A
writ of preliminary injunction is one such provisional relief which a party in a case before the ERC may move for. THCASc

Lastly, the Court herein already declared that the RTC not only lacked the jurisdiction to issue the writ of preliminary
injunction against MERALCO, but that the RTC actually had no jurisdiction at all over the subject matter of the Petition of
BF Homes and PWCC in Civil Case No. 03-0151. Therefore, in addition to the dissolution of the writ of preliminary injunction
issued by the RTC, the Court also deems it appropriate to already order the dismissal of the Petition of BF Homes and
PWCC in Civil Case No. 03-0151 for lack of jurisdiction of the RTC over the subject matter of the same. Although only the
matter of the writ of preliminary injunction was brought before this Court in the instant Petition, the Court is already taking
cognizance of the issue on the jurisdiction of the RTC over the subject matter of the Petition. The Court may motu proprio
consider the issue of jurisdiction. The Court has discretion to determine whether the RTC validly acquired jurisdiction over
Civil Case No. 03-0151 since, to reiterate, jurisdiction over the subject matter is conferred only by law. Jurisdiction over the
subject matter cannot be acquired through, or waived by, any act or omission of the parties. Neither would the active
participation of the parties nor estoppel operate to confer jurisdiction on the RTC where the latter has none over a cause of
action. 29 Indeed, when a court has no jurisdiction over the subject matter, the only power it has is to dismiss the action.
30

WHEREFORE, the instant Petition for Review is DENIED. The Decision dated October 27, 2005 of the Court of Appeals in
CA-G.R. SP No. 82826 is AFFIRMED with the MODIFICATION that the Regional Trial Court, Branch 202 of Las Piñas City,
is ORDERED to dismiss the Petition [With Prayer for the Issuance of Writ of Preliminary Injunction and for the Immediate
Issuance of Restraining Order] of BF Homes, Inc. and Philippine Waterworks and Construction Corporation in Civil Case
No. 03-0151. Costs against BF Homes, Inc. and Philippine Waterworks and Construction Corporation. AaHDSI

SO ORDERED.

Corona, C.J., Del Castillo, Abad * and Perez, JJ., concur.

10. Republic v. Estipular, 336 SCRA 333 (2000)

THIRD DIVISION

[G.R. No. 136588. July 20, 2000.]

REPUBLIC OF THE PHILIPPINES, petitioner, vs. PILAR ESTIPULAR, respondent.

The Solicitor General for petitioner.

Public Attorney's Office for respondent.

SYNOPSIS

Respondent filed a Petition for Reconstitution before the Regional Trial Court of La Union alleging, among others, that the
original title was either destroyed or burned in the Register of Deeds of La Union during the World War. Upon Order of the
trial court, a Certificate of Posting was submitted by the Branch Sheriff proving that the copies of the Petition and Notice of
Hearing were posted at the main entrance of the Municipal Building of Caba, La Union, the place where the property is
located. The National Printing Office likewise issued a Certificate of Publication showing that the said petition for
reconstitution was published in the Official Gazette for two successive weeks. After the presentation of evidence, the lower
court rendered a decision granting the petition. The Court of Appeals granted in toto the decision of the trial court, ruling
that there was substantial compliance with the requirements of the law. SDTcAH

Republic Act No. 26 requires that a petition for reconstitution of a lost or destroyed certificate of title must be published in
the Official Gazette and posted at the main entrance of the provincial and the municipal buildings of the place where the

60
property is situated. This requirement is mandatory and strict compliance therewith is jurisdictional. Thus, before the trial
court can acquire jurisdiction to hear and decide a reconstitution case, compliance with the requisites is imperative.

In the present case, it is undisputed that the Notice of Hearing of respondent's Petition for Reconstitution was not posted
at the main entrance. Clearly, the trial court did not acquire jurisdiction over the case. The publication of the Notice of
Hearing in the Official Gazette did not justify the respondent's failure to comply with the legal requirement of posting the
notice at the main entrance of both the municipal and the provincial buildings. The posting of said notice at the main
entrances of both the municipal and the provincial buildings is another equally vital requisite. The principle of substantial
compliance cannot be applied to the present case, as the trial court's acquisition of jurisdiction over the petition hinged on
a strict compliance with the requirements of the law. Without such publication and posting at the main entrances of both
the municipal and the provincial edifices, the trial court's decision granting the reconstitution was void.

SYLLABUS

1. CIVIL LAW; LAND TITLES AND DEEDS; PETITION FOR RECONSTITUTION OF TITLE; REPUBLIC ACT NO.
26; SECTION 13 THEREOF; REQUIREMENTS AND PROCEDURES PROVIDED THEREIN ARE MANDATORY AND
COMPLIANCE WITH THEM IS JURISDICTIONAL. — Section 13 of Republic Act No. 26 lays down the special
requirements and procedure that must be followed before jurisdiction may be acquired over a petition for reconstitution of
title. These requirements are mandatory and compliance with them is jurisdictional. IACDaS

2. ID.; ID.; ID.; ID.; REQUISITES BEFORE TRIAL COURT MAY ACQUIRE JURISDICTION OVER
RECONSTITUTION CASES. — Before the trial court can acquire jurisdiction to hear and decide a reconstitution case,
compliance with the following requisites is imperative: "1. [That] the notice of the petition be published, at the expense of
the petitioner, twice in successive issues of the Official Gazette, and posted on the main entrance of the provincial building
and of the municipal building of the municipality or city in which the land is situated, at least thirty days prior to the date of
hearing; 2. [That] the notice state among other things, the number of the lost or destroyed certificates of title if known, the
name of the registered owner, the name of the occupants or persons in possession of the property, the owner of the
adjoining properties and all other interested parties, the location, area and boundaries of the property, and the date on
which all persons having any interest therein must appear and file their claim of objection to the petition; "3. [That] a copy
of the notice also be sent, by registered mail or otherwise, at the expense of the petitioner, to every person named therein
(i.e. the occupants or persons in possession of the property, the owner of the adjoining properties and all other interested
parties) whose address is known at least thirty days prior to the date of the hearing; and "4. [That] at the hearing, petitioner
submit proof of publication, posting and service of the notice as directed by the court." In the present case, it is undisputed
that the Notice of Hearing of respondent's Petition for Reconstitution was not posted at the main entrance of the provincial
building. Clearly, the trial court did not acquire jurisdiction over the case.

3. ID.; ID.; ID.; ID.; PUBLICATION OF NOTICE OF HEARING IN OFFICIAL GAZETTE DOES NOT JUSTIFY
FAILURE OF PARTY TO COMPLY WITH REQUIREMENT OF POSTING OF NOTICE AT MAIN ENTRANCES OF BOTH
MUNICIPAL AND PROVINCIAL BUILDINGS; REQUIREMENTS OF THE LAW MUST BE STRICTLY COMPLIED WITH
BY THE TRIAL COURT. — It must be emphasized that under the law, the publication of a notice of hearing in the Official
Gazette is not enough. The posting of said notice at the main entrances of both the municipal and the provincial building is
another equally vital requisite. The purposes of the stringent and mandatory character of the legal requirements of
publication, posting and mailing are to safeguard against spurious and unfounded land ownership claims, to apprise all
interested parties of the existence of such action, and to give them enough time to intervene in the proceeding. The
publication of the Notice of Hearing in the Official Gazette does not justify the respondent's failure to comply with the legal
requirement of posting the Notice at the main entrance of both the municipal and the provincial buildings. The principle of
substantial compliance cannot be applied to the present case, as the trial court's acquisition of jurisdiction over the Petition
hinged on a strict compliance with the requirements of the law. CAacTH

DECISION

PANGANIBAN, J p:

Republic Act No. 26 requires that a petition for reconstitution of a lost or destroyed certificate of title must be published in
the Official Gazette and posted at the main entrance of the provincial and the municipal buildings of the place where the
property is situated. This requirement is mandatory; strict compliance therewith is jurisdictional. Without such publication
and posting at the main entrances of both the municipal and the provincial edifices, the trial court Decision granting the
reconstitution is void. HCSEcI

The Case

61
This is the principle used by this Court in granting the Petition for Review before us, assailing the December 9, 1998
Decision 1 of the Court of Appeals 2 (CA) in CA-GR CV No. 53846. The dispositive portion of the challenged Decision
reads as follows:

"WHEREFORE, premises considered, the appealed judgment is hereby AFFIRMED in toto." 3

The decretal part of the Decision 4 of the Regional Trial Court affirmed by the CA is worded thus:

"WHEREFORE, the Court finds the petition to be well-taken and supported by evidence. Hence, the petition is hereby
GRANTED. The destroyed/burned original copy of Certificate of Title No. 154 is declared cancelled and the Register of
Deeds of La Union is hereby directed to reconstitute in lieu thereof, the Original Certificate of Title No. 154, in favor of
Fermin Estipular, which shall bear the annotation that the same is being issued in place of the destroyed/burned original
copy in exactly the same terms and conditions using as basis the corresponding Owner's Duplicate Certificate of Title
previously issued by the Registry of Deeds of La Union but shall in all respects be entitled to like faith and credit as the
destroyed/burned original copy filed with the Registry Office, and shall thereafter be regarded as such for all purposes of
the Property Registration Decree." 5

The Facts

This case is rooted in a Petition for Reconstitution of Title filed by Pilar Estipular before the Regional Trial Court of La Union.
The factual and the procedural antecedents of the case are summarized in the assailed CA Decision as follows: IDTcHa

"In her Petition for Reconstitution of Title, the petitioner, Pilar Estipular, declared that she [was] the only surviving legal heir
of the late Fermin Estipular, who died intestate in Caba, La Union. During his lifetime, Fermin was issued Certificate of Title
No. 154 duly registered in his own name by the Register of Deeds of La Union covering a parcel of land located at Barrio
Liquicia, Caba, La Union, with an area of 6.1253 hectares. The said Certificate of Title was either destroyed or burned as
a result of the burning of the Register of Deeds of La Union during the last World War. Further, it was alleged that the
aforesaid parcel of land was declared for taxation purposes by Fermin and his heirs; that said estate is not mortgaged to
any financial institution; nor is there any document pending registration affecting the said land. As the land was already
declared and distributed to ten persons who have succeeded him, the petitioner prayed that the said Certificate of Title be
reconstituted in accordance with law.

"On June 15, 1994, the court a quo ordered that a Notice of Hearing be published for two successive issues of the Official
Gazette and be posted at the main entrance of the Municipal Building of Caba, La Union at least thirty (30) days from the
initial hearing set for September 8, 1994 (Records, p. 8). A Certificate of Posting was submitted by Branch Sheriff Romeo
Obiena proving that copies of the Petition and Notice of Hearing were posted at the main entrance of Municipal Building of
Caba, La Union (Records, p. 9). However, the National Printing Office advised the lower court to reschedule its original
date of hearing as it could not meet the schedule of publication (Records, p. 11). On August 12, 1994, another Notice of
Hearing was issued by the trial court, resetting the initial hearing to December 7, 1994. (Records, p. 13). In view thereof, a
second Certificate of Posting was issued by Branch Sheriff concerning the administrative case (Records, p. 16). In the
same manner, the National Printing Office issued a Certificate of Publication showing that the said petition for reconstitution
was published in the Official Gazette for two successive weeks on October 17 and 24, 1994.

"On November 2, 1994, the Office of the Solicitor General entered its appearance as counsel for the respondent Republic
and deputized the Provincial Prosecutor of La Union to appear [o]n its behalf in connection with the subject case (Records,
p. 20).

"The initial hearing materialized on December 7, 1994. The petitioner and the public prosecutor appeared [i]n such hearing.
The case was called to invite private oppositors to come forthwith, but nobody registered his/her opposition. Due to the
absence of the counsel for the petitioner, the latter was allowed to establish jurisdictional facts at the next hearing date,
January 24, 1995. On the latter date, the petitioner presented the jurisdictional facts with the corresponding documentary
requirements prescribed by law, to wit:

"Exhibit "A" — Petition dated June 9, 1994;

Exhibit "A-1" — Verification of petition;

Exhibit "B" — Certified True Copy of Certificate of Title No. 154; IaCHTS

Exhibit "C" — Survey Plan for the Titles;

Exhibit "D" — Technical Description;

Exhibit "E" — Certification of the Provincial Assessor;


62
Exhibit "F" — Notice of Hearing;

Exhibit "G" — Certificate of Publication issued by the National Printing Office;

Exhibit "H" — Certificate of Posting;

Exhibit "I" — Notice of Appearance of the Solicitor General."

"When the Exhibits were offered in evidence, the Public Prosecutor never interposed any objection, hence, all the exhibits
were admitted. Petitioner Pilar Estipular's testimony was offered to prove that she caused the reconstitution of Certificate
of Title No. 154 of the Register of Deeds of La Union. aCIHAD

"Two (2) other witnesses, Davidson Estipular and Juvenal Estacio, testified for the petitioner. The grandson of the petitioner,
Davidson Estipular, stated that the land covered by the title in question (owner's duplicate) [was] existing and that the
original title was burned in the Register of Deeds of La Union. Mr. Juvenal Estacio, the representative of the Register of
Deeds of La Union, testified that all the pre-war records in the said office were either burned, destroyed or stolen during
the last World War.

"After the presentation of evidence, the lower court rendered the questioned decision."

The CA Ruling

Although the Notice of Hearing had not been posted at the main entrance of the provincial building, the CA held that there
was substantial compliance with the requirements of the law. It ruled:

"It is a settled rule that proceedings for judicial reconstitution of certificates of title are proceedings in rem. Thus, NOTICE
OF HEARING BY PROPER PUBLICATION IS SUFFICIENT TO CLOTHE THE COURT WITH JURISDICTION (Calalang
vs. Register of Deeds of Quezon City, 231 SCRA 88, emphasis ours). The purpose of such publication is to apprise the
whole world that such a petition has been filed and that whoever is minded to oppose it for good cause may do so within
thirty (30) days before the date set by the court for hearing the petition. It is the publication of such notice that brings in the
whole world as a party in the case and vests the court with jurisdiction to hear and decide it (Republic vs. Court of Appeals,
218 SCRA 773). Since there was a valid publication of the Notice of Hearing in the Official Gazette, then it is sufficient to
vest jurisdiction upon the court to hear and determine the petition." 6

xxx xxx xxx

"Viewed in proper perspective, the failure of the petitioner to post the Notice of Hearing at the main entrance of the provincial
capitol building does not detract from the fact that there was a substantial compliance with the provisions of the law. It must
be noted that the Branch Sheriff issued two (2) Certificates of Posting (Records, pp. 9 and 16) at the main entrance of the
municipal building where the land [lay]. Coupled with the successive publications in the Official Gazette, it was more than
enough to serve the purpose of notifying all the parties concerned that a petition ha[d] been filed and that whoever ha[d]
an interest therein to oppose it for good cause should come to court and prove his claim. As it [was], no private parties
opposed the petition. No other claimant . . . came forward. On the other hand, the government was ably represented by the
Public Prosecutor so the appellant Republic was not in any manner deprived of the opportunity to protect its rights or
interests over the land subject of the petition." 7

Hence, this recourse by the Republic. 8

The Issue

Petitioner submits this lone issue for the resolution of this Court:

"The sole issue for resolution is whether or not supposed substantial compliance with the requirements of Republic Act No.
26 is sufficient to confer jurisdiction on the trial court over the case." 9

The Court's Ruling

The Petition is meritorious. DaEcTC

Main Issue:

Requirements for Reconstitution of Title Are Mandatory and Jurisdictional

Jurisdiction over the subject matter or nature of the action is conferred only by the Constitution or by law. It cannot be (1)
granted by the agreement of the parties; (2) acquired, waived, enlarged or diminished by any act or omission of the parties;

63
or (3) conferred by the acquiescence of the courts. 10 Republic Act No. 26 11 lays down the special requirements and
procedure that must be followed before jurisdiction may be acquired over a petition for reconstitution of title. In Section 13
of said Act, these requirements and procedure are provided as follows: DTaAHS

"SECTION 13. The Court shall cause a notice of the petition, filed under the preceding section, to be published, at the
expense of the petitioner, twice in successive issues of the Official Gazette, and to be posted on the main entrance of the
provincial building and of the municipal building of the municipality or city in which the land is situated, at least thirty days
prior to the date of hearing. The court shall likewise cause a copy of the notice to be sent, by registered mail or otherwise,
at the expense of the petitioner, to every person named therein whose address is known, at least thirty days prior to the
date of the hearing. Said notice shall state, among other things, the number of the lost or destroyed certificate of title if
known, the name of the registered owner, the name of the occupants or persons in possession of the property, the owner
of the adjoining properties, the location, area and boundaries of the property, and the date on which all persons having any
interest therein must appear and file their claim or objection to the petition. The petitioner shall, at the hearing, submit proof
of publication, posting and service of the notice as directed by the court."

`. In Republic v. Court of Appeals, 12 the Court held:

"Reconstitution of a certificate of title, in the context of Republic Act No. 26, denotes the restoration in the original form and
condition of a lost or destroyed instrument attesting [to] the title of a person to a piece of land. The purpose of the
reconstitution is to have, after observing the procedures prescribed by law, the title reproduced in exactly the same way it
has been when the loss or destruction occurred. Among the conditions explicitly required by the law is publication of the
petition twice in successive issues of the Official Gazette, and its posting at the main entrance of the provincial building and
of the municipal building of the municipality or city in which the land is situated, at least thirty days prior to the date of
hearing. This directive is mandatory; indeed, its compliance has been held to be jurisdictional. . . . ."

Thus, before the trial court can acquire jurisdiction to hear and decide a reconstitution case, compliance with the following
requisites is imperative:

"1. [That] the notice of the petition be published, at the expense of the petitioner, twice in successive issues of the
Official Gazette, and posted on the main entrance of the provincial building and of the municipal building of the municipality
or city in which the land is situated, at least thirty days prior to the date of hearing;

"2. [That] the notice state among other things, the number of the lost or destroyed certificates of title if known, the
name of the registered owner, the name of the occupants or persons in possession of the property, the owner of the
adjoining properties and all other interested parties, the location, area and boundaries of the property, and the date on
which all persons having any interest therein must appear and file their claim of objection to the petition; cCSHET

"3. [That] a copy of the notice also be sent, by registered mail or otherwise, at the expense of the petitioner, to every
person named therein (i.e. the occupants or persons in possession of the property, the owner of the adjoining properties
and all other interested parties) whose address is known at least thirty days prior to the date of the hearing; and

"4. [That] at the hearing, petitioner submit proof of publication, posting and service of the notice as directed by the
court." 13

In the present case, it is undisputed that the Notice of Hearing of respondent's Petition for Reconstitution was not posted
at the main entrance of the provincial building. Clearly, the trial court did not acquire jurisdiction over the case.

But the appellate court, citing Calalang v. Register of Deeds, 14 opined that the publication of the Notice of Hearing in the
Official Gazette was "sufficient to vest jurisdiction upon the court to hear and determine the Petition." 15

We disagree. The Court in Calalang did not rule on whether the posting requirement was mandatory. It merely held that
the absence of personal notice to a person purporting to have a legitimate claim on the property was not a sufficient ground
to invalidate the proceedings. 16

It must be emphasized that under the law, the publication of a notice of hearing in the Official Gazette is not enough. The
posting of said notice at the main entrances of both the municipal and the provincial building is another equally vital requisite.
The purposes of the stringent and mandatory character of the legal requirements of publication, posting and mailing are to
safeguard against spurious and unfounded land ownership claims, to apprise all interested parties of the existence of such
action, and to give them enough time to intervene in the proceeding. 17

The publication of the Notice of Hearing in the Official Gazette does not justify the respondent's failure to comply with the
legal requirement of posting the Notice at the main entrance of both the municipal and the provincial buildings. The principle

64
of substantial compliance cannot be applied to the present case, as the trial court's acquisition of jurisdiction over the
Petition hinged on a strict compliance with the requirements of the law. ACETIa

True, the root of this failure may be traced to the June 15, 1994 Order of the trial court, However, this oversight cannot
excuse noncompliance with the requirements of RA No. 26. Under the circumstances, it is clear that the trial court did not
acquire jurisdiction over the case because of its own lapse, which respondent failed to cure.

WHEREFORE, the Petition is hereby GRANTED. The assailed Decision of the Court of Appeals is REVERSED and SET
ASIDE. No costs. EAISDH

SO ORDERED.

Melo, Vitug, Purisima, and Gonzaga-Reyes, JJ., concur.

11. Tolentino v. Social Security Commission, No. L-28870, September 6, 1985, 138 SCRA 428

SECOND DIVISION

[G.R. No. L-28870. September 6, 1985.]

AMADO D. TOLENTINO, petitioner appellant, vs. SOCIAL SECURITY COMMISSION, GILBERTO TEODORO and ANGEL
PENANO, respondents-appellees.

[G.R. No. L-39149. September 6, 1985.]

SOCIAL SECURITY SYSTEM and GILBERTO TEODORO, petitioner appellant, vs. THE HONORABLE COURT OF
INDUSTRIAL RELATIONS, THE SSS EMPLOYEES' LABOR UNION-NLU, and AMADO TOLENTINO, respondents-
appellees.

Ernesto Duran and Mercedes Bala for petitioners SSS and Teodoro.

Eulogio R. Lerum for respondents SS Labor Union and A. Tolentino.

DECISION

MAKASIAR, J p:

The present petitions for review by certiorari involve two different decisions of two different tribunals. prLL

In G.R. No. L-28870, appellant Amado Tolentino seeks the annulment and setting aside of the order of the Court of First
Instance (CFI) of Rizal (Branch IX, Quezon City) dated June 5, 1967 in Civil Case No. Q-10566 dismissing said appellant's
petition for mandamus with preliminary mandatory injunction, thus —

". . . this Court deems it unnecessary to consider the other grounds raised, and is constrained to dismiss the herein petition
for lack of jurisdiction to issue either preliminarily or permanently the writ sought in this petition, considering that the person
or body against whom the writ is sought to be applied is of the same rank as this Court" (pp. 97-98, Record on Appeal, p.
13, rec. of G.R. No. L-28870).

This appeal also seeks the annulment of the same Court's order dated December 1, 1967, denying herein petitioner-
appellant's motion for reconsideration.

In G.R. No. L-39149, petitioners Gilberto Teodoro and the Social Security System seek the annulment of the decision of
the Court of Industrial Relations (CIR) in Case No. 5042-ULP, entitled "SSS Employees' Labor Union — NLU and Amado
Tolentino vs. Social Security System and Gilberto Teodoro", the dispositive portion of which reads —

"WHEREFORE, respondents are hereby found to have committed unfair labor practice as charged and are ordered to
cease and desist therefrom, reinstate complainant Amado Tolentino to his former position, with back wages from the date
of dismissal up to actual reinstatement, and without loss of seniority and other privileges.

"SO ORDERED" (p. 109, rec. of G.R. No. L-39149).

G.R. NO. L-28870

"Petitioner was employed as an Editorial Assistant in the SSS before April 14, 1961 with a salary of P2,400.00 per annum.
His appointment as such was duly approved by the Civil Service Commission.
65
"On April 14, 1961, petitioner was given a promotion in salary from P2,400.00 per annum to P2,580.00 per annum effective
March 1, 1961. This promotion in salary was likewise duly approved by the Civil Service Commission. The copy of
petitioner's aforesaid promotional appointment is hereto attached and made an integral part hereof as Annex 'A'.

"On March 16, 1962, petitioner's designation was changed from 'Editorial Assistant' to 'Credit Analyst.' This appointment
was also duly approved by the Civil Service Commission. The copy of this appointment is hereto attached and made an
integral part hereof as Annex 'B'.

"On June 15, 1964, petitioner was given an appointment reinstating him to his former position as 'Credit Analyst.' This
reappointment was extended to petitioner following his resignation from the SSS to run for a municipal position in his
municipality in the 1961 elections. The copy of this appointment is hereto attached and made an integral part hereof as
Annex 'C'.

"On June 16, 1964, petitioner took his Oath of Office. The copy of the Oath of Office appearing in C.S. Form No. 32 of the
Philippine Civil Service is hereto attached and made an integral part hereof as Annex 'D'.

"On May 11, 1965, petitioner's designation was changed from 'Credit Analyst' to 'Technical Assistant' effective January 1,
1965, with an increase in salary from P2,580.00 per annum to P4,200 per annum. The copy of this appointment is hereto
attached and made an integral part hereof as Annex 'E'.

"It was the position of 'Technical Assistant' (Executive Assistant) that petitioner was holding when respondent Commission
passed Resolution No. 1003 on September 15, 1966 affirming the decision of respondent Administrator Gilberto Teodoro
'finding petitioner guilty of dishonesty, as charged, and imposing upon him the penalty of dismissal from the service,
effective on the first day of his preventive suspension (July 6, 1966) with prejudice to reinstatement.'

"Under dates of May 23 and 24, 1966, respectively, respondent Administrator filed charges against petitioner for dishonesty
and electioneering.

"In two (2) separate letters, dated July 2, 1966, to respondent Administrator, petitioner answered and denied the charges
against him.

"On July 6, 1966, petitioner received a copy of a memorandum of the same date signed by Mr. Reynaldo Gregorio as Acting
Administrator of the SSS informing him that his answer was not satisfactory and therefore, the charges against him would
be formally investigated by a committee composed of Attys. Ernesto D. Duran (as Chairman), Fabiana J. Patag and
Florencio Ongkingko theretofore constituted by virtue of Personnel Order 52-G. The same memorandum placed petitioner
on preventive suspension effective July, 1966.

"On July 12, 1966, said committee began investigation of the charges against petitioner officially terminating the same on
September 7, 1966.

"On September 30, 1966, petitioner received a letter dated September 20, 1966 from respondent Administrator informing
him, among others, of his dismissal from the service by virtue of Resolution No. 1003 of respondent Commission" (Brief for
Petitioner-Appellant, pp. 3-5, p. 19, rec. of L-28870).

On November 10, 1966, appellant Amado Tolentino filed with the Court of First Instance of Rizal (Quezon City, Branch IX)
a petition for mandamus with preliminary mandatory injunction questioning the validity of Resolution No. 1003. His theory
was "that Resolution No. 1003 of respondent Commission and the decision of respondent Administrator which it affirmed,
is null and void and of no effect whatever, for lack of jurisdiction because the power, nay, jurisdiction to decide administrative
cases against civil service employees like petitioner is vested by the Civil Service Act of 1959 and the Civil Service Rules
exclusively in the Civil Service Commissioner" (pp. 5-6, Record on Appeal, p. 13, rec. of G.R. no. L-28870). cdrep

"Under date of December 8, 1966, respondents filed their answer raising, among others, the affirmative defense of lack of
jurisdiction of the lower court over respondent Social Security Commission (Commission, for short), the latter being of the
same rank as the former.

"On June 5, 1967, after the parties had submitted memoranda to support their respective contentions on the question raised
by the pleadings, among others — whether respondents Social Security Administrator (Administrator, for short) and Social
Security Commission have the jurisdiction and authority to decide cases of administrative discipline against employees of
the Social Security System (SSS, for short) — the lower court rendered an order dismissing petitioner's petition for lack of
jurisdiction over respondent Commission because the latter ranks with the Court of First Instance in the exercise of the
quasi-judicial powers granted to it by the Social Security Act of 1954, as amended, following the decision of this Honorable
Tribunal in Poblete Construction Co., et al. vs. Social Security Commission, et al. (G.R. No. L-17605, promulgated January
22, 1964).

66
"On August 12, 1967 — within the reglementary period to appeal — petitioner filed a motion for reconsideration of the
above-mentioned order of the lower court, which the latter denied in an order dated December 1, 1967" (Brief for Petitioner-
Appellant, pp. 23, p. 19, rec. of L-28870).

Not satisfied with the last two mentioned orders, petitioner-appellant Tolentino elevated the case to this Court.

G.R. No. L-39149

"On May 7, 1968, the Prosecution Division of the CIR filed with said court a complaint . . . on motion of the SSS Employees
Labor Union — NLU and Amado Tolentino charging the SSS and Gilberto Teodoro with commission of unfair labor
practices. This case, docketed as Case No. 5042 — ULP, was entitled The Employees' Labor Union — NLU and Amado
Tolentino, petitioners, versus Social Security System and Gilberto Teodoro, respondents.

"On May 16, 1968, in answer to the complaint filed before the CIR, the herein petitioner Social Security System (hereinafter
referred to as SSS, for short) denied the charges of unfair labor practices and asserted that Amado Tolentino was dismissed
from the service after being charged and found guilty of Dishonesty on Two counts which was preceded by a formal
investigation.

"On March 5, 1974, the CIR rendered a decision declaring the SSS and Gilberto Teodoro guilty of unfair labor practice and
ordering the reinstatement of herein respondent-appellee Tolentino with back wages.

"On August 13, 1974, the CIR en banc denied the motion for reconsideration dated March 12, 1974 filed by the SSS, hence,
this petition for review on certiorari" (Brief for Petitioners, pp. 2-3).

On January 13, 1975, this Court issued a resolution in G.R. No. L-39149 consolidating the two appeal cases as both involve
the same parties and substantially the same issues.

The primordial question is one of jurisdiction — whether or not the Social Security Commission has jurisdiction over
administrative actions filed before it against its own erring employees.

Jurisdiction over the subject matter is vested by law. It is not acquired by the consent or acquiescence of the parties, nor
the unilateral assumption thereof by any tribunal (Bacalso vs. Ramolete, G.R. No. L-22488, October 26, 1967; De Jesus
vs. Garcia, L-26816, February 28, 1967). The settled rule is that jurisdiction of a court or tribunal is determined by the
statute in force at the time of the commencement of the action (Aquisap vs. Basilio, L-21293, December 29, 1967; Rilloraza
vs. Arciaga, L-23848, October 31, 1967; People vs. Pegarum, 58 Phil. 715). And once acquired, jurisdiction continues,
regardless of "subsequent happenings", until the case is finally terminated (People vs. Pegarum, 57 Phil. 715).

WE recall that the petition before US originated from administrative charges of dishonesty and electioneering filed by the
Administrator of the Social Security Commission before the same office on May 23 and 24, 1966. The Commission's
Resolution No. 1003, the validity of which is questioned here in G.R. No. 28870 for jurisdictional reasons, was promulgated
on September 15, 1966.

G.R. No. L-28870 was submitted for decision on January 21, 1969.

The pertinent laws under the circumstances are the Social Security Act of 1954 (R.A. 1161), as amended by R.A. 2658
(which took effect June 18, 1960) and the Civil Service Act of 1959 (R.A. 2260).

Re: G.R. No. L-28870

The question posed in this appeal is not an untrodden path. Mendoza vs. Social Security Commission, et al. (L-29189, April
11, 1972, 44 SCRA 373) penned by Justice J.B.L. Reyes, is in point.

"The case arose in this wise: appellee herein, Victor D. Mendoza, then Manager of the Commercial and Industrial Loans
Department in the Social Security System was subjected to formal investigation of various irregularities allegedly committed
by him. Hearings were duly conducted by the Investigating Committee on the formal charges filed and the explanations
offered by appellee. The records and proceedings were reviewed by a Committee of Commissioners that found Mendoza
guilty on four charges, and recommended his separation from the service. But the Commission en banc decided instead
that said officer be only demoted in rank and salary, from that of Manager, Commercial and Industrial Loans Department,
to that of Division Chief. The decision was embodied in Resolution No. 198 dated 9 February 1967.

"Mendoza then resorted to the Court a quo in quest of a writ of prohibition, with preliminary injunction, contending that the
Commission had no authority to impose the penalties embodied in its Resolution No. 198, the petitioner being covered by
the Civil Service Law of 1962 (Republic Act 2260), Section 33 whereof provided the following:

67
'Sec. 23. Administrative Jurisdiction for Disciplining Officers and Employees. — The Commissioner may, for
dishonesty, oppression, misconduct, neglect of duty, conviction of a crime involving moral turpitude, notoriously disgraceful
or immoral conduct, improper or unauthorized solicitation of contributions from subordinate employees and by teachers or
school officials from school children, violation of the existing Civil Service Law and rules of reasonable office regulations,
or in the interest of the service, remove any subordinate officer or employee from the service, demote him in rank, suspend
him for not more than one year without pay or fine him in an amount not exceeding six months' salary.

'In meting out punishment, like penalties shall be imposed for like offenses and only one penalty shall be imposed in each
case.'

"The Commission answered the petition averring that it was empowered by section 5 of the Social Security Act (Republic
Act No. 1161); . . . and that the Court of First Instance had no jurisdiction to review the resolutions of the Commission.

"After due trial, the court below, by decision of 13 January 1968, ruled that section 5 of the Social Security Act was
inapplicable to the case since it did not involve a settlement of benefit claims; that the assailed Resolution No. 198 was
void ab initio, the Commission having no power to discipline and penalize civil service officers to the exclusion of the
Commissioner of Civil Service . . .. The latter duly appealed to this Court.

"It is clear that under section 33 of the Civil Service Act (Republic Act 2260) heretofore quoted, before the amendments
introduced therein by Republic Act No. 6040, the sole power to impose disciplinary sanctions on civil service employees
was vested exclusively in the Commissioner of Civil Service. This is emphasized by the provisions of section 27 of the Civil
Service Rules requiring the Department Head concerned —

'within 15 days from receipt of the complete record of the case, to forward such record with his comment and
recommendation to the Commissioner for decision',

so that, as pointed out by the court below, the Department Head's powers were purely recommendatory; it had no power
to decide nor impose any penalty, much less to implement the decision or carry it out into execution.

"That the petitioner, as Manager of the Loans Department in the Social Security System, was and is a civil service official
is plain from Article II (section 3) of the Civil Service Act of 1959 as well as from section 3 of the Social Security Act of 1954,
as amended by Republic Act No. 2658.

"'ARTICLE II. — Scope of the Civil Service —

"SEC. 3. Positions embraced in the Civil Service. — The Philippine Civil Service shall embrace all branches,
subdivisions and instrumentalities of the Government, including government-owned or controlled corporations, and
appointments therein except as to those which are policy-determining, primarily confidential or highly technical in nature,
shall be made only according to merit and fitness, to be determined as far as practicable by competitive examination.
Positions included in the civil service fall into three categories; namely, competitive or classified service, noncompetitive or
unclassified service and exempt service. The exempt service does not fall within the scope of this law (emphasis supplied)".

"'SEC. 3. Social Security System. — . . .

'(c) The Commission, upon the recommendation of the Administrator, shall appoint an actuary, medical director, and
such other personnel as may be deemed necessary, shall fix their compensation, prescribe their duties and establish such
methods and procedures as may insure the efficient, honest and economical administration of the provisions and purpose
of this Act: Provided, however, That the personnel of the system shall be selected only from civil service eligibles certified
by the Commissioner of Civil Service and shall be subject to civil service rules and regulations' (emphasis supplied)."

"By way of avoidance of the application of the legal dispositions cited, the respondent Social Security Commission contends
that its officers and employees are not covered by the Civil Service Law and Rules, invoking the memorandum circulars of
Commissioner Abelardo Subido, dated 8 January 1964 and 23 January 1964 (Exhibits '4' and '5', pages 217 and 218,
Record), that —

'officers and employees of government owned or controlled corporations performing proprietary functions who have entered
into collective bargaining contracts with the management of their respective corporations, through their labor unions, fall in
the exempt service and are not covered or protected by the Civil Service Act' (Exhibit '4').

In the memorandum of 28 January 1964 (Exhibit '5'), the Commissioner of Civil Service further broadened the preceding
circular, remarking that —

'This office has observed that there are officers and employees of these corporations who, while not union members, accept
benefits under the collective bargaining contract between union and management and in so doing, have by implied
68
acquiescence, become parties to said collective bargaining contract. They, therefore, likewise fall under the exempt service
and are not governed or protected by the Civil Service Act of 1959.'

"We entertain serious doubts on the validity of the foregoing circulars, in view of the fact that under section 6 of the Civil
Service Act of 1959, the Exempt Service —

'shall consist of the following:

'(a) Elective officers

'(b) Members of the commissioned and enlisted service of the Army, Navy and Air Force of the Philippines.

'(c) Persons employed on a contract basis.'

"It is clear from the foregoing that in order to belong to the exempt service and thus forfeit the protection of the Civil Service
Law, a civilian non-elective officer must have obtained employment through a contract. In fact, section 2, paragraph (j), of
the Civil Service Rules, interpreting section 6 of the Act, declares that —

'(j) . . . the term 'persons employed on a contract basis' refers to independent contractors and those who may be
employed by them; it does not include employees or laborers who serve under the direction and supervision of a
governmental agency, except aliens who may be thus employed on a contract basis when the exigencies of the service so
require. In this context, the term 'independent contractor' refers to one who undertakes to do a piece of work for the
government under his own responsibility, with minimum interference on the part of any governmental agency in the
performance or accomplishment thereof.'

"Plainly, the circulars in question invoked by respondent Commission improperly attempted to broaden the scope of the
exempt service under the law, which was not within the powers of the Civil Service Commissioners to do.

xxx xxx xxx

"We are not unmindful of the fact that by Republic Act No. 6040 the Legislature extended the scope of the exempt service
to persons employed in government owned or controlled corporations primarily performing proprietary functions with
collective bargaining agreements; and that furthermore, the same Act also amended section 33 of the Civil Service Act by
adding at the end of the original section the following provisos:

'Provided, however, that heads of departments, agencies and instrumentalities, provinces and chartered cities, shall have
original jurisdiction to investigate and decide on matters involving disciplinary action. Provided further, that when the penalty
imposed is a reprimand or a fine not exceeding one month salary or suspension without pay for a period not exceeding one
month the decision of the aforementioned heads shall be final; but if the penalty imposed is heavier the decision shall be
appealable to the Commission as provided in this Act: Provided finally, that a decision imposing removal shall always be
subject to review by the Commission.'

"Had the present case arisen, therefore, under Republic Act 6040, the Social Security Commission would have had
jurisdiction, after due investigation, to impose the penalty of demotion subject only to appeal by the officer or employee
affected to the Civil Service Commission. Unfortunately for appellant Commission, Republic Act No. 6040 was enacted on
4 August 1969, and the case at bar had been litigated, decided and appealed to the Supreme Court and submitted for
decision as of February, 1969. Republic Act No. 6040 cannot be retroactively applied to the case, specially since the same
act expressly provides in its section 47 that —

'rights and privileges vested or acquired under the provisions of the Civil Service Law, rules and regulations prior to the
effectivity of this Act shall remain in force and effect'" (emphasis supplied).

Clearly at the time the questioned Resolution No. 1003 was promulgated and implemented dismissing petitioner-appellant
Amado Tolentino, the respondents-appellees Social Security Commission, Gilberto Teodoro and Angel Penano did not
have the power to hear and decide administrative and disciplinary charges filed against erring employees of the
Commission.

WE are not, however, ready to dismiss the questioned Resolution No. 1003 as inutile. The Social Security Commission, as
an agency of the government, may be considered a department and respondent Gilberto Teodoro, its department head.
Resolution No. 1003 may be treated as the recommendation of the department head which may be submitted to the Civil
Service Commission for decision and or appropriate action. LexLib

69
At this juncture, and to pave the way for a complete resolution of the case at bar, WE must consider the constitutionality of
the amendments to the Civil Service Act of 1959 as contained in Republic Act No. 6040, enacted ten years after or on
August 4, 1969.

As earlier noted, by Republic Act No. 6040, the legislature extended the scope of the exempt service to persons employed
in government owned or controlled corporations primarily performing proprietary functions with collective bargaining
agreements; in addition, it appended the following proviso to section 33 of the Civil Service Act —

"Provided, however, that heads of departments, agencies and instrumentalities, provinces and chartered cities, shall have
original jurisdiction to investigate and decide on matters involving disciplinary action: Provided further, that when the penalty
imposed is a reprimand or a fine not exceeding one month salary or suspension without pay for a period not exceeding one
month, the decision of the aforementioned heads shall be final; but if the penalty imposed is heavier the decision shall be
appealable to the Commission as provided in this Act: Provided finally, that a decision imposing removal shall always be
subject to review by the Commission."

However, Section 1(1), Article XII (B) of the 1973 Constitution

reads —

"The Civil Service embraces every branch, agency, subdivision, and instrumentality of the Government, including every
government-owned or controlled corporation . . .."

Clearly, insofar as Republic Act No. 6040 insulates government-owned or controlled-corporations with collective bargaining
agreements with their employees from the embrace of the Civil Service Commission, said statute is inconsistent with the
fundamental law of the land. As such, it is void (Article 7, New Civil Code).

RE: G.R. No. L-39149

In view of OUR decision in G.R. No. L-28870, WE rule to set aside as null and void the decision of respondent Court of
Industrial Relations dated March 5, 1974, and its subsequent en banc resolutions dated August 13, 1974 the same having
been issued without jurisdiction.

At the time Amado Tolentino was charged with and convicted of dishonesty in 1966 up to the time the Prosecution Division
of the Court of Industrial Relations filed with said court the unfair labor suit docketed as Case No. 5042-ULP on May 7,
1968, the power to impose disciplinary sanctions on erring employees of the Social Security Commission was vested
exclusively in the Commissioner of Civil Service, without prejudice to appeal to the Civil Service Board of Appeals (sections
18 and 36, R.A. 2260). Consequently, the Court of Industrial Relations, created under Commonwealth Act No. 103, a statute
of earlier vintage, had no jurisdiction over Case No. 5042-ULP. Again, jurisdiction of a court is determined by the statute in
force at the time of the commencement of the action (Aquisap vs. Basilio, supra; Rilloraza vs. Arciaga, L-23848, October
31, 1967; People vs. Pegarum, supra).

WE find no further need to scrutinize the findings of the Court of Industrial Relations. To do so would benefit no one.

WHEREFORE, THE QUESTIONED RESOLUTION NO. 1003 TOGETHER WITH THE RECORDS THEREOF ARE
HEREBY REMANDED TO THE OFFICE OF THE COMMISSIONER OF CIVIL SERVICE FOR APPROPRIATE ACTION.

THE DECISION AND RESOLUTION APPEALED FROM IN G.R. NO. L-39149 ARE HEREBY SET ASIDE AS NULL AND
VOID FOR HAVING BEEN RENDERED WITHOUT JURISDICTION. LexLib

NO COSTS.

Concepcion, Jr., Escolin, Cuevas and Alampay, JJ., concur.

Aquino (Chairman) and Abad Santos, JJ., are on leave.

12. Gustilo vs Gustilo 659 SCRA 428, G.R. No. 175497, October 19, 2011.

THIRD DIVISION

[G.R. No. 175497. October 19, 2011.]

MARY JOY ANNE GUSTILO and BONIFACIO M. PEÑA, petitioners, vs. JOSE VICENTE GUSTILO III and TERESITA
YOUNG also known as TITA SY YOUNG, respondents.

70
DECISION

ABAD, J p:

This case is about the proper characterization of a dispute between the president of a corporation and a stockholder, both
heirs to the corporation's controlling shares of stock, over the lease of a property that the president agreed to assign to the
stockholder as her inheritance. HIAcCD

The Facts and the Case

Petitioner Mary Joy Anne Gustilo and respondent Jose Vicente Gustilo III are heirs of their natural father, the late Atty.
Armando Gustilo (they have different mothers), who owned several properties and was, prior to his death, the president of
A.G. Agro-Industrial Corporation (A.G. Agro) in Cadiz City, Negros Occidental. Petitioner Bonifacio Peña is Mary Joy's
attorney-in-fact whom she authorized to exercise general control and supervision of her real properties.

On August 31, 1993, following their father's death, Mary Joy and Jose Vicente entered into a Memorandum of Agreement
(MOA), adjudicating between themselves their father's properties. One of these was Hacienda Imelda which the MOA
assigned to Mary Joy. As it happened, however, the hacienda's title remained in the name of A.G. Agro. Mary Joy
immediately took possession of the land through Mila Barco, her mother and natural guardian, and planted sugarcane on
it. TcCEDS

Over three years later or in 1997 Jose Vicente, as president of A.G. Agro, leased Hacienda Imelda and its farm implements
to respondent Tita Sy Young for five agricultural crop years from 1997-1998 to 2001-2002. Being financially hard up, Mary
Joy and her mother were pained to watch Young take over the land.

When the lease contract was about to expire, however, Mary Joy had her lawyer advise Young to surrender the land to
her. But the latter refused to yield possession and continued to cultivate the same for sugarcane. This prompted Mary Joy
to file an action against Jose Vicente and Young for recovery of possession of the hacienda, cancellation of the lease
contract, and damages before the Regional Trial Court (RTC) of Cadiz City. Jose Vicente filed a motion to dismiss mainly
on the ground that the Cadiz RTC had no jurisdiction to hear and decide intra-corporate disputes, the proper forum being
a specially designated commercial court.

On June 15, 2004 the RTC granted Jose Vicente's motion and dismissed the complaint for lack of jurisdiction, without
prejudice to its refiling in the proper court. On August 11, 2006 the Court of Appeals (CA) affirmed the RTC decision,
prompting Mary Joy to file the present petition. IcTCHD

The Issue Presented

The only issue presented in this case is whether or not Mary Joy's action presents an intra-corporate dispute that belongs
to the jurisdiction of a specially designated commercial court.

The Ruling of the Court

It is a basic rule that jurisdiction over the subject matter is determined by the allegations in the complaint. 1 It can be gleaned
from Mary Joy's allegations in her complaint that her case is principally one for recovery of possession. Immediately upon
the execution of the MOA in 1993, Mary Joy took possession of Hacienda Imelda, through her mother, and started planting
sugarcane on it. In 1997 Young, with the use of force, took over the property with the farm equipment and implements.
Despite several demands to vacate and surrender Hacienda Imelda, Young continued to cultivate and plant sugarcanes
on the property up to 2002, and even entered into a new lease contract with Jose Vicente. It must be stated that regardless
of the actual condition of the title to the property, the party in peaceable quiet possession shall not be turned out by a strong
hand, violence or terror. Thus, a party who can prove prior possession, can recover such possession even against the
owner himself. Whatever may be the character of his prior possession, if he has in his favor priority in time, he is entitled to
remain on the property until he is lawfully ejected by a person having a better right. 2 AEIcTD

Here, Jose Vicente and Young mainly argued in their Motion to Dismiss that inasmuch as the subject property is in the
name of A.G. Agro, the nature of the claim or controversy is one of intra-corporate. The Court has ruled in the past that an
action to recover possession is a plenary action in an ordinary civil proceeding to determine the better and legal right to
possess, independently of title. 3 But where the parties raise the issue of ownership, as in this case, the courts may pass
upon such issue to determine who between the parties has the right to possess the property. This adjudication, however,
is not final and binding as regards the issue of ownership; it is merely for the purpose of resolving the issue of possession
when it is inseparably connected to the issue of ownership. The adjudication on the issue of ownership, being provisional,
is not a bar to an action between the same parties involving title to the property. 4 Also, any intra-corporate issues that may

71
be involved in determining the real owner of the property may be threshed out in a separate proceeding in the proper
commercial court.

WHEREFORE, the Court GRANTS the petition and REVERSES and SETS ASIDE the Decision of the Court of Appeals in
CA-G.R. SP 85887 dated August 11, 2006. The Court likewise ORDERS Jose Vicente Gustilo III and Teresita Young to
answer the complaint in Civil Case 723-C, Regional Trial Court Negros Occidental, Branch 60. CTIDcA

SO ORDERED.

Velasco, Jr., Peralta, Mendoza and Perlas-Bernabe, JJ., concur.

13. Delos Reyes v. Odones, G.R. No. 178096, March 23, 2011, 646 SCRA 328

SECOND DIVISION

[G.R. No. 178096. March 23, 2011.]

ROSA DELOS REYES, petitioner, vs. SPOUSES FRANCISCO ODONES and ARWENIA ODONES, NOEMI OTALES, and
GREGORIO RAMIREZ, respondents.

DECISION

NACHURA, J p:

This petition for certiorari under Rule 45 of the Rules of Court seeks the reversal of the February 19, 2007 Decision 1 and
the May 22, 2007 Resolution 2 of the Court of Appeals (CA), affirming the June 20, 2006 decision 3 of the Regional Trial
Court (RTC), Branch 68, Camiling, Tarlac, which in turn set aside the March 28, 2006 decision 4 of the Municipal Trial Court
(MTC) of Camiling, Tarlac, in a complaint for unlawful detainer, disposed as follows: SIcCTD

WHEREFORE, judgment is hereby rendered in favor of the plaintiff and against defendants, ordering defendants, spouses
Arwenia Odones and Francisco Odones, their heirs and assigns and all persons acting in their behalves to vacate the
premises and to surrender possession thereof to the plaintiff. Defendants are likewise ordered to pay One Thousand
(P1,000.00) Pesos as reasonable compensation for the use of the land and Attorney's fees in the amount of Five Thousand
(P5,000.00) Pesos.

SO ORDERED. 5

The Facts

This case emanated from a complaint for Unlawful Detainer with Preliminary Injunction 6 filed by petitioner Rosa delos
Reyes (petitioner) against respondents spouses Arwenia and Francisco Odones, Noemi Otales, and Gregorio Ramirez
(respondents) before the MTC of Camiling, Tarlac, on July 12, 2005. The complaint alleged these material facts:

3. That [petitioner] is the owner of a parcel of land covered . . . by Transfer Certificate of Title No. 392430, of the Land
Records for the Province of Tarlac, located at Pao, Camiling, Tarlac, . . .;

4. That even before the document upon which the title was based, [petitioner] has long been the owner thereof;

5. That [respondents] are staying on the said property with a house/improvements therein, with the mere tolerance
of [petitioner] only without any contract whatsoever and for which there is an implied understanding to vacate upon the
demand;

6. That [petitioner] previously demanded verbally upon [respondents] to vacate which they refused and for which a
written notice was sent advising them to vacate the said property within fifteen (15) days from receipt of the letter to vacate
. . . .;

7. That the said letter was sent by registered mail on June 17, 2005, which was duly received . . . . 7

In their Answer with Counterclaim, 8 respondents claimed that they are the owners of the lot, having purchased the same
by virtue of an Extrajudicial Succession of Estate and Sale 9 dated January 29, 2004, executed by the heirs of Donata
Lardizabal, the land's original owner. Respondents denied that their occupancy of the property was by virtue of petitioner's
tolerance. 10

Respondents further argued that the basis of petitioner's Transfer Certificate of Title (TCT), which is a Deed of Absolute
Sale dated April 18, 1972, 11 was a forgery because the purported vendors therein, Donata Lardizabal and Francisco

72
Razalan, died on June 30, 1926 12 and June 5, 1971, 13 respectively. Incidentally, the said TCT and Deed of Absolute
Sale are the subject of a pending case for annulment of title before the RTC, Branch 68, Camiling, Tarlac. 14 SaICcT

In a decision dated March 28, 2006, the MTC ruled in favor of petitioner, and ordered respondents to vacate the property
and to pay rent for the use and occupation of the same, plus attorney's fees.

Respondents appealed 15 to the RTC, arguing that since the complaint failed to allege how respondents entered the
property or when they erected their houses thereon, it is an improper action for unlawful detainer, and the MTC had no
jurisdiction over the same. 16

In its June 20, 2006 decision, 17 the RTC set aside the MTC's judgment and dismissed the complaint. The RTC held that
the complaint failed to aver acts constitutive of forcible entry or unlawful detainer since it did not state how entry was effected
or how and when the dispossession started. Hence, the remedy should either be accion publiciana or accion reivindicatoria
in the proper RTC.

Aggrieved, petitioner sought recourse with the CA, asseverating that the RTC misappreciated the allegations in the
complaint and that respondents were estopped from assailing the MTC's jurisdiction because they did not raise such issue
in the proceedings before that court. Petitioner insisted that, as the registered owner of the lot, she has a preferential right
of possession over it. 18

On February 19, 2007, the CA affirmed the judgment of the RTC, adding that, as pronounced in Go, Jr. v. Court of Appeals,
19 in order to justify an action for unlawful detainer, the owner's permission or tolerance must be present at the beginning
of the possession. 20 Petitioner moved for reconsideration, 21 but the motion was denied in a Resolution dated May 22,
2007. 22 Hence, the instant petition 23 ascribing the following errors to the CA:

THE HON. COURT OF APPEALS ERRED IN APPLYING THE CASE OF GO, JR. v. COURT OF APPEALS. AETcSa

THE HON. COURT OF APPEALS ERRED IN HOLDING THAT THE HON. MUNICIPAL TRIAL COURT OF CAMILING,
TARLAC NEVER ACQUIRED JURISDICTION OVER THE CASE.

THE HON. COURT OF APPEALS ERRED IN NOT HOLDING THAT THE RESPONDENTS ARE ALREADY ESTOPPED
FROM RAISING THE ISSUE OF JURISDICTION.

THE HON. COURT OF APPEALS ERRED IN NOT APPLYING THE PRINCIPLE OF STARE DECISIS. 24

The petition is meritorious.

Well-settled is the rule that what determines the nature of the action, as well as the court which has jurisdiction over the
case, are the allegations in the complaint. In ejectment cases, the complaint should embody such statement of facts as to
bring the party clearly within the class of cases for which the statutes provide a remedy, as these proceedings are summary
in nature. The complaint must show enough on its face to give the court jurisdiction without resort to parol evidence. 25

Unlawful detainer is an action to recover possession of real property from one who illegally withholds possession after the
expiration or termination of his right to hold possession under any contract, express or implied. The possession by the
defendant in unlawful detainer is originally legal but became illegal due to the expiration or termination of the right to
possess. 26 The proceeding is summary in nature, jurisdiction over which lies with the proper MTC or metropolitan trial
court. The action must be brought up within one year from the date of last demand, and the issue in the case must be the
right to physical possession. 27

A complaint sufficiently alleges a cause of action for unlawful detainer if it recites the following: cSATDC

1. initially, possession of property by the defendant was by contract with or by tolerance of the plaintiff;

2. eventually, such possession became illegal upon notice by plaintiff to defendant of the termination of the latter's
right of possession;

3. thereafter, the defendant remained in possession of the property and deprived the plaintiff of the enjoyment thereof;
and

4. within one year from the last demand on defendant to vacate the property, the plaintiff instituted the complaint for
ejectment. 28

Contrary to the findings of the RTC and the CA, petitioner's allegations in the complaint clearly makes out a case for unlawful
detainer, essential to confer jurisdiction over the subject matter on the MTC. Petitioner alleges that she is the owner of the
lot, as shown by TCT No. 392430, issued by the Registry of Deeds of Tarlac; that respondents are occupying the lot by
73
virtue of petitioner's tolerance; and that petitioner sent a letter to respondents on June 17, 2005, demanding that they vacate
the property, but they failed and refused to do so. The complaint was filed on July 12, 2005, or within one year from the
time the last demand to vacate was made.

Firm is the rule that as long as these allegations demonstrate a cause of action for unlawful detainer, the court acquires
jurisdiction over the subject matter. SEACTH

The CA misapplied the ruling in Go 29 that tolerance must be present right from the start of possession, which possession
is sought to be recovered. The CA, in affirming the RTC, likewise erroneously applied the rule that jurisdictional facts must
appear on the face of the complaint for ejectment, such that when the complaint fails to faithfully aver facts constitutive of
unlawful detainer, as where it does not state when and how entry was effected, or how and when dispossession started,
the remedy should either be accion publiciana or accion reivindicatoria in the proper RTC.

The requirement that the complaint should aver, as jurisdictional facts, when and how entry into the property was made by
the defendants applies only when the issue is the timeliness of the filing of the complaint before the MTC, and not when
the jurisdiction of the MTC is assailed because the case is one for accion publiciana cognizable by the RTC. 30 This is
because, in forcible entry cases, the prescriptive period is counted from the date of defendants' actual entry into the
property; whereas, in unlawful detainer cases, it is counted from the date of the last demand to vacate. Hence, to determine
whether the case was filed on time, there is a necessity to ascertain whether the complaint is one for forcible entry or for
unlawful detainer; and since the main distinction between the two actions is when and how defendant entered the property,
the determinative facts should be alleged in the complaint. 31

In Go, there was evidence that the possession by the defendant was illegal at the inception and not merely tolerated as
alleged in the complaint. No such similar finding is extant in this case. Further, one of the factual issues raised in Go was
whether the action was filed within one (1) year from the date the last demand was made. Here, it is beyond dispute that
the complaint for unlawful detainer was filed within one (1) year from the date the demand letter was sent on June 17, 2005.
ASHaTc

Based on the foregoing, the MTC validly acquired jurisdiction over the complaint and we agree with its conclusion that
petitioner is entitled to the physical possession of the lot, she having been able to prove by preponderance of evidence,
through the TCT registered in her name, that she is entitled to possession of the property as owner. The countervailing
evidence presented by respondents that sought to dispute the authenticity of petitioner's TCT cannot be given weight in
this case. Settled is the rule that the validity of a certificate of title cannot be attacked in an action for ejectment. 32

This notwithstanding, the determination made herein as regards petitioner's ownership of the lot by virtue of TCT No.
392430 is only prima facie and only for purposes of resolving the issue of physical possession. These pronouncements are
without prejudice to the case of annulment of the deed of sale and TCT filed by respondents against petitioner. 33 Lastly,
these pronouncements are not binding on respondents Noemi Otales and Gregorio Ramirez over whose persons no
jurisdiction was acquired by the MTC. 34

WHEREFORE, the petition is GRANTED. The February 19, 2007 Decision and the May 22, 2007 Resolution of the Court
of Appeals are hereby REVERSED and SET ASIDE. The March 28, 2006 decision of the Municipal Trial Court of Camiling,
Tarlac, is REINSTATED and AFFIRMED.

SO ORDERED. EcaDCI

Carpio, Brion, * Peralta and Abad, JJ., concur.

14. Tomas Claudio Memorial College, Inc. v. Court of Appeals, 316 SCRA 502, 509 (1999)

SECOND DIVISION

[G.R. No. 124262. October 12, 1999.]

TOMAS CLAUDIO MEMORIAL COLLEGE, INC., petitioner, vs. COURT OF APPEALS, HON. ALEJANDRO S. MARQUEZ,
CRISANTA DE CASTRO, ELPIDIA DE CASTRO, EFRINA DE CASTRO, IRENEO DE CASTRO and ARTEMIO DE
CASTRO ADRIANO, respondents.

Aladdin F. Trinidad for petitioner.

Felix E. Mendiola for private respondents.

SYNOPSIS

74
Private respondents filed an action for Partition before the RTC of Morong, Rizal, alleging that the land they inherited from
their father was sold by their brother Mariano to herein petitioner without their knowledge and consent. Private respondents
contended that the sale affected only the undivided share of Mariano. Petitioner filed a motion to dismiss for lack of
jurisdiction and prescription. Allegedly, the RTC had no jurisdiction to try the case as the causes of action therein had
already been decided with finality by the Supreme Court in another case. When the RTC still took cognizance of the case
and the Court of Appeals allowed the same, this special civil action of certiorari was availed of.

No grave abuse of discretion was committed by the public respondent. Jurisdiction over the subject matter of a case is
conferred by law and is determined by the allegations of the complaint irrespective of whether plaintiff is entitled to the
claims asserted therein. Acquiring jurisdiction over the subject matter of a case does not necessarily mean that the lower
court meant to reverse the decision of the Supreme Court in the earlier case. And, when the court acts within its jurisdiction,
any alleged errors committed in the exercise thereof will amount to nothing more than errors of judgment which are
reversible by timely appeal and not by a special civil action of certiorari. THaAEC

SYLLABUS

1. REMEDIAL LAW; APPEAL AND CERTIORARI, WHEN PROPER. — As long as a court acts within its jurisdiction,
any alleged errors committed in the exercise thereof will amount to nothing more than errors of judgment which are revisable
by timely appeal and not by a special civil action of certiorari; because certiorari is not available to correct errors of procedure
or mistakes in the judge's findings and conclusions. And for a petition for certiorari to be granted, it must be shown that the
respondent court committed grave abuse of discretion equivalent to lack or excess of jurisdiction. By grave abuse of
discretion is meant such capricious and whimsical exercise of judgment as is equivalent to lack of jurisdiction, and mere
abuse of discretion is not enough — it must be grave.

2. ID.; JURISDICTION OVER THE SUBJECT MATTER OF A CASE; DISCUSSED. — Jurisdiction over the subject
matter of a case is conferred by law and is determined by the allegations of the complaint irrespective of whether the plaintiff
is entitled to all or some of the claims asserted therein. Acquiring jurisdiction over the subject matter of a case does not
necessarily mean that the lower court meant to reverse the decision of the Supreme Court in the land registration case
mentioned by the petitioner. Moreover, settled is the rule that the jurisdiction of the court over the subject matter is
determined by the allegations of the complaint, hence the court's jurisdiction cannot be made to depend upon defenses set
up in the answer or in a motion to dismiss. This has to be so, for were the principle otherwise, the ends of justice would be
frustrated by making the sufficiency of this kind of action dependent upon the defendant in all cases. AaEcDS

3. ID.; PARTY WHO INVOKES THE JURISDICTION OF A COURT CANNOT THEREAFTER CHALLENGE THE
SAME. — It is now too late for petitioner to question the jurisdiction of the Court of Appeals because it was petitioner who
elevated the instant controversy to the Court of Appeals via a petition for certiorari. In effect, petitioner submitted itself to
the jurisdiction of the Court of Appeals by seeking affirmative relief therefrom. If a party invokes the jurisdiction of a court,
he cannot thereafter challenge that court's jurisdiction in the same case. To do otherwise would amount to speculating on
the fortune of litigation, which is against the policy of the Court.

4. CIVIL LAW; PROPERTY; CO-OWNERSHIP; ACTION FOR PARTITION IS IMPRESCRIPTIBLE. — On the issue
of prescription, if a co-owner sells the whole property as his, the sale will affect only of his own share and not those of the
other co-owners who did not consent to the sale. Under Article 493 of the Civil Code, the transferee becomes a co-owner
and gets only what corresponds to his grantor's share in the partition of the property owned in common. Hence, the sale is
not null and void and the proper action is the division or partition of the entire property if it continued to remain in the
possession of the co-owners who possessed and administered it. Such partition should result in segregating the portion
belonging to the seller and its delivery to the buyer. Pursuant to Article 494 of the Civil Code, "no co-owner shall be obliged
to remain in the co-ownership. Such co-owner may demand at anytime the partition of the thing owned in common, insofar
as his share is concerned." In Budlong vs. Bondoc, this Court has interpreted said provision of law to mean that the action
for partition is imprescriptible. It cannot be barred by prescription. For Article 494 of the Civil Code explicitly declares: "No
prescription shall lie in favor of a co-owner or co-heirs as long as he expressly or impliedly recognizes the co-ownership."
EcIaTA

DECISION

QUISUMBING, J p:

This special civil action for certiorari seeks to set aside the Decision of the Court of Appeals dated August 14, 1995, in CA-
G.R. SP No. 36349, and its Resolution dated March 15, 1996, which denied petitioner's motion for reconsideration. cdrep

On December 13, 1993, private respondents filed an action for Partition before the Regional Trial Court of Morong, Rizal.
They alleged that their predecessor-in-interest, Juan De Castro, died intestate in 1993 and they are his only surviving and

75
legitimate heirs. They also alleged that their father owned a parcel of land designated as Lot No. 3010 located at Barrio
San Juan, Morong, Rizal, with an area of two thousand two hundred sixty nine (2,269) square meters more or less. They
further claim that in 1979, without their knowledge and consent, said lot was sold by their brother Mariano to petitioner. The
sale was made possible when Mariano represented himself as the sole heir to the property. It is the contention of private
respondents that the sale made by Mariano affected only his undivided share to the lot in question but not the shares of
the other co-owners equivalent to four fifths (4/5) of the property.

Petitioner filed a motion to dismiss contending, as its special defense, lack of jurisdiction and prescription and/or laches.
The trial court, after hearing the motion, dismissed the complaint in an Order dated August 18, 1984. On motion for
reconsideration, the trial court, in an Order dated October 4, 1994, reconsidered the dismissal of the complaint and set
aside its previous order. Petitioner filed its own motion for reconsideration but it was denied in an Order dated January 5,
1995.

Aggrieved, petitioner filed with the Court of Appeals a special civil action for certiorari anchored on the following grounds:
a) the RTC has no jurisdiction to try and take cognizance of the case as the causes of actions have been decided with
finality by the Supreme Court, and b) the RTC acted with grave abuse of discretion and authority in taking cognizance of
the case.

After the parties filed their respective pleadings, the Court of Appeals, finding no grave abuse of discretion committed by
the lower court, dismissed the petition in a Decision dated August 14, 1995. Petitioner filed a timely motion for
reconsideration but it was denied in a Resolution dated March 15, 1996. Hence this petition. cdasia

Petitioner submits the following grounds to support the granting of the writ of certiorari in the present case:

FIRST GROUND

THE HON. COURT OF APPEALS AND THE REGIONAL TRIAL COURT (BR. 79) HAD NO JURISDICTION TO TRY
SUBJECT CASE (SP. PROC. NO. 118-M). THE "CAUSES OF ACTION" HEREIN HAVE BEEN FINALLY DECIDED BY
THE HON. COURT OF FIRST INSTANCE OF RIZAL (BR. 31) MAKATI, METRO MANILA, AND SUSTAINED IN A FINAL
DECISION BY THE HON. SUPREME COURT.

SECOND GROUND

THE HON. COURT OF APPEALS GRAVELY ABUSED ITS DISCRETION AND AUTHORITY WHEN IT SUSTAINED THE
ORDERS OF THE HON. REGIONAL TRIAL COURT (BR. 79) DATED OCTOBER 4, 1994, AND THE ORDER DATED
JANUARY 5, 1995, WHEN SAID RTC (BR. 79) INSISTED IN TRYING THIS CASE AGAINST TCMC WHEN IT HAS RULED
ALREADY IN A FINAL ORDER THAT PETITIONER IS NOT A "REAL PARTY" IN INTEREST BY THE HON. REGIONAL
TRIAL COURT (BR. 79) IN CIVIL CASE NO. 170, ENTITLED ELPIDIA A. DE CASTRO, ET. AL. vs. TOMAS CLAUDIO
MEMORIAL COLLEGE, ET. AL., WHICH CASE INVOLVED THE SAME RELIEF, SAME SUBJECT MATTER AND THE
SAME PARTIES.

THIRD GROUND

THE HON. COURT OF APPEALS GRAVELY ABUSED ITS DISCRETION AND AUTHORITY WHEN IT CAPRICIOUSLY
AND WHIMSICALLY DISREGARDED THE EXISTENCE OF RES JUDICATA IN THIS CASE.

The pivotal issues to be resolved in this case are: whether or not the Regional Trial Court and/or the Court of Appeals had
jurisdiction over the case, and if so, whether or not the Court of Appeals committed grave abuse of discretion in affirming
the decision of the Regional Trial Court.

In assailing the Orders of the appellate court, petitioner invokes Rule 65 of the Rules of Court as its mode in obtaining a
reversal of the assailed Decision and Resolution. Before we dwell on the merits of this petition, it is worth noting, that for a
petition for certiorari to be granted, it must be shown that the respondent court committed grave abuse of discretion
equivalent to lack or excess of jurisdiction and not mere errors of judgment, for certiorari is not a remedy for errors of
judgment, which are correctible by appeal. 1 By grave abuse of discretion is meant such capricious and whimsical exercise
of judgment as is equivalent to lack of jurisdiction, and mere abuse of discretion is not enough — it must be grave. 2

In the case at hand, there is no showing of grave abuse of discretion committed by the public respondent. As correctly
pointed out by the trial court, when it took cognizance of the action for partition filed by the private respondents, it acquired
jurisdiction over the subject matter of the case. 3 Jurisdiction over the subject matter of a case is conferred by law and is
determined by the allegations of the complaint irrespective of whether the plaintiff is entitled to all or some of the claims
asserted therein. 4 Acquiring jurisdiction over the subject matter of a case does not necessarily mean that the lower court
meant to reverse the decision of the Supreme Court in the land registration case mentioned by the petitioner. Cdpr

76
Moreover, settled is the rule that the jurisdiction of the court over the subject matter is determined by the allegations of the
complaint, hence the court's jurisdiction cannot be made to depend upon defenses set up in the answer or in a motion to
dismiss. 5 This has to be so, for were the principle otherwise, the ends of justice would be frustrated by making the
sufficiency of this kind of action dependent upon the defendant in all cases.

Worth stressing, as long as a court acts within its jurisdiction any alleged errors committed in the exercise thereof will
amount to nothing more than errors of judgment which are revisable by timely appeal and not by a special civil action of
certiorari. 6 Based on the foregoing, even assuming for the sake of argument that the appellate court erred in affirming the
decision of the trial court, which earlier denied petitioner's motion to dismiss, such actuation on the part of the appellate
court cannot be considered as grave abuse of discretion, hence not correctible by certiorari, because certiorari is not
available to correct errors of procedure or mistakes in the judge's findings and conclusions.

In addition, it is now too late for petitioner to question the jurisdiction of the Court of Appeals. It was petitioner who elevated
the instant controversy to the Court of Appeals via a petition for certiorari. In effect, petitioner submitted itself to the
jurisdiction of the Court of Appeals by seeking affirmative relief therefrom. If a party invokes the jurisdiction of a court, he
cannot thereafter challenge that court's jurisdiction in the same case. 7 To do otherwise would amount to speculating on
the fortune of litigation, which is against the policy of the Court.

On the issue of prescription, we have ruled that even if a co-owner sells the whole property as his, the sale will affect only
his own share but not those of the other co-owners who did not consent to the sale. 8 Under Article 493 of the Civil Code,
the sale or other disposition affects only the seller's share pro indiviso, and the transferee gets only what corresponds to
his grantor's share in the partition of the property owned in common. Since a co-owner is entitled to sell his undivided share,
a sale of the entire property by one co-owner without the consent of the other co-owners is not null and void. However, only
the rights of the co-owner/seller are transferred, thereby making the buyer a co-owner of the property. The proper action in
a case like this, is not for the nullification of the sale, or for the recovery of possession of the property owned in common
from the third person, but for division or partition of the entire property if it continued to remain in the possession of the co-
owners who possessed and administered it. 9 Such partition should result in segregating the portion belonging to the seller
and its delivery to the buyer. cdrep

In the light of the foregoing, petitioner's defense of prescription against an action for partition is a vain proposition. Pursuant
to Article 494 of the Civil Code, "no co-owner shall be obliged to remain in the co-ownership. Such co-owner may demand
at anytime the partition of the thing owned in common, insofar as his share is concerned." In Budlong vs. Bondoc, 10 this
Court has interpreted said provision of law to mean that the action for partition is imprescriptible. It cannot be barred by
prescription. For Article 494 of the Civil Code explicitly declares: "No prescription shall lie in favor of a co-owner or co-heirs
as long as he expressly or impliedly recognizes the co-ownership."

WHEREFORE, the instant petition is DENIED. The assailed decision of the Court of Appeals is hereby AFFIRMED. Costs
against petitioners.

SO ORDERED.

Mendoza and Buena, JJ., concur.

Bellosillo, J., is on official leave.

15. Javellana v. Presiding Judge, RTC, Branch 30, Manila, G.R. No. 139067, 23 November 2004, 443 SCRA 497,
506.

SECOND DIVISION

[G.R. No. 139067. November 23, 2004.]

SPS. MA. CARMEN L. JAVELLANA and VICTOR JAVELLANA, petitioners, vs. HON. PRESIDING JUDGE, Regional Trial
Court, Branch 30, Manila and BENITO LEGARDA, respondents.

Santiago Cruz & Sarte Law Offices for petitioners.

Manuel Y. Macias for private respondent.

SYNOPSIS

77
Private respondent Benito Legarda filed before the Regional Trial Court, Branch 30, Manila, a complaint for Accion
Publiciana and Sum of Money against petitioners for the restoration of possession to private respondent of Lot No. 44, Plan
15, situated in the District of Sampaloc, Manila, which was the subject of a Contract to Sell between the herein parties.
Petitioners then filed a motion to dismiss alleging that the trial court had no jurisdiction over the case since the subject
property is a subdivision lot, hence, it is cognizable by the Housing and Land Use Regulatory Board (HLURB), but it was
denied by the trial court. Petitioners then filed their Answer Ad Abundante Cautelam with Compulsory Counterclaim.
Thereafter, petitioners filed the petition for certiorari with the Court of Appeals, but was denied by the latter for having been
filed out of time. Hence, this petition.

The Court ruled that during the pendency of this case, A.M. No. 00-2-03-SC amended Sec. 4, Rule 65 of the Rules of Court
and the amendment was deemed applicable to the instant case. Consequently, counting the 60 days from the time
petitioners received the Order of the trial court denying their motion for reconsideration on January 18, 1999 the petition
filed with the Court of Appeals on March 19, 1999, was within the reglementary period of filing the same. The Court of
Appeals could thus take cognizance of the petition for certiorari filed before it. However, instead of remanding the case to
it which would unduly prolong the trial of the main case, the Court resolved to act on the issue presented, i.e., whether the
subject matter of this case falls under the exclusive jurisdiction of the HLURB, which is a question of law.

A reading of the complaint does not show that the subject lot was a subdivision lot which would fall under the jurisdiction of
the HLURB. The complaint clearly described the subject lot as Lot No. 44, Plan 15, with an area of 139.4 sq. meters situated
in the District of Sampaloc covered by Transfer Certificate of Title No. 131305 of the Registry of Deeds of Manila. Such
description was used when referring to the subject lot. What appeared from the complaint was the fact that the subject lot
was sold to petitioners in an ordinary sale of a lot on installment basis; that petitioners allegedly defaulted in the payment
of their monthly installments for which reason respondent sought to recover possession thereof. Thus, the trial court has
jurisdiction over the case. TaDSCA

SYLLABUS

1. REMEDIAL LAW; SPECIAL CIVIL ACTIONS; CERTIORARI; 60-DAY PERIOD WITHIN WHICH TO FILE THE
PETITION STARTS TO RUN FROM RECEIPT OF NOTICE OF THE DENIAL OF THE MOTION FOR
RECONSIDERATION. — We rule that the amendment is deemed applicable to the instant case. As held in San Luis vs.
CA: Under this amendment, the 60-day period within which to file the petition starts to run from receipt of notice of the
denial of the motion for reconsideration, if one is filed. In our decision in Systems Factors Corporation and Modesto Dean
vs. NLRC, et al. reiterated in Unity Fishing Development Corp. and/or Antonio Dee vs. Court of Appeals, et al. the new
period was made applicable to pending cases, such as in the case at bar. Settled is the rule that remedial statutes or
statutes relating to remedies or modes of procedure, which do not create new rights or take away vested rights but only
operate in furtherance of the remedy or confirmation of rights already existing, do not come within the purview of the general
rule against the retroactive operation of statutes. Procedural laws are construed to be applicable to actions pending and
undetermined at the time of their passage, and are deemed retroactive in that sense and to that extent. As a general rule,
the retroactive application of procedural laws cannot be considered violative of any personal rights because no vested right
may attach to nor arise therefrom. Consequently, counting the 60 days from the time petitioners received the Order of the
trial court denying their motion for reconsideration on January 18, 1999, the petition filed with the Court of Appeals on March
19, 1999, was within the reglementary period of filing the same. STcAIa

2. ID.; CIVIL PROCEDURE; JURISDICTION OVER THE SUBJECT MATTER; DETERMINED BY THE
ALLEGATIONS IN THE COMPLAINT. — It is a settled rule that jurisdiction over the subject matter is determined by the
allegations in the complaint. Jurisdiction is not affected by the pleas or the theories set up by the defendant in an answer
or a motion to dismiss. Otherwise, jurisdiction would become dependent almost entirely upon the whims of the defendant.

3. POLITICAL LAW; ADMINISTRATIVE LAW; PRESIDENTIAL DECREE NO. 1344; JURISDICTION OF THE
HOUSING AND LAND USE REGULATORY BOARD (HLURB); DOES NOT COVER ORDINARY SALE OF A LOT ON
INSTALLMENT BASIS. — A reading of the complaint does not show that the subject lot was a subdivision lot which would
fall under the jurisdiction of the HLURB. The complaint clearly described the subject lot as Lot No. 44, Plan 15 with an area
of 139.4 sq. meters situated in the District of Sampaloc covered by Transfer Certificate of Title No. 131305 of the Registry
of Deeds of Manila. We note that such description was used when referring to the subject lot. What appears from the
complaint was the fact that the subject lot was sold to petitioners in an ordinary sale of a lot on installment basis; that
petitioners allegedly defaulted in the payment of their monthly installments for which reason respondent seeks to recover
possession thereof. Thus, the trial court has jurisdiction over the case.

4. ID.; ID.; ID.; ID.; MERE ALLEGATIONS IN THE COMPLAINT THAT THE SUBJECT LOT IS A "SUBDIVISION
LOT" WERE NOT SUFFICIENT TO VEST HLURB OF JURISDICTION OVER THE CASE. — Both petitioners and
respondent failed to attach in their pleadings filed before the trial court, a copy of the Contract To Sell to show the terms

78
and conditions embodied therein. A mere reference to the above-quoted paragraph, standing alone, does not establish that
the subject lot is a subdivision lot. What it merely states is that petitioners may invoke the grounds of lack of development
of respondent's property as a regular subdivision project and within the time limit set, if such requirements are applicable
to the subject lot, for voluntarily desisting from further payments and their installments paid would not be forfeited. There is
not even a certainty that those grounds apply to the subject lot. In fact, the use of the phrase "regular subdivision project"
does not automatically make the instant case fall under the jurisdiction of the HLURB. In Sps. Kakilala vs. Faraon
notwithstanding the allegations of petitioners in their complaint that the subject lot is "a subdivision lot" in a "subdivision
project," we held that such allegations were not sufficient to vest the HLURB of jurisdiction over the case[.] aACHDS

DECISION

AUSTRIA-MARTINEZ, J p:

Before us is a petition for review on certiorari filed by petitioners spouses Ma. Carmen and Victor Javellana, assailing the
Resolution dated April 30, 1999 1 of the Court of Appeals dismissing their petition for certiorari, docketed as CA-G.R. SP
No. 51833, for being filed out of time; and the Resolution dated June 9, 1999 2 denying petitioners' motion for
reconsideration. THaAEC

The factual background of the case is as follows:

On December 6, 1996, private respondent Benito Legarda filed before the Regional Trial Court of Manila, Branch 30, a
complaint for Accion Publiciana and sum of money 3 against petitioners, portions of which read:

xxx xxx xxx

3. On December 11, 1992, defendants MA. CARMEN L. JAVELLANA and VICTOR C. JAVELLANA entered into a
Contract To Sell with plaintiff whereby subject to the terms and conditions therein provided, plaintiff agreed to sell to them
its property identified as Lot No. 44, Plan 15 with an area of 139.4 square meters situated in the District of Sampaloc, Manila
and covered by Transfer Certificate of Title No. 131305 of the Registry of Deeds of Manila in plaintiff's name for the total
sum of P836,400.00 which after a down payment of P83,640.00 the balance of P752,760.00 was to be paid within five (5)
years by means of 60 equal monthly installments of P19,943.57 each which included the stipulated interest of 20% per
annum. The installments were to be paid every 30th of each month beginning February, 1993.

xxx xxx xxx

4. Upon the execution of the Contract To Sell, ANNEX "A", defendants MA. CARMEN L. JAVELLANA and VICTOR
C. JAVELLANA were placed in possession of the aforementioned lot.

5. Nevertheless, since February, 1995 defendants spouses have defaulted in the payment of the monthly
installments.

6. After the grace period allowed and provided in the Contract To Sell, ANNEX "A", plaintiff exercised its right to
cancel the contract by executing a "RESCISSION OF CONTRACT" on October 16, 1996. . . Formal notice and copy of the
"RESCISSION OF THE CONTRACT," Annex "B", have(sic) duly received by defendants.

7. As defendants have made total payments in the sum of P546,453.18 on the "CONTRACT TO SELL", ANNEX "A",
up to its rescission on October 16, 1996, ANNEX "B", defendants spouses are entitled to the refund of the cash surrender
value equivalent to fifty percent (50%) of the total payments or the sum of P270,726.59 in accordance with the provisions
of Section 3(b) of Republic Act No. 6552 (the MACEDA LAW).

8. Plaintiff is ready to pay to defendants spouses the said cash surrender value in the sum of P270,726.59
immediately after the restoration to plaintiff of the possession of Lot No. 44, Plan 15, District of Sampaloc, Manila.

9. Restoration of possession of the lot to plaintiff should be effected not later than thirty (30) days from the date of
service upon defendants spouses of the Honorable Court's judgment —

a. directing plaintiff to pay defendant spouses the sum of P270,726.59 representing the cash surrender value of the
total payments made by them; ACTaDH

b. ordering defendants to vacate forthwith Lot No. 44, Plan 15, District of Sampaloc, Manila and restore possession
to plaintiff. 4

On March 16, 1998, petitioners filed a motion to dismiss 5 alleging that the trial court has no jurisdiction over the case.
Private respondent filed an opposition thereto 6 and a reply was filed by petitioners. 7

79
In an Order dated September 30, 1998, 8 the trial court denied petitioners' motion to dismiss, a copy of which was received
by petitioners on November 3, 1998. Petitioners' motion for reconsideration was likewise denied in an Order dated
December 28, 1998, 9 and received by petitioners on January 18, 1999. Subsequently, petitioners filed their Answer Ad
Abundante Cautelam with Compulsory Counterclaim for damages and attorney's fees. 10

Petitioners then filed the subject petition for certiorari under Rule 65 11 with the Court of Appeals raising this issue:

WHETHER OR NOT PUBLIC RESPONDENT COMMITTED GRAVE ABUSE OF DISCRETION AMOUNTING TO LACK
OR EXCESS OF JURISDICTION IN HOLDING THAT THE REGIONAL TRIAL COURT OF MANILA (BRANCH 30) HAS
JURISDICTION OVER THE SUBJECT MATTER OF THE COMPLAINT FILED BY THE PRIVATE RESPONDENT.

On April 30, 1999, the Court of Appeals issued its assailed Resolution dismissing CA-G.R. SP No. 51833 for being filed out
of time. Petitioners' motion for reconsideration was denied in a Resolution dated June 9, 1999.

Hence, the present petition which raises the following issues:

I. WHETHER OR NOT THE FAILURE OF THE PETITIONERS TO TIMELY FILE THE PETITION FOR CERTIORARI
AMOUNTS TO ESTOPPEL DESPITE THE FACT THAT THE ISSUE TO BE RESOLVED INVOLVES THE JURISDICTION
OF THE RESPONDENT COURT.

II. WHETHER OR NOT THE REGIONAL TRIAL COURT OF MANILA (BRANCH 30) HAS JURISDICTION OVER
THE SUBJECT MATTER OF THE COMPLAINT FILED BY PRIVATE RESPONDENT.

Petitioners submit that there is a need to reconsider the resolutions of the Court of Appeals since the controversy involves
the jurisdiction of the trial court; that rules of procedure should not be applied in a very rigid and technical sense so as not
to override substantial justice; that the subject property is a subdivision lot as expressly stipulated in their Contract to Sell;
that the dispute between petitioners and respondent involves a subdivision project as defined under Section 2 of P.D. No.
957, hence it is cognizable by the National Housing Authority, now Housing and Land Use Regulatory Board (HLURB), 12
which has exclusive jurisdiction to regulate the real estate trade and business; 13 that HLURB has jurisdiction even over
complaints instituted by developers against subdivision buyers.

In his Comment, private respondent alleges: The title of the case given by petitioners is misleading since it should be Benito
Legarda, Inc. and not Benito Legarda; that nowhere in their petition did petitioners challenge the findings of the Court of
Appeals that they filed their petition six days late; that they are estopped from questioning the jurisdiction of the trial court
since after their motion to dismiss was denied by the trial court, they filed their (1) Answer Ad Abundante Cautelam with
Compulsory Counterclaim for damages and attorney's fees; and (2) Pre-trial brief where their counterclaim for damages
and attorney's fees were also enumerated; that respondent being the lot owner seeking to enforce the terms and conditions
of the Contract To Sell with petitioners is not one of those instances that would fall within the jurisdiction of the HLURB.
Petitioners filed their Reply. IEHTaA

We gave due course to the petition and as required, the parties submitted their respective memoranda.

There is no question that at the time petitioners filed CA-G.R. SP No. 51833 on March 19, 1999, the applicable law was
Section 4, Rule 65 of the 1997 Rules of Civil Procedure, as amended by the Resolution of July 21, 1998, which provides:

Sec. 4. Where petition filed. — The petition shall be filed not later than sixty (60) days from notice of the judgment, order
or resolution sought to be assailed in the Supreme Court or, if it relates to the acts or omissions of a lower court or of a
corporation, board, officer or person in the Regional Trial Court exercising jurisdiction over the territorial area as defined by
the Supreme Court. It may also be filed in the Court of Appeals whether or not the same is in aid of its appellate jurisdiction.
If it involves the acts or omissions of a quasi-judicial agency, and unless otherwise provided by law or these Rules, the
petition shall be filed in and cognizable only by the Court of Appeals.

If the petitioner had filed a motion for new trial or reconsideration in due time after notice of said judgment, order or
resolution, the period herein fixed shall be interrupted. If the motion is denied, the aggrieved party may file the petition
within the remaining period, but which shall not be less than five (5) days in any event, reckoned from notice of such denial.
No extension of time to file the petition shall be granted except for the most compelling reason and in no case exceeding
fifteen (15) days. (Emphasis ours.)

On the basis thereof, the Court of Appeals found the petition belatedly filed, thus:

Applying the aforequoted provision of the rule, since petitioners received a copy of the Order dated September 30, 1998
on NOVEMBER 3, 1998 and they filed a Motion for Reconsideration thereof on NOVEMBER 9, 1998, six (6) days had
elapsed; hence petitioners have a remaining period of FIFTY-FOUR (54) DAYS from receipt of the denial of their Motion
for Reconsideration within which to file petition for certiorari with this Court.
80
They received a copy of the Order dated December 28, 1998, denying their Motion for Reconsideration on January 18,
1998; hence, they have until MARCH 13, 1999 within which to file a petition for certiorari. However, the present petition for
certiorari was filed only on MARCH 19, 1999, or six (6) days late. 14

However, during the pendency of this case, A.M. No. 00-2-03-SC amended Section 4, Rule 65 which took effect on
September 1, 2000, as follows:

Sec. 4. When and where petition filed. — The petition shall be filed not later than sixty (60) days from notice of the
judgment, order or resolution. In case a motion for reconsideration or new trial is timely filed, whether such motion is
required or not, the sixty (60) day period shall be counted from notice of the denial of said motion.

The petition shall be filed in the Supreme Court or, if it relates to the acts or omissions of a lower court or of a corporation,
board, officer or person, in the Regional Trial Court exercising jurisdiction over the territorial area as defined by the Supreme
Court. It may also be filed in the Court of Appeals whether or not the same is in aid of its appellate jurisdiction. If it involves
the acts or omissions of a quasi-judicial agency, unless otherwise provided by law or these rules, the petition shall be filed
in and cognizable only by the Court of Appeals. aCcSDT

No extension of time to file the petition shall be granted except for compelling reason and in no case exceeding fifteen (15)
days. (Emphasis ours.)

We rule that the amendment is deemed applicable to the instant case. As held in San Luis vs. CA: 15

Under this amendment, the 60-day period within which to file the petition starts to run from receipt of notice of the denial of
the motion for reconsideration, if one is filed. In our decision in Systems Factors Corporation and Modesto Dean vs. NLRC,
et al. reiterated in Unity Fishing Development Corp. and/or Antonio Dee vs. Court of Appeals, et al. the new period was
made applicable to pending cases, such as in the case at bar. Settled is the rule that remedial statutes or statutes relating
to remedies or modes of procedure, which do not create new rights or take away vested rights but only operate in
furtherance of the remedy or confirmation of rights already existing, do not come within the purview of the general rule
against the retroactive operation of statutes. Procedural laws are construed to be applicable to actions pending and
undetermined at the time of their passage, and are deemed retroactive in that sense and to that extent. As a general rule,
the retroactive application of procedural laws cannot be considered violative of any personal rights because no vested right
may attach to nor arise therefrom. (Emphasis ours.)

Consequently, counting the 60 days from the time petitioners received the Order of the trial court denying their motion for
reconsideration on January 18, 1999, the petition filed with the Court of Appeals on March 19, 1999, was within the
reglementary period of filing the same.

Considering the foregoing, the Court of Appeals could take cognizance of the petition for certiorari filed before it. However,
instead of remanding the case to it which would unduly prolong the trial of the main case, we shall act on the issue
presented, i.e., whether the subject matter of this case falls under the exclusive jurisdiction of the HLURB, which is a
question of law.

We rule in the negative.

Under Section 1 of P.D. No. 1344, the National Housing Authority (now HLURB) has exclusive jurisdiction to hear and
decide the following, thus:

SECTION 1. In the exercise of its functions to regulate the real estate trade and business and in addition to its powers
provided for in Presidential Decree No. 957, the National Housing Authority shall have exclusive jurisdiction to hear and
decide cases of the following nature:

A. Unsound real estate business practices;

B. Claims involving refund and any other claims filed by subdivision lot or condominium unit buyer against the project
owner, developer, dealer, broker or salesman; and

C. Cases involving specific performance of contractual and statutory obligations filed by buyers of subdivision lot or
condominium unit against the owner, developer, dealer, broker or salesman.

It is a settled rule that jurisdiction over the subject matter is determined by the allegations in the complaint. Jurisdiction is
not affected by the pleas or the theories set up by the defendant in an answer or a motion to dismiss. Otherwise, jurisdiction
would become dependent almost entirely upon the whims of the defendant. 16 Accordingly, we can resolve the issue
presented before us by examining the allegations in the complaint filed by respondent in the trial court, docketed as Civil
Case No. 96-81225, for accion publiciana and sum of money. CEcaTH
81
A reading of the complaint does not show that the subject lot was a subdivision lot which would fall under the jurisdiction of
the HLURB. The complaint clearly described the subject lot as Lot No. 44, Plan 15 with an area of 139.4 sq. meters situated
in the District of Sampaloc covered by Transfer Certificate of Title No. 131305 of the Registry of Deeds of Manila. We note
that such description was used when referring to the subject lot. What appears from the complaint was the fact that the
subject lot was sold to petitioners in an ordinary sale of a lot on installment basis; that petitioners allegedly defaulted in the
payment of their monthly installments for which reason respondent seeks to recover possession thereof. Thus, the trial
court has jurisdiction over the case.

Petitioners' insistence that the subject lot is a subdivision lot, thus cognizable by HLURB is anchored on paragraph 6 of
their Contract To Sell which provides:

xxx xxx xxx

Notwithstanding the foregoing provisions of the first paragraph of this Sixth Clause no installment payment make (sic) by
the SECOND PARTY shall be forfeited in favor of the FIRST PARTY, when the SECOND PARTY, after giving notice to the
FIRST PARTY, voluntarily desists from further payment on grounds of lack of development of the FIRST PARTY'S property
as a regular subdivision project and within the time limit that had been set for such development, insofar as this requirement
may apply to the FIRST PARTY'S property considering that the lots being sold by the FIRST PARTY had been inherited
by the FIRST PARTY'S immediate predecessors in interest and constituted scattered fragments of widely separated pre-
war subdivisions approved according to the official pre-requisites in force at the time.

We are not convinced.

Both petitioners and respondent failed to attach in their pleadings filed before the trial court, a copy of the Contract To Sell
to show the terms and conditions embodied therein. A mere reference to the above-quoted paragraph, standing alone,
does not establish that the subject lot is a subdivision lot. What it merely states is that petitioners may invoke the grounds
of lack of development of respondent's property as a regular subdivision project and within the time limit set, if such
requirements are applicable to the subject lot, for voluntarily desisting from further payments and their installments paid
would not be forfeited. There is not even a certainty that those grounds apply to the subject lot. In fact, the use of the phrase
"regular subdivision project" does not automatically make the instant case fall under the jurisdiction of the HLURB. In Sps.
Kakilala vs. Faraon, 17 notwithstanding the allegations of petitioners in their complaint that the subject lot is "a subdivision
lot" in a "subdivision project," we held that such allegations were not sufficient to vest the HLURB of jurisdiction over the
case, thus:

Jurisdiction is determined by the averments of the complaint and not by the defense contained in the answer. Hence, the
jurisdictional issue involved here shall be determined on the basis of the allegations of petitioner's complaint before the
HLURB. Petitioners simply alleged therein that the subject lot is "a subdivision lot" in "a subdivision project." Under Section
2(d) and (e) of PD 957, "subdivision project" and "subdivision lot" are defined as follows:

d) Subdivision project — "Subdivision project" shall mean a tract or a parcel of land registered under Act No. 496
which is partitioned primarily for residential purposes into individual lots with or without improvements thereon, and offered
to the public for sale, in cash or in installment terms. It shall include all residential, commercial, industrial and recreational
areas as well as open spaces and other community and public areas in the project. IEDHAT

e) Subdivision lot. — "Subdivision lot" shall mean any of the lots, whether residential, commercial, industrial, or
recreational, in a subdivision project.

There is no allegation in the complaint that the lot purchased by petitioners is part of a tract of land partitioned primarily for
residential purposes into individual lots and offered to the public for sale. There is likewise no allegation that the tract of
land includes recreational areas and open spaces. Nor does the "Contract to Sell", which forms part of the complaint,
describe the subject property as a subdivision lot. What the contract strongly suggests is that the property is simply a lot
offered by respondents, as vendors, to the petitioners, as vendees, for sale on installment. As can be clearly gleaned from
the same contract, respondents are not acting as subdivision owners, developers, brokers or salesmen, nor are they
engaged in the real estate business. What is plain is that the parties are acting only as ordinary sellers and buyers of a
specific lot, a portion of a big tract of land co-owned by the heirs of Mariano Faraon. Neither are there undertakings specified
in the contract that respondents shall develop the land, like providing for the subdivision concrete roads and sidewalks,
street lights, curbs and gutters, underground drainage system, independent water system, landscaping, developed park,
and 24-hour security guard service. Even the rights and obligations of the sellers and buyers of a subdivision lot are not
provided in the agreement. All these provisions are usually contained in a standard contract involving a sale of a subdivision
lot.

82
WHEREFORE, the Resolutions of the Court of Appeals dated April 30, 1999 and June 9, 1999 in CA-G.R. SP No. 51833
are SET ASIDE. The Orders dated September 30, 1998 of the trial court denying petitioners' motion to dismiss and
December 28, 1998 denying their motion for reconsideration are hereby AFFIRMED.

SO ORDERED.

Puno, Callejo, Sr., Tinga and Chico-Nazario, JJ ., concur.

16. Hilado v. Chavez, 438 SCRA 623, 641 (2004)

SECOND DIVISION

[G.R. No. 134742. September 22, 2004.]

MELCHOR HILADO, CESAR ARAL, ADELA ARAL, ARTURO VILLARENA, TARCELO MIRANO, ROBERTO PEDUHAN,
ANTONIO SOLITO, MANUEL CANIENDO, FELIX ORTEGA, ANTONIO BALLENTOS, SALVADOR MIRANO, VICENTE
ONLAYAO, FEDERICO ORLANO, ROGELIO SEMILLANO, SALVADOR DE GUZMAN, PACIFICO TALIBUTAB, NESTOR
BELLIRAN, SALUSTIANO BELLIRAN, EDGARDO CABRA, and YOLANDA LESTINO, petitioners, vs. HON. ROLANDO
CHAVEZ, PERPETUAL HELP DEVELOPMENT AND REALTY CORP., represented by the Chairman of the Board
JULIETA C. SALGADO, respondents.

Entila & Entila Law Offices for petitioner.

Manalo Puno Jocson & Placido Law Offices and Vicente Garaygay, Jr. for private respondent.

SYNOPSIS

Petitioners were tenants of a large parcel of land owned by Celso Zayco. The said property was subsequently foreclosed
by the Pacific Banking Corporation for failure of Zayco to pay his loan. The bank then sold the said property to Julian C.
Salgado. In 1997, Salgado filed a complaint for unlawful detainer against the petitioners. She claimed that petitioners were
not agricultural tenants under the agrarian reform laws because they entered the property without his consent and did not
pay any consideration for the use of the land they occupied, and the property was partly for light industry and partly
residential as resolved by the Sangguniang Bayan. In their answer with motion to dismiss, the petitioners alleged that the
landholding had been long placed under Operation Land Transfer and that they became the owners thereof under
Presidential Decree No. 27. After due proceedings, the court a quo rendered judgment in favor of the respondent. The
petitioners filed a notice of appeal but was denied by the court a quo. Thus, the petitioners filed before the Regional Trial
Court (RTC) a petition for the annulment of the decision of the Municipal Trial Court (MTC) on the ground that the MTC had
no jurisdiction over the subject matter since it is an agrarian dispute and it is the Provincial Agrarian Reform Adjudicatory
Board that has exclusive jurisdiction over the action. The RTC, however, rendered judgment dismissing the petition on the
ground that the MTCC had exclusive jurisdiction over the action and over the persons of the defendants and petitioners
were estopped from assailing the jurisdiction of the MTCC since they failed to file a motion to dismiss the complaint and
even participated in the proceedings therein. Hence, this petition. TADcCS

In granting the petition, the Court ruled that the petitioners appended to their petition in the RTC a Certification of the
Register of Deeds indicating that thirteen (13) of the petitioners were issued transfer certificates of title based on the
Emancipation Patents filed with said office, made of record in the Primary Entry Book on September 16, 20, and 22, 1998;
and an LBP certificate stating that eighteen (18) of the petitioners had made advance payments for the portions of the
landholding occupied by them. It is evident from the face of the complaint and the pleadings of the parties and the
appendages thereof that the issue of possession of the subject property was inextricably interwoven with the issue of
whether the Emancipation Patents issued by the DAR to the petitioners were valid. Under the DAR Rules of Procedure,
the DARAB has primary and exclusive original jurisdiction over cases involving the issuance and cancellation of
Emancipation Patents. Moreover, the respondent claimed possession over the property based on TCT No. 133298, which
had already been partially cancelled by the Emancipation Patents and Torrens titles issued to the petitioners.

SYLLABUS

1. REMEDIAL LAW; CIVIL PROCEDURE; APPEAL BY WRIT OF ERROR; PROPER REMEDY OF A PARTY
AGGRIEVED BY THE DECISION OF THE REGIONAL TRIAL COURT, IN THE EXERCISE OF ITS ORIGINAL
JURISDICTION. — We agree with the respondent that the remedy of a party aggrieved by the decision of the RTC, in the
exercise of its original jurisdiction, is to appeal by writ of error to the Court of Appeals under Rule 41 of the Rules of Court,
in which questions of facts and/or of law may be raised by the parties. However, under Section 2(c), Rule 41 of the Rules
of Court, where only questions of law are raised or are involved, the appeal shall be to the Supreme Court by petition for
83
review on certiorari under Rule 45 of the Rules. However, even if only questions or issues are raised by the party in his
appeal, it should be made to the Court of Appeals and not to the Supreme Court, unless there are compelling reasons to
allow such appeal.

2. ID.; ID.; ID.; QUESTION OF LAW AND QUESTION OF FACT; DIFFERENTIATED. — In Reyes v. Court of
Appeals, we held that "for a question to be one of law, it must involve no examination of the probative value of the evidence
presented by the litigants or any one of them". In an avuncular case, we held that there is a question of law in a given case
when the doubt or difference arises as to what the law is pertaining to a certain set of facts, and there is a question of fact
when the doubt arises as to the truth or the falsity of alleged facts. ACTIHa

3. ID.; ID.; ACTIONS; AGGRIEVED PARTIES MAY AGAIN FILE SUITS WHERE THE VALIDITY OF THE DECISION
OF THE LOWER COURT IS ASSAILED FOR LACK OF JURISDICTION. — [U]nless the issues, which are cogent and
substantial, are resolved, in all likelihood, suits may again be filed by the aggrieved parties in suits involving landholdings
where the validity of the decision of the MTCC is assailed for lack of jurisdiction.

4. ID.; ID.; JURISDICTION; REGULAR COURTS HAVE NO ORIGINAL JURISDICTION TO DETERMINE AND
ABDICATE AGRARIAN DISPUTES. — On the issue of jurisdiction, Section 33, paragraph 2 of Batas Pambansa Blg. 129,
as amended by Section 3 of Rep. Act No. 7691 provides that Municipal Trial Court, Municipal Circuit Trial Court and
Metropolitan Trial Court, have exclusive original jurisdiction over cases for unlawful detainer. The proceedings in ejectment
cases are covered by Rule 70 of the Rules of Court and the Rules on Summary Procedure. However, such courts have no
original jurisdiction to determine and adjudicate agrarian disputes under Rep. Act No. 6657, as amended, and the Rules of
Procedure issued by the DARAB implementing said laws, which are within the exclusive original and appellate jurisdiction
of the DARAB. . . . The DAR is vested with primary jurisdiction to determine and adjudicate agrarian reform matters and
shall have exclusive jurisdiction over all matters involving the implementation of agrarian reform programs. The rule is that
the DARAB has jurisdiction to try and decide any agrarian dispute or any incident involving the implementation of the
Comprehensive Agrarian Reform Program. In Tirona v. Alejo, we held that the MTCC has no jurisdiction over an ejectment
case where the issue of possession is inextricably interwoven with an agrarian dispute.

5. ID.; ID.; ID.; DETERMINED BY THE MATERIAL ALLEGATIONS OF THE COMPLAINT AND THE LAW. — The
well-entrenched principle is that the jurisdiction of the court over the subject matter of the action is determined by the
material allegations of the complaint and the law, irrespective of whether or not the plaintiff is entitled to recover all or some
of the claims or reliefs sought therein. In Movers-Baseco Integrated Port Services, Inc. v. Cyborg Leasing Corporation, we
ruled that the jurisdiction of the court over the nature of the action and the subject matter thereof cannot be made to depend
upon the defenses set up in the court or upon a motion to dismiss for, otherwise, the question of jurisdiction would depend
almost entirely on the defendant. Once jurisdiction is vested, the same is retained up to the end of the litigation. We also
held in Arcelona v. Court of Appeals that in American jurisprudence, the nullity of a decision arising from lack of jurisdiction
may be determined from the record of the case, not necessarily from the face of the judgment only. HCEcaT

6. ID.; ID.; ID.; IF THE DEFENDANT IN THE EJECTMENT CASE ALLEGED TENANCY RELATIONSHIP BETWEEN
THE PARTIES AS A DEFENSE, THE LOWER COURT SHOULD RECEIVE EVIDENCE TO DETERMINE THE
EXISTENCE THEREOF. — The MTCC does not lose its jurisdiction over an ejectment case by the simple expedient of a
party raising as a defense therein the alleged existence of a tenancy relationship between the parties. But it is the duty of
the court to receive evidence to determine the allegations of tenancy. If after hearing, tenancy had in fact been shown to
be the real issue, the court should dismiss the case for lack of jurisdiction. Earlier in Bayog v. Natino, we held that if a
defendant in an action for ejectment interposed the defense of being the agricultural tenant in the property subject of the
complaint, the MTCC should hear and receive the evidence for the purpose of determining whether or not it possessed
jurisdiction over the case, and if, upon such hearing, tenancy is shown to be the issue, the MTCC should dismiss the case
for lack of jurisdiction. Our ruling in said case is a reiteration of our rulings in Ignacio v. CFI, and in Concepcion v. Presiding
Judge of CFI, Bulacan, Br. V.

7. POLITICAL LAW; ADMINISTRATIVE LAW; REPUBLIC ACT NO. 6657 (COMPREHENSIVE AGRARIAN
REFORM LAW); WHEN THE SUBJECT PROPERTY WAS COVERED BY AN EMANCIPATION PATENT, THE
DEPARTMENT OF AGRARIAN REFORM ADJUDICATION BOARD (DARAB) HAS PRIMARY AND ORIGINAL
JURISDICTION OVER THE ACTION. — In this case, even on the basis of the material allegations of the complaint, more
so if the answer with motion to dismiss the petition and position papers of the parties are considered, the DARAB, and not
the MTCC, had primary and original jurisdiction over the action of the respondent. The latter alleged, in its complaint, that
seven (7) of the petitioners were issued Emancipation Patents which were annotated at the dorsal portion of TCT No.
133298, a copy of which is appended to the complaint. . . . The foregoing annotation confirmed the claim of the petitioners
in their answer with motion to dismiss that the entirety of the landholding had been placed under the Operation Land
Transfer program under P.D. No. 27 and that the petitioners to whom the said patents were granted by the government
became the owners of the property covered by the said patents. In fact, TCT No. 133298 had been partially cancelled by
84
the said patents. Consequently, the petitioners who were the beneficiaries under the Emancipation Patents are entitled to
possess the property covered by said patents. SHTcDE

8. ID.; ID.; ID.; ID.; THE REGULAR COURT SHOULD HAVE DISMISSED THE COMPLAINT FOR LACK OF
JURISDICTION OR SHOULD HAVE PROCEEDED TO HEAR THE PARTIES ON THE MOTION TO DISMISS. — It bears
stressing that before Emancipation Patents are issued to farmers-beneficiaries, the DAR is mandated to comply with the
requirements of P.D. No. 266 and the procedural requirements set forth by Rep. Act No. 6657, otherwise known as the
Comprehensive Agrarian Reform Law (CARL) of 1988. Conformably to our ruling in Bayog v. Natino, the MTCC should not
have applied the Rules on Summary Procedure; it should have dismissed the complaint for lack of jurisdiction; or, at the
very least, should have proceeded to hear the parties on the petitioners' motion to dismiss and receive their respective
evidence on the issue of whether or not it had jurisdiction over the subject matter of the action. Had the MTCC followed our
ruling in Bayog, it would have confirmed that, before the respondent filed its complaint, the property had long been brought
under Operation Land Transfer and that the Register of Deeds had issued to all the petitioners their respective transfer
certificates of title based on the Emancipation Patents issued by the President of the Philippines, through the DAR.

9. ID.; ID.; ID.; VALIDITY AND EFFICACY OF THE EMANCIPATION PATENTS WERE NOT NEGATED BY THE
REGISTER OF DEEDS' DELAY IN THE ANNOTATION THEREOF. — The validity and efficacy of the Emancipation
Patents were not negated by the Register of Deeds' delay in the annotation thereof at the dorsal portion of TCT No. 133298.
As certified by the Register of Deeds, the failure to make the annotations in the . . . patents earlier were inadvertent[.]

10. ID.; ID.; ID.; CONVERSION OF LANDS; AGRICULTURAL LANDS MAY BE RECLASSIFIED ONLY BY THE
DEPARTMENT OF AGRARIAN REFORM (DAR) AFTER THE LAPSE OF FIVE YEARS FROM ITS AWARD TO THE
FARMERS-BENEFICIARIES. — Under Section 65 of Rep. Act No. 6657 which took effect on June 15, 1988, agricultural
lands may be reclassified only by the DAR after the lapse of five (5) years from its award to the farmers-beneficiaries. . . .
In this case, there is no showing that the DAR ever approved the reclassification of the property. It appears that the
reclassification of the landholding was unilaterally made by the Sangguniang Bayan despite the issuance to the petitioners
of Emancipation Patents and transfer certificates of title in their names over the portions of the landholdings respectively
occupied by them. ACcHIa

11. ID.; ID.; ID.; DEPARTMENT OF AGRARIAN REFORM ADJUDICATION BOARD HAS PRIMARY AND
EXCLUSIVE ORIGINAL JURISDICTION OVER CASES INVOLVING THE ISSUANCE AND CANCELLATION OF
EMANCIPATION PATENTS. — The petitioners appended to their petition in the RTC a Certification of the Register of
Deeds indicating that thirteen (13) of the petitioners were issued transfer certificates of title based on the Emancipation
Patents filed with said office, made of record in the Primary Entry Book on September 16, 20, and 22, 1998; and an LBP
certificate stating that eighteen (18) of the petitioners had made advance payments for the portions of the landholding
occupied by them. And yet, the RTC dismissed the petition and affirmed the ruling of the MTCC that it had jurisdiction over
the subject matter of the complaint. It is evident from the face of the complaint and the pleadings of the parties and the
appendages thereof that the issue of possession of the subject property was inextricably interwoven with the issue of
whether the Emancipation Patents issued by the DAR to the petitioners were valid. Under the DAR Rules of Procedure,
the DARAB has primary and exclusive original jurisdiction over cases involving the issuance and cancellation of
Emancipation Patents. Moreover, the respondent claimed possession over the property based on TCT No. 133298, which
had already been partially cancelled by the Emancipation Patents and Torrens titles issued to the petitioners.

12. REMEDIAL LAW; CIVIL PROCEDURE; JUDGMENT; WANT OF JURISDICTION OVER THE SUBJECT MATTER
RENDERS THE COURT'S JUDGMENT VOID. — As we held in Arevalo v. Benedicto: [F]urthermore, the want of jurisdiction
by a court over the subject-matter renders its judgment void and a mere nullity, and considering that a void judgment is in
legal effect no judgment, by which no rights are divested, from which no rights can be obtained, which neither binds nor
bars any one, and under which all acts performed and all claims flowing out of are void, and considering further, that the
decision, for want of jurisdiction of the court, is not a decision in contemplation of law, and, hence, can never become
executory, it follows that such a void judgment cannot constitute a bar to another case by reason of res judicata. Our ruling
in Abbain v. Chua is also instructive: In varying language, this Court has expressed its reprobation for judgments rendered
by a court without jurisdiction. Such a judgment is held to be "a dead limb on the judicial tree, which should be lopped off
or wholly disregarded as the circumstances require". In the language of Mr. Justice Street: "Where a judgment or judicial
order is void in this sense it may be said to be a lawless thing, which can be treated as an outlaw and slain at sight, or
ignored wherever and whenever it exhibits its head." And in Gomez vs. Concepcion, this Court quoted with approval the
following from Freeman on Judgments: "A void judgment is in legal effect no judgment. By it no rights are divested. From
it no rights can be obtained. Being worthless in itself, all proceedings found upon it are equally worthless. It neither binds
nor bars any one. All acts performed under it and all claims flowing out of it are void. The parties attempting to enforce it
may be responsible as trespassers. The purchaser at a sale by virtue of its authority finds himself without title and without
redress." Since the judgment here on its face is void ab initio, the limited periods for relief from judgment in Rule 38 are

85
inapplicable. That judgment is vulnerable to attack "in any way and at any time, even when no appeal has been taken".
IcADSE

13. ID.; ID.; ID.; JURISDICTION OVER THE JUDGMENT CANNOT BE CHANGED BY AGREEMENT OF THE
PARTIES OR BY THE ACT OR OMISSION OF EACH OF THEM THAT CONTRAVENE THE LEGISLATIVE INTENT. —
It is settled that jurisdiction over the judgment cannot be changed by agreement of the parties or by the act or omission of
each of them that will contravene the legislative will. A party should not be allowed to divest a competent court of its
jurisdiction, whether erroneously or even deliberately in derogation of the law. In this case, the counsel of the petitioners
opted to assail in a direct action the decision of the MTCC, instead of perfecting their appeal or assailing the decision of
the MTCC disallowing their appeal. The petitioners believed that the decision of the MTCC was null and void for want of
jurisdiction over the subject matter of the action filed therein; hence, they are not proscribed from assailing such decision
in a direct action. The remedy resorted to by their counsel should not prejudice and bar them from assailing the MTCC
decision before the RTC on a petition to annul the same for lack of jurisdiction. Neither are they estopped from assailing
the decision, simply because they filed their answer and motion to dismiss the complaint on the ground of lack of jurisdiction
over the subject matter of the action. After all, the only relief prayed for by them in their answer was the dismissal of the
complaint.

14. ID.; ID.; JUSTNESS OF THE RELIEF SOUGHT BY THE PARTIES MAY NOT BE IGNORED OR RENDERED
FUTILE BY REASON OF A DOCTRINE WHICH IS OF HIGHLY DOUBTFUL APPLICABILITY. — It bears stressing that
the petitioners are now the registered owners of the portions of the landholding and entitled to the possession thereof. For
us to deny the petition and affirm the decision of the RTC would be to sanction the eviction of the petitioners who are the
registered owners of the landholding and, as such, are entitled to the possession thereof and allow the respondent to take
possession thereof in derogation of law. Not too long ago in Calimlim v. Ramirez we held that: The inequity of barring the
petitioners from vindicating their right over their property in Civil Case No. SCC-180 is rendered more acute in the face of
the undisputed fact that the property in question admittedly belonged to the petitioners, and that the title in the name of the
private respondent was the result of an error committed by the Provincial Sheriff in issuing the deed of sale in the execution
proceeding. The justness of the relief sought by herein petitioners may not be ignored or rendered futile by reason of a
doctrine which is of highly doubtful applicability herein. HAaDcS

DECISION

CALLEJO, SR., J p:

Before us is a petition for review on certiorari under Rule 45 of the 1997 Rules of Civil Procedure, as amended, of the
Decision 1 and the Resolution 2 of the Regional Trial Court (RTC) of Kabankalan City, Branch 61, in Civil Case No. 830.
ATcaHS

The Antecedents

Celso "Nene" Zayco was the owner of a large parcel of agricultural land with an area of 540,248 square meters, located in
Kabankalan Poblacion, Negros Occidental, now Kabankalan City. The property was identified as Lot No. 343 and was
covered by Transfer Certificate of Title (TCT) No. 133298, and portions thereof were occupied and cultivated by tenants.
Zayco mortgaged the property to the Pacific Banking Corporation as security for a loan; however, the bank foreclosed the
mortgage upon Zayco's failure to pay his account. When the property was sold at public auction by the sheriff, the bank
was adjudged as the highest bidder. Zayco failed to redeem the property, and the bank consolidated its title thereon; TCT
No. 115264 was issued in its favor on March 20, 1980. 3

On December 21, 1984, the bank sold the property to Julieta C. Salgado, the Chairman of the Board of the respondent,
Perpetual Help Development and Realty Corporation (PHDRC). TCT No. 133298 was, thereafter, issued in favor of PHDRC
on January 18, 1985. No liens or encumbrances whatsoever or any notice that the property had been placed under the
agrarian reform laws were annotated at the dorsal portion thereof. 4

Subsequently, the Department of Agrarian Reform (DAR) granted Emancipation Patents to the twenty (20) tenants on the
property from April 28, 1988 to July 1, 1988 on the basis of which titles were issued in their favor during the period of
September 16, 1988 to August 24, 1990. 5

The foregoing notwithstanding, the Sangguniang Bayan ng Kabankalan approved, on February 14, 1996, Resolution No.
96-39, reclassifying the property partly as property for light industry, and the rest as residential. 6

On August 26, 1997, the respondent filed a complaint for unlawful detainer against the twenty (20) petitioners, who were
all occupants-farmers on the property, with the Municipal Trial Court in Cities (MTCC) of Kabankalan City, docketed as Civil
Case No. 034-97. AIHDcC

86
The respondent alleged, inter alia, in its complaint that on May 27, 1997, it obtained a certified xerox copy of TCT No.
133298 from the Register of Deeds and discovered that of the twenty (20) petitioners, seven (7) had been issued
Emancipation Patents on July 1, 1988 which were inscribed at the dorsal portion of said title. Nevertheless, according to
the respondent, the petitioners were not agricultural tenants under the agrarian reform laws because (a) they entered the
property without its consent and did not pay any consideration for the use of the land they occupied; and (b) the property
was, as resolved by the Sangguniang Bayan under Resolution No. 96-39 in 1996, partly for light industry and partly
residential.

The respondent prayed that, after due proceedings, judgment be rendered in its favor, thus:

WHEREFORE, premises considered, it is respectfully prayed of the Honorable Court that, after due notice and hearing,
judgment be rendered in favor of plaintiff and against defendants as follows:

1. Ordering defendants to vacate the areas they respectively occupy in Lot No. 343 and to return the same to plaintiff;

2. Ordering defendants to pay to plaintiff P10,000.00 as litigation expenses; P50,000.00 as attorney's fees, plus
P1,000.00 for every court appearance and P20,000.00 as exemplary damages.

Plaintiff prays for such other reliefs and remedies just and equitable in the premises. 7

Appended to the complaint was a photocopy of TCT No. 133298. EaCDAT

In their answer with motion to dismiss the complaint, the petitioners, who were represented by DAR lawyer Atty. Quirico
Infante, alleged that the landholding had long been placed under Operation Land Transfer, and that they became the
owners thereof under Presidential Decree No. 27. They interposed the following special and affirmative defenses:

12. That the landholding, subject matter of the case more particularly described as Lot No. 343, has been placed within
the ambit of the Operation Land Transfer program per P.D. 27 as amended;

13. That defendants herein are actual-occupants and tenant-tillers of the land wayback in [the] 1970's, having been
installed thereat by Lorenzo Zayco, the original landowner with the defendants sharing [with] the landowner the produce of
the land up to 1981 as evidenced by receipts which will be presented in due time;

14. That being bonafide tenant-tillers thereat, they were identified by the Department of Agrarian Reform as qualified
farmers-beneficiaries and consequently were issued Emancipation Patents;

15. That in 1982-83, the subject landholding was conveyed by way of sale by the former landowner to J. Salgado;

16. That despite the sale thereof to the plaintiff herein, defendants continued to cultivate the land and extend payment
of lease rentals to the new landowner thereof;

17. That in 1988-1989, being beneficiaries of the program, caused to directly pay their amortization to the Land Bank
of the Philippines (LBP);

18. That defendants, having been issued EPs, are therefore considered owners of the land and now deemed full
owners of the land they till as provided for under P.D. 27 and E.O. 328 respectively, thus, cannot be ejected/ousted
therefrom without circumventing their right to [s]ecurity of tenure as amplified in the case of De Jesus vs. IAC, SCRA 559
(sic) and Dolorfino vs. CA, 191 SCRA 880, Dec. 3, 1990, where the Highest Tribunal ruled that:

"Once a leasehold relation has been established, the agricultural lessee is entitled to security of tenure. He has a right to
continue working on the land and he may not be ejected therefrom except for cause as provided by law. AcCTaD

The agricultural relationship is not extinguished by the sale, alienation or transfer of the legal possession of the landholding.
The purchaser or transferee is simply subjugated to the rights and substituted to the obligations of the agricultural lessor."
(Sec. 10, R.A. 3844) (Dolorfino vs. Court of Appeals, supra).

19. That granting that the property in question has already been classified as residential, commercial and industrial
zone per Res. No. 96-39, dated February 14, 1996, however, the landowner has failed to present a Conversion Order to
be issued by the DAR Secretary, thus, in absence thereof, the subject landholding remains agricultural, in the light of A.O.
12, Series of 1994 the same provides and we quote:

"II. LEGAL MANDATE

87
A. The Department of Agrarian Reform (DAR) is mandated to "approved (sic) or disapproved (sic) applications for
conversion, restructuring or readjustment of agricultural lands into non-agricultural uses," pursuant to Section 4(j) of
Executive Order No. 129-A, Series of 1987.

B. Section 5(1) of E.O. No. 129-A, Series of 1987, vests in the DAR, exclusive authority to approve or disapprove
applications for conversion of agricultural lands for residential, commercial, industrial and other land uses." 8

The petitioners prayed that the complaint be dismissed for lack of jurisdiction over the subject matter of the action. aEcADH

The court a quo applied the Rules of Summary Procedure. Instead of ruling on the motion to dismiss, it ordered the parties
to file their position papers. The petitioners appended to their position paper, as Annexes "1" to "36," photocopies of sample
receipts purportedly signed by Celso Zayco and Julieta Salgado, acknowledging receipt of their respective shares in the
produce of the landholding. 9

On December 18, 1997, after due proceedings, the court a quo rendered judgment in favor of the respondent. The decretal
portion of the decision reads:

WHEREFORE and upon the foregoing disquisitions, the Court hereby renders judgment in favor of the plaintiff and against
the defendants, to wit:

1. Ordering the named defendants to vacate the portions they had been occupying of Lot No. 343, situated in Sitio
Lapui, Barangay Hilamanan, Kabankalan City, and to turn-over the possession thereof to the plaintiff; and

2. Ordering the DISMISSAL of plaintiff's claims for litigation expenses, attorney's fees and exemplary damages.

SO ORDERED. 10

The court a quo ruled that the petitioners failed to prove that they were farmers-beneficiaries on the landholding and that
based on Resolution No. 96-39 of the Municipal Council, the said property had already been reclassified as part residential
and part industrial/commercial areas. The court a quo also ruled that thirteen (13) of the petitioners occupied portions of
the landholding only by tolerance of the respondent and its predecessors, and failed to pay any amount as consideration
for their occupancy of the petitioners' property. It rejected the petitioners' contention that the Department of Agrarian Reform
Adjudication Board (DARAB) had exclusive original jurisdiction over the subject matter of the action, ruling that the action
was one for unlawful detainer over which it had exclusive original jurisdiction. EScaIT

Aggrieved, the petitioners filed a notice of appeal dated January 19, 1998 on the ground that grave errors were committed
by the court a quo in its findings of facts and conclusions of law in its decision. 11

On February 6, 1998, a motion to disapprove the notice of appeal and for execution of final judgment was filed by the
respondent on its claim that the required appellate docket and other lawful fees had not been paid to the clerk of court
within the reglementary period therefor. The court a quo issued an Order on March 31, 1998, granting the motion of the
respondent and disapproved the notice of appeal filed by the petitioners. 12 It also ordered the issuance of a writ of
execution on its finding that its decision had become final and executory, following the failure of the petitioners to perfect
their appeal to the RTC. On April 2, 1998 and April 21, 1998, writs of execution were issued by the MTCC.

The petitioners did not assail the order of the MTCC. Instead, the petitioners filed on April 23, 1998 a petition with the
Regional Trial Court against the respondent for the annulment of the decision of the MTC in Civil Case No. 034-97. TaEIcS

The petitioners alleged, inter alia, that they were agricultural tenants of the late Cesar Zayco as evidenced by the receipts
signed by him, where he acknowledged receipt of their rentals over the portions of the landholding tenanted by them,
respectively, and the affidavit of Lorenzo Zayco, the son of Cesar Zayco, appended to the petition as Annex "J" thereof.
They also alleged, thus:

2. That subject landholding has the following farmers-beneficiaries, whose respective farmholdings were identified
under [the] Operation Land Transfer (OLT) program of the government pursuant to PD 27 as follows:

1. Melchor Hilado 11. Federico Orlano

2. Cesar Aral 12. Felix Ortega

3. Adela Aral 13. Rogelio Semillano

4. Arturo Villarena 14. Antonio Ballentos

5. Salvador Mirano 15. Pacifico Talibutab


88
6. Tarcelo Mirano 16. Nestor Belliran

7. Roberto Pedulan 17. Salustiano Belliran

8. Antonio Solito 18. Salvador de Guzman

9. Vicente Onlayao 19. Edgardo Cabra

10. Manuel Caniendo 20. Yolanda Lestino

3. That when my father Celso "Nene" Zayco, during his lifetime, administered subject landholding, he received yearly
rental consideration in the amount fixed by humber (sic) of cavans of palay, from the above-named farmer-beneficiaries. In
fact, when my father was already sickly I was assigned to collect yearly rentals from said farmers-beneficiaries;

4. That in 1981, unfortunately, the Pacific Banking Corporation foreclosed said farmholding of my late father and
before the Zayco family knew it, it was already purchased by Julieta C. Salgado, the latter likewise, as far as my knowledge
is concerned, also collected yearly rental consideration from the above-named parties. 13

The petitioners also averred that, after she purchased the property in 1981, Julieta Salgado received rentals over the
landholding from them, as evidenced by the receipts 14 signed by her. They averred that in 1988, Emancipation Patents
15 over their respective landholdings were issued in their favor. The petitioners also claimed that they had made partial
payments to the Land Bank of the Philippines (LBP) for the price of the lots covered by their respective patents, as
evidenced by the certification attached as Annex "BBB" of the petition. They also appended a Certification by the Register
of Deeds that thirteen (13) of them were issued Transfer Certificates of Title based on the Emancipation Patents executed
in their favor, viz:

EP TITLE NO. LOT NO. NAME OF FARMERS AREA/SQ.M.

EP-1716 343-9 Pacifico B. Talibutab 8,735

EP-1717 343-15 Felix S. Ortega 8,106

EP-1718 343-22 Roberto D. Peduhan 7,779

EP-1719 343-25 Arturo T. Villarena 8,346

EP-1720 343-19 Vicente C. Onlayao 7,709

EP-1722 343-11 Antonio E. Ballentos 9,066

EP-1723 343-29 Cesar C. Aral 8,485

EP-1724 343-18 Manuel P. Caniendo 10,110

EP-1725 343-24 Salvador G. Mirano 8,215

EP-1740 343-8 Salustiano P. Billeran 23,391

EP-1751 343-16 Federico L. Orlano 10,453

EP-1754 343-14 Rogelio U. Semillano 7,668

EP-1813 343-23 Tarcelo S. Mirano 7,920 16

The petitioners asserted that the MTC had no jurisdiction over the subject matter of the action of the respondent in Civil
Case No. 034-97, it being an agrarian dispute between the petitioners, as patentees, and the respondent; hence, the court
a quo's decision was null and void. They contended that the Provincial Agrarian Reform Adjudicatory Board (PARAD) had
exclusive jurisdiction over the action in Civil Case No. 034-97. DEIHSa

The RTC found the petition sufficient in form and substance and directed the respondent to file its comment on or answer
to the petition. 17

In its verified answer to the petition, the respondent averred that the receipts purportedly signed by Julieta C. Salgado were
spurious. It also asserted that the petitioners failed to submit the said receipts to the court a quo and, as such, they were
barred from submitting the same before the RTC. The respondent also countered that the petitioners participated in the
proceedings in the MTCC and were, thus, estopped from assailing the jurisdiction of the court a quo. It posited that the
petitioners were not entitled to injunctive relief because the decision of the MTC had become final and executory.
89
On April 29, 1998, the RTC issued an Order declaring that the case involved only questions of law and not of facts, and
ordered the parties to file their respective memoranda. 18 On May 26, 1998, the RTC rendered judgment dismissing the
petition on the ground that the MTCC had exclusive jurisdiction over the action of the plaintiff in Civil Case No. 034-97 and
over the persons of the defendants therein. 19 The RTC also held that the petitioners failed to file a motion to dismiss the
complaint in the MTCC and even participated in the proceedings therein; hence, they were estopped from assailing the
jurisdiction of the MTCC. The petitioners filed a motion for reconsideration of the decision, but on June 26, 1998, the RTC
issued an order denying the same.

The Present Petition

Instead of appealing the decision to the Court of Appeals by writ of error, the petitioners filed their petition with this Court,
under Rule 45 of the Rules of Court, as amended, assailing the decision of the RTC on questions of law. They appended
to their petition, as Annexes "Q" to "JJ" thereof, certified true copies of the transfer certificates of title issued to each of them
during the period of September 16, 1988 to August 24, 1990 by the Register of Deeds based on Emancipation Patents
executed by the President of the Philippines, through the Secretary of Agrarian Reform, to prove that long before the
respondent filed its complaint with the MTCC, the Register of Deeds had issued such titles to each of them, thus:

OWNER EMANCIPATION DATE TCT DATE

PATENT OF APPROVAL NUMBER OF ISSUE

Federico L. Orlano EP-1751 April 28, 1988 A-192817 September 20, 1988

Ma. Yolanda S. Lestino EP-5656 July 1, 1988 A-192802 August 24, 1990

Felix S. Ortega EP-1717 April 28, 1988 A-192816 September 16, 1988

Melchor T. Hilado EP-5139 July 1, 1988 A-192831 March 21, 1990

Antonio D. Solito EP-5414 July 1, 1988 A-192821 July 12, 1990

Arturo T. Vellarena EP-1719 April 28, 1988 A-192826 September 16, 1988

Antonio E. Ballentos EP-1722 April 28, 1988 A-192812 September 16, 1988

Salvador J. De Guzman EP-5415 July 1, 1988 A-192808 July 12, 1990

Rogelio U. Semellano EP-1754 April 28, 1988 A-192815 September 20, 1988

Salustiano P. Billeran EP-1740 April 28, 1988 A-192809 September 20, 1988

Vicente Onlayao EP-1720 April 28, 1988 A-192820 September 16, 1988

Salvador G. Mirano EP-1725 April 28, 1988 A-192825 September 16, 1988

Nestor P. Billeran EP-5416 July 1, 1988 A-192804 July 12, 1990

Tarcelo S. Mirano EP-1813 April 28, 1988 A-192824 September 22, 1988

Pacifico P. Talibutab EP-1716 April 28, 1988 A-192810 September 16, 1988

Edgardo D. Cabra EP-5417 July 1, 1988 A-192807 July 12, 1990

Manuel P. Caniendo EP-1724 April 28, 1988 A-192819 September 16, 1988

Adela O. Aral EP-5657 July 1, 1988 A-192827 August 24, 1990

Roberto D. Peduhan EP-1718 April 28, 1988 A-192823 September 16, 1988

Cesar C. Aral EP-1723 April 28, 1988 A-192830 September 16, 1988

The issues for resolution are the following:

1. Whether it is proper for the petitioners to file a petition for review under Rule 45 of the Rules of Court with this
Court from the decision of the RTC;

2. Whether the MTCC had exclusive jurisdiction over the action of the respondent; and,

90
3. Whether the decision of the MTCC is null and void

On the first issue, the petitioners assert that, instead of appealing the decision of the RTC to the Court of Appeals, they
filed their petition under Rule 45 of the Rules of Court because the issues raised by them are only legal issues. They aver
that the RTC erred when it declared that instead of filing their answer to the respondent's complaint in the MTCC,
participating in the proceedings and praying for reliefs therein, the petitioners as defendants in said case, should have filed
a motion to dismiss the complaint. The petitioners posit that they could not have filed such motion because the proceedings
in ejectment cases are summary in nature and such motion to dismiss the complaint is a prohibited pleading. The petitioners
point out that they incorporated in their answer to the complaint a motion to dismiss the complaint for lack of jurisdiction
over the subject matter of the action which should be considered as a motion for a hearing on their affirmative defenses.
They also aver that the action filed before the MTCC is an agrarian case involving agricultural land placed under Operation
Land Transfer, for which they were issued Emancipation Patents by the Secretary of Agrarian Reform even before such
complaint for ejectment was filed against them. They note that the issue involved the validity of the Emancipation Patents
issued to them, and was decisive of the issue of jurisdiction in the MTCC; hence, the MTCC had no original jurisdiction over
the action of the respondent, conformably to Republic Act No. 6657, as amended, and the DARAB Rules of Procedure.
The petitioners assert that by virtue of Presidential Decree (P.D.) No. 27 and the Emancipation Patents issued to them,
they became owners of the property and were entitled to the possession thereof. ACaEcH

For its part, the respondent asserts that, if at all, the RTC decision is appealable, the proper remedy of the petitioners from
the said decision was to appeal, by writ of error, to the Court of Appeals under Rule 41 of the Rules of Court, as amended,
and not via a petition for review on certiorari to this Court under Rule 45 of the said Rules. It contends that, as gleaned from
the petition, the petitioners raised factual issues. It notes that the petitioners assailed the factual findings made by the
MTCC that they (the petitioners) are not agricultural tenants of the late Cesar Zayco and Julieta C. Salgado, and that the
subject landholding is residential and not agricultural.

In reply, the petitioners contend that this appeal via Rule 45 of the Rules of Court is proper, considering that they raised
purely legal issues in their petition. They note that the RTC itself, per its Order on April 29, 1998, succinctly states that the
issue raised by the parties is one of law, namely, whether the MTCC had jurisdiction over the action of the respondent
against the petitioners. They point out that the RTC required the parties to merely file their respective memoranda, instead
of adducing evidence in their favor.

We agree with the respondent that the remedy of a party aggrieved by the decision of the RTC, in the exercise of its original
jurisdiction, is to appeal by writ of error to the Court of Appeals under Rule 41 20 of the Rules of Court, in which questions
of facts and/or of law may be raised by the parties. However, under Section 2(c), 21 Rule 41 of the Rules of Court, where
only questions of law are raised or are involved, the appeal shall be to the Supreme Court by petition for review on certiorari
under Rule 45 of the Rules. However, even if only questions or issues are raised by the party in his appeal, it should be
made to the Court of Appeals and not to the Supreme Court, unless there are compelling reasons to allow such appeal.

In Reyes v. Court of Appeals, 22 we held that "for a question to be one of law, it must involve no examination of the probative
value of the evidence presented by the litigants or any one of them." In an avuncular case, we held that there is a question
of law in a given case when the doubt or difference arises as to what the law is pertaining to a certain set of facts, and there
is a question of fact when the doubt arises as to the truth or the falsity of alleged facts. 23

We agree with the petitioners and the RTC that the issues before it were legal: (1) whether the MTCC had properly exercised
jurisdiction over the subject matter of the action of the respondent based on the material allegations of said complaint, as
well as the relevant pleadings of the parties in said case; and (2) whether the petitioners were estopped from assailing the
decision of the MTCC on the ground of lack of jurisdiction. DTIACH

We resolved to give due course to the petition. Indeed, unless the issues, which are cogent and substantial, are resolved,
in all likelihood, suits may again be filed by the aggrieved parties in suits involving landholdings where the validity of the
decision of the MTCC is assailed for lack of jurisdiction.

On the issue of jurisdiction, Section 33, paragraph 2 of Batas Pambansa Blg. 129, as amended by Section 3 of Rep. Act
No. 7691 provides that Municipal Trial Court, Municipal Circuit Trial Court and Metropolitan Trial Court, have exclusive
original jurisdiction over cases for unlawful detainer. The proceedings in ejectment cases are covered by Rule 70 of the
Rules of Court and the Rules on Summary Procedure. However, such courts have no original jurisdiction to determine and
adjudicate agrarian disputes under Rep. Act No. 6657, as amended, and the Rules of Procedure issued by the DARAB
implementing said laws, which are within the exclusive original and appellate jurisdiction of the DARAB, thus:

Section 1. Primary And Exclusive Original and Appellate Jurisdiction. — The Board shall have primary and exclusive
jurisdiction, both original and appellate, to determine and adjudicate all agrarian disputes involving the implementation of
the Comprehensive Agrarian Reform Program (CARP) under Republic Act No. 6657, Executive Order Nos. 228, and 129-
91
A, Republic Act No. 3844 as amended by Republic Act No. 6389, Presidential Decree No. 27 and other agrarian laws and
their implementing rules and regulations. Specifically, such jurisdiction shall include but not be limited to cases involving
the following:

a) The rights and obligations of persons, whether natural or juridical, engaged in the management, cultivation and
use of all agricultural lands covered by the CARP and other agrarian laws;

xxx xxx xxx

f) Those involving the issuance, correction and cancellation of Certificates of Land Ownership Award (CLOAs) and
Emancipation Patents (EPs) which are registered with the Land Registration Authority;

g) Those cases previously falling under the original and exclusive jurisdiction of the defunct Court of Agrarian
Relations under Section 12 of Presidential Decree No. 946, except sub-paragraph (Q) thereof and Presidential Decree No.
1815. CHEDAc

It is understood that the aforementioned cases, complaints or petitions were filed with the DARAB after August 29, 1987.

Matters involving strictly the administrative implementation of Republic Act No. 6657, otherwise known as the
Comprehensive Agrarian Reform Law (sic) (CARP) of 1988 and other agrarian laws as enunciated by pertinent rules shall
be the exclusive prerogative of and cognizable by the Secretary of the DAR.

h) And such other agrarian cases, disputes, matters or concerns referred to it by the Secretary of the DAR.

The DAR is vested with primary jurisdiction to determine and adjudicate agrarian reform matters and shall have exclusive
jurisdiction over all matters involving the implementation of agrarian reform programs. The rule is that the DARAB has
jurisdiction to try and decide any agrarian dispute or any incident involving the implementation of the Comprehensive
Agrarian Reform Program. 24 In Tirona v. Alejo, 25 we held that the MTCC has no jurisdiction over an ejectment case
where the issue of possession is inextricably interwoven with an agrarian dispute.

The well-entrenched principle is that the jurisdiction of the court over the subject matter of the action is determined by the
material allegations of the complaint and the law, irrespective of whether or not the plaintiff is entitled to recover all or some
of the claims or reliefs sought therein. 26 In Movers-Baseco Integrated Port Services, Inc. v. Cyborg Leasing Corporation,
27 we ruled that the jurisdiction of the court over the nature of the action and the subject matter thereof cannot be made to
depend upon the defenses set up in the court or upon a motion to dismiss for, otherwise, the question of jurisdiction would
depend almost entirely on the defendant. 28 Once jurisdiction is vested, the same is retained up to the end of the litigation.
We also held in Arcelona v. Court of Appeals 29 that in American jurisprudence, the nullity of a decision arising from lack
of jurisdiction may be determined from the record of the case, not necessarily from the face of the judgment only. IDcHCS

The MTCC does not lose its jurisdiction over an ejectment case by the simple expedient of a party raising as a defense
therein the alleged existence of a tenancy relationship between the parties. 30 But it is the duty of the court to receive
evidence to determine the allegations of tenancy. 31 If after hearing, tenancy had in fact been shown to be the real issue,
the court should dismiss the case for lack of jurisdiction. 32

Earlier in Bayog v. Natino, 33 we held that if a defendant in an action for ejectment interposed the defense of being the
agricultural tenant in the property subject of the complaint, the MTCC should hear and receive the evidence for the purpose
of determining whether or not it possessed jurisdiction over the case, and if, upon such hearing, tenancy is shown to be the
issue, the MTCC should dismiss the case for lack of jurisdiction. Our ruling in said case is a reiteration of our rulings in
Ignacio v. CFI, 34 and in Concepcion v. Presiding Judge of CFI, Bulacan, Br. V . 35

In this case, even on the basis of the material allegations of the complaint, more so if the answer with motion to dismiss the
petition and position papers of the parties are considered, the DARAB, and not the MTCC, had primary and original
jurisdiction over the action of the respondent. The latter alleged, in its complaint, that seven (7) of the petitioners were
issued Emancipation Patents which were annotated at the dorsal portion of TCT No. 133298, a copy of which is appended
to the complaint. Indeed, the title contains the following annotations:

ENTRY NO. EP LOT NO. AREA/SQ.M NAME OF FARMER

EP-1539 343-30 8,597 Melchor T. Hilado

The property described in this Title has been partially cancelled Emancipation Patent Issued By Department of Agrarian
Reform, containing an Area as stated to above.

Date of Instrument July 1, 1988.


92
Date of Inscription March 21, 1990.

(Sgd.) Illegible

Register of Deeds

ENTRY NO. EP LOT NO. AREA/SQ.M NAME OF FARMERS

EP-5414 343-20 7,232 Antonio D. Solito

EP-5415 343-7 7,518 Salvador J. de Guzman

EP-5416 343-3 6,531 Nestor P. Billeran

EP-5417 343-6 14,529 Edgardo D. Cabra

The property described in this Transfer Certificate of Title has been PARTIALLY CANCELLED by Emancipation Patent
issued by Department of Agrarian Reform containing an area of 35,810 SQ/M. as stated above.

Date of Instrument July 1, 1988. Date of Inscription July 12, 1990.

(Sgd.) Illegible

Register of Deeds

ENTRY NO. EP LOT NO. AREA/SQ.M NAME OF FARMERS

EP-5656 343-1 14,916 Maria Yolanda S. Lestino

EP-5657 343-26 9,558 Adela O. Aral

The property described in this Transfer Certificate of Title has been PARTIALLY CANCELLED by Emancipation Patent
issued by the Department of Agrarian Reform containing an area of TWENTY-FOUR THOUSAND FOUR HUNDRED
SEVENTY-FOUR (24,474) SQ. METERS as stated to above. DHESca

Date of Instrument July 1, 1988.

Date of Inscription August 24, 1990.

(Sgd.) Illegible

Register of Deeds 36

The foregoing annotation confirmed the claim of the petitioners in their answer with motion to dismiss that the entirety of
the landholding had been placed under the Operation Land Transfer program under P.D. No. 27 and that the petitioners to
whom the said patents were granted by the government became the owners of the property covered by the said patents.
In fact, TCT No. 133298 had been partially cancelled by the said patents. Consequently, the petitioners who were the
beneficiaries under the Emancipation Patents are entitled to possess the property covered by said patents. 37

It bears stressing that before Emancipation Patents are issued to farmers-beneficiaries, the DAR is mandated to comply
with the requirements of P.D. No. 266 and the procedural requirements set forth by Rep. Act No. 6657, otherwise known
as the Comprehensive Agrarian Reform Law (CARL) of 1988. 38 Conformably to our ruling in Bayog v. Natino, 39 the
MTCC should not have applied the Rules on Summary Procedure; it should have dismissed the complaint for lack of
jurisdiction; or, at the very least, should have proceeded to hear the parties on the petitioners' motion to dismiss and receive
their respective evidence on the issue of whether or not it had jurisdiction over the subject matter of the action. Had the
MTCC followed our ruling in Bayog, it would have confirmed that, before the respondent filed its complaint, the property
had long been brought under Operation Land Transfer and that the Register of Deeds had issued to all the petitioners their
respective transfer certificates of title based on the Emancipation Patents issued by the President of the Philippines, through
the DAR. However, the MTCC rendered judgment against the petitioners and ordered their eviction on the following findings:

Firstly, they asseverate that they had been giving the original landowner Lorenzo Zayco (but the owner was Celso Zayco)
his shares of the produce from 191970 (sic) to 1981 and that in 1982-83, they had been paying the lease rentals to the new
landowner Julieta C. Salgado. Obviously, defendants predicate their tenurial rights on their perceived landowner Lorenzo
Zayco (sic) and that therefore, the new landowner Julieta C. Salgado has assumed her liabilities to them as her alleged
tenants. On this point, there is not a shred of evidence proving that either Lorenzo Zayco or Julieta C. Salgado had ever
received their respective shares of the harvests. aAcDSC
93
Secondly, how was Celso Zayco (not Lorenzo Zayco, as erroneously claimed by defendants) able to mortgage Lot No. 343
with the Pacific Banking Corporation (PBC), despite this alleged tenancy relationship between him and the defendants?
Equally baffling to the Court is this undisputed fact: although the Emancipation Patents (EPs) in the names of the seven (7)
defendants were issued on July 1, 1988; yet, those were, respectively, registered on plaintiff's title only on March 21, 1990,
July 12, 1990 and August 24, 1990.

Thirdly, of the other thirteen (13) defendants, plaintiff's Exhibit "A" clearly shows that they had not been cultivating personally
the portions occupied by them or with the help of the immediate members of their families; but that they had been leasing
such portions to several persons. 40

The MTCC even ignored the receipts appended by the petitioners to their position paper showing that the landowner and/or
Julieta C. Salgado received their share of the produce of the landholding as "rental" of the petitioners.

The validity and efficacy of the Emancipation Patents were not negated by the Register of Deeds' delay in the annotation
thereof at the dorsal portion of TCT No. 133298. As certified by the Register of Deeds, the failure to make the annotations
in the following patents earlier were inadvertent:

EP TITLE NO. LOT NO. NAME OF FARMERS AREA/SQ.M.

EP-1716 343-9 Pacifico P. Talibutab 8,735

EP-1717 343-15 Felix S. Ortega 8,106

EP-1718 343-22 Roberto D. Peduhan 7,779

EP-1719 343-25 Arturo T. Villarena 8,346

EP-1720 343-19 Vicente C. Onlayao 7,709

EP-1722 343-11 Antonio E. Ballentos 9,066

EP-1723 343-29 Cesar C. Aral 8,485

EP-1724 343-18 Manuel P. Caniendo 10,110

EP-1725 343-24 Salvador G. Mirano 8,215

EP-1740 343-8 Salustiano P. Billeran 23,391

EP-1751 343-16 Federico L. Orlano 10,453

EP-1754 343-14 Rogelio U. Semellano 7,668

EP-1813 343-23 Tarcelo S. Mirano 7,920 41

It must be underscored that the said patents were already annotated at the dorsal portion of TCT No. 133298 long before
the respondent filed its complaint with the MTCC against the petitioners. ASTIED

The MTCC also took into account and gave emphasis to Resolution No. 96-39 approved by the Sangguniang Bayan on
February 14, 1996, thus:

At this point, it bears stressing that in its Comprehensive Land Use Plan (CLUP), per Resolution No. 96-39 dated February
14, 1996, the then Sangguniang Bayan of Kabankalan, Negros Occidental, now a component city under R.A. No. 8297,
had reclassified Lot No. 343 into light industrial, commercial and residential areas. To the mind of the Court, this
reclassification falls squarely within the ambit of Title VI, B.2.b of Administrative Order No. 07, Series of 1997 dated October
29, 1997 on the subject: "Omnibus Rules and Procedures Governing Conversion of Agricultural Lands to Non-Agricultural
Uses." This provision is as follows:

B. General Guidelines

1. ...

2. ...

a) ...

94
b) Conversion may be allowed if at the time of the application, the lands are reclassified as commercial, industrial,
residential or other non-agricultural in the new or revised town plans promulgated by the Local Government Unit (LGU) and
approved by the Housing and Land Use Regulatory Board (HLURB) or by the Sangguniang Panglalawigan (SP) after June
15, 1988 in accordance with Section 20 of R.A. No. 7160, as implemented by M.C. No. 54, and Executive Order No. 72,
Series of 1993 of the Office of the President. 42

The ruling of the MTCC is erroneous. Under Section 65 of Rep. Act No. 6657 which took effect on June 15, 1988, agricultural
lands may be reclassified only by the DAR after the lapse of five (5) years from its award to the farmers-beneficiaries:

Section 65. Conversion of Lands. — After the lapse of five (5) years from its award, when the land ceases to be
economically feasible and sound for agricultural purposes, or the locality has become urbanized and the land will have
greater economic value for residential, commercial or industrial purposes, the DAR, upon application of the beneficiary or
the landowner, with due notice to the affected parties, and subject to existing laws, may authorize the reclassification of
conversion of the land and its disposition: Provided, That the beneficiary shall have fully paid his obligation. EHcaAI

In this case, there is no showing that the DAR ever approved the reclassification of the property. It appears that the
reclassification of the landholding was unilaterally made by the Sangguniang Bayan despite the issuance to the petitioners
of Emancipation Patents and transfer certificates of title in their names over the portions of the landholdings respectively
occupied by them.

The petitioners appended to their petition in the RTC a Certification of the Register of Deeds indicating that thirteen (13) of
the petitioners were issued transfer certificates of title based on the Emancipation Patents filed with said office, made of
record in the Primary Entry Book on September 16, 20, and 22, 1998; and an LBP certificate stating that eighteen (18) of
the petitioners had made advance payments for the portions of the landholding occupied by them. And yet, the RTC
dismissed the petition and affirmed the ruling of the MTCC that it had jurisdiction over the subject matter of the complaint.

It is evident from the face of the complaint and the pleadings of the parties and the appendages thereof that the issue of
possession of the subject property was inextricably interwoven with the issue of whether the Emancipation Patents issued
by the DAR to the petitioners were valid. Under the DAR Rules of Procedure, the DARAB has primary and exclusive original
jurisdiction over cases involving the issuance and cancellation of Emancipation Patents. Moreover, the respondent claimed
possession over the property based on TCT No. 133298, which had already been partially cancelled by the Emancipation
Patents and Torrens titles issued to the petitioners.

On the third issue, we reject the contention of the respondent that the decision of the MTCC had become final and executory
because of the petitioners' failure to perfect the appeal therefrom; hence, immutable. Neither do we agree with the
respondent's contention that by participating in the proceedings before the MTCC, the petitioners were estopped from
assailing the jurisdiction of the MTCC. As we held in Arevalo v. Benedicto:43

[F]urthermore, the want of jurisdiction by a court over the subject-matter renders its judgment void and a mere nullity, and
considering that a void judgment is in legal effect no judgment, by which no rights are divested, from which no rights can
be obtained, which neither binds nor bars any one, and under which all acts performed and all claims flowing out of are
void, and considering further, that the decision, for want of jurisdiction of the court, is not a decision in contemplation of law,
and, hence, can never become executory, it follows that such a void judgment cannot constitute a bar to another case by
reason of res judicata. caHASI

Our ruling in Abbain v. Chua 44 is also instructive:

In varying language, this Court has expressed its reprobation for judgments rendered by a court without jurisdiction. Such
a judgment is held to be "a dead limb on the judicial tree, which should be lopped off or wholly disregarded as the
circumstances require." In the language of Mr. Justice Street: "Where a judgment or judicial order is void in this sense it
may be said to be a lawless thing, which can be treated as an outlaw and slain at sight, or ignored wherever and whenever
it exhibits its head." And in Gomez vs. Concepcion, this Court quoted with approval the following from Freeman on
Judgments: "A void judgment is in legal effect no judgment. By it no rights are divested. From it no rights can be obtained.
Being worthless in itself, all proceedings found upon it are equally worthless. It neither binds nor bars any one. All acts
performed under it and all claims flowing out of it are void. The parties attempting to enforce it may be responsible as
trespassers. The purchaser at a sale by virtue of its authority finds himself without title and without redress."

Since the judgment here on its face is void ab initio, the limited periods for relief from judgment in Rule 38 are inapplicable.
That judgment is vulnerable to attack "in any way and at any time, even when no appeal has been taken."

It is settled that jurisdiction over the judgment cannot be changed by agreement of the parties or by the act or omission of
each of them that will contravene the legislative will. A party should not be allowed to divest a competent court of its
jurisdiction, whether erroneously or even deliberately in derogation of the law. 45
95
In this case, the counsel of the petitioners opted to assail in a direct action the decision of the MTCC, instead of perfecting
their appeal or assailing the decision of the MTCC disallowing their appeal. The petitioners believed that the decision of the
MTCC was null and void for want of jurisdiction over the subject matter of the action filed therein; hence, they are not
proscribed from assailing such decision in a direct action. The remedy resorted to by their counsel should not prejudice and
bar them from assailing the MTCC decision before the RTC on a petition to annul the same for lack of jurisdiction. Neither
are they estopped from assailing the decision, simply because they filed their answer and motion to dismiss the complaint
on the ground of lack of jurisdiction over the subject matter of the action. After all, the only relief prayed for by them in their
answer was the dismissal of the complaint. A propos is our ruling in Calimlim v. Ramirez: 46

It is neither fair nor legal to bind a party by the result of a suit or proceeding which was taken cognizance of in a court which
lacks jurisdiction over the same irrespective of the attendant circumstances. The equitable defense of estoppel requires
knowledge or consciousness of the facts upon which it is based. The same thing is true with estoppel by conduct which
may be asserted only when it is shown, among others, that the representation must have been made with knowledge of
the facts and that the party to whom it was made is ignorant of the truth of the matter. (De Castro vs. Gineta, 27 SCRA
623.) The filing of an action or suit in a court that does not possess jurisdiction to entertain the same may not be presumed
to be deliberate and intended to secure a ruling which could later be annulled if not favorable to the party who filed such
suit or proceeding. Instituting such an action is not a one-sided affair. It can just as well be prejudicial to the one who filed
the action or suit in the event that he obtains a favorable judgment therein which could also be attacked for having been
rendered without jurisdiction. The determination of the correct jurisdiction of a court is not a simple matter. It can raise highly
debatable issues of such importance that the highest tribunal of the land is given the exclusive appellate jurisdiction to
entertain the same. The point simply is that when a party commits error in filing his suit or proceeding in a court that lacks
jurisdiction to take cognizance of the same, such act may not at once be deemed sufficient basis of estoppel. It could have
been the result of an honest mistake, or of divergent interpretations of doubtful legal provisions. If any fault is to be imputed
to a party taking such course of action, part of the blame should be placed on the court which shall entertain the suit,
thereby lulling the parties into believing that they pursued their remedies in the correct forum. Under the rules, it is the duty
of the court to dismiss an action "whenever it appears that the court has no jurisdiction over the subject matter." (Sec. 2,
Rule 9, Rules of Court.) Should the court render a judgment without jurisdiction, such judgment may be impeached or
annulled for lack of jurisdiction (Sec. 30, Rule 132, Ibid.), within ten (10) years from the finality of the same. (Art. 1144, par.
3, Civil Code.) CETDHA

It bears stressing that the petitioners are now the registered owners of the portions of the landholding and entitled to the
possession thereof. For us to deny the petition and affirm the decision of the RTC would be to sanction the eviction of the
petitioners who are the registered owners of the landholding and, as such, are entitled to the possession thereof and allow
the respondent to take possession thereof in derogation of law. Not too long ago in Calimlim v. Ramirez 47 we held that:

The inequity of barring the petitioners from vindicating their right over their property in Civil Case No. SCC-180 is rendered
more acute in the face of the undisputed fact that the property in question admittedly belonged to the petitioners, and that
the title in the name of the private respondent was the result of an error committed by the Provincial Sheriff in issuing the
deed of sale in the execution proceeding. The justness of the relief sought by herein petitioners may not be ignored or
rendered futile by reason of a doctrine which is of highly doubtful applicability herein.

IN LIGHT OF ALL THE FOREGOING, the petition is GRANTED. The decisions of the Municipal Trial Court in Cities and
the Regional Trial Court are SET ASIDE and declared NULL and VOID. The writ of execution issued by the MTCC is also
set aside. No costs.

SO ORDERED.

Austria-Martinez and Tinga, JJ ., concur.

Puno, J ., took no part.

Chico-Nazario, J ., is on leave.

17. Echegaray v. Secretary of Justice, G.R. No. 132601, January 19, 1999, 301 SCRA 96, 106.

EN BANC

[G.R. No. 132601. January 19, 1999.]

LEO ECHEGARAY, petitioner, vs. SECRETARY OF JUSTICE, ET AL., respondents.

Atty. Theodore O. Te for petitioner.


96
The Solicitor General for respondents.

SYNOPSIS

This is the Urgent Motion for Reconsideration and the Supplement thereto of the Resolution of the Supreme Court dated
January 4, 1999 temporarily restraining the execution of the death convict Leo Echegaray by lethal injection. It is the main
submission of public respondents that the Decision of the case having become final and executory, its execution enters the
exclusive ambit of authority of the executive authority. caHASI

The Court ruled that the power to control the execution of its decision is an essential aspect of jurisdiction. It cannot be the
subject of substantial subtraction for our Constitution vests the entirety of judicial power in one Supreme Court and in such
lower courts as may be established by law. To be sure, the most important part of a litigation, whether civil or criminal, is
the process of execution of decisions where supervening events may change the circumstance of the parties and compel
courts to intervene and adjust the rights of the litigants to prevent unfairness. It is because of these unforeseen, supervening
contingencies that courts have been conceded the inherent and necessary power of control of its processes and orders to
make them conformable to law and justice. For this purpose, Section 6 of Rule 135 provides that "when by law jurisdiction
is conferred on a court or judicial officer, all auxiliary writs, processes and other means necessary to carry it into effect may
be employed by such court or officer and if the procedure to be followed in the exercise of such jurisdiction is not specifically
pointed out by law or by these rules, any suitable process or mode of proceedings may be adopted which appears
conformable to the spirit of said law or rules." It bears repeating that what the Court restrained temporarily is the execution
of its own Decision to give it reasonable time to check its fairness in light of supervening events in Congress as alleged by
petitioner. The Court, contrary to popular misimpression, did not restrain the effectivity of a law enacted by Congress.
CHIEDS

Moreover, the temporary restraining order of this Court has produced its desired result, i.e., the crystallization of the issue
whether Congress is disposed to review capital punishment. The public respondents, thru the Solicitor General, cite
posterior events that negate beyond doubt the possibility that Congress will repeal or amend the death penalty law. In light
of these developments, the Court's TRO should now be lifted as it has served its legal and humanitarian purpose.

The instant motion is GRANTED. IaAHCE

SYLLABUS

1. REMEDIAL LAW; ACTIONS; RULE ON FINALITY OF JUDGMENT; CANNOT DIVEST COURT OF ITS
JURISDICTION. — The rule on finality of judgment cannot divest this Court of its jurisdiction to execute and enforce the
same judgment. Retired Justice Camilo Quiason synthesized the well established jurisprudence on this issue as follows: .
. . "the finality of a judgment does not mean that the Court has lost all its powers nor the case. By the finality of the judgment,
what the court loses is its jurisdiction to amend, modify or alter the same. Even after the judgment has become final the
court retains its jurisdiction to execute and enforce it. There is a difference between the jurisdiction of the court to execute
its judgment and its jurisdiction to amend, modify or alter the same. The former continues even after the judgment has
become final for the purpose of enforcement of judgment; the latter terminates when the judgment becomes final. . . . For
after the judgment has become final facts and circumstances may transpire which can render the execution unjust or
impossible.

2. ID.; SUPREME COURT; FINALITY OF DECISION IN CRIMINAL CASES; PARTICULAR OF EXECUTION ITSELF
STILL UNDER CONTROL OF JUDICIAL AUTHORITY. — In criminal cases, after the sentence has been pronounced and
the period for reopening the same has elapsed, the court cannot change or alter its judgment, as its jurisdiction has
terminated . . . When in cases of appeal or review the cause has been returned thereto for execution, in the event that the
judgment has been affirmed, it performs a ministerial duty in issuing the proper order. But it does not follow from this
cessation of functions on the part of the court with reference to the ending of the cause that the judicial authority terminates
by having then passed completely to the Executive. The particulars of the execution itself, which are certainly not always
included in the judgment and writ of execution, in any event are absolutely under the control of the judicial authority, while
the executive has no power over the person of the convict except to provide for carrying out of the penalty and to pardon.
(Director of Prisons v. Judge of First Instance, 26 Phil. 267 [1915])

3 ID.; CRIMINAL PROCEDURE; EXECUTION OF SENTENCE; GROUNDS FOR POSTPONEMENT. —


Notwithstanding the order of execution and the executory nature thereof on the date set or at the proper time, the date
therefor can be postponed, even in sentences of death. Under the common law this postponement can be ordered in three
ways: (1) By command of the King; (2) by discretion (arbitrio) of the court; and (3) by mandate of the law. It is sufficient to
state this principle of the common law to render impossible that assertion in absolute terms that after the convict has once
been placed in jail the trial court can not reopen the case to investigate the facts that show the need for postponement.
IDTcHa
97
4. ID.; ACTIONS; JURISDICTION; POWER TO CONTROL EXECUTION OF DECISION, AN ESSENTIAL ASPECT
THEREOF. — The power to control the execution of its decision is an essential aspect of jurisdiction. It cannot be the
subject of substantial subtraction for our Constitution vests the entirety of judicial power in one Supreme Court and in such
lower courts as may be established by law. To be sure, the most important part of a litigation, whether civil or criminal, is
the process of execution of decisions where supervening events may change the circumstance of the parties and compel
courts to intervene and adjust the rights of the litigants to prevent unfairness. It is because of these unforeseen, supervening
contingencies that courts have been conceded the inherent and necessary power of control of its processes and orders to
make them conformable to law and justice.

5. ID.; SUPREME COURT; JURISDICTION OF THIS COURT DOES NOT DEPEND ON CONVENIENCE OF
LITIGANTS. — The same motion to compel Judge Ponferrada to reveal the date of execution of petitioner Echegaray was
filed by his counsel, Atty. Theodore Te, on December 7, 1998. He invoked his client's right to due process and the public's
right to information. The Solicitor General, as counsel for public respondents, did not oppose petitioner's motion on the
ground that this Court has no more jurisdiction over the process of execution of Echegaray. This Court granted the relief
prayed for by the Secretary of Justice and by the counsel of the petitioner in its Resolution of December 15, 1998. There
was not a whimper of protest from the public respondents and they are now estopped from contending that this Court has
lost its jurisdiction to grant said relief. The jurisdiction of this Court does not depend on the convenience of litigants.

6. ID.; ID.; POWER TO SUSPEND EXECUTION OF CONVICTS DOES NOT VIOLATE CO-EQUAL AND
COORDINATE POWERS OF BRANCHES OF GOVERNMENT. — The text and tone of this provision will not yield to the
interpretation suggested by the public respondents. The provision is simply the source of power of the President to grant
reprieves, commutations, and pardons and remit fines and forfeitures after conviction by final judgment. It also provides the
authority for the President to grant amnesty with the concurrence of a majority of all the members of the Congress. The
provision, however, cannot be interpreted as denying the power of courts to control the enforcement of their decisions after
their finality. In truth, an accused who has been convicted by final judgment still possesses collateral rights and these rights
can be claimed in the appropriate courts. For instance, a death convict who becomes insane after his final conviction cannot
be executed while in a state of insanity. As observed by Antieau, "today, it is generally assumed that due process of law
will prevent the government from executing the death sentence upon a person who is insane at the time of execution." The
suspension of such a death sentence is undisputably an exercise of judicial power. It is not a usurpation of the presidential
power of reprieve though its effect is the same — the temporary suspension of the execution of the death convict. In the
same vein, it cannot be denied that Congress can at any time amend R. A. No. 7659 by reducing the penalty of death to
life imprisonment. The effect of such an amendment is like that of commutation of sentence. But by no stretch of the
imagination can the exercise by Congress of its plenary power to amend laws be considered as a violation of the power of
the President to commute final sentences of conviction. The powers of the Executive, the Legislative and the Judiciary to
save the life of a death convict do not exclude each other for the simple reason that there is no higher right than the right
to life. Indeed, in various States in the United States, laws have even been enacted expressly granting courts the power to
suspend execution of convicts and their constitutionality has been upheld over arguments that they infringe upon the power
of the President to grant reprieves. For the public respondents therefore to contend that only the Executive can protect the
right to life of an accused after his final conviction is to violate the principle of co-equal and coordinate powers of the three
branches of our government.

VITUG, J., separate opinion:

1. CONSTITUTIONAL LAW; R.A. NO. 7659, UNCONSTITUTIONAL. — Let me state at the outset that I have humbly
maintained that Republic Act No. 7659, insofar as it prescribes the death penalty, falls short of the strict norm set forth by
the Constitution. I and some of my brethren on the Court, who hold similarly, have consistently expressed this stand in the
affirmance by the Court of death sentences imposed by Regional Trial Courts. Until the exacting standards of the
Constitution are clearly met as so hereinabove expressed, I will have to disagree, most respectfully, with my colleagues in
the majority who continue to hold the presently structured Republic Act. No. 7659 to be in accord with the Constitution, an
issue that is fundamental, constant and inextricably linked to the imposition each time of the death penalty and, like the
instant petition, to the legal incidents pertinent thereto. HAaDTI

2. REMEDIAL LAW; SUPREME COURT; WITH JURISDICTION TO ISSUE "TEMPORARY RESTRAINING ORDER"
ON EXECUTION OF DEATH PENALTY. — In its resolution of 04 January 1999, the Court resolved to issue in the above-
numbered petition a temporary restraining order ("TRO") because, among other things, of what had been stated to be
indications that Congress would re-examine the death penalty law. The Court, it must be stressed, did not, by issuing the
TRO, thereby reconsider its judgment convicting the accused or recall the imposition of the death penalty. The doctrine has
almost invariably been that after a decision becomes final and executory, nothing else is further done except to see to its
compliance since for the Court to adopt otherwise would be to put no end to litigations. The rule notwithstanding, the Court
retains control over the case until the full satisfaction of the final judgment conformably with established legal processes.
Hence, the Court has taken cognizance of the petition assailing before it the use of lethal injection by the State to carry out
98
the death sentence. In fine, the authority of the Court to see to the proper execution of its final judgment, the power of the
President to grant pardon, commutation or reprieve, and the prerogative of Congress to repeal or modify the law that could
benefit the convicted accused are not essentially preclusive of one another nor constitutionally incompatible and may each
be exercised within their respective spheres and confines. Thus, the stay of execution issued by the Court would not prevent
either the President from exercising his pardoning power or Congress from enacting a measure that may be advantageous
to the adjudged offender.

3. ID.; ACTIONS; JUDGMENT; IMMUTABILITY OF FINAL AND EXECUTORY JUDGMENTS; EXCEPTIONS. — In


any event, jurisprudence teaches that the rule of immutability of final and executory judgments admits of settled exceptions.
Concededly, the Court may, for instance, suspend the execution of a final judgment when it becomes imperative in the
higher interest of justice or when supervening events warrant it. Certainly, this extraordinary relief cannot be denied any
man, whatever might be his station, whose right to life is the issue at stake.

PANGANIBAN, J., separate opinion:

1. CONSTITUTIONAL LAW; R.A. NO. 7659 (DEATH PENALTY LAW), UNCONSTITUTIONAL. — R.A. 7659 (the
Death Penalty Law) is unconstitutional insofar as some parts thereof prescribing the capital penalty fail to comply with the
requirements of "heinousness" and "compelling reasons" prescribed by the Constitution of the Philippines.

2. ID.; R.A. NO. 8177 (LETHAL INJECTION LAW), UNCONSTITUTIONAL. — R.A. 8177 (the Lethal Injection Law)
is likewise unconstitutional since it merely prescribes the manner in which R.A. 7659 (the Death Penalty Law) is to be
implemented. DTAaCE

RESOLUTION

PUNO, J p:

For resolution are public respondents' Urgent Motion for Reconsideration of the Resolution of this Court dated January 4,
1999 temporarily restraining the execution of petitioner and Supplemental Motion to Urgent Motion for Reconsideration. It
is the submission of public respondents that: cdasia

"(1) The Decision in this case having become final and executory, its execution enters the exclusive ambit of authority
of the executive authority. The issuance of the TRO may be construed as trenching on that sphere of executive authority;

(2) The issuance of the temporary restraining order . . . creates dangerous precedent as there will never be an end to
litigation because there is always a possibility that Congress may repeal a law.

(3) Congress had earlier deliberated extensively on the death penalty bill. To be certain, whatever question may now
be raised on the Death Penalty Law before the present Congress within the 6-month period given by this Honorable Court
had in all probability been fully debated upon . . ..

(4) Under the time honored maxim lex futuro, judex praeterito, the law looks forward while the judge looks at the past,
. . . the Honorable Court in issuing the TRO has transcended its power of judicial review.

(5) At this moment, certain circumstances/supervening events transpired to the effect that the repeal or modification
of the law imposing death penalty has become nil, to wit:

a. The public pronouncement of President Estrada that he will veto any law imposing the death penalty involving
heinous crimes.

b. The resolution of Congressman Golez, et al., that they are against the repeal of the law;

c. The fact that Senator Roco's resolution to repeal the law only bears his signature and that of Senator Pimentel."

In their Supplemental Motion to Urgent Motion for Reconsideration, public respondents attached a copy of House
Resolution No. 629 introduced by Congressman Golez entitled "Resolution expressing the sense of the House of
Representative to reject any move to review Republic Act No. 7659 which provided for the re-imposition of death penalty,
notifying the Senate, the Judiciary and the Executive Department of the position of the House of Representatives on this
matter, and urging the President to exhaust all means under the law to immediately implement the death penalty law." The
Resolution was concurred in by one hundred thirteen (113) congressmen.

In their Consolidated Comment, petitioner contends: (1) the stay order . . . is within the scope of judicial power and duty
and does not trench on executive powers nor on congressional prerogatives; (2) the exercise by this Court of its power to
stay execution was reasonable; (3) the Court did not lose jurisdiction to address incidental matters involved or arising from

99
the petition; (4) public respondents are estopped from challenging the Court's jurisdiction; and (5) there is no certainty that
the law on capital punishment will not be repealed or modified until Congress convenes and considers all the various
resolutions and bills filed before it.

Prefatorily, the Court likes to emphasize that the instant motions concern matters that are not incidents in G.R. No. 117472,
where the death penalty was imposed on petitioner on automatic review of his conviction by this Court. The instant motions
were filed in this case, G.R. No. 132601, where the constitutionality of R.A. No. 8177 (Lethal Injection Law) and its
implementing rules and regulations was assailed by petitioner. For this reason, the Court in its Resolution of January 4,
1999 merely noted the Motion to Set Aside of Rodessa "Baby" R. Echegaray dated January 7, 1999 and Entry of
Appearance of her counsel dated January 5, 1999. Clearly, she has no legal standing to intervene in the case at bar, let
alone the fact that the interest of the State is properly represented by the Solicitor General.

We shall now resolve the basic issues raised by the public respondents.

First. We do not agree with the sweeping submission of the public respondents that this Court lost its jurisdiction over the
case at bar and hence can no longer restrain the execution of the petitioner. Obviously, public respondents are invoking
the rule that final judgments can no longer be altered in accord with the principle that "it is just as important that there should
be a place to end as there should be a place to begin litigation." 1 To start with, the Court is not changing even a comma
of its final Decision. It is appropriate to examine with precision the metes and bounds of the Decision of this Court that
became final. These metes and bounds are clearly spelled out in the Entry of Judgment in this case, viz: LLjur

"ENTRY OF JUDGMENT

This is to certify that on October 12, 1998 a decision rendered in the above-entitled case was filed in this Office, the
dispositive part of which reads as follows:

'WHEREFORE, the petition is DENIED insofar as petitioner seeks to declare the assailed statute (Republic Act No. 8177)
as unconstitutional; but GRANTED insofar as Sections 17 and 19 of the Rules and Regulations to Implement Republic Act
No. 8177 are concerned, which are hereby declared INVALID because (a) Section 17 contravenes Article 83 of the Revised
Penal Code, as amended by Section 25 of Republic Act No. 7659; and (b) Section 19 fails to provide for review and approval
of the Lethal Injection Manual by the Secretary of Justice, and unjustifiably makes the manual confidential, hence
unavailable to interested parties including the accused/convict and counsel. Respondents are hereby enjoined from
enforcing and implementing Republic Act No. 8177 until the aforesaid Sections 17 and 19 of the Rules and Regulations to
Implement Republic Act No. 8177 are appropriately amended, revised and/or corrected in accordance with this Decision.

SO ORDERED.'

and that the same has, on November 6, 1998 become final and executory and is hereby recorded in the Book of Entries of
Judgment.

Manila, Philippines.

Clerk of Court

By: (SGD) TERESITA G. DIMAISIP

Acting Chief

Judicial Records Office"

The records will show that before the Entry of Judgment, the Secretary of Justice, the Honorable Serafin Cuevas, filed with
this Court on October 21, 1998 a Compliance where he submitted the Amended Rules and Regulations implementing R.A.
No. 8177 in compliance with our Decision. On October 28, 1998, Secretary Cuevas submitted a Manifestation informing
the Court that he has caused the publication of the said Amended Rules and Regulations as required by the Administrative
Code. It is crystalline that the Decision of this Court that became final and unalterable mandated: (1) that R.A. No. 8177 is
not unconstitutional; (2) that sections 17 and 19 of the Rules and Regulations to Implement R.A. No. 8177 are invalid, and
(3) R.A. No. 8177 cannot be enforced and implemented until sections 17 and 19 of the Rules and Regulations to Implement
R.A. No. 8177 are amended. It is also daylight clear that this Decision was not altered a whit by this Court. Contrary to the
submission of the Solicitor General, the rule on finality of judgment cannot divest this Court of its jurisdiction to execute and
enforce the same judgment. Retired Justice Camilo Quiason synthesized the well established jurisprudence on this issue
as follows: 2

100
xxx xxx xxx

"the finality of a judgment does not mean that the Court has lost all its powers nor the case. By the finality of the judgment,
what the court loses is its jurisdiction to amend, modify or alter the same. Even after the judgment has become final the
court retains its jurisdiction to execute and enforce it. 3 There is a difference between the jurisdiction of the court to execute
its judgment and its jurisdiction to amend, modify or alter the same. The former continues even after the judgment has
become final for the purpose of enforcement of judgment; the latter terminates when the judgment becomes final. 4 . . . For
after the judgment has become final facts and circumstances may transpire which can render the execution unjust or
impossible. 5

In truth, the argument of the Solicitor General has long been rejected by this Court. As aptly pointed out by the petitioner,
as early as 1915, this Court has unequivocably ruled in the case of Director of Prisons v. Judge of First Instance, 6 viz:

"This Supreme Court has repeatedly declared in various decisions, which constitute jurisprudence on the subject, that in
criminal cases, after the sentence has been pronounced and the period for reopening the same has elapsed, the court
cannot change or alter its judgment, as its jurisdiction has terminated . . . When in cases of appeal or review the cause has
been returned thereto for execution, in the event that the judgment has been affirmed, it performs a ministerial duty in
issuing the proper order. But it does not follow from this cessation of functions on the part of the court with reference to the
ending of the cause that the judicial authority terminates by having then passed completely to the Executive. The particulars
of the execution itself, which are certainly not always included in the judgment and writ of execution, in any event are
absolutely under the control of the judicial authority, while the executive has no power over the person of the convict except
to provide for carrying out of the penalty and to pardon. cdasia

Getting down to the solution of the question in the case at bar, which is that of execution of a capital sentence, it must be
accepted as a hypothesis that postponement of the date can be requested. There can be no dispute on this point. It is a
well-known principle that notwithstanding the order of execution and the executory nature thereof on the date set or at the
proper time, the date therefor can be postponed, even in sentences of death. Under the common law this postponement
can be ordered in three ways: (1) By command of the King; (2) by discretion (arbitrio) of the court; and (3) by mandate of
the law. It is sufficient to state this principle of the common law to render impossible that assertion in absolute terms that
after the convict has once been placed in jail the trial court can not reopen the case to investigate the facts that show the
need for postponement. If one of the ways is by direction of the court, it is acknowledged that even after the date of the
execution has been fixed, and notwithstanding the general rule that after the (court) has performed its ministerial duty of
ordering the execution . . . and its part is ended, if however a circumstance arises that ought to delay the execution, and
there is an imperative duty to investigate the emergency and to order a postponement. Then the question arises as to
whom the application for postponing the execution ought to be addressed while the circumstances is under investigation
and as to who has jurisdiction to make the investigation."

The power to control the execution of its decision is an essential aspect of jurisdiction. It cannot be the subject of substantial
subtraction for our Constitution 7 vests the entirety of judicial power in one Supreme Court and in such lower courts as may
be established by law. To be sure, the most important part of a litigation, whether civil or criminal, is the process of execution
of decisions where supervening events may change the circumstance of the parties and compel courts to intervene and
adjust the rights of the litigants to prevent unfairness. It is because of these unforeseen, supervening contingencies that
courts have been conceded the inherent and necessary power of control of its processes and orders to make them
conformable to law and justice. 8 For this purpose, Section 6 of Rule 135 provides that "when by law jurisdiction is conferred
on a court or judicial officer, all auxiliary writs, processes and other means necessary to carry it into effect may be employed
by such court or officer and if the procedure to be followed in the exercise of such jurisdiction is not specifically pointed out
by law or by these rules, any suitable process or mode of proceeding may be adopted which appears conformable to the
spirit of said law or rules." It bears repeating that what the Court restrained temporarily is the execution of its own Decision
to give it reasonable time to check its fairness in light of supervening events in Congress as alleged by petitioner. The
Court, contrary to popular misimpression, did not restrain the effectivity of a law enacted by Congress.

The more disquieting dimension of the submission of the public respondents that this Court has no jurisdiction to restrain
the execution of petitioner is that it can diminish the independence of the judiciary. Since the implant of republicanism in
our soil, our courts have been conceded the jurisdiction to enforce their final decisions. In accord with this unquestioned
jurisdiction, this Court promulgated rules concerning pleading, practice and procedure which, among others, spelled out
the rules on execution of judgments. These rules are all predicated on the assumption that courts have the inherent,
necessary and incidental power to control and supervise the process of execution of their decisions. Rule 39 governs
execution, satisfaction and effects of judgments in civil cases. Rule 120 governs judgments in criminal cases. It should be
stressed that the power to promulgate rules of pleading, practice and procedure was granted by our Constitutions to this
Court to enhance its independence, for in the words of Justice Isagani Cruz "without independence and integrity, courts will
lose that popular trust so essential to the maintenance of their vigor as champions of justice." 9 Hence, our Constitutions
101
continuously vested this power to this Court for it enhances its independence. Under the 1935 Constitution, the power of
this Court to promulgate rules concerning pleading, practice and procedure was granted but it appeared to be co-existent
with legislative power for it was subject to the power of Congress to repeal, alter or supplement. Thus, its Section 13, Article
VIII provides:

"SECTION. 13. The Supreme Court shall have the power to promulgate rules concerning pleading, practice and procedure
in all courts, and the admission to the practice of law. Said rules shall be uniform for all courts of the same grade and shall
not diminish, increase, or modify substantive rights. The existing laws on pleading, practice and procedure are hereby
repealed as statutes, and are declared Rules of Court, subject to the power of the Supreme Court to alter and modify the
same. The Congress shall have the power to repeal, alter or supplement the rules concerning pleading, practice and
procedure, and the admission to the practice of law in the Philippines."

The said power of Congress, however, is not as absolute as it may appear on its surface. In In re Cunanan 10 Congress in
the exercise of its power to amend rules of the Supreme Court regarding admission to the practice of law, enacted the Bar
Flunkers Act of 1953 11 which considered as a passing grade, the average of 70% in the bar examinations after July 4,
1946 up to August 1951 and 71% in the 1952 bar examinations. This Court struck down the law as unconstitutional. In his
ponencia, Mr. Justice Diokno held that ". . . the disputed law is not a legislation; it is a judgment — a judgment promulgated
by this Court during the aforecited years affecting the bar candidates concerned; and although this Court certainly can
revoke these judgments even now, for justifiable reasons, it is no less certain that only this Court, and not the legislative
nor executive department, that may do so. Any attempt on the part of these departments would be a clear usurpation of its
function, as is the case with the law in question." 12 The venerable jurist further ruled: "It is obvious, therefore, that the
ultimate power to grant license for the practice of law belongs exclusively to this Court, and the law passed by Congress
on the matter is of permissive character, or as other authorities say, merely to fix the minimum conditions for the license."
By its ruling, this Court qualified the absolutist tone of the power of Congress to "repeal, alter or supplement the rules
concerning pleading, practice and procedure, and the admission to the practice of law in the Philippines. cdasia

The ruling of this Court in In re Cunanan was not changed by the 1973 Constitution. For the 1973 Constitution reiterated
the power of this Court "to promulgate rules concerning pleading, practice and procedure in all courts, . . . which, however,
may be repealed, altered or supplemented by the Batasang Pambansa . . . ."

More completely, Section 5(2)5 of its Article X provided:

xxx xxx xxx

"SECTION. 5. The Supreme Court shall have the following powers.

xxx xxx xxx

(5) Promulgate rules concerning pleading, practice, and procedure in all courts, the admission to the practice of law,
and the integration of the Bar, which, however, may be repealed, altered, or supplemented by the Batasang Pambansa.
Such rules shall provide a simplified and inexpensive procedure for the speedy disposition of cases, shall be uniform for all
courts of the same grade, and shall not diminish, increase, or modify substantive rights."

Well worth noting is that the 1973 Constitution further strengthened the independence of the judiciary by giving to it the
additional power to promulgate rules governing the integration of the Bar. 13

The 1987 Constitution molded an even stronger and more independent judiciary. Among others, it enhanced the rule
making power of this Court. Its Section 5(5), Article VIII provides:

xxx xxx xxx

"SECTION 5. The Supreme Court shall have the following powers:

xxx xxx xxx

(5) Promulgate rules concerning the protection and enforcement of constitutional rights, pleading, practice and
procedure in all courts, the admission to the practice of law, the Integrated Bar, and legal assistance to the underprivileged.
Such rules shall provide a simplified and inexpensive procedure for the speedy disposition of cases, shall be uniform for all
courts of the same grade, and shall not diminish, increase, or modify substantive rights. Rules of procedure of special courts
and quasi-judicial bodies shall remain effective unless disapproved by the Supreme Court."

The rule making power of this Court was expanded. This Court for the first time was given the power to promulgate rules
concerning the protection and enforcement of constitutional rights. The Court was also granted for the first time the power
to disapprove rules of procedure of special courts and quasi-judicial bodies. But most importantly, the 1987 Constitution
102
took away the power of Congress to repeal, alter, or supplement rules concerning pleading, practice and procedure. In fine,
the power to promulgate rules of pleading, practice and procedure is no longer shared by this Court with Congress, more
so with the Executive. If the manifest intent of the 1987 Constitution is to strengthen the independence of the judiciary, it is
inutile to urge, as public respondents do, that this Court has no jurisdiction to control the process of execution of its
decisions, a power conceded to it and which it has exercised since time immemorial. LLphil

To be sure, it is too late in the day for public respondents to assail the jurisdiction of this Court to control and supervise the
implementation of its decision in the case at bar. As aforestated, our Decision became final and executory on November 6,
1998. The records reveal that after November 6, 1998, or on December 8, 1998, no less than the Secretary of Justice
recognized the jurisdiction of this Court by filing a Manifestation and Urgent Motion to compel the trial judge, the Honorable
Thelma A. Ponferrada, RTC, Br. 104, Quezon City to provide him ". . . a certified true copy of the Warrant of Execution
dated November 17, 1998 bearing the designated execution day of death convict Leo Echegaray and allow (him) to reveal
or announce the contents thereof, particularly the execution date fixed by such trial court to the public when requested."
The relevant portions of the Manifestation and Urgent Motion filed by the Secretary of Justice beseeching this Court "to
provide the appropriate relief" state:

xxx xxx xxx

5. Instead of filing a comment on Judge Ponferrada's Manifestation however, herein respondent is submitting the
instant Manifestation and Motion (a) to stress, inter alia, that the non-disclosure of the date of execution deprives herein
respondent of vital information necessary for the exercise of his statutory powers, as well as renders nugatory the
constitutional guarantee that recognizes the people's right to information of public concern, and (b) to ask this Honorable
Court to provide the appropriate relief.

6. The non-disclosure of the date of execution deprives herein respondent of vital information necessary for the
exercise of his power of supervision and control over the Bureau of Corrections pursuant to Section 39, Chapter 8, Book
IV of the Administrative Code of 1987, in relation to Title III, Book IV of such Administrative Code, insofar as the enforcement
of Republic Act No. 8177 and the Amended Rules and Regulations to Implement Republic Act No. 8177 is concerned and
for the discharge of the mandate of seeing to it that laws and rules relative to the execution of sentence are faithfully
observed.

7. On the other hand, the willful omission to reveal the information about the precise day of execution limits the
exercise by the President of executive clemency powers pursuant to Section 19, Article VII (Executive Department) of the
1987 Philippine Constitution and Article 81 of the Revised Penal Code, as amended, which provides that the death sentence
shall be carried out 'without prejudice to the exercise by the President of his executive clemency powers at all times."
(Underscoring supplied) For instance, the President cannot grant reprieve, i.e., postpone the execution of a sentence to a
day certain (People v. Vera, 65 Phil. 56, 110 [1937]) in the absence of a precise date to reckon with. The exercise of such
clemency power, at this time, might even work to the prejudice of the convict and defeat the purpose of the Constitution
and the applicable statute as when the date of execution set by the President would be earlier than that designated by the
court.

8. Moreover, the deliberate non-disclosure of information about the date of execution to herein respondent and the
public violates Section 7, Article III (Bill of Rights) and Section 28, Article II (Declaration of Principles and State Policies) of
the 1987 Philippine Constitution which read:

SECTION 7. The right of the people to information on matters of public concern shall be recognized. Access to official
records, and to documents and papers pertaining to official acts, transactions, or decisions, as well as to government
research data used as basis for policy development, shall be afforded the citizen, subject to such limitations as may be
provided by law.

SECTION 28. Subject to reasonable conditions prescribed by law, the State adopts and implements a policy of full public
disclosure of all its transactions involving public interest.

9. The 'right to information' provision is self-executing. It supplies 'the rules by means of which the right to information
may be enjoyed (Cooley, A Treatise on the Constitutional Limitations, 167 [1972]) by guaranteeing the right and mandating
the duty to afford access to sources of information. Hence, the fundamental right therein recognized may be asserted by
the people upon the ratification of the Constitution without need for any ancillary act of the Legislature (Id., at p. 165) What
may be provided for by the Legislature are reasonable conditions and limitations upon the access to be afforded which
must, of necessity, be consistent with the declared State policy of full public disclosure of all transactions involving public
interest (Constitution, Art. II, Sec. 28). However, it cannot be overemphasized that whatever limitation may be prescribed
by the Legislature, the right and the duty under Art. III, Sec. 7 have become operative and enforceable by virtue of the

103
adoption of the New Charter." (Decision of the Supreme Court En Banc in Legaspi v. Civil Service Commission, 150 SCRA
530, 534-535 [1987]."

The same motion to compel Judge Ponferrada to reveal the date of execution of petitioner Echegaray was filed by his
counsel, Atty. Theodore Te, on December 7, 1998. He invoked his client's right to due process and the public's right to
information. The Solicitor General, as counsel for public respondents, did not oppose petitioner's motion on the ground that
this Court has no more jurisdiction over the process of execution of Echegaray. This Court granted the relief prayed for by
the Secretary of Justice and by the counsel of the petitioner in its Resolution of December 15, 1998. There was not a
whimper of protest from the public respondents and they are now estopped from contending that this Court has lost its
jurisdiction to grant said relief. The jurisdiction of this Court does not depend on the convenience of litigants. cdtai

II

Second. We likewise reject the public respondents' contention that the "decision in this case having become final and
executory, its execution enters the exclusive ambit of authority of the executive department . . .. By granting the TRO, the
Honorable Court has in effect granted reprieve which is an executive function." 14 Public respondents cite as their authority
for this proposition, Section 19, Article VII of the Constitution which reads:

"Except in cases of impeachment, or as otherwise provided in this Constitution, the President may grant reprieves,
commutations, and pardons, and remit fines and forfeitures after conviction by final judgment. He shall also have the power
to grant amnesty with the concurrence of a majority of all the members of the Congress."

The text and tone of this provision will not yield to the interpretation suggested by the public respondents. The provision is
simply the source of power of the President to grant reprieves, commutations, and pardons and remit fines and forfeitures
after conviction by final judgment. It also provides the authority for the President to grant amnesty with the concurrence of
a majority of all the members of the Congress. The provision, however, cannot be interpreted as denying the power of
courts to control the enforcement of their decisions after their finality. In truth, an accused who has been convicted by final
judgment still possesses collateral rights and these rights can be claimed in the appropriate courts. For instance, a death
convict who becomes insane after his final conviction cannot be executed while in a state of insanity. 15 As observed by
Antieau, "today, it is generally assumed that due process of law will prevent the government from executing the death
sentence upon a person who is insane at the time of execution." 16 The suspension of such a death sentence is
undisputably an exercise of judicial power. It is not a usurpation of the presidential power of reprieve though its effect is the
same — the temporary suspension of the execution of the death convict. In the same vein, it cannot be denied that Congress
can at any time amend R.A. No. 7659 by reducing the penalty of death to life imprisonment. The effect of such an
amendment is like that of commutation of sentence. But by no stretch of the imagination can the exercise by Congress of
its plenary power to amend laws be considered as a violation of the power of the President to commute final sentences of
conviction. The powers of the Executive, the Legislative and the Judiciary to save the life of a death convict do not exclude
each other for the simple reason that there is no higher right than the right to life. Indeed, in various States in the United
States, laws have even been enacted expressly granting courts the power to suspend execution of convicts and their
constitutionality has been upheld over arguments that they infringe upon the power of the President to grant reprieves. For
the public respondents therefore to contend that only the Executive can protect the right to life of an accused after his final
conviction is to violate the principle of co-equal and coordinate powers of the three branches of our government.

III

Third. The Court's resolution temporarily restraining the execution of petitioner must be put in its proper perspective as it
has been grievously distorted especially by those who make a living by vilifying courts. Petitioner filed his Very Urgent
Motion for Issuance of TRO on December 28, 1998 at about 11:30 p.m. He invoked several grounds, viz: (1) that his
execution has been set on January 4, the first working day of 1999; (b) that members of Congress had either sought for his
executive clemency and/or review or repeal of the law authorizing capital punishment; (b.1) that Senator Aquilino Pimentel's
resolution asking that clemency be granted to the petitioner and that capital punishment be reviewed has been concurred
by thirteen (13) other senators; (b.2) Senate President Marcelo Fernan and Senator Miriam S. Defensor have publicly
declared they would seek a review of the death penalty law; (b.3) Senator Raul Roco has also sought the repeal of capital
punishment, and (b.4) Congressman Salacrib Baterina, Jr., and thirty five (35) other congressmen are demanding review
of the same law.

When the Very Urgent Motion was filed, the Court was already in its traditional recess and would only resume session on
January 18, 1999. Even then, Chief Justice Hilario Davide, Jr. called the Court to a Special Session on January 4, 1999 17
at 10 a.m. to deliberate on petitioner's Very Urgent Motion. The Court hardly had five (5) hours to resolve petitioner's motion
as he was due to be executed at 3 p.m. Thus, the Court had the difficult problem of resolving whether petitioner's allegations
about the moves in Congress to repeal or amend the Death Penalty Law are mere speculations or not. To the Court's
majority, there were good reasons why the Court should not immediately dismiss petitioner's allegations as mere
104
speculations and surmises. They noted that petitioner's allegations were made in a pleading under oath and were widely
publicized in the print and broadcast media. It was also of judicial notice that the 11th Congress is a new Congress and
has no less than one hundred thirty (130) new members whose views on capital punishment are still unexpressed. The
present Congress is therefore different from the Congress that enacted the Death Penalty Law (R.A. No. 7659) and the
Lethal Injection Law (R.A. No. 8177). In contrast, the Court's minority felt that petitioner's allegations lacked clear factual
bases. There was hardly a time to verify petitioner's allegations as his execution was set at 3 p.m. And verification from
Congress was impossible as Congress was not in session. Given these constraints, the Court's majority did not rush to
judgment but took an extremely cautious stance by temporarily restraining the execution of petitioner. The suspension was
temporary — "until June 15, 1999, coeval with the constitutional duration of the present regular session of Congress, unless
it sooner becomes certain that no repeal or modification of the law is going to be made." The extreme caution taken by the
Court was compelled, among others, by the fear that any error of the Court in not stopping the execution of the petitioner
will preclude any further relief for all rights stop at the graveyard. As life was at stake, the Court refused to constitutionalize
haste and the hysteria of some partisans. The Court's majority felt it needed the certainty that the legislature will not change
the circumstance of petitioner as alleged by his counsel. It was believed that law and equitable considerations demand no
less before allowing the State to take the life of one of its citizens.

The temporary restraining order of this Court has produced its desired result, i.e., the crystallization of the issue whether
Congress is disposed to review capital punishment. The public respondents, thru the Solicitor General, cite posterior events
that negate beyond doubt the possibility that Congress will repeal or amend the death penalty law. He names these
supervening events as follows: cda

xxx xxx xxx

"a. The public pronouncement of President Estrada that he will veto any law repealing the death penalty involving
heinous crimes.

b. The resolution of Congressman Golez, et al., that they are against the repeal of the law;

c. The fact that Senator Roco's resolution to repeal the law only bears his signature and that of Senator Pimentel."
18

In their Supplemental Motion to Urgent Motion for Reconsideration, the Solicitor General cited House Resolution No. 629
introduced by Congressman Golez entitled "Resolution expressing the sense of the House of Representatives to reject any
move to review R.A. No. 7659 which provided for the reimposition of death penalty, notifying the Senate, the Judiciary and
the Executive Department of the position of the House of Representatives on this matter and urging the President to exhaust
all means under the law to immediately implement the death penalty law." The Golez resolution was signed by 113
congressmen as of January 11, 1999. In a marathon session yesterday that extended up to 3 o'clock in the morning, the
House of Representatives with minor amendments formally adopted the Golez resolution by an overwhelming vote. House
Resolution No. 25 expressed the sentiment that the House ". . . does not desire at this time to review Republic Act 7659."
In addition, the President has stated that he will not request Congress to ratify the Second Protocol in view of the prevalence
of heinous crimes in the country. In light of these developments, the Court's TRO should now be lifted as it has served its
legal and humanitarian purpose.

A last note. In 1922, the famous Clarence Darrow predicted that ". . . the question of capital punishment has been the
subject of endless discussion and will probably never be settled so long as men believe in punishment." 19 In our clime
and time when heinous crimes continue to be unchecked, the debate on the legal and moral predicates of capital
punishment has been regrettably blurred by emotionalism because of the unfaltering faith of the pro and anti-death partisans
on the right and righteousness of their postulates. To be sure, any debate, even if it is no more than an exchange of epithets
is healthy in a democracy. But when the debate deteriorates to discord due to the overuse of words that wound, when
anger threatens to turn the majority rule to tyranny, it is the especial duty of this Court to assure that the guarantees of the
Bill of Rights to the minority fully hold. As Justice Brennan reminds us ". . . it is the very purpose of the Constitution — and
particularly the Bill of Rights — to declare certain values transcendent, beyond the reach of temporary political majorities."
20 Man has yet to invent a better hatchery of justice than the courts. It is a hatchery where justice will bloom only when we
can prevent the roots of reason to be blown away by the winds of rage. The flame of the rule of law cannot be ignited by
rage, especially the rage of the mob which is the mother of unfairness. The business of courts in rendering justice is to be
fair and they can pass their litmus test only when they can be fair to him who is momentarily the most hated by society. 21

IN VIEW WHEREOF, the Court grants the public respondents' Urgent Motion for Reconsideration and Supplemental Motion
to Urgent Motion for Reconsideration and lifts the Temporary Restraining Order issued in its Resolution of January 4, 1999.

105
The Court also orders respondent trial court judge (Hon. Thelma A. Ponferrada, Regional Trial Court, Quezon City, Branch
104) to set anew the date for execution of the convict/petitioner in accordance with applicable provisions of law and the
Rules of Court, without further delay. cdasia

SO ORDERED.

Davide, Jr., C.J., Romero, Bellosillo, Melo, Kapunan, Mendoza, Martinez, Quisumbing, Purisima and Pardo, JJ., concur.

Vitug and Panganiban, JJ., please see separate opinion.

Buena and Gonzaga-Reyes, JJ., took no part.

Separate Opinions

VITUG, J.:

Let me state at the outset that I have humbly maintained that Republic Act No. 7659, insofar as it prescribes the death
penalty, falls short of the strict norm set forth by the Constitution. I and some of my brethren on the Court, who hold similarly,
have consistently expressed this stand in the affirmance by the Court of death sentences imposed by Regional Trial Courts.

In its resolution of 04 January 1999, the Court resolved to issue in the above-numbered petition a temporary restraining
order ("TRO") because, among other things, of what had been stated to be indications that Congress would re-examine
the death penalty law. It was principally out of respect and comity to a co-equal branch of the government, i.e., to reasonably
allow it that opportunity if truly minded, that motivated the Court to grant, after deliberation, a limited time for the purpose.

The Court, it must be stressed, did not, by issuing the TRO, thereby reconsider its judgment convicting the accused or
recall the imposition of the death penalty. cdtai

The doctrine has almost invariably been that after a decision becomes final and executory, nothing else is further done
except to see to its compliance since for the Court to adopt otherwise would be to put no end to litigations. The rule
notwithstanding, the Court retains control over the case until the full satisfaction of the final judgment conformably with
established legal processes. Hence, the Court has taken cognizance of the petition assailing before it the use of lethal
injection by the State to carry out the death sentence. In any event, jurisprudence teaches that the rule of immutability of
final and executory judgments admits of settled exceptions. Concededly, the Court may, for instance, suspend the execution
of a final judgment when it becomes imperative in the higher interest of justice or when supervening events warrant it. 1
Certainly, this extraordinary relief cannot be denied any man, whatever might be his station, whose right to life is the issue
at stake. The pronouncement in Director of Prisons vs. Judge of First Instance of Cavite, 2 should be instructive. Thus —

"This Supreme Court has repeatedly declared in various decisions, which constitute jurisprudence on the subject, that in
criminal cases, after the sentence has been pronounced and the period for reopening the same has elapsed, the court can
not change or alter its judgment, as its jurisdiction has terminated, functus est officio suo, according to the classical phrase.
When in cases of appeal or review the cause has been returned thereto for execution, in the event that the judgment has
been affirmed, it performs a ministerial duty in issuing the proper order. But it does not follow from this cessation of functions
on the part of the court with reference to the ending of the cause that the judicial authority terminates by having then passed
completely to the executive. The particulars of the execution itself, which are certainly not always included in the judgment
and writ of execution, in any event are absolutely under the control of the judicial authority, while the executive has no
power over the person of the convict except to provide for carrying out the penalty and to pardon.

"Getting down to the solution of the question in the case at bar, which is that of execution of a capital sentence, it must be
accepted as a hypothesis that postponement of the date can be requested. There can be no dispute on this point. It is a
well-known principle that, notwithstanding the order of execution and the executory nature thereof on the date set or at the
proper time, the date therefor can be postponed, even in sentences of death. Under the common law this postponement
can be ordered in three ways: (1) By command of the King; (2) by discretion (arbitrio) of the court; and (3) by mandate of
the law. It is sufficient to state this principle of the common law to render impossible the assertion in absolute terms that
after the convict has once been placed in jail the trial court can not reopen the case to investigate the facts that show the
need for postponement. If one of the ways is by direction of the court, it is acknowledged that even after the date of the
execution has been fixed, and notwithstanding the general rule that after the Court of First Instance has performed its
ministerial duty of ordering the execution, functus est officio suo, and its part is ended, if however a circumstance arises
that ought to delay the execution, there is an imperative duty to investigate the emergency and to order a postponement .
. ."

In fine, the authority of the Court to see to the proper execution of its final judgment, the power of the President to grant
pardon, commutation or reprieve, and the prerogative of Congress to repeal or modify the law that could benefit the

106
convicted accused are not essentially preclusive of one another nor constitutionally incompatible and may each be
exercised within their respective spheres and confines. Thus, the stay of execution issued by the Court would not prevent
either the President from exercising his pardoning power or Congress from enacting a measure that may be advantageous
to the adjudged offender.

The TRO of this Court has provided that it shall be lifted even before its expiry date of 15 June 1999, "coeval with the
duration of the present regular session of Congress," if it "sooner becomes certain that no repeal or modification of the law
is going to be made." The "Urgent Motion for Reconsideration" filed by the Office of the Solicitor General states that as of
the moment, "certain circumstances/supervening events (have) transpired to the effect that the repeal or modification of
the law imposing death penalty has become nil . . ." If, indeed, it would be futile to yet expect any chance for a timely 3 re-
examination by Congress of the death penalty law, then I can appreciate why the majority of the Justices on the Court feel
rightly bound even now to lift the TRO.

I am hopeful, nevertheless, that Congress will in time find its way clear to undertaking a most thorough and dispassionate
re-examination of the law not so much for its questioned wisdom as for the need to have a second look at the conditions
sine qua non prescribed by the Constitution in the imposition of the death penalty. In People vs. Masalihit, 4 in urging, with
all due respect, Congress to consider a prompt re-examination of the death penalty law, I have said:

"The determination of when to prescribe the death penalty lies, in the initial instance, with the law-making authority, the
Congress of the Philippines, subject to the conditions that the Constitution itself has set forth; viz: (1) That there must be
compelling reasons to justify the imposition of the death penalty; and (2) That the capital offense must involve a heinous
crime. It appears that the fundamental law did not contemplate a simple 'reimposition' of the death penalty to offenses
theretofore already provided in the Revised Penal Code or, let alone, just because of it. The term 'compelling reasons'
would indicate to me that there must first be a marked change in the milieu from that which has prevailed at the time of
adoption of the 1987 Constitution, on the one hand, to that which exists at the enactment of the statute prescribing the
death penalty, upon the other hand, that would make it distinctively inexorable to allow the re-imposition of the death
penalty. Most importantly, the circumstances that would characterize the 'heinous nature' of the crime and make it so
exceptionally offensive as to warrant the death penalty must be spelled out with great clarity in the law, albeit without
necessarily precluding the Court from exercising its power of judicial review given the circumstances of each case. To
venture, in the case of murder, the crime would become 'heinous' within the Constitutional concept, when, to exemplify, the
victim is unnecessarily subjected to a painful and excruciating death or, in the crime of rape, when the offended party is
callously humiliated or even brutally killed by the accused. The indiscriminate imposition of the death penalty could
somehow constrain courts to apply, perhaps without consciously meaning to, stringent standards for conviction, not too
unlikely beyond what might normally be required in criminal cases, that can, in fact, result in undue exculpation of offenders
to the great prejudice of victims and society."

Today, I reiterate the above view and until the exacting standards of the Constitution are clearly met as so hereinabove
expressed, I will have to disagree, most respectfully, with my colleagues in the majority who continue to hold the presently
structured Republic Act No. 7659 to be in accord with the Constitution, an issue that is fundamental, constant and
inextricably linked to the imposition each time of the death penalty and, like the instant petition, to the legal incidents
pertinent thereto.

Accordingly, I vote against the lifting of the restraining order of the Court even as I, like everyone else, however, must
respect and be held bound by the ruling of the majority.

PANGANIBAN, J.:

I agree with the Court's Resolution that, without doubt, this Court has jurisdiction to issue the disputed Temporary
Restraining Order (TRO) on January 4, 1999. I will not repeat its well-reasoned disquisition. I write only to explain my vote
in the context of the larger issue of the death penalty.

Since the solicitor general has demonstrated that Congress will not repeal or amend RA 7659 during its current session
which ends on June 15, 1999 and that, in any event, the President will veto any such repeal or amendment, the TRO should
by its own terms be deemed lifted now. However, my objections to the imposition of the death penalty transcend the TRO
and permeate its juridical essence. cdasia

I maintain my view that RA 7659 (the Death Penalty Law) is unconstitutional insofar as some parts thereof prescribing the
capital penalty fail to comply with the requirements of "heinousness" and "compelling reasons" prescribed by the
Constitution of the Philippines. * This I have repeatedly stated in my Dissenting Opinions in various death cases decided
by the Court, as well as during the Court's deliberation on this matter on January 4, 1999. For easy reference, I hereby
attach a copy of my Dissent promulgated on February 7, 1997.

107
Consequently, I cannot now vote to lift the TRO, because to do so would mean the upholding and enforcement of a law (or
the relevant portions thereof) which, I submit with all due respect, is unconstitutional and therefore legally nonexistent. I
also reiterate that, in my humble opinion, RA 8177 (the Lethal Injection Law) is likewise unconstitutional since it merely
prescribes the manner in which RA 7659 (the Death Penalty Law) is to be implemented.

Having said that, I stress, however, that I defer to the rule of law and will abide by the ruling of the Court that both RA 7659
and RA 8177 are constitutional and that the death penalty should, by majority vote, be implemented by means of lethal
injection.

FOR THE ABOVE REASONS, I vote to deny the solicitor general's Motion for Reconsideration.

Death Penalty Law Unconstitutional

In his Supplemental Motion for Reconsideration 1 dated August 22, 1996 filed by his newly-retained counsel, 2 the accused
raises for the first time a very crucial ground for his defense: that Republic Act No. 7659, the law reimposing the death
penalty, is unconstitutional. In the Brief and (original) Motion for Reconsideration filed by his previous counsel, 3 this
transcendental issue was not brought up. Hence, it was not passed upon by this Court in its Decision affirming the trial
court's sentence of death. 4

The Constitution Abolished Death Penalty

Section 19, Article III of the 1987 Constitution provides:

"Sec. 19. (1) Excessive fines shall not be imposed, nor cruel, degrading or inhuman punishment inflicted. Neither
shall death penalty be imposed, unless for compelling reasons involving heinous crimes, the Congress hereafter provides
for it. Any death penalty already imposed shall be reduced to reclusion perpetua." (Underscoring supplied)

The second and third sentences of the above provision are new and had not been written in the 1935, 1973 or even in the
1986 "Freedom Constitution." They proscribe the imposition 5 of the death penalty "unless for compelling reasons involving
heinous crimes, Congress provides for it," and reduced "any death penalty already imposed" to reclusion perpetua. The
provision has both a prospective aspect (it bars the future imposition of the penalty) and a retroactive one (it reduces
imposed capital sentences to the lesser penalty of imprisonment).

This two-fold aspect is significant. It stresses that the Constitution did not merely suspend the imposition of the death
penalty, but in fact completely abolished it from the statute books. The automatic commutation or reduction to reclusion
perpetua of any death penalty extant as of the effectivity of the Constitution clearly recognizes that, while the conviction of
an accused for a capital crime remains, death as a penalty ceased to exist in our penal laws and thus may no longer be
carried out. This is the clear intent of the framers of our Constitution. As Comm. Bernas exclaimed, 6 "(t)he majority voted
for the constitutional abolition of the death penalty."

Citing this and other similar pronouncements of the distinguished Concom delegate, Mme. Justice Ameurfina Melencio-
Herrera emphasized, 7 "It is thus clear that when Fr. Bernas sponsored the provision regarding the non-imposition of the
death penalty, what he had in mind was the total abolition and removal from the statute books of the death penalty. This
became the intent of the framers of the Constitution when they approved the provision and made it a part of the Bill of
Rights." With such abolition as a premise, restoration thereof becomes an exception to a constitutional mandate. Being an
exception and thus in derogation of the Constitution, it must then be strictly construed against the State and liberally in
favor of the people. 8 In this light, RA 7659 enjoys no presumption of constitutionality.

The Constitution Strictly Limits

Congressional Prerogative to Prescribe Death

To me, it is very clear that the Constitution (1) effectively removed the death penalty from the then existing statutes but (2)
authorized Congress to restore it at some future time to enable or empower courts to re-impose it on condition that it
(Congress) 9 finds "compelling reasons, involving heinous crimes." The language of the Constitution is emphatic (even if
"awkward" 10): the authority of Congress to "provide for it" is not absolute. Rather, it is strictly limited:

(1) by "compelling reasons" that may arise after the Constitution became effective; and

(2) to crimes which Congress should identify or define or characterize as "heinous."

The Constitution inexorably placed upon Congress the burden of determining the existence of "compelling reasons" and of
defining what crimes are "heinous" before it could exercise its law-making prerogative to restore the death penalty. For

108
clarity's sake, may I emphasize that Congress, by law; prescribes the death penalty on certain crimes; and courts, by their
decisions, impose it on individual offenders found guilty beyond reasonable doubt of committing said crimes.

In the exercise of this fundamental mandate, Congress enacted RA 7659 11 to "provide for it" (the death penalty) (1) by
amending certain provisions of the Revised Penal Code; 12 (2) by incorporating a new article therein; 13 and (3) by
amending certain special laws. 14

But RA 7659 did not change the nature or the elements of the crimes stated in the Penal Code and in the special laws. It
merely made the penalty more severe. Neither did its provisions (other than the preamble, which was cast in general terms)
discuss or justify the reasons for the more severe sanction, either collectively for all the offenses or individually for each of
them. LLjur

Generally, it merely reinstated the concept of and the method by which the death penalty had been imposed until February
2, 1987, when the Constitution took effect as follows: (1) a person is convicted of a capital offense; and (2) the commission
of which was accompanied by aggravating circumstances not outweighed by mitigating circumstances.

The basic question then is: In enacting RA 7659, did Congress exceed the limited authority granted it by the Constitution?
More legally put: In reviving the death penalty, did Congress act with grave abuse of discretion or in excess of the very
limited power or jurisdiction conferred on it by Art. III, Sec. 19? The answer, I respectfully submit, is YES.

Heinous Crimes

To repeat, while the Constitution limited the power of Congress to prescribe the death penalty ONLY to "heinous" crimes,
it did not define or characterize the meaning of "heinous". Neither did Congress. As already stated, RA 7659 itself merely
selected some existing crimes for which it prescribed death as an applicable penalty. It did not give a standard or a
characterization by which courts may be able to appreciate the heinousness of a crime. I concede that Congress was only
too well aware of its constitutionally limited power. In deference thereto, it included a paragraph in the preambular or
"whereas" clauses of RA 7659, as follows:

"WHEREAS, the crimes punishable by death under this Act are heinous for being grievous, odious and hateful offenses
and which, by reason of their inherent or manifest wickedness, viciousness, atrocity and perversity are repugnant and
outrageous to the common standards and norms of decency and morality in a just, civilized and ordered society."

In my humble view, however, the foregoing clause is clearly an insufficient definition or characterization of what a heinous
crime is. It simply and gratuitously declared certain crimes to be "heinous" without adequately justifying its bases therefor.
It supplies no useful, workable, clear and unambiguous standard by which the presence of heinousness can be determined.
Calling the crimes "grievous, odious and hateful" is not a substitute for an objective juridical definition. Neither is the
description "inherent or manifest wickedness, viciousness, atrocity and perversity." Describing blood as blue does not
detract from its being crimson in fact; and renaming gumamela as rose will not arm it with thorns.

Besides, a preamble is really not an integral part of a law. It is merely an introduction to show its intent or purposes. It
cannot be the origin of rights and obligations. Where the meaning of a statute is clear and unambiguous, the preamble can
neither expand nor restrict its operation, much less prevail over its text. 15 In this case, it cannot be the authoritative source
to show compliance with the Constitution.

As already alluded to, RA 7659 merely amended certain laws to prescribe death as the maximum imposable penalty once
the court appreciates the presence or absence of aggravating circumstances. 16 There's nothing really new that Congress
did which it could not have otherwise done had such provision not been included in our fundamental law. cdt

In other words, it just reinstated capital punishment for crimes which were already punishable with death prior to the
effectivity of the 1987 Constitution. With the possible exception of plunder and qualified bribery, 17 no new crimes were
introduced by RA 7659. The offenses punished by death under said law were already so punishable by the Revised Penal
Code 18 and by special laws. In short, Sec. 19, Article III of the Constitution did not have any impact upon the legislative
action. It was effectively ignored by Congress in enacting the capital punishment law.

During the debate on Senate Bill No. 891 which later became RA 7659, Sen. Jose Lina, in answer to a question of Sen.
Ernesto Maceda, wryly said: 19

"So we did not go that far from the Revised Penal Code, Mr. President, and from existing special laws which, before abolition
of the death penalty, had already death as the maximum penalty."

By merely reimposing capital punishment on the very same crimes which were already penalized with death prior to the
charter's effectivity, Congress I submit has not fulfilled its specific and positive constitutional duty. If the Constitutional
Commission intended merely to allow Congress to prescribe death for these very same crimes, it would not have written
109
Sec. 19 of Article III into the fundamental law. But the stubborn fact is it did. Verily, the intention to 1) delete the death
penalty from our criminal laws and 2) make its restoration possible only under and subject to stringent conditions is evident
not only from the language of the Constitution but also from the charter debates on this matter.

The critical phrase "unless for compelling reasons involving heinous crimes" was an amendment introduced by Comm.
Christian Monsod. In explaining what possible crimes could qualify as heinous, he and Comm. Jose Suarez agreed on
"organized murder" or "brutal murder of a rape victim". 20 Note that the honorable commissioners did not just say "murder"
but organized murder; not just rape but brutal murder of a rape victim. While the debates were admittedly rather scanty, I
believe that the available information shows that, when deliberating on "heinousness", the Constitutional Commission did
not have in mind the offenses already existing and already penalized with death. I also believe that the heinousness clause
requires that:

1) the crimes should be entirely new offenses, the elements of which have an inherent quality, degree or level of
perversity, depravity or viciousness unheard of until then; or

2) even existing crimes, provided some new element or essential ingredient like "organized" or "brutal" is added to
show their utter perversity, odiousness or malevolence; or

3) the means or method by which the crime, whether new or old, is carried out evinces a degree or magnitude of
extreme violence, evil, cruelty, atrocity, viciousness as to demonstrate its heinousness. 21

For this purpose, Congress could enact an entirely new set of circumstances to qualify the crime as "heinous", in the same
manner that the presence of treachery in a homicide aggravates the crime to murder for which a heavier penalty is
prescribed.

Compelling Reasons

Quite apart from requiring the attendant element of heinousness, the Constitution also directs Congress to determine
"compelling reasons" for the revival of the capital penalty. It is true that paragraphs 3 and 4 of the preamble of RA 7659 22
made some attempt at meeting this requirement. But such effort was at best feeble and inconsequential. It should be
remembered that every word or phrase in the Constitution is sacred and should never be ignored, cavalierly-treated or
brushed aside. Thus, I believe that the compelling reasons and the characterization of heinousness cannot be done
wholesale but must be shown for each and every crime, individually and separately.

The words "compelling reasons" were included in the Charter because, in the words of Comm. Monsod, "in the future,
circumstances may arise which we should not preclude today . . . and that the conditions and the situation (during the
deliberations of the Constitutional Commission) might change for very specific reasons" requiring the return of the
constitutionally-abhorred penalty.

In his sponsorship of House Bill No. 62 which later evolved into RA 7659, Congressman Pablo Garcia, in answer to
questions raised by Representative Edcel Lagman tried to explain these compelling reasons: 23

"MR. LAGMAN:

So what are the compelling reasons now, Mr. Speaker? . . .

MR. GARCIA (P.).

The worsening peace and order condition in the country, Mr. Speaker. That is one.

MR. LAGMAN.

So the compelling reason which the distinguished sponsor would like to justify or serve as an anchor for the
justification of the reimposition of the death penalty is the alleged worsening peace and order situation. The Gentleman
claims that that is one of the compelling reasons. But before we dissect this particular "compelling reason," may we know
what are the other compelling reasons, Mr. Speaker?

MR. GARCIA (P.)

Justice, Mr. Speaker.

MR. LAGMAN.

Justice.

110
MR. GARCIA (P.).

Yes, Mr. Speaker.

MR. LAGMAN.

Justice is a compelling reason, Mr. Speaker? Could the Gentleman kindly elaborate on that answer? Why is justice
a compelling reason as if justice was not obtained at the time the Constitution abolished the death penalty? Any compelling
reason should be a supervening circumstances after 1987.

MR. GARCIA (P.).

Mr. Speaker, I have repeatedly said again and again that if one lives in an organized society governed by law,
justice demands that crime be punished and that the penalty imposed be commensurate with the offense committed.

MR. LAGMAN.

The Gentleman would agree with me that when the Constitution speaks of the compelling reasons to justify the
reimposition of death penalty, it refers to reasons which would supervene or come after the approval of the 1987
Constitution. Is he submitting that justice, in his own concept of a commensurate penalty for the offense committed, was
not obtained in 1987 when the Constitution abolished the death penalty and the people ratified it?

MR. GARCIA (P.).

That is precisely why we are saying that now, under present conditions, because of the seriousness of the offenses
being committed at this time, justice demands that the appropriate penalty must be meted out for those who have committed
heinous crimes.

xxx xxx xxx

In short, Congressman Garcia invoked the preambular justifications of "worsening peace and order" and "justice". With all
due respect I submit that these grounds are not "compelling" enough to justify the revival of state-decreed deaths. In fact,
I dare say that these "reasons" were even non-existent. Statistics from the Philippine National Police show that the crime
volume and crime rate particularly on those legislated capital offenses did not worsen but in fact declined between 1987,
the date when the Constitution took effect, and 1993, the year when RA 7659 was enacted. Witness the following debate
24 also between Representatives Garcia and Lagman:

"MR. LAGMAN.

Very good, Mr. Speaker. Now, can we go to 1987. Could the Gentleman from Cebu inform us the volume of the
crime of murder in 1987?

MR. GARCIA (P.).

The volume of the crime of murder in 1987 is 12,305.

MR. LAGMAN.

So, the corresponding crime rate was 21 percent.

MR. GARCIA (P.).

Yes, Mr. Speaker.

MR. LAGMAN.

That was in 1987. Mr. Speaker, could the distinguished chairman inform us the volume of murder in 1988?

MR. GARCIA (P.).

It was 10,521, Mr. Speaker.

MR. LAGMAN.

Or it was a reduction from 12,305 in 1987 to 10,521 in 1988. Correspondingly, the crime rate in the very year after
the abolition of the death penalty was reduced from 21 percent to 18 percent. Is that correct, Mr. Speaker?

111
MR. GARCIA (P.).

That is correct, Mr. Speaker. Those are the statistics supplied by the PC.

MR. LAGMAN.

Now can we go again to 1987 when the Constitution abolished the death penalty? May we know from the
distinguished Gentleman the volume of robbery in 1987?

MR. GARCIA (P.).

Will the Gentleman state the figure? I will confirm it.

MR. LAGMAN.

No. Mr. Speaker, I am asking the question.

MR. GARCIA (P.).

It was 22,942, Mr. Speaker, and the crime rate was 40 percent.

MR. LAGMAN.

This was the year immediately after the abolition of the death penalty. Could the Gentleman tell us the volume of
robbery cases in 1988?

MR. GARCIA (P.).

It was 16,926, Mr. Speaker.

MR. LAGMAN.

Obviously, the Gentleman would agree with me, Mr. Speaker that the volume of robbery cases declined from
22,942 in 1987 or crime rate of 40 percent to 16,926 or a crime rate of 29 percent. Would the Gentleman confirm that, Mr.
Speaker?

MR. GARCIA (P.).

This is what the statistics say. I understand we are reading now from the same document.

MR. LAGMAN.

Now, going to homicide, the volume 1987 was 12,870 or a crime rate of 22 percent. The volume in 1988 was
11,132 or a crime rate of 19 percent. Would the Gentleman confirm that, Mr. Speaker?

MR. GARCIA (P.).

As I Said, Mr. Speaker, we are reading from the same document and I would not want to say that the Gentleman
is misreading the document that I have here.

MR. LAGMAN.

But would the Gentleman confirm that?

MR. GARCIA (P.).

The document speaks for itself."

When interpellated by Sen. Arturo Tolentino, Sen. Jose Lina gave some figures on the number of persons arrested in
regard to drug-related offenses in the year 1987 as compared to 1991: 25

"Let me cite this concrete statistics by the Dangerous Drug Board.

In 1987 — this was the year when the death penalty was abolished — the persons arrested in drug-related cases were
3,062, and the figure dropped to 2,686 in 1988. cdtai

By the way, I will furnish my Colleagues with a photocopy of this report.

112
From 3,062 in 1987, it dropped to 2,686. Again, it increased a bit to 2,862 in 1989. It still decreased to 2,202 in 1990, and
it increased again to 2,862 in 1991.

But in 1987, when the death penalty was abolished, as far as the drug-related cases are concerned, the figure continued a
downward trend, and there was no death penalty in this time from, 1988 to 1991."

In a further attempt to show compelling reasons, the proponents of the death penalty argue that its reimposition "would
pose as an effective deterrent against heinous crimes." 26 However no statistical data, no sufficient proof, empirical or
otherwise, have been submitted to show with any conclusiveness the relationship between the prescription of the death
penalty for certain offenses and the commission or non-commission thereof. This is a theory that can be debated on and
on, 27 in the same manner that another proposition — that the real deterrent to crime is the certainty of immediate arrest,
prosecution and conviction of the culprit without unnecessary risk, expense and inconvenience to the victim, his heirs or
his witnesses — can be argued indefinitely. 28 This debate can last till the academics grow weary of the spoken word, but
it would not lessen the constitutionally-imposed burden of Congress to act within the "heinousness" and "compelling
reasons" limits of its death-prescribing power.

Other Constitutional Rights

Militate Against RA 7659

It should be emphasized that the constitutional ban against the death penalty is included in our Bill of Rights. As such, it
should — like any other guarantee in favor of the accused — be zealously protected, 29 and any exception thereto
meticulously screened. Any doubt should be resolved in favor of the people, particularly where the right pertains to persons
accused of crimes. 30 Here the issue is not just crimes — but capital crimes!

So too, all our previous Constitutions, including the first one ordained at Malolos, guarantee that "(n)o person shall be
deprived of life, liberty or property without due process of law." 31 This primary right of the people to enjoy life — life at its
fullest, life in dignity and honor — is not only reiterated by the 1987 Charter but is in fact fortified by its other pro-life and
pro-human rights provisions. Hence, the Constitution values the dignity of every human person and guarantees full respect
for human rights, 32 expressly prohibits any form of torture 33 which is arguably a lesser penalty than death, emphasizes
the individual right to life by giving protection to the life of the mother and the unborn from the moment of conception 34
and establishes the people's rights to health, a balanced ecology and education. 35

This Constitutional explosion of concern for man more than property, for people more than the state, and for life more than
mere existence augurs well for the strict application of the constitutional limits against the revival of death penalty as the
final and irreversible exaction of society against its perceived enemies.

Indeed, volumes have been written about individual rights to free speech, assembly and even religion. But the most basic
and most important of these rights is the right to life. Without life, the other rights cease in their enjoyment, utility and
expression.

This opinion would not be complete without a word on the wrenching fact that the death penalty militates against the poor,
the powerless and the marginalized. The "Profile of 165 Death Row Convicts" submitted by the Free Legal Assistance
Group 36 highlights this sad fact:

"(1) Since the reimposition of the death penalty, 186 persons 37 have been sentenced to death. At the end of 1994,
there were 24 death penalty convicts, at the end of 1995, the number rose to 90; an average of seven (7) convicts per
month; double the monthly average of capital sentences imposed the prior year. From January to June 1996, the number
of death penalty convicts reached 72, an average of 12 convicts per month, almost double the monthly average of capital
sentences imposed in 1995.

(2) Of the 165 convicts polled, approximately twenty one percent (21%) earn between P200 to P2,900 monthly; while
approximately twenty seven percent (27%) earn between P3,000 to P3,999 monthly. Those earning above P4,000 monthly
are exceedingly few: seven percent (7%) earn between P4,000 to P4,999, four percent (4%) earn between P5,000 to
P5,999, seven percent (7%) earn between P6,000 to P6,999, those earning between P7,000 to P15,000 comprise only four
percent (4%), those earning P15,000 and above only one percent (1%). Approximately thirteen percent (13%) earn nothing
at all, while approximately two percent (2%) earn subsistence wages with another five percent (5%) earning variable
income. Approximately nine percent (9%) do not know how much they earn in a month. llibris

(3) Thus, approximately two-thirds of the convicts, about 112 of them, earn below the government-mandated minimum
monthly wage of P4,290; ten (10) of these earn below the official poverty line set by government. Twenty six (26) earn
between P4,500.00 and P11,000.00 monthly, indicating they belong to the middle class; only one (1) earns P30,000.00

113
monthly. Nine (9) convicts earn variable income or earn on a percentage or allowance basis; fifteen (15) convicts do not
know or are unsure of their monthly income. Twenty two (22) convicts earn nothing at all.

(4) In terms of occupation, approximately twenty one percent (21%) are agricultural workers or workers in animal
husbandry; of these, thirty (30), or almost one-fifth thereof, are farmers. Thirty five percent (35%) are in the transport and
construction industry, with thirty one (31) construction workers or workers in allied fields (carpentry, painting, welding) while
twenty seven (27) are transport workers (delivery, dispatcher, mechanic, tire man, truck helper) with sixteen (16) of them
drivers. Eighteen percent (18%) are in clerical, sales and service industries, with fourteen (14) sales workers (engaged in
buy and sell or fish, cigarette or rice vendors), twelve (12) service workers (butchers, beauticians, security guards,
shoemakers, tour guides, computer programmers, radio technicians) and four (4) clerks, (janitors, MERALCO employee
and clerk). About four percent (4%) are government workers, with six (6) persons belonging to the armed services (AFP,
PNP and even CAFGU). Professionals, administrative employee and executives comprise only three percent (3%), nine
percent (9%) are unemployed.

(5) None of the DRC's use English as their medium of communication. About forty four percent (44%), or slightly less
than half speak and understand Tagalog; twenty six percent (26%), or about one-fourth, speak and understand Cebuano.
The rest speak and understand Bicolano, Ilocano, Ilonggo, Kapampangan, Pangasinense and Waray. One (1) convict is a
foreign national and speaks and understand Niponggo.

(6) Approximately twelve percent (12%) graduated from college, about forty seven percent (47%) finished varying
levels of elementary education with twenty seven (27) graduating from elementary. About thirty five percent (35%), fifty
eight (58) convicts, finished varying levels of high school, with more than half of them graduating from high school. Two (2)
convicts finished vocational education; nine (9) convicts did not study at all."

The foregoing profile based on age, language and socio-economic situations sufficiently demonstrates that RA 7659 has
militated against the poor and the powerless in society — those who cannot afford the legal services necessary in capital
crimes, where extensive preparation, investigation, research and presentation are required. The best example to show the
sad plight of the underprivileged is this very case where the crucial issue of constitutionality was woefully omitted in the
proceedings in the trial court and even before this Court until the Free Legal Assistance Group belatedly brought it up in
the Supplemental Motion for Reconsideration.

To the poor and unlettered, it is bad enough that the law is complex and written in a strange, incomprehensible language.
Worse still, judicial proceedings are themselves complicated, intimidating and damning. The net effect of having a death
penalty that is imposed more often than not upon the impecunious is to engender in the minds of the latter, a sense —
unfounded, to be sure, but unhealthy nevertheless — of the unequal balance of the scales of justice. cdtai

Most assuredly, it may be contended that the foregoing arguments, and in particular, the statistics above-cited, are in a
very real sense prone to be misleading, and that regardless of the socio-economic profile of the DRCs, the law reviving
capital punishment does not in any way single out or discriminate against the poor, the unlettered or the underprivileged.
To put it in another way, as far as the disadvantaged are concerned, the law would still be complex and written in a strange
and incomprehensible language, and judicial proceedings complicated and intimidating, whether the ultimate penalty
involved be life (sentence) or death. Another aspect of the whole controversy is that, whatever the penalties set by law, it
seems to me that there will always be a certain class or classes of people in our society who, by reason of their poverty,
lack of educational attainment and employment opportunities, are consequently confined to living, working and subsisting
in less-than-ideal environments, amidst less-than-genteel neighbors similarly situated as themselves, and are therefore
inherently more prone to be involved (as victims or perpetrators) in vices, violence and crime. So from that perspective, the
law reviving the death penalty neither improves nor worsens their lot substantially. Or, to be more precise, such law may
even be said to help improve their situation (at least in theory) by posing a much stronger deterrent to the commission of
heinous crimes.

However, such a viewpoint simply ignores the very basic differences that exist in the situations of the poor and the non-
poor. Precisely because the underprivileged are what they are, they require and deserve a greater degree of protection
and assistance from our laws and Constitution, and from the courts and the State, so that in spite of themselves, they can
be empowered to rise above themselves and their situation. The basic postulates for such a position are, I think, simply
that everyone ultimately wants to better himself and that we cannot better ourselves individually to any significant degree if
we are unable to advance as an entire people and nation. All the pro-poor provisions of the Constitution point in this
direction. Yet we are faced with this law that effectively inflicts the ultimate punishment on none other than the poor and
disadvantaged in the greater majority of cases, and which penalty, being so obviously final and so irreversibly permanent,
erases all hope of reform, of change for the better. This law, I submit, has no place in our legal, judicial and constitutional
firmament.

Epilogue
114
In sum, I respectfully submit that:

(1) The 1987 Constitution abolished the death penalty from our statute books. It did not merely suspend or prohibit its
imposition.

(2) The Charter effectively granted a new right: the constitutional right against the death penalty, which is really a
species of the right to life.

(3) Any law reviving the capital penalty must be strictly construed against the State and liberally in favor of the accused
because such a statute denigrates the Constitution, impinges on a basic right and tends to deny equal justice to the
underprivileged. liblex

(4) Every word or phrase in the Constitution is sacred and should never be ignored, cavalierly-treated or brushed
aside.

(5) Congressional power to prescribe death is severely limited by two concurrent requirements:

(a) First, Congress must provide a set of attendant circumstances which the prosecution must prove beyond
reasonable doubt, apart from the elements of the crime and itself. Congress must explain why and how these circumstances
define or characterize the crime as "heinous".

(b) Second, Congress has also the duty of laying out clear and specific reasons which arose after the effectivity of the
Constitution compelling the enactment of the law. It bears repeating that these requirements are inseparable. They must
both be present in view of the specific constitutional mandate — "for compelling reasons involving heinous crimes." The
compelling reason must flow from the heinous nature of the offense.

(6) In every law reviving the capital penalty, the heinousness and compelling reasons must be set out for each and
every crime, and not just for all crimes generally and collectively.

"Thou shall not kill" is a fundamental commandment to all Christians, as well as to the rest of the "sovereign Filipino people"
who believe in Almighty God. 38 While the Catholic Church, to which the vast majority of our people belong, acknowledges
the power of public authorities to prescribe the death penalty, it advisedly limits such prerogative only to "cases of extreme
gravity." 39 To quote Pope John Paul II in his encyclical Evangelium Vitae (A Hymn to Life), 40 "punishment must be
carefully evaluated and decided upon, and ought not go to the extreme of executing the offender except in cases of absolute
necessity: in other words, when it would not be possible otherwise to defend society . . . (which is) very rare, if not practically
non-existent."

Although not absolutely banning it, both the Constitution and the Church indubitably abhor the death penalty. Both are pro-
people and pro-life. Both clearly recognize the primacy of human life over and above even the state which man created
precisely to protect, cherish and defend him. The Constitution reluctantly allows capital punishment only for "compelling
reasons involving heinous crimes" just as the Church grudgingly permits it only for reasons of "absolute necessity" involving
crimes of "extreme gravity", which are very rare and practically non-existent.

In the face of these evident truisms, I ask: Has Congress, in enacting RA 7659, amply discharged its constitutional burden
of proving the existence of "compelling reasons" to prescribe death against well-defined "heinous" crimes?

I respectfully submit it has not.

WHEREFORE, premises considered, I respectfully vote to grant partially the Supplemental Motion for Reconsideration and
to modify the dispositive portion of the decision of the trial court by deleting the words "DEATH, as provided for under RA
7659," and substitute therefor reclusion perpetua.

I further vote to declare RA 7659 unconstitutional insofar as it prescribes the penalty of death for the crimes mentioned in
its text. Cdasia

18. Victory Liner vs. Bellosillo, A.M. MTJ-00-1321, 10 March 2004, 425 SCRA 79.

FIRST DIVISION

[A.M. No. MTJ-00-1321. March 10, 2004.]

VICTORY LINER, INC., represented by JOHNNY T. HERNANDEZ, President, complainant, vs. JUDGE REYNALDO B.
BELLOSILLO, respondent.

Songko Kho & Lepasura for complainants.


115
SYNOPSIS

In a criminal case for reckless imprudence resulting in homicide pending before the respondent judge, complainant Victory
Liner, Inc. (VLI) owner of the bus which accidentally hit and fatally injured Marciana Bautista Morales assails the acts of
respondent judge: in ordering the impounding of the bus involved in the accident and in requiring the posting of a cash
bond for its release; in fixing an excessive bail bond for driver Reino de la Cruz in Criminal Case No. 10512; and in
increasing the bail bond of one of VLI's driver Edwin Serrano in Criminal Case No. 9373 from P60,000 to P350,000 and
requiring that it be posted in cash. Despite the resignation of respondent judge, the OCA Consultant recommended
imposition of a fine of P20,000 against him, finding the abovementioned acts erroneous. The other charges against him for
inaction on a pending motion and for compelling the police authorities to file a criminal case against De la Cruz were
dismissed for lack of evidence. TAEDcS

On appeal, the Supreme Court held that absence of a Court ruling on the legality of impounding vehicles involved in an
accident, in addition to the bail bond required for the provisional liberty of the accused drivers, implies that there is yet no
clear-cut policy or rule on the matter. This negates a finding of gross ignorance of the law or grave abuse of authority of
respondent judge. Moreover, even assuming that said acts were erroneous, such are errors of judgment that cannot be the
subject of a disciplinary action absent proof of fraud, dishonesty, corruption or bad faith. The right forum to determine the
legality of respondent's order requiring VLI to post a cash bond for the release of the impounded vehicle is in the proper
court, not by way of this administrative case. The Supreme Court found the respondent judge administratively liable for
imposing excessive cash bail bonds on accused Reino de la Cruz and Edwin Serrano, violating the constitutional right of
the accused to bail. SEcADa

SYLLABUS

1. JUDICIAL ETHICS; JUDGES; ADMINISTRATIVE COMPLAINT AGAINST JUDGES, NOT RENDERED MOOT
AND ACADEMIC BY RESPONDENT JUDGE'S RESIGNATION; CASE AT BAR. — Verily, the resignation of respondent
Judge Bellosillo does not render moot and academic the instant administrative case. The jurisdiction that the Court had at
the time of the filing of the administrative complaint is not lost by the mere fact that the respondent judge ceased to be in
office during the pendency of this case. The Court retains its jurisdiction to pronounce the respondent official innocent or
guilty of the charges against him. A contrary rule would be fraught with injustice and pregnant with dreadful and dangerous
implications.

2. REMEDIAL LAW; COURTS; HIERARCHY OF COURTS; DETERMINATION OF ISSUES MUST BE RAISED IN


PROPER COURTS, NOT DIRECTLY TO THE SUPREME COURT AND BY WAY OF AN ADMINISTRATIVE CASE; CASE
AT BAR. — This administrative case is not the right forum to determine the issue of the legality of respondent's order
requiring VLI to post a cash bond for the release of its impounded vehicle. VLI should have raised that issue in the proper
courts and not directly to us, and much less by way of an administrative case. There is after all a hierarchy of courts. As
we have said in Santiago v. Vasquez, the propensity of litigants and lawyers to disregard the hierarchy of courts in our
judicial system by seeking a ruling directly from us must be put to a halt. . . . To allow VLI to raise that issue before us and
obtain a ruling thereon directly from us through an administrative case would be to countenance a disregard of the
established rules of procedure and of the hierarchy of courts. VLI would thus be able to evade compliance with the
requirements inherent in the filing of a proper petition, including the payment of docket fees. Hence, we shall shun from
passing upon that issue in this case. ISDCHA

3. JUDICIAL ETHICS; JUDGES; ADMINISTRATIVE COMPLAINT AGAINST JUDGES; JUDGE NOT LIABLE FOR
ERRONEOUS RULING IN THE ABSENCE OF PROOF OF FRAUD, DISHONESTY, CORRUPTION OR BAD FAITH;
CASE AT BAR. — The absence of a ruling in Cañas v. Castigador on the legality of the impounding of vehicles involved in
an accident, as well as the foregoing statements of VLI in its Motion to Resolve, implies that there is yet no clear-cut policy
or rule on the matter. They would, therefore, negate a finding of gross ignorance of the law or grave abuse of authority on
the part of respondent Judge. Moreover, even assuming that the acts of the respondent in ordering the impounding and
subsequent re-impounding of the subject vehicle and in requiring the posting of a cash bond for its release were erroneous,
as found by OCA Consultant Atienza, such are errors of judgment that cannot be the subject of a disciplinary action absent
proof of fraud, dishonesty, corruption, or bad faith. A judge may not be held administratively liable for every erroneous order
or decision he renders.

4. CONSTITUTIONAL LAW; RIGHTS OF THE ACCUSED; RIGHT TO BAIL; AMOUNT OF BAIL SHOULD BE
REASONABLE; CASE AT BAR. — The Constitution guarantees to every person under legal custody the right to bail except
those charged with offenses punishable with reclusion perpetua when evidence of guilt is strong. Section 9, Rule 114 of
the 1985 Rules on Criminal Procedure, as amended, provides that in fixing the amount of bail . . . The amount of bail should,
therefore, be reasonable at all times. It should be high enough to assure the presence of the accused when required, but
no higher than is reasonably calculated to serve this purpose. Excessive bail shall not be required. In implementing this

116
mandate, the accused's financial capability should particularly be considered. What is reasonable to a wealthy person may
not be so to a man charged with a like offense. Where the right to bail exists, it should not be rendered nugatory by requiring
a sum that is excessive. . . . De la Cruz and Serrano were both charged with the offense of reckless imprudence resulting
in homicide. Although permanently employed as drivers of VLI, it could not be said that each was capable of posting a cash
bail bond of P50,000 and P350,000, respectively. In fixing such amounts, the respondent apparently did not take into
account the gravity of the offense charged and the financial capability of the accused. He thereby willfully disregarded the
guidelines under Section 9, Rule 114 of the 1985 Rules on Criminal Procedure, as amended, and the 2000 Bail Bond Guide
of the DOJ. In effect, he violated the constitutional right of the accused to bail, as well as the prohibition against excessive
bail, making the right, in the words of Justice Jackson, "a teasing illusion like a munificent bequest in a pauper's will."
TEAcCD

5. ID.; ID.; ID.; ID.; EXCESSIVE BAIL FIXED IN THE FORM OF CASH IS A DENIAL OF ACCUSED'S RIGHT TO
BAIL; CASE AT BAR. — The bail fixed by the respondent is all the more excessive because it was in the form of cash. The
posting of a cash bond would entail a transfer of assets into the possession of the court, and its procurement could work
untold hardship on the part of the accused as to have the effect of altogether denying the accused's constitutional right to
bail. On the other hand, a surety bond may be obtained by the accused upon the payment of a relatively small premium. A
surety or property bond does not require an actual financial outlay on the part of the bondsman or the property owner. Only
the reputation or credit standing of the bondsman or the expectancy of the price at which the property can be sold is placed
in the hands of the court to guarantee the production of the body of the accused at the various proceedings leading to
conviction or acquittal. While cash bail is authorized under our rules, the option to deposit cash in lieu of a surety bond
primarily belongs to the accused, as can be gleaned from the language of Section 14, Rule 114 of the 1985 Rules on
Criminal Procedure, as amended.

6. JUDICIAL ETHICS; JUDGES; ADMINISTRATIVE COMPLAINT AGAINST JUDGES; GROSS IGNORANCE OF


THE LAW; FAILURE OF JUDGE TO APPLY A BASIC LAW OR RULE, A CASE OF. — It bears repeating that judges
should exhibit more than cursory acquaintance with the basic legal norms and precepts, as well as with statutes and
procedural rules. As advocates of justice and visible representations of the law, they are expected to keep abreast with the
law and jurisprudence, and be proficient in the application and interpretation thereof. When the law or rule is basic, judges
owe it to their office to simply apply it; anything less than that is gross ignorance of the law. In light of our current
jurisprudence, the respondent should be fined in the amount of P10,000 for his act of imposing on accused De la Cruz and
Serrano an excessive bail to be posted in cash in violation of pertinent rules and guidelines, as well as the constitutional
right of the accused to bail and the proscription against excessive bail. EcIaTA

DECISION

DAVIDE, JR., C.J p:

For our resolution is the verified complaint of Victory Liner, Inc. (VLI) against respondent Judge Reynaldo B. Bellosillo, then
Presiding Judge of the Municipal Circuit Trial Court (MCTC) of Orani, Bataan, and Acting Presiding Judge of the MCTC of
Dinalupihan-Hermosa, Bataan, for gross ignorance of the law, grave abuse of authority, oppression, and inaction on a
pending motion. prLL

The antecedent facts are as follows:

On 2 March 2000, while a Victory Liner bus bearing Plate No. CWF-935 was cruising along the National Highway of
Dinalupihan, Bataan, it accidentally hit and fatally injured Marciana Bautista Morales. Marciana died the following day. VLI
shouldered all the funeral and burial expenses of Marciana. Subsequently, on 6 March 2000, VLI and the heirs of the victim
entered into an Agreement/Undertaking. 1 On 14 March 2000, after payment by VLI of the claims, Faustina M. Antonio, the
authorized and designated representative of the heirs of the victim, executed a Release of Claim 2 and an Affidavit of
Desistance 3 in favor of VLI and the driver Reino de la Cruz.

However, earlier or on 3 March 2000, two of Marciana’s sons Rolando B. Soriano and Jimmy B. Morales, who were also
signatories to the Agreement/Undertaking, executed a Pinagsamang Salaysay 4 against Reino de la Cruz. On the strength
of that document, a criminal complaint was filed with the MCTC of Dinalupihan-Hermosa, Bataan, for reckless imprudence
resulting in homicide, 5 which was docketed as Criminal Case No. 10512.

After preliminary examination, or on 13 March 2000, respondent Judge Bellosillo ordered the immediate issuance of a
warrant of arrest against De la Cruz and fixed his bail at P50,000 to be posted in cash. He further directed the Chief of
Police of Dinalupihan, Bataan, to immediately impound the bus involved in the accident, which could be released only upon
the posting of a cash bond in the amount of P50,000. 6

117
On 30 March 2000, VLI filed a Manifestation and Motion 7 manifesting that it was depositing to the court under protest a
cash bond of P50,000 for the release of its bus. After making the deposit, VLI’s counsel presented the receipt issued by
the Clerk of Court of MCTC, Dinalupihan, to the Chief of Police of Dinalupihan, Bataan, who then released the bus.

On 4 April 2000, VLI filed with respondent’s court a petition 8 to declare null and void the order directing it to post bond for
the release of its bus. This petition was, however, dismissed for improper venue and lack of jurisdiction.

On that same day also, respondent Judge Bellosillo issued an order directing the Chief of Police of Dinalupihan, Bataan,
and his deputies and investigators to explain in writing why they should not be held in contempt of court for, and be
administratively charged with, having released without a court order the Victory Liner bus involved in Criminal Case No.
10512. Thus, the bus was re-impounded by the police authorities of Dinalupihan, Bataan.

Subsequently, on 18 April 2000, respondent Judge acted on VLI’s Manifestation and Motion dated 30 March 2000 and
issued an order 9 for the release of the bus.

On 23 June 2000, VLI filed a verified complaint 10 with the Office of the Court Administrator (OCA) claiming that the
respondent (a) is guilty of gross ignorance of the law in impounding its bus and requiring it to post a cash bond for the
release of the bus; (b) gravely abused his authority when it revoked the surety bond of one of VLI’s driver Edwin Serrano
in Criminal Case No. 9373; (c) knowingly rendered an unjust and oppressive order when he increased the bond to P350,000
and required that it be posted in cash; (d) gravely abused his authority when he ordered the police authorities of Dinalupihan,
Bataan, to file a case against Reino de la Cruz; and (e) is guilty of inaction or dereliction of duty in failing to resolve, despite
the lapse of two months, VLI's petition for the nullification of the order requiring the posting of a cash bond for the release
of the bus involved in the accident. Later, VLI filed with the Office of the Chief Justice a verified supplemental complaint
against the respondent, which was forthwith indorsed to the OCA.

In his comment, 11 respondent Judge Bellosillo explains that in the exercise of his sound discretion and in the greater
interest of justice and fair play, he required a cash bond of P50,000 for the release of the police-impounded vehicle to
answer for damages by way of subsidiary liability in case of accused’s insolvency. The requirement of a bond for the release
of impounded vehicles involved in reckless imprudence cases is practiced not only by him but by other judges throughout
the country.

As for his order for the re-impounding of the Victory Liner bus, respondent Judge claims that it was just under the
circumstances considering that its prior release was illegal. The payment of cash bond for the release of the impounded
vehicle was made by the VLI when respondent Judge was at his official station in the MCTC of Orani-Samal, Bataan. Thus,
in his absence, no order could have been issued for the release of the impounded vehicle. If ever said vehicle had to be
re-impounded, it was the fault of VLI’s counsel, as he was the one who misled the police authorities into believing that with
the payment of the bond, the bus could already be released.

The respondent justifies the substitution of the surety bond of accused Edwin Serrano in Criminal Case No. 9373 with a
cash bond on the strength of the prayer of the prosecutor that the bond be posted in cash in view of the gravity of the
offense. The Rules of Court leave to the discretion of trial judges the question of whether a bail should be posted in the
form of a corporate surety bond, property bond, cash deposit, or personal recognizance. Having found that Serrano's surety
bond, which was not even attached to the information but merely noted on the third page thereof, was in a minimal amount
and had expired already, he required a cash bond. He increased the bond after considering that Serrano was a fugitive
from justice.

Respondent Judge Bellosillo denies that he ordered the police authorities of Dinalupihan to file the criminal case against
Reino de la Cruz. He points to (a) the Pinagsamang Salaysay dated 3 March 2000 of Rolando B. Soriano and Jimmy B.
Morales, which was the basis for the filing of the criminal complaint by the police investigator and; (b) the fact that said
criminal complaint filed by the police investigator was duly approved by the Chief of Police. Thus, with these circumstances,
it could not be said that he compelled the police authorities into filing the criminal case.

As to the charge of dereliction of duty for failure to act on the petition for the nullification of the order requiring a bond for
the release of VLI's bus, respondent Judge avers that the same is baseless. Contrary to VLI's contention, he acted on that
petition as early as 10 April 2000, which was the date set by VLI's counsel for the hearing of such petition. 12 VLI's counsel
did not appear on that date and refused to accept or receive notices of hearing and court orders from court personnel.
AEIDTc

In his Report and Recommendation, retired Justice Narciso T. Atienza, the OCA Consultant to whom this case was referred
by the Court, submits that Judge Bellosillo's resignation, which was accepted by the Court En Banc effective 27 March
2002, does not render moot and academic the instant administrative complaint. He finds that the respondent Judge erred
in ordering the impounding of the Victory Liner bus and in requiring a cash bond of P50,000 for its release; in fixing an

118
excessive bail bond for Reino de la Cruz in Criminal Case No. 10512; and in increasing the bail bond of Edwin Serrano in
Criminal Case No. 9373 unconscionably from P60,000 to P350,000. He then recommends that the respondent Judge be
penalized with a fine of P20,000. But for lack of evidence, he exonerates respondent Judge from complainant's charge that
he compelled the police authorities into filing the criminal case against De la Cruz. As to respondent's alleged inaction on
VLI's petition to declare null and void the order requiring a bond for the release of the subject bus, Justice Atienza finds
that the said petition was resolved on 10 April 2000, right on the day it was submitted for resolution. Likewise, he disregards
the additional charges in the supplemental complaint, there being no showing that the respondent received a copy thereof.

Justice Atienza also notes that in A.M. No. 00-1293, promulgated on 5 July 2000, respondent Judge was reprimanded for
issuing a policy action and an order beyond the scope of his authority; and in MTJ No. 00-1308, promulgated on 16
December 2002, respondent Judge was found guilty of undue delay in rendering a decision and was ordered to pay a fine
of P11,000 to be taken from his retirement benefits. He further notes the pending administrative cases against respondent
Judge: (1) OCA IPI No. 96-232-MTJ for conduct unbecoming a judge; (2) OCA IPI No. 98-533-MTJ for ignorance of the
law, grave abuse of discretion, and gross misconduct; (3) OCA IPI No. 96-203-MTJ for issuing an unjust interlocutory order
and gross ignorance of the law; (4) A.M. No. 99-1222 for violation of the constitutional rights to information and to speedy
trial; and (5) undocketed cases for unprofessional and ill-mannered conduct, refusing to receive documents, and illegal
possession of firearms.

Verily, the resignation of respondent Judge Bellosillo does not render moot and academic the instant administrative case.
The jurisdiction that the Court had at the time of the filing of the administrative complaint is not lost by the mere fact that
the respondent judge ceased to be in office during the pendency of this case. The Court retains its jurisdiction to pronounce
the respondent official innocent or guilty of the charges against him. A contrary rule would be fraught with injustice and
pregnant with dreadful and dangerous implications. 13

We agree with Justice Atienza in exonerating the respondent from the charges of inaction on a pending motion and of
compelling the police authorities to file a criminal case against De la Cruz. We, however, hesitate to hold the respondent
administratively accountable for gross ignorance of the law in ordering (1) the impounding of the vehicle involved in the
vehicular accident and (2) the posting of a P50,000 bond for the release of the vehicle, both of which were found by OCA
Consultant Atienza to be erroneous.

Notably, in its Motion to Resolve, VLI submits that this case presents a good occasion for us to resolve, among other issues,
"the legality of the imposition by trial judges on bus operators to post bail bond for their impounded vehicles in accident
cases, in addition to the bail bond required for the provisional liberty of accused-drivers." According to VLI, our ruling on
this matter would guide trial court judges nationwide in accident cases so that bus operators and their personnel would not
be at the mercy of judges like the respondent in this case, who during his incumbency had been requiring vehicle owners
involved in accidents to post cash bonds for the release of impounded vehicles.

In Lacadin v. Mangino, 14 the respondent Judge therein was sought to be administratively liable for extending the lifetime
of a search warrant issued by him. We held that even if he may have committed an error of judgment or an abuse of
discretion for such act, he cannot be punished administratively therefor in the absence of proof that he was motivated by
ignominy or ill-will. Moreover, we ruled that the administrative case is not the right forum to determine whether the life of a
search warrant may be extended by the court upon proper motion filed before the expiration of the 10-day period.

Worth noting also is the case of Cañas v. Castigador. 15 In that case, an Isuzu trailer truck involved in a vehicular mishap
was ordered impounded in an Order of 11 September 1996 of the trial court where the criminal case against its driver was
pending. That order was addressed to the Chief of Police of General Trias, Cavite, or any officer of the law. In an earlier
order of 14 August 1996, the vehicle owner was required to surrender the truck to the court. Subsequently, on motion of
the prosecutor, the trial court declared the vehicle owner guilty of indirect contempt for continued defiance of the 11
September 1996 Order. However, upon the vehicle owner’s petition, we found respondent's order holding the petitioner
therein guilty of indirect contempt to be highly improper for several reasons. But we did not pass upon the issue of the
legality of the impounding of the vehicle involved in the vehicular accident. We did not declare the order for the impounding
of the vehicle to be illegal or unauthorized. If it were so, it could have been one of the several reasons for admonishing the
respondent Judge therein.

In the same vein, this administrative case is not the right forum to determine the issue of the legality of respondent's order
requiring VLI to post a cash bond for the release of its impounded vehicle. VLI should have raised that issue in the proper
courts and not directly to us, and much less by way of an administrative case. There is after all a hierarchy of courts. As
we have said in Santiago v. Vasquez, 16 the propensity of litigants and lawyers to disregard the hierarchy of courts in our
judicial system by seeking a ruling directly from us must be put to a halt. 17

It must be recalled that on 4 April 2000, VLI filed with respondent judge's court a Petition to Declare Order Directing Victory
Liner, Inc., to Post Bond for the Release of the Bus Null and Void. 18 In that petition, VLI submitted that there is no legal
119
basis for the order directing the impounding of the bus and the posting by the bus owner of a cash bond for its release, and
hence that order is void ab initio. 19 However, despite notice, VLI’s counsel Atty. Reynaldo R. Romero did not appear on
10 April 2000, the schedule 20 for the hearing of that petition as set by him. 21 The respondent thereupon issued an order
22 dismissing the petition outright on grounds of improper venue and lack of jurisdiction, and ordering that a copy of the
said order be furnished VLI’s counsel at his given address. However, VLI's counsel reportedly refused to accept or receive
from court personnel notices of hearing and court orders. And, according to respondent Judge, he (VLI's counsel) never
appeared and continued not to appear before the respondent for reasons known only to him. 23 VLI cannot, therefore,
resurrect that issue directly before us, and much less through a mere verified administrative complaint or motion to resolve.

To allow VLI to raise that issue before us and obtain a ruling thereon directly from us through an administrative case would
be to countenance a disregard of the established rules of procedure and of the hierarchy of courts. VLI would thus be able
to evade compliance with the requirements inherent in the filing of a proper petition, including the payment of docket fees.
Hence, we shall shun from passing upon that issue in this case. DEHaTC

In any event, the absence of a ruling in Cañas v. Castigador on the legality of the impounding of vehicles involved in an
accident, as well as the foregoing statements of VLI in its Motion to Resolve, implies that there is yet no clear-cut policy or
rule on the matter. They would, therefore, negate a finding of gross ignorance of the law or grave abuse of authority on the
part of respondent Judge. Moreover, even assuming that the acts of the respondent in ordering the impounding and
subsequent re-impounding of the subject vehicle and in requiring the posting of a cash bond for its release were erroneous,
as found by OCA Consultant Atienza, such are errors of judgment that cannot be the subject of a disciplinary action absent
proof of fraud, dishonesty, corruption, or bad faith. 24 A judge may not be held administratively liable for every erroneous
order or decision he renders. To hold otherwise would be to render a judicial office unbearable, for no one called upon to
try the facts or interpret the law in the process of administering justice can be infallible in rendering a judgment. For a judge
to be held administratively liable for ignorance of the law, it is necessary that the law be sufficiently basic that all that the
judge must do is to simply apply it; 25 or that the error must be gross or patent, deliberate and malicious, or incurred with
evident bad faith. 26

We, however, find respondent administratively liable for imposing excessive cash bail bonds on accused Reino de la Cruz
in Criminal Case No. 10512 and Edwin Serrano in Criminal Case No. 9373.

The Constitution guarantees to every person under legal custody the right to bail except those charged with offenses
punishable with reclusion perpetua when evidence of guilt is strong. 27 Section 9, Rule 114 of the 1985 Rules on Criminal
Procedure, as amended, 28 provides that in fixing the amount of bail, the judge must primarily consider the following factors:

a) Financial ability of the accused to give bail;

b) Nature and circumstances of the offense;

c) Penalty for the offense charged;

d) Character and reputation of the accused;

e) Age and health of the accused;

f) The weight of the evidence against the accused;

g) Probability of the accused appearing in trial;

h) Forfeiture of the bonds;

i) The fact that the accused was a fugitive from justice when arrested; and

j) The pendency of other cases in which the accused is under bond.

The amount of bail should, therefore, be reasonable at all times. It should be high enough to assure the presence of the
accused when required, but no higher than is reasonably calculated to serve this purpose. Excessive bail shall not be
required. 29 In implementing this mandate, the accused’s financial capability should particularly be considered. What is
reasonable to a wealthy person may not be so to a man charged with a like offense. Where the right to bail exists, it should
not be rendered nugatory by requiring a sum that is excessive. 30

Moreover, under the 2000 Bail Bond Guide of the Department of Justice (DOJ), crimes of reckless imprudence resulting in
homicide and with violation of the Land Transportation and Traffic Code, bail shall be P30,000 regardless of the number of
deaths. 31

120
De la Cruz and Serrano were both charged with the offense of reckless imprudence resulting in homicide. Although
permanently employed as drivers of VLI, it could not be said that each was capable of posting a cash bail bond of P50,000
and P350,000, respectively. In fixing such amounts, the respondent apparently did not take into account the gravity of the
offense charged and the financial capability of the accused. He thereby willfully disregarded the guidelines under Section
9, Rule 114 of the 1985 Rules on Criminal Procedure, as amended, and the 2000 Bail Bond Guide of the DOJ. In effect,
he violated the constitutional right of the accused to bail, as well as the prohibition against excessive bail, making the right,
in the words of Justice Jackson, “a teasing illusion like a munificent bequest in a pauper’s will.” 32

The bail fixed by the respondent is all the more excessive because it was in the form of cash. The posting of a cash bond
would entail a transfer of assets into the possession of the court, and its procurement could work untold hardship on the
part of the accused as to have the effect of altogether denying the accused’s constitutional right to bail. On the other hand,
a surety bond may be obtained by the accused upon the payment of a relatively small premium. A surety or property bond
does not require an actual financial outlay on the part of the bondsman or the property owner. Only the reputation or credit
standing of the bondsman or the expectancy of the price at which the property can be sold is placed in the hands of the
court to guarantee the production of the body of the accused at the various proceedings leading to conviction or acquittal.
33

While cash bail is authorized under our rules, the option to deposit cash in lieu of a surety bond primarily belongs to the
accused, 34 as can be gleaned from the language of Section 14, Rule 114 of the 1985 Rules on Criminal Procedure, as
amended, 35 which read:

SEC. 14. Deposit of cash as bail. — The accused or any person acting in his behalf may deposit in cash with the
nearest collector of internal revenue or provincial, city, or municipal treasurer the amount of bail fixed by the court or
recommended by the prosecutor who investigated or filed the case, and upon submission of a proper certificate of deposit
and of a written undertaking showing compliance with the requirements of Section 2 hereof, the accused shall be discharged
from custody . . .

The respondent judge, therefore, grossly erred in converting Serrano's surety bond to cash bond and in demanding that
De la Cruz post a cash bond to obtain their provisional liberty. 36

It bears repeating that judges should exhibit more than cursory acquaintance with the basic legal norms and precepts, as
well as with statutes and procedural rules. As advocates of justice and visible representations of the law, they are expected
to keep abreast with the law and jurisprudence, and be proficient in the application and interpretation thereof. When the
law or rule is basic, judges owe it to their office to simply apply it; anything less than that is gross ignorance of the law. 37

In light of our current jurisprudence, 38 the respondent should be fined in the amount of P10,000 for his act of imposing on
accused De la Cruz and Serrano an excessive bail to be posted in cash in violation of pertinent rules and guidelines, as
well as the constitutional right of the accused to bail and the proscription against excessive bail.

WHEREFORE, for gross ignorance of the law and oppression in imposing excessive cash bail bonds on Reino de la Cruz
in Criminal Case No. 10512 and Edwin Serrano in Criminal Case No. 9373, respondent Judge Reynaldo B. Bellosillo is
hereby ORDERED to pay a fine of Ten Thousand Pesos (P10,000) to be taken from his retirement benefits. TSHIDa

SO ORDERED.

Ynares-Santiago, Carpio and Azcuna, JJ., concur.

Panganiban, J., is on official leave.

19. Calimlim, et al. v. Ramirez, et al., G.R. No. 34362, November 12, 1982, 118 SCRA 399.

FIRST DIVISION

[G.R. No. L-34362. November 19, 1982.]

MODESTA CALIMLIM AND LAMBERTO MAGALI IN HIS CAPACITY AS ADMINISTRATOR OF THE ESTATE OF
DOMINGO MAGALI, petitioner, vs. HON. PEDRO A. RAMIREZ IN HIS CAPACITY AS PRESIDING JUDGE OF THE
COURT OF FIRST INSTANCE OF PANGASINAN, BRANCH I, and FRANCISCO RAMOS, respondents.

Eugenio Ramos for petitioners.

Rogelio P. Closa for respondents.

SYNOPSIS

121
Petitioners Modesta Calimlim, surviving spouse of Domingo Magali, and Lamberto Magali, in his capacity as administrator
of the estate of the deceased, upon learning that Domingo's title over a parcel of land had been cancelled, filed a petition
with respondent Court of First Instance, sitting as a cadastral court, praying for the cancellation of TCT No. 68568 issued
in favor of Independent Mercantile Corporation. Petitioners alleged therein that they are the true owners of the property,
and that TCT No. 68568 which they sought to cancel was issued as a result of errors which were not of their own making.
The cadastral court, however, dismissed the petition for lack of merit on the basis only of memoranda of the parties.
Petitioners did not appeal. Two-and-a-half years after said dismissal, petitioners filed Civil Case No. SCC-180 praying for
the cancellation of the conveyances and sales that had been made with respect to the subject parcel of land previously
registered in the name of Domingo Magali and covered by TCT No. 9138. Private respondent Francisco Ramos who
claimed to have bought the property from Independent Mercantile Corporation was named a defendant in said civil suit.
Private respondent moved for dismissal which was granted by respondent court on the ground of res judicata.

On appeal by certiorari, the Supreme Court reversed and set aside the order of dismissal and ordered respondent court to
conduct further proceedings in the case. The Court held that the defense of res judicata does not obtain to bar the second
suit since the judgment rendered by the Court of First Instance in the first case was null and void because when sitting as
a land registration court, it had no jurisdiction to pass upon controversial issues involving ownership or title to real property.

Assailed order reversed and set aside.

SYLLABUS

1. REMEDIAL LAW; JUDGMENTS; RES JUDICATA OR BAR BY PRIOR JUDGMENT; THE ESSENTIAL ELEMENT
THAT THE JUDGMENT IN THE PRIOR ACTION MUST HAVE BEEN RENDERED BY A COURT WITH THE PROPER
JURISDICTION TO TAKE COGNIZANCE OF THE PROCEEDING IN WHICH THE PRIOR JUDGMENT OR ORDER WAS
RENDERED DOES NOT EXIST IN CASE AT BAR. — It is error to consider the dismissal of the petition filed by the herein
petitioner in LRC Record No. 39492 for the cancellation of TCT. No. 68568 as a bar by prior judgment against the filing of
Civil Case No. SCC-180. In order to avail of the defense of res judicata, it must be shown, among others, that the judgment
in the prior action must have been rendered by a court with the proper jurisdiction to take cognizance of the proceeding in
which the prior judgment or order was rendered. If there is lack of jurisdiction over the subject matter of the suit or of the
parties, the judgment or order cannot operate as an adjudication of the controversy. (2 Moran Comments on the Rules of
Court, 1970 Edition, p. 364.) This essential element of the defense of bar by prior judgment or res judicata does not exist
in the case presently considered.

2. CIVIL LAW; LAND REGISTRATION; A COURT OF FIRST INSTANCE, ACTING AS A LAND REGISTRATION
COURT, CANNOT PASS UPON QUESTIONS INVOLVING OWNERSHIP OR TITLE TO REAL PROPERTY, OR ANY
INCIDENT WHERE THE ISSUES INVOLVED HAVE BECOME CONTROVERSIAL. — It has been settled by consistent
rulings of this Court that a court of first instance, acting as a land registration court, is a court of limited and special
jurisdiction. As such, its proceedings are not adequate for the litigation of issues pertaining to an ordinary civil action, such
as, questions involving ownership or title to real property. (Bareng vs. Shintoist Shrine and Japanese Charity Bureau, 83
SCRA 418; Manalo vs. Mariano. 69 SCRA 80; In re: Nicanor T. Santos, 102 SCRA 474.) In Hu Chon Sunpongco vs. Heirs
of Nicolas Ronquillo, L-27040, December 19, 1970, 36 SCRA 395, we have held that: "Section 112 of Act 496 confers
authority upon the land registration court to order the cancellation, alteration or amendment of a certificate of title but
withdraws from the Court the power to pass upon any question concerning ownership of the registered property, or any
incident where the issues involved have become controversial."

3. ID.; ID.; ID.; CASE AT BAR. — It may hardly be questioned that the issues raised by the petitioners in their petition
to cancel TCT No. 68568 refer to the ownership or title over the property covered thereby. The said petition presented
before the respondent Court in the exercise of its limited jurisdiction as a cadastral court, the question of who should be
considered the true and lawful owner of the parcel of land embraced in said title. The petitioners alleged therein that they
are the true owners of the property, and that TCT No. 68568 which they sought to cancel was issued as a result of the
errors which were not of their own making. In short, the petition raised a highly controversial matter which is beyond the
judicial competence of a cadastral court to pass upon or to adjudicate.

4. REMEDIAL LAW; COURTS; JURISDICTION; CONFERRED BY LAW AS A RULE AND MAY NOT BE WAIVED
BY CONSENT OR AGREEMENT OF THE PARTIES; LACK OF JURISDICTION MAY BE RAISED AT ANY STAGE OF
THE PROCEEDINGS, EVEN ON APPEAL. — A rule that had been settled by unquestioned acceptance and upheld in
decisions so numerous to cite is that the jurisdiction of a court over the subject matter of the action is a matter of law and
may not be conferred by consent or agreement of the parties. The lack of jurisdiction of a court may be raised at any stage
of the proceedings, even on appeal.

5. ID.; ID.; ID.; ID.; RULING IN TIJAM VS. SIBONGHANOY SHOULD BE REGARDED AS A MERE EXCEPTION
TO THE RULE. — This doctrine has been qualified by recent pronouncements which stemmed principally from the ruling
122
in the cited case of Tijam vs. Sibonghanoy, 23 SCRA 29. It is to be regretted, however, that the holding in said case had
been applied to situations which were obviously not contemplated therein. The exceptional circumstance involved in
Sibonghanoy which justified the departure from the accepted concept of non-waivability of objection to jurisdiction has been
ignored and, instead a blanket doctrine had been repeatedly upheld that rendered the supposed ruling in Sibonghanoy not
as the exception, but rather the general rule, virtually overthrowing altogether the time-honored principle that the issue of
jurisdiction is not lost by waiver or by estoppel. In Sibonghanoy, the defense of lack of jurisdiction of the court that rendered
the questioned ruling was held to be barred by estoppel by laches. It was ruled that the lack of jurisdiction having been
raised for the first time in a motion to dismiss filed almost fifteen (15) years after the questioned ruling had been rendered,
such plea may no longer be raised for being barred by laches.

6. ID.; CIVIL PROCEDURE; ESTOPPEL BY LACHES; PETITIONERS CANNOT BE FAULTED WITH LACHES AS
THEY IMMEDIATELY FILED A PETITION IN COURT AFTER LEARNING THAT THEIR TITLE HAD BEEN CANCELLED.
— The petitioners in the instant case may not be faulted with laches. When they learned that the title to the property owned
by them had erroneously and illegally been cancelled and registered in the name of another entity or person who had no
right to the same. they filed a petition to cancel the latter's title. It is unfortunate that in pursuing said remedy, their counsel
had to invoke the authority of the respondent Court as a cadastral court, instead of its capacity as a court of general
jurisdiction. Their petition to cancel the title in the name of Independent Mercantile Corporation was dismissed upon a
finding by the respondent Court that the same was "without merit." No explanation was given for such dismissal nor why
the petition lacked merit. There was no hearing, and the petition was resolved solely on the basis of memoranda filed by
the parties which do not appear of record. It is even a possibility that such dismissal was in view of the realization of the
respondent Court that, sitting as a cadastral court, it lacked the authority to entertain the petition involving as it does a highly
controversial issue. Upon such petition being dismissed, the petitioners instituted Civil Case No. SCC-180 on January 1,
1971, or only two and one-half years after the dismissal of their petition in LRC Record No. 39492. Hence, we see no
unreasonable delay in the assertion by the petitioners of their right to claim the property which rightfully belongs to them.
They can hardly be presumed to have abandoned or waived such right by inaction within an unreasonable length of time
or inexcusable negligence. In short, their filing of Civil Case No. SCC-180 which in itself is an implied non- acceptance of
the validity of the proceedings had in LRC Record No. 39492 may not be deemed barred by estoppel by laches.

7. ID.; ID.; ESTOPPEL; WHEN IT MAY BE INVOKED. — It is neither fair nor legal to bind a party by the suit or
proceeding which was taken cognizance of in a court which lacks jurisdiction over the same irrespective of the attendant
circumstances. The equitable defense of estoppel requires knowledge or consciousness of the facts upon which it is based.
The same thing is true with estoppel by conduct which may be asserted only when it is shown, among others, that the
representation must have been made with knowledge of the facts and that the party to whom it was made is ignorant of the
truth of the matter. (De Castro vs. Gineta, 27 SCRA 623.) The filing of an action or suit in a court that does not possess
jurisdiction to entertain the same may not be presumed to be deliberate and intended to secure a ruling which could later
be annulled if not favorable to the party who filed such suit or proceeding. Instituting such an action is not a one-sided affair.
It can just as well be prejudicial to the one who filed the action or suit in the event that he obtains a favorable judgment
therein which could also be attacked for having been rendered without jurisdiction.

8. ID.; COURTS; JURISDICTION; FILING OF PETITION FOR COMPLAINT IN COURT DOES NOT RESULT IN
WAIVER ON LACK OF COURT'S JURISDICTION. — The determination of the correct jurisdiction of a court is not a simple
matter. It can raise highly debatable issues of such importance that the highest tribunal of the land is given the exclusive
appellate jurisdiction to entertain the same. The point simply is that when a party commits error in filing his suit or proceeding
in a court that lacks jurisdiction to take cognizance of the same, such act may not at once be deemed sufficient basis of
estoppel. It could have been the result of an honest mistake, or of divergent interpretations of doubtful legal provisions. If
any fault is to be imputed to a party taking such course of action, part of the blame should be placed on the court which
shall entertain the suit, thereby lulling the parties into believing that they pursued their remedies in the correct forum. Under
the rules, it is the duty of the court to dismiss an action "whenever it appears that the court has no jurisdiction over the
subject-matter." (Sec. 2, Rule 9, Rules of Court.) Should the court render a judgment without jurisdiction, such judgment
may be impeached or annulled for lack of jurisdiction (Sec. 30, Rule 132, ibid), within ten (10) years from the finality of the
same. (Art. 1144, par. 3, Civil Code.)

DECISION

VASQUEZ, J p:

The dismissal of Civil Case No. SCC-180 filed by the herein petitioners in the respondent Court against the private
respondent is sought to be annulled and set aside by this Petition For Review On Certiorari.

The antecedent material facts are not disputed. Sometime in 1961, a judgment for a sum of money was rendered in favor
of Independent Mercantile Corporation against a certain Manuel Magali by the Municipal Court of Manila in Civil Case No.

123
85136. After said judgment became final, a writ of execution was issued on July 31, 1961. The Notice of Levy made on
September 21, 1961 on a parcel of land covered by Transfer Certificate of Title No. 9138 registered in the name of "Domingo
Magali, married to Modesta Calimlim", specified that the said levy was only against "all rights, title, action, interest and
participation of the defendant Manuel Magali over the parcel of land described in this title." The Certificate of Sale executed
by the Provincial Sheriff of Pangasinan on October 17, 1961 in favor of Independent Mercantile Corporation also stated
that the sale referred only to the rights and interest of Manuel Magali over the land described in TCT No. 9138. Manuel
Magali is one of the several children of Domingo Magali who had died in 1940 and herein petitioner Modesta Calimlim.

However, when the Sheriff issued the final Deed of Sale on January 26, 1963, it was erroneously stated therein that the
sale was with respect to "the parcel of land described in this title" (referring to TCT No. 9138) and not only over the rights
and interest of Manuel Magali in the same. The execution of the said final Deed of Sale was annotated at the back of said
title.

On February 23, 1967, Independent Mercantile Corporation filed a petition in the respondent Court to compel Manuel Magali
to surrender the owner's duplicate of TCT No. 9138 in order that the same may be cancelled and a new one issued in the
name of the said corporation. Not being the registered owner and the title not being in his possession, Manuel Magali failed
to comply with the order of the Court directing him to surrender the said title. On June 20, 1967, Independent Mercantile
Corporation filed an ex-parte petition to declare TCT No. 9138 as cancelled and to issue a new title in its name. The said
petition was granted by the respondent Court and in its Order dated July 13, 1967, it directed the issuance of a new
certificate of title in the name of the Independent Mercantile Corporation and the cancellation of TCT No. 9138. By virtue of
said Order, the Register of Deeds of Pangasinan issued a new title in the name of the corporation, identified as TCT No.
68568.

On November 21, 1967, petitioner Modesta Calimlim, surviving spouse of Domingo Magali, upon learning that her
husband's title over the parcel of land had been cancelled, filed a petition with the respondent Court, sitting as a cadastral
court, praying for the cancellation of TCT No. 68568. An opposition to the said petition was filed by Independent Mercantile
Corporation. After the parties submitted their respective Memoranda, the respondent Court issued an Order dated July 3,
1968 dismissing the petition. (Rollo, pp. 31-38.)

The herein petitioners did not appeal the dismissal of the petition they filed in LRC Record No. 39492 for the cancellation
of TCT No. 68568. Instead, on January 11, 1971, they filed the complaint in Civil Case No. SCC-180 praying for the
cancellation of the conveyances and sales that had been made with respect to the property covered by TCT No. 9138
previously registered in the name of Domingo Magali, married to Modesta Calimlim. Named as defendant in said civil case
was herein private respondent Francisco Ramos who claimed to have bought the property from Independent Mercantile
Corporation on July 25, 1967. Private respondent Francisco Ramos, however, failed to obtain a title over the property in
his name in view of the existence of an adverse claim annotated on the title thereof at the instance of the herein petitioners.

Private respondent Francisco Ramos filed a Motion To Dismiss Civil Case No. SCC-180 on the ground that the same is
barred by prior judgment or by the statute of limitations (Rollo, pp. 42-45). Resolving the said Motion, the respondent Court,
in its Order dated April 21, 1971, dismissed Civil Case No. SCC-180 on the ground of estoppel by prior judgment. (Ibid, pp.
10-13.) A Motion For Reconsideration filed by the petitioners was denied by the respondent Judge in his Order of September
2, 1971. (Ibid, pp. 13-15.) A second Motion For Reconsideration was similarly denied in the Order dated September 29,
1971. (Rollo, pp. 16-17.) Hence, this Petition. cdll

We find merit in this appeal.

It is error to consider the dismissal of the petition filed by the herein petitioner in LRC Record No. 39492 for the cancellation
of TCT No. 68568 as a bar by prior judgment against the filing of Civil Case No. SCC-180. In order to avail of the defense
of res judicata it must be shown, among others, that the judgment in the prior action must have been rendered by s court
with the proper jurisdiction to take cognizance of the proceeding in which the prior judgment or order was rendered. If there
is lack of jurisdiction over the subject-matter of the suit or of the parties, the judgment or order cannot operate as an
adjudication of the controversy. (2 Moran Comments on the Rules of Court, 1970 Edition, p. 364.) This essential element
of the defense of bar by prior judgment or res judicata does not exist in the case presently considered.

The petition filed by the herein petitioners in LRC Record No. 39492 was an apparent invocation of the authority of the
respondent Court sitting as a land registration court. Although the said petition did not so state, that reliance was apparently
placed on Section 112 of the Land Registration Act. It has been settled by consistent rulings of this Court that a court of
first instance, acting as a land registration court, is a court of limited and special jurisdiction. As such, its proceedings are
not adequate for the litigation of issues pertaining to an ordinary civil action, such as, questions involving ownership or title
to real property. (Bareng vs. Shintoist Shrine and Japanese Charity Bureau, 83 SCRA 418; Manalo vs. Mariano, 69 SCRA
80; In re: Nicanor T. Santos, 102 SCRA 747; Santos vs. Aquino, 101 SCRA 377.) In Hu chon Sunpongco vs. Heirs of
Nicolas Ronquillo, L-27040, December 19, 1970, 36 SCRA 395, we have held that:
124
"Section 112 of Act 496 confers authority upon the land registration court to order the cancellation, alteration or amendment
of a certificate of title but withdraws from the Court the power to pass upon any question concerning ownership of the
registered property, or any incident where the issues involved have become controversial."

It may hardly be questioned that the issues raised by the petitioners in their petition to cancel TCT No. 68568 refer to the
ownership or title over the property covered thereby. The said petition presented before the respondent Court in the exercise
of its limited jurisdiction as a cadastral court, the question of who should be considered the true and lawful owner of the
parcel of land embraced in said title. The petitioners alleged therein that they are the true owners of the property, and that
TCT No. 68568 which they sought to cancel was issued as a result of the errors which were not of their own making. In
short, the petition raised a highly controversial matter which is beyond the judicial competence of a cadastral court to pass
upon or to adjudicate.

It may neither be claimed that the parties have mutually agreed to submit the aforesaid issues for the determination by the
court, it being a fact that herein private respondent was not a party in the petition in LRC Record No. 39492. Incidentally,
although the said petition was filed by the herein petitioners on November 21, 1967, the Opposition filed by Independent
Mercantile Corporation to the said petition made no mention of the alleged sale of the property in question in favor of private
respondent Francisco Ramos on July 5, 1967. This circumstance places in grave doubt the sincerity of said sale and the
claim that the private respondent was an innocent purchaser for value of the property in question. Cdpr

In the order of the respondent Judge dated September 29, 1971 denying the second motion for reconsideration, he cited
the case of Tijam vs. Sibonghanoy, 23 SCRA 29, to uphold the view that the petitioners are deemed estopped from
questioning the jurisdiction of the respondent Court in having taken cognizance of the petition for cancellation of TCT No.
68568, they being the ones who invoked the jurisdiction of the said Court to grant the affirmative relief prayed for therein.
We are of the opinion that the ruling laid down in Sibonghanoy may not be applied herein. Neither its factual backdrop nor
the philosophy of the doctrine therein expounded fits the case at bar.

A rule that had been settled by unquestioned acceptance and upheld in decisions so numerous to cite is that the jurisdiction
of a court over the subject-matter of the action is a matter of law and may not be conferred by consent or agreement of the
parties. The lack of jurisdiction of a court may be raised at any stage of the proceedings, even on appeal. This doctrine has
been qualified by recent pronouncements which stemmed principally from the ruling in the cited case of Sibonghanoy. It is
to be regretted, however, that the holding in said case had been applied to situations which were obviously not contemplated
therein. The exceptional circumstance involved in Sibonghanoy which justified the departure from the accepted concept of
non-waivability of objection to jurisdiction has been ignored and, instead a blanket doctrine had been repeatedly upheld
that rendered the supposed ruling in Sibonghanoy not as the exception, but rather the general rule, virtually overthrowing
altogether the time-honored principle that the issue of jurisdiction is not lost by waiver or by estoppel.

In Sibonghanoy, the defense of lack of jurisdiction of the court that rendered the questioned ruling was held to be barred
by estoppel by laches. It was ruled that the lack of jurisdiction having been raised for the first time in a motion to dismiss
filed almost fifteen (15) years after the questioned ruling had been rendered, such a plea may no longer be raised for being
barred by laches. As defined in said case, laches is "failure or neglect, for an unreasonable and unexplained length of time,
to do that which, by exercising due diligence, could or should have been done earlier; it is negligence or omission to assert
a right within a reasonable time, warranting a presumption that the party entitled to assert has abandoned it or declined to
assert it."

The petitioners in the instant case may not be faulted with laches. When they learned that the title to the property owned
by them had erroneously and illegally been cancelled and registered in the name of another entity or person who had no
right to the same, they filed a petition to cancel the latter's title. It is unfortunate that in pursuing said remedy, their counsel
had to invoke the authority of the respondent Court as a cadastral court, instead of its capacity as a court of general
jurisdiction. Their petition to cancel the title in the name of Independent Mercantile Corporation was dismissed upon a
finding by the respondent Court that the same was "without merit." No explanation was given for such dismissal nor why
the petition lacked merit. There was no hearing, and the petition was resolved solely on the basis of memoranda filed by
the parties which do not appear of record. It is even a possibility that such dismissal was in view of the realization of the
respondent Court that, sitting as a cadastral court, it lacked the authority to entertain the petition involving as it does a highly
controversial issue. Upon such petition being dismissed, the petitioners instituted Civil Case No. SCC-180 on January 1,
1971, or only two and one-half years after the dismissal of their petition in LRC Record No. 39492. Hence, we see no
unreasonable delay in the assertion by the petitioners of their right to claim the property which rightfully belongs to them.
They can hardly be presumed to have abandoned or waived such right by inaction within an unreasonable length of time
or inexcusable negligence. In short, their filing of Civil Case No. SCC-180 which in itself is an implied non-acceptance of
the validity of the proceedings had in LRC Record No. 39492 may not be deemed barred by estoppel by laches. llcd

125
It is neither fair nor legal to bind a party by the result of a suit or proceeding which was taken cognizance of in a court which
lacks jurisdiction over the came irrespective of the attendant circumstances. The equitable defense of estoppel requires
knowledge or consciousness of the facts upon which it is based. The same thing is true with estoppel by conduct which
may be asserted only when it is shown, among others, that the representation must have been made with knowledge of
the facts and that the party to whom it was made is ignorant of the truth of the matter. (De Castro vs. Gineta, 27 SCRA
623.) The filing of an action or suit in a court that does not possess jurisdiction to entertain the same may not be presumed
to be deliberate and intended to secure a ruling which could later be annulled if not favorable to the party who filed such
suit or proceeding. Instituting such an action is not a one-sided affair. It can just as well be prejudicial to the one who filed
the action or suit in the event that he obtains a favorable judgment therein which could also be attacked for having been
rendered without jurisdiction. The determination of the correct jurisdiction of a court is not a simple matter. It can raise highly
debatable issues of such importance that the highest tribunal of the land is given the exclusive appellate jurisdiction to
entertain the same. The point simply is that when a party commits error in filing his suit or proceeding in a court that lacks
jurisdiction to take cognizance of the same, such act may not at once be deemed sufficient basis of estoppel. It could have
been the result of an honest mistake, or of divergent interpretations of doubtful legal provisions. If any fault is to be imputed
to a party taking such course of action, part of the blame should be placed on the court which shall entertain the suit,
thereby lulling the parties into believing that they pursued their remedies in the correct forum. Under the rules, it is the duty
of the court to dismiss an action "whenever it appears that the court has no jurisdiction over the subject matter." (Sec. 2,
Rule 9, Rules of Court.) Should the court render a judgment without jurisdiction, such judgment may be impeached or
annulled for lack of jurisdiction (Sec. 30, Rule 132, Ibid.), within ten (10) years from the finality of the same. (Art. 1144, par.
3, Civil Code.)

The inequity of barring the petitioners from vindicating their right over their property in Civil Case No. SCC-180 is rendered
more acute in the face of the undisputed fact that the property in question admittedly belonged to the petitioners, and that
the title in the name of the private respondent was the result of an error committed by the Provincial Sheriff in issuing the
deed of sale in the execution proceeding. The justness of the relief sought by herein petitioners may not be ignored or
rendered futile by reason of a doctrine which is of highly doubtful applicability herein. LLjur

WHEREFORE, the Orders appealed from are hereby REVERSED and SET ASIDE. The Motion To Dismiss filed by the
private respondent in Civil Case No. SCC-180 shall be deemed denied and the respondent Court is ordered to conduct
further proceedings in the case. With costs against the private respondent.

SO ORDERED.

Teehankee (Chairman), Melencio-Herrera, Plana, Relova and Gutierrez, Jr., JJ., concur.

20. Davao Light & Power., Inc. vs. Court of Appeals, et al., G.R. No. 93262, 204 SCRA 343 (1991)

EN BANC

[G.R. No. 93262. November 29, 1991.]

DAVAO LIGHT & POWER CO., INC., petitioner, vs. THE COURT OF APPEALS, QUEENSLAND HOTEL or MOTEL or
QUEENSLAND TOURIST INN, and TEODORICO ADARNA, respondents.

Breva & Breva Law Offices for petitioner.

Goc-Ong & Associates for private respondents.

SYLLABUS

1. REMEDIAL LAW; ACTIONS; JURISDICTION; HOW ACQUIRED. — An action or proceeding is commenced by


the filing of the complaint or other initiatory pleading. By that act, the jurisdiction of the court over the subject matter or
nature of the action or proceeding is invoked or called into activity; and it is thus that the court acquires jurisdiction over
said subject matter or nature of the action. And it is by that self-same act of the plaintiff (or petitioner) of filing the complaint
(or other appropriate pleading) — by which he signifies his submission to the court's power and authority — that jurisdiction
is acquired by the court over his person. On the other hand, jurisdiction over the person of the defendant is obtained, as
above stated, by the service of summons or other coercive process upon him or by his voluntary submission to the authority
of the court.

2. ID.; PROVISIONAL REMEDIES; PRELIMINARY ATTACHMENT; DEFINED. — A preliminary attachment may be


defined, paraphrasing the Rules of Court, as the provisional remedy in virtue of which a plaintiff or other proper party may,
at the commencement of the action or at any time thereafter, have the property of the adverse party taken into the custody
126
of the court as security for the satisfaction of any judgment that may be recovered. It is a remedy which is purely statutory
in respect of which the law requires a strict construction of the provisions granting it. Withal no principle, statutory or
jurisprudential, prohibits its issuance by any court before acquisition of jurisdiction over the person of the defendant.

3. ID.; ID.; ID.; PHRASE "AT THE COMMENCEMENT OF THE ACTION," CONSTRUED. — Rule 57 in fact speaks
of the grant of the remedy "at the commencement of the action or at any time thereafter." The phrase, "at the
commencement of the action," obviously refers to the date of the filing of the complaint — which, as above pointed out, is
the date that marks "the commencement of the action;" and the reference plainly is to a time before summons is served on
the defendant, or even before summons issues.

4. ID.; ID.; ID.; WRIT MAY BE ISSUED EX-PARTE. — What the rule is saying quite clearly is that after an action is
properly commenced — by the filing of the complaint and the payment of all requisite docket and other fees — the plaintiff
may apply for and obtain a writ of preliminary attachment upon fulfillment of the pertinent requisites laid down by law, and
that he may do so at any time, either before or after service of summons on the defendant. And this indeed, has been the
immemorial practice sanctioned by the courts: for the plaintiff or other proper party to incorporate the application for
attachment in the complaint or other appropriate pleading (counterclaim, cross-claim, third-party claim) and for the Trial
Court to issue the writ ex-parte at the commencement of the action if it finds the application otherwise sufficient in form and
substance.

5. ID.; ID.; ID.; HEARING ON APPLICATION THEREON, GENERALLY NOT NECESSARY. — In Toledo v. Burgos
this Court ruled that a hearing on a motion or application for preliminary attachment is not generally necessary unless
otherwise directed by the Trial Court in its discretion. And in Filinvest Credit Corporation v. Relova, the Court declared that
"(n)othing in the Rules of Court makes notice and hearing indispensable and mandatory requisites for the issuance of a
writ of attachment."

6. ID.; ID.; ID.; ID.; BASIS OF GRANT. — The only pre-requisite is that the Court be satisfied, upon consideration of
"the affidavit of the applicant or of some other person who personally knows the facts, that a sufficient cause of action
exists, that the case is one of those mentioned in Section 1 . . . (Rule 57), that there is no other sufficient security for the
claim sought to be enforced by the action, and that the amount due to the applicant, or the value of the property the
possession of which he is entitled to recover, is as much as the sum for which the order (of attachment) is granted above
all legal counterclaims." If the court be so satisfied, the "order of attachment shall be granted," and the writ shall issue upon
the applicant's posting of a bond executed to the adverse party in an amount to be fixed by the judge, not exceeding the
plaintiff's claim, conditioned that the latter will pay all the costs which may be adjudged to the adverse party and all damages
which he may sustain by reason of the attachment, if the court shall finally adjudge that the applicant was not entitled
thereto."

7. ID.; ID.; ID.; ID.; REASON. — In Mindanao Savings & Loan Association, Inc. v. Court of Appeals, decided on April
18, 1989, decided on April 18, 1989, this Court had occasion to emphasize the postulate that no hearing is required on an
application for preliminary attachment, with notice to the defendant, for the reason that this "would defeat the objective of
the remedy . . . (since the) time which such a hearing would take, could be enough to enable the defendant to abscond or
dispose of his property before a writ of attachment issues." As observed by a former member of this Court, such a procedure
would warn absconding debtors-defendants of the commencement of the suit against them and the probable seizure of
their properties, and thus give them the advantage of time to hide their assets, leaving the creditor-plaintiff holding the
proverbial empty bag; it would place the creditor-applicant in danger of losing any security for a favorable judgment and
thus give him only an illusory victory.

8. ID.; ID.; ID.; HOW DISCHARGED. — There are two (2) ways of discharging an attachment: first, by the posting of
a counterbond; and second, by a showing of its improper or irregular issuance.

9. ID.; ID.; ID.; ID.; BY COUNTERBOND. — The submission of a counterbond is an efficacious mode of lifting an
attachment already enforced against property, or even of preventing its enforcement altogether. When property has already
been seized under attachment, the attachment may be discharged upon counterbond in accordance with Section 12 of
Rule 57. But even before actual levy on property, seizure under attachment may be prevented also upon counterbond. The
defendant need not wait until his property is seized before seeking the discharge of the attachment by a counterbond. This
is made possible by Section 5 of Rule 57.

10. ID.; ID.; ID.; ID.; BY MOTION TO DISCHARGE ON GROUND THAT THE SAME WAS IRREGULARLY OR
IMPROPERLY ISSUED. — Aside from the filing of a counterbond, a preliminary attachment may also be lifted or discharged
on the ground that it has been irregularly or improperly issued, in accordance with Section 13 of Rule 57. Like the first, this
second mode of lifting an attachment may be resorted to even before any property has beer levied on. Indeed, it may be
availed of after property has been released from a levy on attachment, as is made clear by said Section 13.

127
11. ID.; ID.; ID.; ID.; FIRST MODE SPEEDIER THAN THE SECOND. — The filing of a counterbond is a speedier way
of discharging the attachment writ maliciously sought out by the attaching creditor instead of the other way, which, in most
instances . . . would require presentation of evidence in a fullblown trial on the merits, and cannot easily be settled in a
pending incident of the case.

12. ID.; ID.; ID.; MAY NOT BE DISSOLVED BY A SHOWING OF ITS IRREGULAR OR IMPROPER ISSUANCE. —
(a) When an attachment may not be dissolved by a showing of its irregular or improper issuance:

". . . (W)hen the preliminary attachment is issued upon a ground which is at the same time the applicant's cause of action
e.g., 'an action for money or property embezzled or fraudulently misapplied or converted to his own use by a public officer,
or an officer of a corporation, or an attorney, factor, broker, agent, or clerk, in the course of his employment as such, or by
any other person in a fiduciary capacity, or for a willful violation of duty.' (Sec. 1 [b], Rule 57), or 'an action against a party
who has been guilty of fraud in contracting the debt or incurring the obligation upon which the action is brought' (Sec. 1 [d],
Rule 57), the defendant is not allowed to file a motion to dissolve the attachment under Section 13 of Rule 57 by offering
to show the falsity of the factual averments in the plaintiff's application and affidavits on which the writ was based — and
consequently that the writ based thereon had been improperly or irregularly issued (SEE Benitez v. I.A.C., 154 SCRA 41)
— the reason being that the hearing on such a motion for dissolution of the writ would be tantamount to a trial of the merits
of the action. In other words, the merits of the action would be ventilated at a mere hearing of a motion, instead of at the
regular trial. Therefore, when the writ of attachment is of this nature, the only way it can be dissolved is by a counterbond
(G.B. Inc. v. Sanchez, 98 Phil. 886)."

13. ID.; ID.; ID.; DISSOLUTION OF PRELIMINARY ATTACHMENT DOES NOT DISCHARGE SURETIES ON BOND;
REASON. — ". . . The dissolution of the preliminary attachment upon security given, or a showing of its irregular or improper
issuance, does not of course operate to discharge the sureties on plaintiff's own attachment bond. The reason is simple.
That bond is 'executed to the adverse party, . . . conditioned that the . . . (applicant) will pay all the costs which may be
adjudged to the adverse party and all damages which he may sustain by reason of the attachment, if the court shall finally
adjudge that the applicant was not entitled thereto' (SEC. 4, Rule 57). Hence, until that determination is made, as to the
applicant's entitlement to the attachment, his bond must stand and cannot be withdrawn."

DECISION

NARVASA, J p:

Subject of the appellate proceedings at bar is the decision of the Court of Appeals in CA-G.R. Sp. No. 1967 entitled
"Queensland Hotel, Inc., etc. and Adarna v. Davao Light & Power Co., Inc., promulgated on May 4, 1990. 1 That decision
nullified and set aside the writ of preliminary attachment issued by the Regional Trial Court of Davao City 2 in Civil Case
No. 19513-89 on application of the plaintiff (Davao Light & Power Co.), before the service of summons on the defendants
(herein respondents Queensland Co., Inc. and Adarna).

Following is the chronology of the undisputed material facts culled from the Appellate Tribunal's judgment of May 4, 1990.

1. On May 2, 1989 Davao Light & Power Co., Inc. (hereafter, simply Davao Light) filed a verified complaint for
recovery of a sum of money and damages against Queensland Hotel, etc. and Teodorico Adarna (docketed as Civil Case
No. 19613-89). The complaint contained an ex parte application for a writ of preliminary attachment.

2. On May 3, 1989 Judge Nartatez, to whose branch the case was assigned by raffle, issued an Order granting the
ex parte application and fixing the attachment bond at P4,600,513.37.

3. On May 11, 1989 the attachment bond having been submitted by Davao Light, the writ of attachment issued.

4. On May 12, 1989, the summons and a copy of the complaint, as well as the writ of attachment and a copy of the
attachment bond, were served on defendants Queensland and Adarna; and pursuant to the writ, the sheriff seized
properties belonging to the latter. LibLex

5. On September 6, 1989, defendants Queensland and Adarna filed a motion to discharge the attachment for lack of
jurisdiction to issue the same because at the time the order of attachment was promulgated (May 3, 1989) and the
attachment writ issued (May 11, 1989), the Trial Court had not yet acquired jurisdiction over the cause and over the persons
of the defendants.

6. On September 14, 1989, Davao Light filed an opposition to the motion to discharge attachment.

7. On September 19, 1989, the Trial Court issued an Order denying the motion to discharge.

128
This Order of September 19, 1989 was successfully challenged by Queensland and Adarna in a special civil action of
certiorari instituted by them in the Court of Appeals. The Order was, as aforestated, annulled by the Court of Appeals in its
Decision of May 4, 1990. The Appellate Court's decision closed with the following disposition:

". . . the Orders dated May 3, 1989 granting the issuance of a writ of preliminary attachment, dated September 19, 1989
denying the motion to discharge attachment; dated November 7, 1989 denying petitioner's motion for reconsideration; as
well as all other orders emanating therefrom, specially the Writ of Attachment dated May 11, 1989 and Notice of Levy on
Preliminary Attachment dated May 11, 1989, are hereby declared null and void and the attachment hereby ordered
DISCHARGED."

The Appellate Tribunal declared that —

". . .While it is true that a prayer for the issuance of a writ of preliminary attachment may be included in the complaint, as is
usually done, it is likewise true that the Court does not acquire jurisdiction over the person of the defendant until he in duly
summoned or voluntarily appears, and adding the phrase that it be issued 'ex parte' does not confer said jurisdiction before
actual summons had been made, nor retroact jurisdiction upon summons being made. . . ."

It went on to say, citing Sievert v. Court of Appeals, 3 that "in a proceedings in attachment," the "critical time which must
be identified is . . . when the trial court acquires authority under law to act coercively against the defendant or his property
. . .;" and that " critical time is the time of the vesting of jurisdiction in the court over the person of the defendant in the main
case."

Reversal of this Decision of the Court of Appeals of May 4, 1990 is what Davao Light seeks in the present appellate
proceedings. cdrep

The question is whether or not a writ of preliminary attachment may issue ex parte against a defendant before acquisition
of jurisdiction of the latter's person by service of summons or his voluntary submission to the Court's authority.

The Court rules that the question must be answered in the affirmative and that consequently, the petition for review will
have to be granted.

It is incorrect to theorize that after an action or proceeding has been commenced and jurisdiction over the person of the
plaintiff has been vested in the court, but before the acquisition of jurisdiction over the person of the defendant (either by
service of summons or his voluntary submission to the court's authority), nothing can be validly done by the plaintiff or the
court. It is wrong to assume that the validity of acts done during this period should be dependent on, or held in suspension
until, the actual obtention of jurisdiction over the defendant's person. The obtention by the court of jurisdiction over the
person of the defendant is one thing; quite another is the acquisition of jurisdiction over the person of the plaintiff or over
the subject-matter or nature of the action, or the res or object thereof.

An action or proceeding is commenced by the filing of the complaint or other initiatory pleading. 4 By that act, the jurisdiction
of the court over the subject matter or nature of the action or proceeding is invoked or called into activity; 5 and it is thus
that the court acquires jurisdiction over said subject matter or nature of the action. 6 And it is by that self-same act of the
plaintiff (or petitioner) of filing the complaint (or other appropriate pleading) — by which he signifies his submission to the
court's power and authority — that jurisdiction is acquired by the court over his person. 7 On the other hand, jurisdiction
over the person of the defendant is obtained, as above stated, by the service of summons or other coercive process upon
him or by his voluntary submission to the authority of the court. 8

The events that follow the filing of the complaint as a matter of routine are well known. After the complaint is filed, summons
issues to the defendant, the summons is then transmitted to the sheriff, and finally, service of the summons is effected on
the defendant in any of the ways authorized by the Rules of Court. There is thus ordinarily some appreciable interval of
time between the day of the filing of the complaint and the day of service of summons of the defendant. During this period,
different acts may be done by the plaintiff or by the Court, which are of unquestionable validity and propriety. Among these,
for example, are the appointment of a guardian ad litem, 9 the grant of authority to the plaintiff to prosecute the suit as a
pauper litigant, 10 the amendment of the complaint by the plaintiff as a matter of right without leave of court, 11 authorization
by the Court of service of summons by publication, 12 the dismissal of the action by the plaintiff on mere notice. 13

This, too, is true with regard to the provisional remedies of preliminary attachment, preliminary injunction, receivership or
replevin. 14 They may be validly and properly applied for and granted even before the defendant is summoned or is heard
from. LibLex

A preliminary attachment may be defined, paraphrasing the Rules of Court, as the provisional remedy in virtue of which a
plaintiff or other proper party may, at the commencement of the action or at any time thereafter, have the property of the
adverse party taken into the custody of the court as security for the satisfaction of any judgment that may be recovered. 15
129
It is a remedy which is purely statutory in respect of which the law requires a strict construction of the provisions granting
it. 16 Withal no principle, statutory or jurisprudential, prohibits its issuance by any court before acquisition of jurisdiction
over the person of the defendant.

Rule 57 in fact speaks of the grant of the remedy "at the commencement of the action or at any time thereafter." 17 The
phrase, "at the commencement of the action," obviously refers to the date of the filing of the complaint — which, as above
pointed out, is the date that marks "the commencement of the action;" 18 and the reference plainly is to a time before
summons is served on the defendant, or even before summons issues. What the rule is saying quite clearly is that after an
action is properly commenced — by the filing of the complaint and the payment of all requisite docket and other fees — the
plaintiff may apply for and obtain a writ of preliminary attachment upon fulfillment of the pertinent requisites laid down by
law, and that he may do so at any time, either before or after service of summons on the defendant. And this indeed, has
been the immemorial practice sanctioned by the courts: for the plaintiff or other proper party to incorporate the application
for attachment in the complaint or other appropriate pleading (counterclaim, cross-claim, third-party claim) and for the Trial
Court to issue the writ ex-parte at the commencement of the action if it finds the application otherwise sufficient in form and
substance.

In Toledo v. Burgos, 19 this Court ruled that a hearing on a motion or application for preliminary attachment is not generally
necessary unless otherwise directed by the Trial Court in its discretion. 20 And in Filinvest Credit Corporation v. Relova, 21
the Court declared that "(n)othing in the Rules of Court makes notice and hearing indispensable and mandatory requisites
for the issuance of a writ of attachment." The only pre-requisite is that the Court be satisfied, upon consideration of "the
affidavit of the applicant or of some other person who personally knows the facts, that a sufficient cause of action exists,
that the case is one of those mentioned in Section 1 . . . (Rule 57), that there is no other sufficient security for the claim
sought to be enforced by the action, and that the amount due to the applicant, or the value of the property the possession
of which he is entitled to recover, is as much as the sum for which the order (of attachment) is granted above all legal
counterclaims." 22 If the court be so satisfied, the "order of attachment shall be granted," 23 and the writ shall issue upon
the applicant's posting of a bond executed to the adverse party in an amount to be fixed by the judge, not exceeding the
plaintiff's claim, conditioned that the latter will pay all the costs which may be adjudged to the adverse party and all damages
which he may sustain by reason of the attachment, if the court shall finally adjudge that the applicant was not entitled
thereto." 24

In Mindanao Savings & Loan Association, Inc. v. Court of Appeals, decided on April 18, 1989, 25 this Court had occasion
to emphasize the postulate that no hearing is required on an application for preliminary attachment, with notice to the
defendant, for the reason that this "would defeat the objective of the remedy . . . (since the) time which such a hearing
would take, could be enough to enable the defendant to abscond or dispose of his property before a writ of attachment
issues." As observed by a former member of this Court, 26 such a procedure would warn absconding debtors-defendants
of the commencement of the suit against them and the probable seizure of their properties, and thus give them the
advantage of time to hide their assets, leaving the creditor-plaintiff holding the proverbial empty bag; it would place the
creditor-applicant in danger of losing any security for a favorable judgment and thus give him only an illusory victory. Cdpr

Withal, ample modes of recourse against a preliminary attachment are secured by law to the defendant. The relative ease
with which a preliminary attachment may be obtained is matched and paralleled by the relative facility with which the
attachment may legitimately be prevented or frustrated. These modes of recourse against preliminary attachments granted
by Rule 57 were discussed at some length by the separate opinion in Mindanao Savings & Loans Asso. Inc. v. C.A., supra.

That separate opinion stressed that there are two (2) ways of discharging an attachment: first, by the posting of a
counterbond; and second, by a showing of its improper or irregular issuance.

1.0. The submission of a counterbond is an efficacious mode of lifting an attachment already enforced against property,
or even of preventing its enforcement altogether.

1.1. When property has already been seized under attachment, the attachment may be discharged upon counterbond
in accordance with Section 12 of Rule 57.

'SECTION 12. Discharge of attachment upon giving counterbond. — At any time after an order of attachment has been
granted, the party whose property has been attached or the person appearing in his behalf, may, upon reasonable notice
to the applicant, apply to the judge who granted the order, or to the judge of the court in which the action is pending, for an
order discharging the attachment wholly or in part on the security given . . . in an amount equal to the value of the property
attached as determined by the judge to secure the payment of any judgment that the attaching creditor may recover in the
action . . .'

130
1.2. But even before actual levy on property, seizure under attachment may be prevented also upon counterbond. The
defendant need not wait until his property is seized before seeking the discharge of the attachment by a counterbond. This
is made possible by Section 5 of Rule 57.

'SECTION 5. Manner of attaching property. — The officer executing the order shall without delay attach, to await
judgment and execution in the action, all the properties of the party against whom the order is issued in the province, not
exempt from execution, or so much thereof as may be sufficient to satisfy the applicant's demand, unless the former makes
a deposit with the clerk or judge of the court from which the order issued, or gives a counter-bond executed to the applicant,
in an amount sufficient to satisfy such demand besides costs, or in an amount equal to the value of the property which is
about to be attached, to secure payment to the applicant of any judgment which he may recover in the action. . . .' (Emphasis
supplied).

2.0. Aside from the filing of a counterbond, a preliminary attachment may also be lifted or discharged on the ground
that it has been irregularly or improperly issued, in accordance with Section 13 of Rule 57. Like the first, this second mode
of lifting ar attachment may be resorted to even before any property has beer levied on. Indeed, it may be availed of after
property has been released from a levy on attachment, as is made clear by said Section 13, viz.: cdphil

'SECTION 13. Discharge of attachment for improper or irregular issuance. — The party whose property has been
attached may also, at any time either BEFORE or AFTER the release of the attached property, or before any attachment
shall have been actually levied, upon reasonable notice to the attaching creditor, apply to the judge who granted the order,
or to the judge of the court in which the action is pending, for an order to discharge the attachment on the ground that the
same was improperly or irregularly issued. If the motion be made on affidavits on the part of the party whose property has
been attached, but not otherwise, the attaching creditor may oppose the same by counter-affidavits or other evidence in
addition to that on which the attachment was made. . . .' (Emphasis supplied).

This is so because "(a)s pointed out in Calderon v. I.A.C., 155 SCRA 531 (1987), 'The attachment debtor cannot be deemed
to have waived any defect in the issuance of the attachment writ by simply availing himself of one way of discharging the
attachment writ, instead of the other. Moreover, the filing of a counterbond is a speedier way of discharging the attachment
writ maliciously sought out by the attaching creditor instead of the other way, which, in most instances . . . would require
presentation of evidence in a fullblown trial on the merits, and cannot easily be settled in a pending incident of the case.'"
27

It may not be amiss to here reiterate other related principles dealt with in Mindanao Savings & Loans Asso. Inc. v. C.A.,
supra., 28 to wit:

(a) When an attachment may not be dissolved by a showing of its irregular or improper issuance:

". . . (W)hen the preliminary attachment is issued upon a ground which is at the same time the applicant's cause of action
e.g., 'an action for money or property embezzled or fraudulently misapplied or converted to his own use by a public officer,
or an officer of a corporation, or an attorney, factor, broker, agent, or clerk, in the course of his employment as such, or by
any other person in a fiduciary capacity, or for a willful violation of duty.' (Sec. 1 [b], Rule 57), or 'an action against a party
who has been guilty of fraud in contracting the debt or incurring the obligation upon which the action is brought' (Sec. 1 [d],
Rule 57), the defendant is not allowed to file a motion to dissolve the attachment under Section 13 of Rule 57 by offering
to show the falsity of the factual averments in the plaintiffs application and affidavits on which the writ was based — and
consequently that the writ based thereon had been improperly or irregularly issued (SEE Benitez v. I.A.C., 154 SCRA 41)
— the reason being that the hearing on such a motion for dissolution of the writ would be tantamount to a trial of the merits
of the action. In other words, the merits of the action would be ventilated at a mere hearing of a motion, instead of at the
regular trial. Therefore, when the writ of attachment is of this nature, the only way it can be dissolved is by a counterbond
(G.B. Inc. v. Sanchez, 98 Phil. 886)."

(b) Effect of the dissolution of a preliminary attachment on the plaintiff's attachment bond:

". . . The dissolution of the preliminary attachment upon security given, or a showing of its irregular or improper issuance,
does not of course operate to discharge the sureties on plaintiff's own attachment bond. The reason is simple. That bond
is 'executed to the adverse party, . . . conditioned that the . . . (applicant) will pay all the costs which may be adjudged to
the adverse party and all damages which he may sustain by reason of the attachment, if the court shall finally adjudge that
the applicant was not entitled thereto' (SEC. 4, Rule 57). Hence, until that determination is made, as to the applicant's
entitlement to the attachment, his bond must stand and cannot be withdrawn." LexLib

With respect to the other provisional remedies, i.e., preliminary injunction (Rule 58), receivership (Rule 59), replevin or
delivery of personal property (Rule 60), the rule is the same: they may also issue ex parte. 29

131
It goes without saying that whatever be the acts done by the Court prior to the acquisition of jurisdiction over the person of
the defendant, as above indicated — issuance of summons, order of attachment and writ of attachment (and/or appointment
of guardian ad litem, or grant of authority to the plaintiff to prosecute the suit as a pauper litigant, or amendment of the
complaint by the plaintiff as a matter of right without leave of court 30 — and however valid and proper they might otherwise
be, these do not and cannot bind and affect the defendant until and unless jurisdiction over his person is eventually obtained
by the court, either by service on him of summons or other coercive process or his voluntary submission to the court's
authority. Hence, when the sheriff or other proper officer commences implementation of the writ of attachment, it is essential
that he serve on the defendant not only a copy of the applicant's affidavit and attachment bond, and of the order of
attachment, as explicitly required by Section 5 of Rule 57, but also the summons addressed to said defendant as well as a
copy of the complaint and order for appointment of guardian ad litem, if any, as also explicitly directed by Section 3, Rule
14 of the Rules of Court. Service of all such documents is indispensable not only for the acquisition of jurisdiction over the
person of the defendant, but also upon considerations of fairness, to apprise the defendant of the complaint against him,
of the issuance of a writ of preliminary attachment and the grounds therefor and thus accord him the opportunity to prevent
attachment of his property by the posting of a counterbond in an amount equal to the plaintiff's claim in the complaint
pursuant to Section 5 (or Section 12), Rule 57, or dissolving it by causing dismissal of the complaint itself on any of the
grounds set forth in Rule 16, or demonstrating the insufficiency of the applicant's affidavit or bond in accordance with
Section 13, Rule 57.

It was on account of the failure to comply with this fundamental requirement of service of summons and the other documents
above indicated that writs of attachment issued by the Trial Court ex parte were struck down by this Court's Third Division
in two (2) cases, namely: Sievert v. Court of Appeals, 31 and BAC Manufacturing and Sales Corporation v. Court of Appeals,
et al. 32 In contrast to the case at bar — where the summons and a copy of the complaint, as well as the order and writ of
attachment and the attachment bond were served on the defendant — in Sievert, levy on attachment was attempted
notwithstanding that only the petition for issuance of the writ of preliminary attachment was served on the defendant, without
any prior or accompanying summons and copy of the complaint; and in BAC Manufacturing and Sales Corporation, neither
the summons nor the order granting the preliminary attachment or the writ of attachment itself was served on the defendant
"before or at the time the levy was made."

For the guidance of all concerned, the Court reiterates and reaffirms the proposition that writs of attachment may properly
issue ex parte provided that the Court is satisfied that the relevant requisites therefor have been fulfilled by the applicant,
although it may, in its discretion, require prior hearing on the application with notice to the defendant; but that levy on
property pursuant to the writ thus issued may not be validly effected unless preceded, or contemporaneously accompanied
by service on the defendant of summons, a copy of the complaint (and of the appointment of guardian ad litem, if any), the
application for attachment (if not incorporated in but submitted separately from the complaint), the order of attachment, and
the plaintiff's attachment bond.

WHEREFORE, the petition is GRANTED; the challenged decision of the Court of Appeals is hereby REVERSED, and the
order and writ of attachment issued by Hon. Milagros C. Nartatez, Presiding Judge of Branch 8, Regional Trial Court of
Davao City in Civil Case No. 19513-89 against Queensland Hotel or Motel or Queensland Tourist Inn and Teodorico Adarna
are hereby REINSTATED. Costs against private respondents.

SO ORDERED.

Melencio-Herrera, Gutierrez, Jr., Cruz, Paras, Feliciano, Padilla, Bidin, Griño-Aquino, Medialdea, Regalado and Romero,
JJ., concur.

Fernan, C.J., is on leave.

Davide, Jr., J., took no part; one of the parties was his client before.

21. Macasaet v. Co, Jr., G.R. No. 156759, June 5, 2013, 697 SCRA 187.

FIRST DIVISION

[G.R. No. 156759. June 5, 2013.]

ALLEN A. MACASAET, NICOLAS V. QUIJANO, JR., ISAIAS ALBANO, LILY REYES, JANET BAY, JESUS R. GALANG,
AND RANDY HAGOS, petitioners, vs. FRANCISCO R. CO, JR., respondent.

DECISION
132
BERSAMIN, J p:

To warrant the substituted service of the summons and copy of the complaint, the serving officer must first attempt to effect
the same upon the defendant in person. Only after the attempt at personal service has become futile or impossible within
a reasonable time may the officer resort to substituted service. EHCaDS

The Case

Petitioners-defendants in a suit for libel brought by respondent — appeal the decision promulgated on March 8, 2002 1 and
the resolution promulgated on January 13, 2003, 2 whereby the Court of Appeals (CA) respectively dismissed their petition
for certiorari, prohibition and mandamus and denied their motion for reconsideration. Thereby, the CA upheld the order the
Regional Trial Court (RTC), Branch 51, in Manila had issued on March 12, 2001 denying their motion to dismiss because
the substituted service of the summons and copies of the complaint on each of them had been valid and effective. 3

Antecedents

On July 3, 2000, respondent, a retired police officer assigned at the Western Police District in Manila, sued Abante Tonite,
a daily tabloid of general circulation; its Publisher Allen A. Macasaet; its Managing Director Nicolas V. Quijano; its
Circulation Manager Isaias Albano; its Editors Janet Bay, Jesus R. Galang and Randy Hagos; and its Columnist/Reporter
Lily Reyes (petitioners), claiming damages because of an allegedly libelous article petitioners published in the June 6, 2000
issue of Abante Tonite. The suit, docketed as Civil Case No. 00-97907, was raffled to Branch 51 of the RTC, which in due
course issued summons to be served on each defendant, including Abante Tonite, at their business address at Monica
Publishing Corporation, 301-305 3rd Floor, BF Condominium Building, Solana Street corner A. Soriano Street, Intramuros,
Manila. 4 TSaEcH

In the morning of September 18, 2000, RTC Sheriff Raul Medina proceeded to the stated address to effect the personal
service of the summons on the defendants. But his efforts to personally serve each defendant in the address were futile
because the defendants were then out of the office and unavailable. He returned in the afternoon of that day to make a
second attempt at serving the summons, but he was informed that petitioners were still out of the office. He decided to
resort to substituted service of the summons, and explained why in his sheriff's return dated September 22, 2000, 5 to wit:

SHERIFF'S RETURN

This is to certify that on September 18, 2000, I caused the service of summons together with copies of complaint and its
annexes attached thereto, upon the following:

1. Defendant Allen A. Macasaet, President/Publisher of defendant Abante Tonite, at Monica Publishing Corporation,
Rooms 301-305 3rd Floor, BF Condominium Building, Solana corner A. Soriano Streets, Intramuros, Manila, thru his
secretary Lu-Ann Quijano, a person of sufficient age and discretion working therein, who signed to acknowledge receipt
thereof. That effort (sic) to serve the said summons personally upon said defendant were made, but the same were
ineffectual and unavailing on the ground that per information of Ms. Quijano said defendant is always out and not available,
thus, substituted service was applied; IDCScA

2. Defendant Nicolas V. Quijano, at the same address, thru his wife Lu-Ann Quijano, who signed to acknowledge
receipt thereof. That effort (sic) to serve the said summons personally upon said defendant were made, but the same were
ineffectual and unavailing on the ground that per information of (sic) his wife said defendant is always out and not available,
thus, substituted service was applied;

3. Defendants Isaias Albano, Janet Bay, Jesus R. Galang, Randy Hagos and Lily Reyes, at the same address, thru
Rene Esleta, Editorial Assistant of defendant Abante Tonite, a person of sufficient age and discretion working therein who
signed to acknowledge receipt thereof. That effort (sic) to serve the said summons personally upon said defendants were
made, but the same were ineffectual and unavailing on the ground that per information of (sic) Mr. Esleta said defendants
is (sic) always roving outside and gathering news, thus, substituted service was applied.

Original copy of summons is therefore, respectfully returned duly served.

Manila, September 22, 2000. IaEHSD

On October 3, 2000, petitioners moved for the dismissal of the complaint through counsel's special appearance in their
behalf, alleging lack of jurisdiction over their persons because of the invalid and ineffectual substituted service of summons.
They contended that the sheriff had made no prior attempt to serve the summons personally on each of them in accordance
with Section 6 and Section 7, Rule 14 of the Rules of Court. They further moved to drop Abante Tonite as a defendant by
virtue of its being neither a natural nor a juridical person that could be impleaded as a party in a civil action.

133
At the hearing of petitioners' motion to dismiss, Medina testified that he had gone to the office address of petitioners in the
morning of September 18, 2000 to personally serve the summons on each defendant; that petitioners were out of the office
at the time; that he had returned in the afternoon of the same day to again attempt to serve on each defendant personally
but his attempt had still proved futile because all of petitioners were still out of the office; that some competent persons
working in petitioners' office had informed him that Macasaet and Quijano were always out and unavailable, and that
Albano, Bay, Galang, Hagos and Reyes were always out roving to gather news; and that he had then resorted to substituted
service upon realizing the impossibility of his finding petitioners in person within a reasonable time. aICcHA

On March 12, 2001, the RTC denied the motion to dismiss, and directed petitioners to file their answers to the complaint
within the remaining period allowed by the Rules of Court, 6 relevantly stating:

Records show that the summonses were served upon Allen A. Macasaet, President/Publisher of defendant Abante Tonite,
through Lu-Ann Quijano; upon defendants Isaias Albano, Janet Bay, Jesus R. Galang, Randy Hagos and Lily Reyes,
through Rene Esleta, Editorial Assistant of defendant Abante Tonite (p. 12, records). It is apparent in the Sheriff's Return
that on several occasions, efforts to served (sic) the summons personally upon all the defendants were ineffectual as they
were always out and unavailable, so the Sheriff served the summons by substituted service.

Considering that summonses cannot be served within a reasonable time to the persons of all the defendants, hence
substituted service of summonses was validly applied. Secretary of the President who is duly authorized to receive such
document, the wife of the defendant and the Editorial Assistant of the defendant, were considered competent persons with
sufficient discretion to realize the importance of the legal papers served upon them and to relay the same to the defendants
named therein (Sec. 7, Rule 14, 1997 Rules of Civil Procedure). CSTEHI

WHEREFORE, in view of the foregoing, the Motion to Dismiss is hereby DENIED for lack of merit.

Accordingly, defendants are directed to file their Answers to the complaint within the period still open to them, pursuant to
the rules.

SO ORDERED.

Petitioners filed a motion for reconsideration, asserting that the sheriff had immediately resorted to substituted service of
the summons upon being informed that they were not around to personally receive the summons, and that Abante Tonite,
being neither a natural nor a juridical person, could not be made a party in the action.

On June 29, 2001, the RTC denied petitioners' motion for reconsideration. 7 It stated in respect of the service of summons,
as follows: HSIDTE

The allegations of the defendants that the Sheriff immediately resorted to substituted service of summons upon them when
he was informed that they were not around to personally receive the same is untenable. During the hearing of the herein
motion, Sheriff Raul Medina of this Branch of the Court testified that on September 18, 2000 in the morning, he went to the
office address of the defendants to personally serve summons upon them but they were out. So he went back to serve said
summons upon the defendants in the afternoon of the same day, but then again he was informed that the defendants were
out and unavailable, and that they were always out because they were roving around to gather news. Because of that
information and because of the nature of the work of the defendants that they are always on field, so the sheriff resorted to
substituted service of summons. There was substantial compliance with the rules, considering the difficulty to serve the
summons personally to them because of the nature of their job which compels them to be always out and unavailable.
Additional matters regarding the service of summons upon defendants were sufficiently discussed in the Order of this Court
dated March 12, 2001.

Regarding the impleading of Abante Tonite as defendant, the RTC held, viz.:

"Abante Tonite" is a daily tabloid of general circulation. People all over the country could buy a copy of "Abante Tonite" and
read it, hence, it is for public consumption. The persons who organized said publication obviously derived profit from it. The
information written on the said newspaper will affect the person, natural as well as juridical, who was stated or implicated
in the news. All of these facts imply that "Abante Tonite" falls within the provision of Art. 44 (2 or 3), New Civil Code.
Assuming arguendo that "Abante Tonite" is not registered with the Securities and Exchange Commission, it is deemed a
corporation by estoppels considering that it possesses attributes of a juridical person, otherwise it cannot be held liable for
damages and injuries it may inflict to other persons. ICHAaT

Undaunted, petitioners brought a petition for certiorari, prohibition, mandamus in the CA to nullify the orders of the RTC
dated March 12, 2001 and June 29, 2001.

Ruling of the CA

134
On March 8, 2002, the CA promulgated its questioned decision, 8 dismissing the petition for certiorari, prohibition,
mandamus, to wit:

We find petitioners' argument without merit. The rule is that certiorari will prosper only if there is a showing of grave abuse
of discretion or an act without or in excess of jurisdiction committed by the respondent Judge. A judicious reading of the
questioned orders of respondent Judge would show that the same were not issued in a capricious or whimsical exercise of
judgment. There are factual bases and legal justification for the assailed orders. From the Return, the sheriff certified that
"effort to serve the summons personally . . . were made, but the same were ineffectual and unavailing . . . .

and upholding the trial court's finding that there was a substantial compliance with the rules that allowed the substituted
service. ASaTHc

Furthermore, the CA ruled:

Anent the issue raised by petitioners that "Abante Tonite is neither a natural or juridical person who may be a party in a
civil case," and therefore the case against it must be dismissed and/or dropped, is untenable.

The respondent Judge, in denying petitioners' motion for reconsideration, held that:

xxx xxx xxx

Abante Tonite's newspapers are circulated nationwide, showing ostensibly its being a corporate entity, thus the doctrine of
corporation by estoppel may appropriately apply.

An unincorporated association, which represents itself to be a corporation, will be estopped from denying its corporate
capacity in a suit against it by a third person who relies in good faith on such representation. IaECcH

There being no grave abuse of discretion committed by the respondent Judge in the exercise of his jurisdiction, the relief
of prohibition is also unavailable.

WHEREFORE, the instant petition is DENIED. The assailed Orders of respondent Judge are AFFIRMED.

SO ORDERED. 9

On January 13, 2003, the CA denied petitioners' motion for reconsideration. 10

Issues

Petitioners hereby submit that:

1. THE COURT OF APPEALS COMMITTED AN ERROR OF LAW IN HOLDING THAT THE TRIAL COURT
ACQUIRED JURISDICTION OVER HEREIN PETITIONERS.

2. THE COURT OF APPEALS COMMITTED REVERSIBLE ERROR BY SUSTAINING THE INCLUSION OF


ABANTE TONITE AS PARTY IN THE INSTANT CASE. 11 acADIT

Ruling

The petition for review lacks merit.

Jurisdiction over the person, or jurisdiction in personam — the power of the court to render a personal judgment or to
subject the parties in a particular action to the judgment and other rulings rendered in the action — is an element of due
process that is essential in all actions, civil as well as criminal, except in actions in rem or quasi in rem. Jurisdiction over
the defendantin an action in rem or quasi in rem is not required, and the court acquires jurisdiction over an action as long
as it acquires jurisdiction over the res that is the subject matter of the action. The purpose of summons in such action is not
the acquisition of jurisdiction over the defendant but mainly to satisfy the constitutional requirement of due process. 12

The distinctions that need to be perceived between an action in personam, on the one hand, and an action in rem or quasi
in rem, on the other hand, are aptly delineated in Domagas v. Jensen, 13 thusly: HATICc

The settled rule is that the aim and object of an action determine its character. Whether a proceeding is in rem, or in
personam, or quasi in rem for that matter, is determined by its nature and purpose, and by these only. A proceeding in
personam is a proceeding to enforce personal rights and obligations brought against the person and is based on the
jurisdiction of the person, although it may involve his right to, or the exercise of ownership of, specific property, or seek to
compel him to control or dispose of it in accordance with the mandate of the court. The purpose of a proceeding in personam
is to impose, through the judgment of a court, some responsibility or liability directly upon the person of the defendant. Of
135
this character are suits to compel a defendant to specifically perform some act or actions to fasten a pecuniary liability on
him. An action in personam is said to be one which has for its object a judgment against the person, as distinguished from
a judgment against the prop[er]ty to determine its state. It has been held that an action in personam is a proceeding to
enforce personal rights or obligations; such action is brought against the person. As far as suits for injunctive relief are
concerned, it is well-settled that it is an injunctive act in personam. In Combs v. Combs, the appellate court held that
proceedings to enforce personal rights and obligations and in which personal judgments are rendered adjusting the rights
and obligations between the affected parties is in personam. Actions for recovery of real property are in personam.
TaDSCA

On the other hand, a proceeding quasi in rem is one brought against persons seeking to subject the property of such
persons to the discharge of the claims assailed. In an action quasi in rem, an individual is named as defendant and the
purpose of the proceeding is to subject his interests therein to the obligation or loan burdening the property. Actions quasi
in rem deal with the status, ownership or liability of a particular property but which are intended to operate on these
questions only as between the particular parties to the proceedings and not to ascertain or cut off the rights or interests of
all possible claimants. The judgments therein are binding only upon the parties who joined in the action. ESTCHa

As a rule, Philippine courts cannot try any case against a defendant who does not reside and is not found in the Philippines
because of the impossibility of acquiring jurisdiction over his person unless he voluntarily appears in court; but when the
case is an action in rem or quasi in rem enumerated in Section 15, Rule 14 of the Rules of Court, Philippine courts have
jurisdiction to hear and decide the case because they have jurisdiction over the res, and jurisdiction over the person of the
non-resident defendant is not essential. In the latter instance, extraterritorial service of summons can be made upon the
defendant, and such extraterritorial service of summons is not for the purpose of vesting the court with jurisdiction, but for
the purpose of complying with the requirements of fair play or due process, so that the defendant will be informed of the
pendency of the action against him and the possibility that property in the Philippines belonging to him or in which he has
an interest may be subjected to a judgment in favor of the plaintiff, and he can thereby take steps to protect his interest if
he is so minded. On the other hand, when the defendant in an action in personam does not reside and is not found in the
Philippines, our courts cannot try the case against him because of the impossibility of acquiring jurisdiction over his person
unless he voluntarily appears in court. 14 aSTHDc

As the initiating party, the plaintiff in a civil action voluntarily submits himself to the jurisdiction of the court by the act of filing
the initiatory pleading. As to the defendant, the court acquires jurisdiction over his person either by the proper service of
the summons, or by a voluntary appearance in the action. 15

Upon the filing of the complaint and the payment of the requisite legal fees, the clerk of court forthwith issues the
corresponding summons to the defendant. 16 The summons is directed to the defendant and signed by the clerk of court
under seal. It contains the name of the court and the names of the parties to the action; a direction that the defendant
answers within the time fixed by the Rules of Court; and a notice that unless the defendant so answers, the plaintiff will
take judgment by default and may be granted the relief applied for. 17 To be attached to the original copy of the summons
and all copies thereof is a copy of the complaint (and its attachments, if any) and the order, if any, for the appointment of a
guardian ad litem. 18

The significance of the proper service of the summons on the defendant in an action in personam cannot be
overemphasized. The service of the summons fulfills two fundamental objectives, namely: (a) to vest in the court jurisdiction
over the person of the defendant; and (b) to afford to the defendant the opportunity to be heard on the claim brought against
him. 19 As to the former, when jurisdiction in personam is not acquired in a civil action through the proper service of the
summons or upon a valid waiver of such proper service, the ensuing trial and judgment are void. 20 If the defendant
knowingly does an act inconsistent with the right to object to the lack of personal jurisdiction as to him, like voluntarily
appearing in the action, he is deemed to have submitted himself to the jurisdiction of the court. 21 As to the latter, the
essence of due process lies in the reasonable opportunity to be heard and to submit any evidence the defendant may have
in support of his defense. With the proper service of the summons being intended to afford to him the opportunity to be
heard on the claim against him, he may also waive the process. 22 In other words, compliance with the rules regarding the
service of the summons is as much an issue of due process as it is of jurisdiction. 23 IDCHTE

Under the Rules of Court, the service of the summons should firstly be effected on the defendant himself whenever
practicable. Such personal service consists either in handing a copy of the summons to the defendant in person, or, if the
defendant refuses to receive and sign for it, in tendering it to him. 24 The rule on personal service is to be rigidly enforced
in order to ensure the realization of the two fundamental objectives earlier mentioned. If, for justifiable reasons, the
defendant cannot be served in person within a reasonable time, the service of the summons may then be effected either
(a) by leaving a copy of the summons at his residence with some person of suitable age and discretion then residing therein,
or (b) by leaving the copy at his office or regular place of business with some competent person in charge thereof. 25 The

136
latter mode of service is known as substituted service because the service of the summons on the defendant is made
through his substitute. DHESca

It is no longer debatable that the statutory requirements of substituted service must be followed strictly, faithfully and fully,
and any substituted service other than that authorized by statute is considered ineffective. 26 This is because substituted
service, being in derogation of the usual method of service, is extraordinary in character and may be used only as prescribed
and in the circumstances authorized by statute. 27 Only when the defendant cannot be served personally within a
reasonable time may substituted service be resorted to. Hence, the impossibility of prompt personal service should be
shown by stating the efforts made to find the defendant himself and the fact that such efforts failed, which statement should
be found in the proof of service or sheriff's return. 28 Nonetheless, the requisite showing of the impossibility of prompt
personal service as basis for resorting to substituted service may be waived by the defendant either expressly or impliedly.
29 CTDAaE

There is no question that Sheriff Medina twice attempted to serve the summons upon each of petitioners in person at their
office address, the first in the morning of September 18, 2000 and the second in the afternoon of the same date. Each
attempt failed because Macasaet and Quijano were "always out and not available" and the other petitioners were "always
roving outside and gathering news." After Medina learned from those present in the office address on his second attempt
that there was no likelihood of any of petitioners going to the office during the business hours of that or any other day, he
concluded that further attempts to serve them in person within a reasonable time would be futile. The circumstances fully
warranted his conclusion. He was not expected or required as the serving officer to effect personal service by all means
and at all times, considering that he was expressly authorized to resort to substituted service should he be unable to effect
the personal service within a reasonable time. In that regard, what was a reasonable time was dependent on the
circumstances obtaining. While we are strict in insisting on personal service on the defendant, we do not cling to such
strictness should the circumstances already justify substituted service instead. It is the spirit of the procedural rules, not
their letter, that governs. 30 SCDaET

In reality, petitioners' insistence on personal service by the serving officer was demonstrably superfluous. They had actually
received the summonses served through their substitutes, as borne out by their filing of several pleadings in the RTC,
including an answer with compulsory counterclaim ad cautelam and a pre-trial brief ad cautelam. They had also availed
themselves of the modes of discovery available under the Rules of Court. Such acts evinced their voluntary appearance in
the action.

Nor can we sustain petitioners' contention that Abante Tonite could not be sued as a defendant due to its not being either
a natural or a juridical person. In rejecting their contention, the CA categorized Abante Tonite as a corporation by estoppel
as the result of its having represented itself to the reading public as a corporation despite its not being incorporated.
Thereby, the CA concluded that the RTC did not gravely abuse its discretion in holding that the non-incorporation of Abante
Tonite with the Securities and Exchange Commission was of no consequence, for, otherwise, whoever of the public who
would suffer any damage from the publication of articles in the pages of its tabloids would be left without recourse. We
cannot disagree with the CA, considering that the editorial box of the daily tabloid disclosed that although Monica Publishing
Corporation had published the tabloid on a daily basis, nothing in the box indicated that Monica Publishing Corporation had
owned Abante Tonite. TIDcEH

WHEREFORE, the Court AFFIRMS the decision promulgated on March 8, 2002; and ORDERS petitioners to pay the costs
of suit.

SO ORDERED.

Sereno, C.J., Leonardo-de Castro, Villarama, Jr. and Reyes, JJ., concur.

22. Navale v. Court of Appeals, 253 SCRA 705 (1996).

SECOND DIVISION

[G.R. No. 109957. February 20, 1996.]

ANTONIO NAVALE, FULGENCIO ABUHAN, LEONARDO NOVO, FILEMON VALDEHUEZA, RODOLFO SANGA,
ROGELIO SANGA, LYNITA VALDEHUEZA QUIRANTE, TEODORA LIAMSO, MONICA NACASABOG, LEONILA NOBLE,
PETRONILA ALLA, GRACE SISON, REBECCA MARAON, APOLONIA SANICO, ROMEO YAMSON, EUFRACIA
REGAHAL, NATIVIDAD MANAGBANAG, JASMIN ANANETA, REX MAXILUM, PERPITUA SALVO, ARNULFO
TINAMPAY, LENY DUGADUGA, FLORITA DELA GANAR, LAURITA MALAZARTE, LOURDES IMAN, ESTER DUHAYA,
MARY CAMPANA, FROSERFIDA NULO, DOLORES AGANA, NELSON MAGMAONG, and CLARITA DONASCO,
petitioners, vs. COURT OF APPEALS, HON. GREGORIO PANTANOSAS, Municipal Trial Court Judge, JOVEN YASAY,
and SAMSON YASAY, respondents.
137
Guerrero A. Adaza & Associates for petitioners.

Frederico M. Gapuz for private respondents.

SYLLABUS

1. REMEDIAL LAW; CIVIL PROCEDURE; SUMMONS; PERSONAL SERVICE. — Under Section 7, Rule 14 of the
Rules of Court, summons may be served personally by handing a copy thereof to the defendant in person or if he refuses
to receive it, by tendering it to him.

2. ID.; ID.; ID.; VOLUNTARY APPEARANCE WHICH IS EQUIVALENT TO SERVICE. — Jurisdiction cannot be
acquired over the defendant without service of summons. However, Section 23 of the Rules provides that the defendant's
voluntary appearance in the action shall be equivalent to service. Instances of actions amounting to voluntary appearance
have been held by this Court to be: when his counsel files the corresponding pleading thereon; when a defendant files a
motion for reconsideration of the judgment by default; when he files a petition to set aside the judgment of default; when
he and the plaintiff jointly submit a compromise agreement for approval of the trial court.

3. ID.; ID.; ID.; WHEN THE COURT ACQUIRES JURISDICTION DESPITE DEFECT IN THE SERVICE OF
SUMMONS. — "Defects of summons are cured by voluntary appearance and by the filing of an answer to the complaint.
A defendant can not be permitted to speculate upon the judgment of the court by objecting to the court's jurisdiction over
its person if the judgment is adverse to it, and acceding to jurisdiction over its person if and when the judgment sustains its
defense." [Republic v. Ker & Company, Ltd., No. L-21609, September 29, 1966, 18 SCRA 208]. Any form of appearance
in court by the defendant, his authorized agent or attorney, is equivalent to service except where such appearance is
precisely to object to the jurisdiction of the court over his person. [Carballo v. Encarnacion, 49 O.G. 1383]

4. ID.; ID.; LACK OF JURISDICTION OVER SUBJECT MATTER OR NATURE OF ACTION; MAY BE RAISED AT
ANYTIME. — Where the court itself clearly has no jurisdiction over the subject matter or nature of the action the invocation
of the defense of lack of jurisdiction may be raised at any time. An example of this is when the case falls within the jurisdiction
of another government agency or quasi-judicial body; in which case, voluntary appearance will not be deemed as a waiver.

5. ID.; EVIDENCE; DISPUTABLE PRESUMPTION; A SHERIFF HAS REGULARLY PERFORMED HIS OFFICIAL
DUTY. — In the absence of contrary evidence, a presumption exists that a sheriff has regularly performed his official duty.
To overcome the presumption arising from the sheriff's certificate, the evidence must be clear and convincing.

DECISION

ROMERO, J p:

Petitioners question the decision of the Court of Appeals 1 affirming the decision of the Regional Trial Court (Branch XX)
of Cagayan de Oro City dismissing the petition for certiorari in Special Civil Action No. 10804.

The facts are the following:

Private respondents filed an action for forcible entry and damages with the Municipal Trial Court in Cities in Cagayan de
Oro City docketed as Civil Case No. 8942 with prayer for the issuance of preliminary mandatory injunction and for the return
of the possession of a portion of the Bagting Estate in Carmen, Cagayan de Oro City over which they claimed absolute
ownership but which had been allegedly occupied by petitioners with force and violence. Moreover, the latter had also
constructed houses thereon against private respondents' will.

The Municipal Trial Court in Cities (MTCC) granted the petition and issued the writ prayed for. However, petitioners ignored
the writ, prompting private respondents to file a motion to have petitioners declared in contempt. The MTCC subsequently
issued an order directing petitioners to comply with the writ of injunction and for private respondents not to demolish the
former's houses pending a decision on the merits.

Petitioners were later declared in default for failure to appear and to present evidence on their behalf. The MTCC then
rendered judgment based on private respondents' evidence declaring them the rightful possessors of the land; ordering
petitioners to immediately vacate the premises and to pay actual damages for destroying the perimeter fence and
guardhouse in the amount of P5,000.00; exemplary damages of P25,000.00 as a deterrent to future unlawful acts of the
same category; attorney's fees of P5,000.00; and litigation expenses of P1,000.00 plus costs.

The above judgment became final and writs of execution and demolition were issued.

138
Petitioners then filed a petition for certiorari with the Regional Trial Court questioning the order of default, the subsequent
judgment and the writ of demolition issued by the MTCC arguing that they had never been summoned to answer the
complaint.

The RTC dismissed the petition finding that, as shown by the Sheriff's return, as of March 29, 1983 summonses were
served on petitioners and several John Does, but that they either refused to receive the same and/or refused to give their
names. These actions prompted the Sheriff to leave copies of the summonses at the residences of petitioners and also
with one Eligio Valdehueza who, petitioners alleged, had allowed them to enter the land in question and to build their
houses thereon in his capacity as judicial administrator of the Bagting Estates.

The RTC further found that answers were filed in the said case by petitioners without any qualification; hence, jurisdiction
was acquired by the MTCC over their persons.

Elevated to the Court of Appeals, the decision of the RTC was affirmed. Hence, this petition.

Petitioners argue that summonses were never validly served on them and that they did not appear voluntarily in the action
as to be covered by Section 23 of Rule 14 of the Rules of Court in what is equivalent to service. They contend that it was
only Eligio Valdehueza who received a copy of the summonses and answered the same and that they never authorized
him to represent them. Thus, they conclude, the MTCC never acquired jurisdiction over them.

We find petitioners' contention to be devoid of merit.

Under Section 7, Rule 14 of the Rules of Court, summons may be served personally by handing a copy thereof to the
defendant in person or if he refuses to receive it, by tendering it to him.

Jurisdiction cannot be acquired over the defendant without service of summons. However, Section 23 of the Rules provides
that the defendant's voluntary appearance in the action shall be equivalent to service. Instances of actions amounting to
voluntary appearance have been held by this Court to be: when his counsel files the corresponding pleading thereon; 2
when a defendant files a motion for reconsideration of the judgment by default; 3 when he files a petition to set aside the
judgment of default; 4 when he and the plaintiff jointly submit a compromise agreement for approval of the trial court. 5

In the present case, the record shows that summonses were duly served on petitioners but that they, not only refused to
receive the same, but that they also declined to give their names.

The Court is aware of the difficulties of serving judicial notices on defendants in unlawful detainer, recovery of possession,
or ejectment cases. Process servers in these cases are often greeted with hostility and suspicion by the occupants of the
subject properties and, sometimes, even threatened with physical violence.

In the absence of contrary evidence, a presumption exists that a sheriff has regularly performed his official duty. 6 To
overcome the presumption arising from the sheriff's certificate, the evidence must be clear and convincing. 7 However, no
such proof of irregularity in the Sheriff's return was ever presented by petitioners.

We have held that the refusal of a defendants (the petitioners in this case) to receive the summons is a technicality resorted
to in an apparent attempt to frustrate the ends of justice. 8

Granting that there was an invalid service of summons, which is not the case here, still the MTCC acquired jurisdiction over
the petitioners through their voluntary appearance thereat.

As the RTC noted:

"On April 11, 1983, defendants, thru counsel and without qualification filed their answer to the contempt charge filed by
private respondents.

On April 12, 1983 all the defendants filed their Answer to the complaint in Civil Case No. 8942 (p. 167, Record in C.C. No.
8942) thru counsel, claiming that the land is part of the Bagting Estate and they were duly authorized by one, Eligio
Valdehueza, administrator of the Bagting Estate pending in the Court of First Instance of Misamis Oriental and duly
appointed as administrator.

During the execution of the writ of injunction and the contempt proceedings all the defendants pleaded that they be allowed
to vacate the premises within an extended time of three (3) months provided the contempt petition be withdrawn but later
the defendants never left the premises, hence, the MTCC issued a writ of demolition.

The defendants, thru counsel even answered not only the contempt charge but also the writ of demolition motion pleading
that the case must be tried and decided on the merits before they will be ejected from the premises.

139
On April 29, 1983 counsel for defendants manifested and volunteered to have an ocular inspection of the premises in
connection with the claim of the plaintiffs in Civil Case No. 8942 that the defendants defied the order of injunction.

On May 3, 1983 the defendants in the civil case below and also accused in the same court for violating P.D. 772 agreed
with plaintiff below to comply with the order by vacating the premises as plaintiffs and complainants agreed to their petition
for contempt and by virtue of said agreement no demolition was effected.

On June 6, 1983 the pre-trial was terminated and some of the defendants were declared in default; on July 20, 1983 Atty.
Fermente P. Dable, counsel for the defendants appeared to explain his non-appearance on the hearing on July 14, 1983.

On December 3, 1983 defendants presented Pricilla Valdehueza as a witness and on January 6, 1984 (p. 294, Record)
Elegio Valdehueza testified and the decision was rendered n June 13, 1983 by the respondent judge." 9

Petitioners' actions, such as filing an answer to the contempt charge (among others), are a clear manifestation that they
voluntarily submitted themselves to the jurisdiction of the MTCC. 10 Even their filing of the petition for certiorari is evidence
of such voluntary submission. 11

There is no showing that petitioners ever questioned the jurisdiction of the MTCC over them, except when a judgment in
default was declared against them. To properly avail of the defense of invalid service of summons, petitioners should have
questioned it and the MTCC's exercise of jurisdiction over them, from the very start.

"Defects of summons are cured by voluntary appearance and by the filing of an answer to the complaint. A defendant can
not be permitted to speculate upon the judgment of the court by objecting to the court's jurisdiction over its person if the
judgment is adverse to it, and acceding to jurisdiction over its person if and when the judgment sustains its defense." 12

Any form of appearance in court by the defendant, his authorized agent or attorney, is equivalent to service except where
such appearance is precisely to object to the jurisdiction of the court over his person. 13

In La Naval Drug Corporation v. Court of Appeals, 14 we held:

"Jurisdiction over the person must be seasonably raised, i.e., that it is pleaded in a motion to dismiss or by way of an
affirmative defense in an answer. Voluntary appearance shall be deemed a waiver of this defense."

However, we also said therein that where the court itself clearly has no jurisdiction over the subject matter or nature of the
action (which is not the case here) the invocation of the defense of lack of jurisdiction may be raised at any time. An example
of this is when the case falls within the jurisdiction of another government agency or quasi-judicial body; in which case,
voluntary appearance will not be deemed as a waiver.

In the instant case, there is no question that the MTCC had jurisdiction over the subject matter of the action. The question
was whether it had jurisdiction over the person of petitioners.

We hold that it did.

Petitioners, having failed to object to the MTCC's jurisdiction from the very beginning, may no longer raise it now as a
ground to set aside the judgment by default. Nor can they claim that they are not bound by the consequences of their own
acts before the Court.

As the RTC aptly stated:

". . . otherwise there will be no end to litigation. There will be anarchy if the petitioners will be allowed to use their own culpa
or violation as a reason or excuse from the impact and sanctions imposed by law. . . ."

WHEREFORE, the decision of the Court of Appeals is hereby AFFIRMED in toto.

SO ORDERED.

Regalado, Puno and Mendoza, JJ., concur.

23. French Oil Mill Machinery Co., Inc. v. Court of Appeals, 295 SCRA 462 (1998)

SECOND DIVISION

[G.R. No. 126477. September 11, 1998.]

140
FRENCH OIL MILL MACHINERY CO., INC., petitioner, vs. COURT OF APPEALS [CA], REGIONAL TRIAL COURT [RTC],
CEBU CITY, BR. 11, and LUDO & LUYM OLEOCHEMICAL CO.), respondents.

Castillo, Laman, Tan, Pantaleon & San Jose for petitioner.

Angara, Abello, Concepcion, Regala & Cruz for private respondents.

SYNOPSIS

Ludo and Luym Oleochemical Co. filed a complaint for breach of contract with damages against French Oil Mill Machinery
Co., Inc. Subsequently, summons for both respondents were served to Trans-World Trading Company. The French Oil Mill
Machinery Co., Inc. filed a special appearance with motion to dismiss on the ground that the lower court had no jurisdiction
over its person due to improper service of summons contending that it is not doing business in the Philippines and that
Trans-World Trading Company is not its agent. Initially, the lower court dismissed the complaint for lack of jurisdiction over
petitioner but when a motion for reconsideration was filed, it reversed its ruling and ruled that summons was properly
served. cdasia

In a petition for certiorari and prohibition filed before the Court of Appeals, the appellate court upheld the ruling of the lower
court. Hence, this petition for review on certiorari raising the issue of whether or not there was proper service of summons
to the petitioner. CHDAaS

It is not enough to merely allege in the complaint that a defendant foreign corporation is doing business. For purposes of
the rule on summons, the fact of doing business must first be "established by appropriate allegations in the complaint" and
the court in determining such fact need not go beyond the allegations therein. In this case, the allegations that petitioner
entered into a contract with private respondent to supply and install various machineries and equipments for the use of the
latter's oil mill factory and that the first shipment of machineries from petitioner was received by private respondent are
sufficient allegations that petitioner is doing business. In any case, the determination that a foreign corporation is doing
business is merely tentative and only to enable the local court to acquire jurisdiction over the person of the foreign
corporation through service of summons.

For purposes of the rules on summons, the determination of principal agent relationship from the allegations in the complaint
is only preliminary and is not even conclusive as to liability. Nothing bars the court from later making a different finding after
the parties had substantiated their respective allegations with respect to the agency should the same be disputed. As found
by both courts below, petitioner treated Trans-World as its Philippine agent in the assailed transaction. AcEIHC

SYLLABUS

1. REMEDIAL LAW; CIVIL PROCEDURE; SUMMONS; SERVICE OF SUMMONS; DETERMINATION THEREOF


CAN BE ESTABLISHED BY APPROPRIATE ALLEGATIONS IN THE COMPLAINT. — It is not enough to merely allege in
the complaint that a defendant foreign corporation is doing business. For purposes of the rule on summons, the fact of
doing business must first be "established by appropriate allegations in the complaint" and the court in determining such fact
need not go beyond the allegations therein. . . . In any case, the determination that a foreign corporation is doing business
is merely tentative and only to enable the local court to acquire jurisdiction over the person of the foreign corporation through
service of summons. It does not foreclose a subsequent finding to the contrary depending on the evidence.

2. ID.; ID.; ID.; ID.; TO A FOREIGN CORPORATION. — Having determined the issue of doing business, the Court
will now inquire on whether petitioner was validly served with summons. Under the Rules of Court, if the defendant is a
foreign corporation doing business in the Philippines, summons may be served on (a) its resident agent designated in
accordance with law; (b) if there is no resident agent, the government official designated by law to that effect, or (c) any of
its officer or agent within the Philippines. acHTIC

3. ID.; ID.; ID.; ID.; ID.; DETERMINATION OF PRINCIPAL-AGENCY RELATIONSHIP; SPECIFIC ALLEGATION IS
NECESSARY; CASE AT BAR. — Private respondent alleged in its complaint that Trans-World is petitioner's agent, so that
the service was made on the latter. Such general allegation is insufficient to show the agency relationship between petitioner
and Trans-World. However, although there is no requirement to first substantiate the allegation of agency yet it is necessary
that there must be specific allegations in the complaint that establishes the connection between the principal foreign
corporation and its alleged agent with respect to the transaction in question.

4. ID.; ID.; COMPLAINT; ALLEGATION THEREOF OF PRINCIPAL-AGENT RELATIONSHIP IS FOR THE


PURPOSE OF SERVICE OF SUMMONS ONLY. — For purposes of the rules on summons, the determination of principal-
agent relationship from the allegations in the complaint is only preliminary and is not even conclusive as to liability. Nothing
bars the court from later making a different finding after the parties had substantiated their respective allegations with
respect to agency should the same be disputed. cCTAIE
141
RESOLUTION

MARTINEZ, J p:

Private respondent filed a complaint for breach of contract with damages against petitioner foreign corporation and the
latter's alleged Philippine agent Trans-World Trading Company. The complaint states in part that: cdrep

"1.2 Defendant French Oil Mill Machinery ('FOMMCO') is a corporation with principal office at, Piqua, Ohio, United
States of America, engaged in business in the Philippines through its agent Trans-World Trading Company. FOMMCO may
be served with summons and other court processes through its agent, Trans-World Trading Company.

"1.3 Defendant Trans-World Trading Company ('Trans-World') is the agent of FOMMCO in the Philippines, with office
at Don Pablo Building, 144 Amorsolo St., Makati, Metro Manila, where it may be served with summons and other court
processes." 1

Summons was served on Trans-World which moved to dismiss the complaint arguing that it is not petitioner's agent.
Petitioner itself filed a special appearance with motion to dismiss contending that the court had no jurisdiction over its
person due to improper service of summons. It argued that (a) it is not doing business in the Philippines and (b) Trans-
World is not its agent, therefore the procedure in Sections 14 2 and 173 , Rule 14 should have been observed. The court
a quo initially dismissed the complaint for lack of jurisdiction over petitioner 4 but on private respondent's motion for
reconsideration, said court reversed the order of dismissal and ruled that summons was properly served on petitioner whom
it found doing business in the Philippines and Trans-World as its agent. Petitioner elevated the case to the Court of Appeals
(CA) via petition for certiorari and prohibition but to no avail. Not satisfied, petitioner filed this petition under Rule 45 which
was initially dismissed for being filed late 5 but on petitioner's motion for reconsideration was reinstated by the Court. 6

Petitioner contends that it is not doing business in the Philippines and that Trans-World is not its agent, and thus, the
summons served on the latter has no effect on the former. The contention is not meritorious. LexLib

It is not enough to merely allege in the complaint that a defendant foreign corporation is doing business. For purposes of
the rule on summons, the fact of doing business must first be "established by appropriate allegations in the complaint" 7
and the court in determining such fact need not go beyond the allegations therein. 8 In this case, the allegations that
petitioner entered into a contract with private respondent to supply and install various machineries and equipments for the
use of the latter's oil mill factory 9 and that the first shipment of machineries from petitioner was received by private
respondent 10 are sufficient allegations that petitioner is doing business for purposes of Section 14, Rule 14. In any case,
the determination that a foreign corporation is doing business is merely tentative and only to enable the local court to
acquire jurisdiction over the person of the foreign corporation through service of summons. It does not foreclose a
subsequent finding to the contrary depending on the evidence. 11

Having determined the issue of doing business, the Court will now inquire on whether petitioner was validly served with
summons. Under the Rules of Court, if the defendant is a foreign corporation doing business in the Philippines, summons
may be served on (a) its resident agent designated in accordance with law; (b) if there is no resident agent the government
official designated by law to that effect or (c) any of its officer or agent within the Philippines. 12 Private respondent alleged
in its complaint that Trans-World is petitioner's agent, so that the service was made on the latter. Such general allegation
is insufficient to show the agency relationship between petitioner and Trans-World. However, although there is no
requirement to first substantiate the allegation of agency yet it is necessary that there must be specific allegations in the
complaint that establishes the connection between the principal foreign corporation and its alleged agent with respect to
the transaction in question. Nowhere in the case of Signetics Corporation v. CA, 13 cited by both parties, did the court state
that if the "complaint alleges that defendant has an agent in the Philippines, summons can validly be served thereto even
without prior evidence of the truth of such factual allegation." It is only in the headnote of the reporter 14 where the quoted
statement appears. Certainly a portion of the decision was paraphrased to convey that statement which is never meant nor
mentioned in the ponencia and thus, was a misinterpretation of the scope of the decision. The headnote or syllabi is not
the work of the court, nor does it state its decision. It is simply the work of the reporter, who gives his understanding of the
decision, and is prepared for the convenience of the profession in the examination of the reports. 15 A headnote is not a
part of the court's decision. cdtai

For purposes of the rules on summons, the determination of principal-agent relationship from the allegations in the
complaint is only preliminary and is not even conclusive as to liability. Nothing bars the court from later making a different
finding after the parties had substantiated their respective allegations with respect to agency should the same be disputed.
As found by both courts below, petitioner treated Trans-World as its Philippine agent in the assailed transaction. 16 Such
factual assessment is binding on this Court 17 and will not be disturbed as no exceptional circumstances 18 nor cogent
reasons 19 were shown to justify its reversal. For it is well-settled that factual findings of the trial court are respected on
appeal when supported by substantial evidence on record 20 and carry more weight when affirmed by the appellate court,
142
21 absent any proof that significant facts or circumstances were overlooked or disregarded which would have varied the
outcome of the case. 22

Finally, petitioner fears that it could no longer contest the jurisdiction of the court once it files an answer instead of a motion
to dismiss, as the filing of the former amounts to voluntary appearance. 23 Suffice it to say that the filing of an answer per
se should not be automatically treated as voluntary appearance by the defendant for purposes of summons. It should be
noted that when the appearance of a defendant is precisely to object to the jurisdiction of the court over his person, it cannot
be considered as appearance in court. 24 The foregoing, however, need not be further discussed in this case as petitioner
did not file any answer.

ACCORDINGLY, the petition is DENIED for lack of merit.

SO ORDERED.

Melo, Puno and Mendoza, JJ ., concur.

Regalado, J ., on official leave.

24. Asiavest Limited v. Court of Appeals, 296 SCRA 539, 552 (1998).

FIRST DIVISION

[G.R. No. 128803. September 25, 1998.]

ASIAVEST LIMITED, petitioner, vs. THE COURT OF APPEALS and ANTONIO HERAS, respondents.

SYLLABUS

1. REMEDIAL LAW; EVIDENCE; PRESUMPTIONS; VALIDITY OF FOREIGN JUDGMENTS. — Under paragraph


(b) of Section 50, Rule 39 of the Rules of Court, which was the governing law at the time this case was decided by the trial
court and respondent Court of Appeals, a foreign judgment against a person rendered by a court having jurisdiction to
pronounce the judgment is presumptive evidence of a right as between the parties and their successors in interest by the
subsequent title. However, the judgment may be repelled by evidence of want of jurisdiction, want of notice to the party,
collusion, fraud, or clear mistake of law or fact. Also, Section 3(n) of Rule 131 of the New Rules of Evidence provides that
in the absence of proof to the contrary, a court, or judge acting as such, whether in the Philippines or elsewhere, is presumed
to have acted in the lawful exercise of jurisdiction. Hence, once the authenticity of the foreign judgment is proved, the
burden to repel it on grounds provided for in paragraph (b) of Section 50, Rule 39 of the Rules of Court is on the party
challenging the foreign judgment. EACTSH

2. ID.; ID.; ID.; ID.; CASE AT BAR. — At the pre-trial conference, HERAS admitted the existence of the Hong Kong
judgment. On the other hand, ASIAVEST presented evidence to prove rendition, existence, and authentication of the
judgment by the proper officials. The judgment is thus presumed to be valid and binding in the country from which it comes,
until the contrary is shown. Consequently, the first ground relied upon by ASIAVEST has merit. The presumption of validity
accorded foreign judgment would be rendered meaningless were the party seeking to enforce it be required to first establish
its validity.

3. ID.; ACTIONS; MATTERS OF REMEDY AND PROCEDURE, GOVERNED BY THE LAW OF THE FORUM. —
Matters of remedy and procedure such as those relating to the service of process upon the defendant are governed by the
lex fori or the law of the forum. SHIETa

4. ID.; EVIDENCE; RECORD OF PUBLIC DOCUMENTS OF A SOVEREIGN AUTHORITY, TRIBUNAL, OFFICIAL


BODY OR PUBLIC OFFICER, HOW PROVED. — Under Sections 24 and 25, Rule 132 of the New Rules of Evidence, the
record of public documents of a sovereign authority, tribunal, official body, or public officer may be proved by (1) an official
publication thereof or (2) a copy attested by the officer having the legal custody thereof which must be accompanied if the
record is not kept in the Philippines, with a certificate that such officer has the custody. The certificate may be issued by a
secretary of the embassy or legation, consul general, consul, vice consul, or consular agent, or any officer in the foreign
service of the Philippines stationed in the foreign country in which the record is kept, and authenticated by the seal of his
office. The attestation must state, in substance, that the copy is a correct copy of the original, or a specific part thereof, as
the case may be, and must be under the official seal of the attesting officer. Nevertheless, the testimony of an expert
witness may be allowed to prove a foreign law. CcAIDa

143
5. ID.; ID.; ID.; HONGKONG LAW ON SERVICE OF SUMMONS PRESUMED SIMILAR TO PHILIPPINE LAW — In
the absence of proof of the Hong Kong law on service of summons, the presumption of identity or similarity or the so-called
processual presumption shall come into play. It will thus be presumed that the Hong Kong law on the matter is similar to
the Philippine law.

6. ID.; ACTIONS; ACTION IN PERSONAM, IN REM AND QUASI IN REM; DISTINGUISHED. — An action in
personam is an action against a person on the basis of his personal liability. An action in rem is an action against the thing
itself instead of against the person. An action quasi in rem is one wherein an individual is named as defendant and the
purpose of the proceeding is to subject his interest therein to the obligation or lien burdening the property.

7. ID.; ID.; ACTION IN PERSONAM; JURISDICTION OVER THE PERSON OF THE DEFENDANT, NECESSARY.
— In an action in personam, jurisdiction over the person of the defendant is necessary for the court to validly try and decide
the case. Jurisdiction over the person of a resident defendant who does not voluntarily appear in court can be acquired by
personal service of summons as provided under Section 7, Rule 14 of the Rules of Court. If he cannot be personally served
with summons within a reasonable time, substituted service may be made in accordance with Section 8 of said Rule. If he
is temporarily out of the country, any of the following modes of service may be resorted to: (1) substituted service set forth
in Section 8; (2) personal service outside the country, with leave of court (3) service by publication, also with leave of court;
or (4) any other manner the court may deem sufficient. However, in an action in personam wherein the defendant is a non-
resident who does not voluntarily submit himself to the authority of the court, personal service of summons within the state
is essential to the acquisition of jurisdiction over her person. This method of service is possible if such defendant is
physically present in the country. If he is not found therein, the court cannot acquire jurisdiction over his person and therefore
cannot validly try and decide the case against him. An exception was laid down in Gemperle v. Schenker wherein a non-
resident was served with summons through his wife, who was a resident of the Philippines and who was his representative
and attorney-in-fact in prior civil case filed by him; moreover, the second case was a mere offshoot of the first case.
HTSaEC

8. ID.; ID.; ACTION IN REM; JURISDICTION OVER THE PERSON OF DEFENDANT, NOT A PREREQUISITE;
SUMMONS MUST BE SERVED UPON DEFENDANT TO SATISFY DUE PROCESS REQUIREMENT. — In a proceeding
in rem or quasi in rem, jurisdiction over the person of the defendant is not a prerequisite to confer jurisdiction on the court
provided that the court acquires jurisdiction over the res. Nonetheless, summons must be served upon the defendant not
for the purpose of vesting the court with jurisdiction but merely for satisfying the due process requirements. Thus, where
the defendant is a non-resident who is not found in the Philippines and (1) the action affects the personal status of the
plaintiff; (2) the action relates to, or the subject matter of which is property in the Philippines in which the defendant has or
claims a lien or interest; (3) the action seeks the exclusion of the defendant from any interest in the property located in the
Philippines; or (4) the property of the defendant has been attached in the Philippines — service of summons may be effected
by (a) personal service out of the country, with leave of court; (b) publication, also with leave of court; or (c) any other
manner the court may deem sufficient.

9. ID.; ID.; ENFORCEMENT OF FOREIGN JUDGMENT; SUMMONS MUST BE SERVED ON DEFENDANT IN


FOREIGN LAND; CASE AT BAR. — In the pre-trial conference, the parties came up with stipulations of facts, among which
was that "the residence of defendant, Antonio Heras, is New Manila, Quezon City." We note that the residence of HERAS
insofar as the action for the enforcement of the Hong Kong court judgment is concerned, was never in issue. He never
challenged the service of summons on him through a security guard in his Quezon City residence and through a lawyer in
his office in that city. In his Motion to Dismiss, he did not question the jurisdiction of the Philippine court over his person on
the ground of invalid service of summons. What was in issue was his residence as far as the Hong Kong suit was concerned.
We therefore conclude that the stipulated fact that HERAS "is a resident of New Manila, Quezon City, Philippines" refers
to his residence at the time jurisdiction over his person was being sought by the Hong Kong court. With that stipulation of
fact, ASIAVEST cannot now claim that HERAS was a resident of Hong Kong at the time. Accordingly, since HERAS was
not a resident of Hong Kong and the action against him was, indisputably, one in personam, summons should have been
personally served on him in Hong Kong. The extraterritorial service in the Philippines was therefore invalid and did not
confer on the Hong Kong court jurisdiction over his person. It follows that the Hong Kong court judgment cannot be given
force and effect here in the Philippines for having been rendered without jurisdiction. Even assuming that HERAS was
formerly as resident of Hong Kong, he was no longer so in November 1984 when the extraterritorial service of summons
was attempted to be made on him; As declared by his secretary, which statement was not disputed by ASIAVEST, HERAS
left Hong Kong in October 1984 "for good." His absence in Hong Kong must have been the reason why summons was not
served on him therein; thus, ASIAVEST was constrained to apply for leave to effect service in the Philippines, and upon
obtaining a favorable action on the matter, it commissioned the Sycip Salazar Hernandez & Gatmaitan law firm to serve
the summons here in the Philippines. HERAS, who was also an absentee, should have been served with summons in the
same manner as a non-resident not found in Hong Kong. Section 17, Rule 14 of the Rules of Court providing for
extraterritorial service will not apply because of the suit against him was in personam. Neither can we apply Section 18,

144
which allows extraterritorial service on a resident defendant who is temporarily absent from the country, because even if
HERAS be considered as a resident of Hong Kong, the undisputed fact remains that he left Hong Kong not only
"temporarily" but "for good." aDHScI

DECISION

DAVIDE, JR, J p:

In issue is the enforceability in the Philippines of a foreign judgment. The antecedents are summarized in the 24 August
1990 Decision 1 of Branch 107 of the Regional Trial Court of Quezon City in Civil Case No. Q-52452; thus:,

The plaintiff Asiavest Limited filed a complaint on December 3, 1987 against the defendant Antonio Heras praying that said
defendant be ordered to pay to the plaintiff the amounts awarded by the Hong Kong Court Judgment dated December 28,
1984 and amended on April 13, 1987 to wit:

1) US$1,810,265.40 or its equivalent in Hong Kong currency at the time of payment with legal interest from December
28, 1984 until fully paid;

2) interest on the sum of US$1,500.00 at 9.875% per annum from October 31, 1984 to December 28, 1984; and

3) HK$905.00 at fixed cost in the action; and

4) at least $80,000.00 representing attorney's fees, litigation expenses and cost, with interest thereon from the date
of the judgment until fully paid.

On March 3, 1988 the defendant filed a Motion to Dismiss. However, before the court could resolve the said motion, a fire
which partially razed the Quezon City Hall Building on June 11, 1988 totally destroyed the office of this Court, together with
all its records, equipment and properties. On July 26, 1988, the plaintiff, through counsel filed a Motion for Reconstitution
of Case Records. The Court, after allowing the defendant to react thereto, granted the said Motion and admitted the
annexes attached thereto as the reconstituted records of this case per Order dated September 6, 1988. Thereafter, the
Motion to Dismiss, the resolution of which had been deferred, was denied by the Court in its Order of October 4, 1988.

On October 19, 1988 defendant filed his Answer. The case was then set for pre-trial conference. At the conference, the
parties could not arrive at any settlement. However, they agreed on the following stipulations of facts:

1. The defendant admits the existence of the judgment dated December 28, 1984 as well as its amendment dated
April 13, 1987, but not necessarily the authenticity or validity thereof;

2. The plaintiff is not doing business and is not licensed to do business in the Philippines;

3. The residence of defendant, Antonio Heras, is New Manila, Quezon City.

The only issue for this Court to determine is, whether or not the judgment of the Hong Kong Court has been repelled by
evidence of want of jurisdiction, want of notice to the party, collusion, fraud or clear mistake of law or fact, such as to
overcome the presumption established in Section 50, Rule 39 of the Rules of Court in favor of foreign judgments.

In view of the admission by the defendant of the existence of the aforementioned judgment (Pls. See Stipulations of Facts
in the Order dated January 5, 1989 as amended by the Order of January 18, 1989) as well as the legal presumption in
favor of the plaintiff as provided for in paragraph (b), Sec. 50, (Ibid.), the plaintiff presented only documentary evidence to
show rendition, existence, and authentication of such judgment by the proper officials concerned (Pls. See Exhibits "A" thru
"B", with their submarkings). In addition, the plaintiff presented testimonial and documentary evidence to show its
entitlement to attorney's fees and other expenses of litigation . . .

On the other hand, the defendant presented two witnesses, namely, Fortunata dela Vega and Russel Warren Lousich.

The gist of Ms. dela Vega's testimony is to the effect that no writ of summons or copy of a statement of claim of Asiavest
Limited was ever served in the office of the Navegante Shipping Agency Limited and/or for Mr. Antonio Heras, and that no
service of the writ of summons was either served on the defendant at his residence in New Manila, Quezon City. Her
knowledge is based on the fact that she was the personal secretary of Mr. Heras during his JD Transit days up to the latter
part of 1972 when he shifted or diversified to shipping business in Hong Kong; that she was in-charge of all his letters and
correspondence, business commitments, undertakings, conferences and appointments, until October 1984 when Mr. Heras
left Hong Kong for good; that she was also the Officer-in-Charge or Office Manager of Navegante Shipping Agency LTD, a
Hong Kong registered and based company acting as ships agent, up to and until the company closed shop sometime in
the first quarter of 1985 when shipping business collapsed worldwide; that the said Company held office at 34-35 Connaught

145
Road, Central Hong Kong and later transferred to Caxton House at Duddel Street, Hong Kong, until the company closed
shop in 1985; and that she was certain of such facts because she held office at Caxton House up to the first quarter of
1985.

Mr. Lousich was presented as an expert on the laws of Hong Kong, and as a representative of the law office of the
defendant's counsel who made a verification of the record of the case filed by the plaintiff in Hong Kong against the
defendant as well as the procedure in serving Court processes in Hong Kong.

In his affidavit (Exh. "2") which constitutes his direct testimony the said witness stated that:

The defendant was sued on the basis of his personal guarantee of the obligations of Compania Hermanos de Navegacion
S.A. There is no record that a writ of summons was served on the person of the defendant in Hong Kong, or that any such
attempt at service was made. Likewise, there is no record that a copy of the judgment of the High Court was furnished or
served on the defendant; anyway, it is not a legal requirement to do so under Hong Kong laws;

a) The writ of summons or claim can be served by the solicitor (lawyer) of the claimant or plaintiff. In Hong Kong there
are no Court personnel who serve writs of summons and/or most other processes.

b) If the writ of summons or claim (or complaint) is not contested, the claimant or the plaintiff is not required to present
proof of his claim or complaint nor present evidence under oath of the claim in order to obtain a Judgment.

c) There is no legal requirement that such a Judgment or decision rendered by the Court in Hong Kong [to] make a
recitation of the facts or the law upon which the claim is based.

d) There is no necessity to furnish the defendant with a copy of the Judgment or decision rendered against him.

e) In an action based on a guarantee, there is no established legal requirement or obligation under Hong Kong laws
that the creditor must first bring proceedings against the principal debtor. The creditor can immediately go against the
guarantor.

On cross examination, Mr. Lousich stated that before he was commissioned by the law firm of the defendant's counsel as
an expert witness and to verify the records of the Hong Kong case he had been acting as counsel for the defendant in a
number of commercial matters; that there was an application for service of summons upon the defendant outside the
jurisdiction of Hong Kong; that there was an order of the Court authorizing service upon Heras outside of Hong Kong,
particularly in Manila or any other place in the Philippines (p. 9, TSN, 2/14/90); that there must be adequate proof of service
of summons otherwise the Hong Kong Court will refuse to render judgment (p. 10, ibid); that the mere fact that the Hong
Kong Court rendered judgment, it can be presumed that there was service of summons; that in this case, it is not just a
presumption because there was an affidavit stating that service was effected in [sic] a particular man here in Manila; that
such affidavit was filed by one Jose R. Fernandez of the firm Sycip Salazar on the 21st of December 1984 and stated in
essence that "on Friday the 23rd of November 1984 he served the 4th defendant at No. 6 First Street, Quezon City by
leaving it at that address with Mr. Dionisio Lopez, the son-in-law of the 4th defendant the copy of the writ and Mr. Lopez
informed me and I barely believed that he would bring the said writ to the attention of the 4th "defendant" (pp. 11-12, ibid.);
that upon filing of that affidavit the Court was asked and granted judgment against the 4th defendant; and that if the
summons or claim is not contested, the claimant of the plaintiff is not required to present proof of his claim or complaint or
present evidence under oath of the claim in order to obtain judgment; and that such judgment can be enforced in the same
manner as a judgment rendered after full hearing.

The trial court held that since the Hong Kong court judgment had been duly proved, it is a presumptive evidence of a right
as between the parties; hence, the party impugning it had the burden to prove want of jurisdiction over his person. HERAS
failed to discharge that burden. He did not testify to state categorically and under oath that he never received summons.
Even his own witness Lousich admitted that HERAS was served with summons in his Quezon City residence. As to De la
Vega's testimony regarding non-service of summons, the same was hearsay and had no probative value. prLL

As to HERAS' contention that the Hong Kong court judgment violated the Constitution and the procedural laws of the
Philippines because it contained no statements of the facts and the law on which it was based, the trial court ruled that
since the issue related to procedural matters, the law of the forum, i.e., Hong Kong laws, should govern. As testified by the
expert witness Lousich, such legalities were not required under Hong Kong laws. The trial court also debunked HERAS'
contention that the principle of excussion under Article 2058 of the Civil Code of the Philippines was violated. It declared
that matters of substance are subject to the law of the place where the transaction occurred; in this case, Hong Kong laws
must govern.

The trial court concluded that the Hong Kong court judgment should be recognized and given effect in this jurisdiction for
failure of HERAS to overcome the legal presumption in favor of the foreign judgment It then decreed; thus:
146
WHEREFORE, judgment is hereby rendered ordering defendant to pay to the plaintiff the following sums or their equivalents
in Philippine currency at the time of payment: US$1,810,265.40 plus interest on the sum of US$1,500,000.00 at 9.875%
per annum from October 31, 1984 to December 28, 1984, and HK$905 as fixed cost, with legal interests on the aggregate
amount from December 28, 1984, and to pay attorneys fees in the sum of P80,000.00.

ASIAVEST moved for the reconsideration of the decision. It sought an award of judicial costs and an increase in attorney's
fees in the amount of US$19,346.45 with interest until full payment of the said obligations. On the other hand, HERAS no
longer opposed the motion and instead appealed the decision to the Court of Appeals, which docketed the appeal as CA-
G.R. CV No. 29513.

In its order 2 of 2 November 1990, the trial court granted ASIAVEST's motion for reconsideration by increasing the award
of attorney's fees to "US$19,345.65 OR ITS EQUIVALENT IN PHILIPPINE CURRENCY, AND TO PAY THE COSTS OF
THIS SUIT," provided that ASIAVEST would pay the corresponding filing fees for the increase. ASIAVEST appealed the
order requiring prior payment of filing fees. However, it later withdrew its appeal and paid the additional filing fees.

On 3 April 1997, the Court of Appeals rendered its decision 3 reversing the decision of the trial court and dismissing
ASIAVEST's complaint without prejudice. It underscored the fact that a foreign judgment does not of itself have any
extraterritorial application. For it to be given effect, the foreign tribunal should have acquired jurisdiction over the person
and the subject matter. If such tribunal has not acquired jurisdiction, its judgment is void.

The Court of Appeals agreed with the trial court that matters of remedy and procedure such as those relating to service of
summons upon the defendant are governed by the lex fori, which was, in this case, the law of Hong Kong. Relative thereto,
it gave weight to Lousich's testimony that under the Hong Kong law, the substituted service of summons upon HERAS
effected the Philippines by the clerk of Sycip Salazar Hernandez & Gatmaitan firm would be valid provided that it was done
in accordance with Philippine laws. It then stressed that where the action is in personam and the defendant is in the
Philippines, the summons should be personally served on the defendant pursuant to Section 7, Rule 14 of the Rules of
Court. 4 Substituted service may only be availed of where the defendant cannot be promptly served in person, the fact of
impossibility of personal service should be explained in the proof of service. It also found as persuasive HERAS' argument
that instead of directly using the clerk of the Sycip Salazar Hernandez & Gatmaitan law office, who was not authorized by
the judge of the court issuing the summons, ASIAVEST should have asked for leave of the local courts to have the foreign
summons served by the sheriff or other court officer of the place where service was to be made, or for special reasons by
any person authorized by the judge. cdasia

The Court of Appeals agreed with HERAS that "notice sent outside the state to a non-resident is unavailing to give
jurisdiction in an action against him personally for money recovery." Summons should have been personally served on
HERAS in Hong Kong, for, as claimed by ASIAVEST, HERAS was physically present in Hong Kong for nearly 14 years.
Since there was not even an attempt to serve summons on HERAS in Hong Kong, the Hong Kong Supreme Court did not
acquire jurisdiction over HERAS. Nonetheless, it did not totally foreclose the claim of ASIAVEST; thus:

While we are not fully convinced that [HERAS] has a meritorious defense against [ASIAVEST's] claims or that [HERAS]
ought to be absolved of any liability, nevertheless, in view of the foregoing discussion, there is a need to deviate from the
findings of the lower court in the interest of justice and fair play This, however, is without prejudice to whatever action
[ASIAVEST] might deem proper in order to enforce its claims against [HERAS].

Finally, the Court of Appeals also agreed with HERAS that it was necessary that evidence supporting the validity of the
foreign judgment be submitted and that our courts are not bound to give effect to foreign judgments which contravene our
laws and the principle of sound morality and public policy.

ASIAVEST forthwith filed the instant petition alleging that the Court of Appeals erred in ruling that

I.

. . . IT WAS NECESSARY FOR [ASIAVEST] TO PRESENT EVIDENCE 'SUPPORTING THE VALIDITY OF THE
JUDGMENT';

II.

. . . THE SERVICE OF SUMMONS ON [HERAS] WAS DEFECTIVE UNDER PHILIPPINE LAW;

III.

. . . SUMMONS SHOULD HAVE BEEN PERSONALLY SERVED ON HERAS IN HONG KONG;

IV.
147
. . . THE HONG KONG SUMMONS SHOULD HAVE BEEN SERVED WITH LEAVE OF PHILIPPINE COURTS;

V.

. . . THE FOREIGN JUDGMENT 'CONTRAVENES PHILIPPINE LAWS, THE PRINCIPLES OF SOUND MORALITY, AND
THE PUBLIC POLICY OF THE PHILIPPINES.

Being interrelated, we shall take up together the assigned errors.

Under paragraph (b) of Section 50, Rule 39 of the Rules of Court, 5 which was the governing law at the time this case was
decided by the trial court and respondent Court of Appeals, a foreign judgment against a person rendered by a court having
jurisdiction to pronounce the judgment is presumptive evidence of a right as between the parties and their successors in
interest by the subsequent title. However, the judgment may be repelled by evidence of want of jurisdiction, want of notice
to the party, collusion, fraud, or clear mistake of law or fact.

Also, Section 3(n) of Rule 131 of the New Rules of Evidence provides that in the absence of proof to the contrary, a court,
or judge acting as such, whether in the Philippines or elsewhere, is presumed to have acted in the lawful exercise of
jurisdiction.

Hence, once the authenticity of the foreign judgment is proved, the burden to repel it on grounds provided for in paragraph
(b) of Section 50, Rule 39 of the Rules of Court is on the party challenging the foreign judgment — HERAS in this case.
cdrep

At the pre-trial conference, HERAS admitted the existence of the Hong Kong judgment. On the other hand, ASIAVEST
presented evidence to prove rendition, existence, and authentication of the judgment by the proper officials. The judgment
is thus presumed to be valid and binding in the country from which it comes, until the contrary is shown. 6 Consequently,
the first ground relied upon by ASIAVEST has merit. The presumption of validity accorded foreign judgment would be
rendered meaningless were the party seeking to enforce it be required to first establish its validity.

The main argument raised against the Hong Kong judgment is that the Hong Kong Supreme Court did not acquire
jurisdiction over the person of HERAS. This involves the issue of whether summons was properly and validly served on
HERAS. It is settled that matters of remedy and procedure such as those relating to the service of process upon the
defendant are governed by the lex fori or the law of the forum, 7 i.e., the law of Hong Kong in this case. HERAS insisted
that according to his witness Mr. Lousich, who was presented as an expert on Hong Kong laws, there was no valid service
of summons on him.

In his counter-affidavit, 8 which served as his direct testimony per agreement of the parties, 9 Lousich declared that the
record of the Hong Kong case failed to show that a writ of summons was served upon HERAS in Hong Kong or that any
such attempt was made. Neither did the record show that a copy of the judgment of the court was served on HERAS. He
stated further that under Hong Kong laws (a) a writ of summons could be served by the solicitor of the claimant or plaintiff;
and (b) where the said writ or claim was not contested, the claimant or plaintiff was not required to present proof under oath
in order to obtain judgment.

On cross-examination by counsel for ASIAVEST, Lousich testified that the Hong Kong court authorized service of summons
on HERAS outside of its jurisdiction, particularly in the Philippines. He admitted also the existence of an affidavit of one
Jose R. Fernandez of the Sycip Salazar Hernandez & Gatmaitan law firm stating that he (Fernandez) served summons on
HERAS on 13 November 1984 at No. 6, 1st St., Quezon City, by leaving a copy with HERAS's son-in-law Dionisio Lopez.
10 On redirect examination, Lousich declared that such service of summons would be valid under Hong Kong laws provided
that it was in accordance with Philippine laws. 11

We note that there was no objection on the part of ASIAVEST on the qualification of Mr. Lousich as an expert on the Hong
Kong law. Under Sections 24 and 25, Rule 132 of the New Rules of Evidence, the record of public documents of a sovereign
authority, tribunal, official body, or public officer may be proved by (1) an official publication thereof or (2) a copy attested
by the officer having the legal custody thereof, which must be accompanied, if the record is not kept in the Philippines, with
a certificate that such officer has the custody. The certificate may be issued by the secretary of the embassy or legation,
consul general, consul, vice consul, or consular agent, or any officer in the foreign service of the Philippines stationed in
the foreign country in which the record is kept, and authenticated by the seal of his office. The attestation must state, in
substance, that the copy is a correct copy of the original, or a specific part thereof, as the case may be, and must be under
the official seal of the attesting officer.

Nevertheless, the testimony of an expert witness may be allowed to prove a foreign law. An authority 12 on private
international law thus noted:

148
Although it is desirable that foreign law be proved in accordance with the above rule, however, the Supreme Court held in
the case of Willamettee Iron and Steel Works v. Muzzal, 13 that Section 41, Rule 123 (Section 25, Rule 132 of the Revised
Rules of Court) does not exclude the presentation of other competent evidence to prove the existence of a foreign law. In
that case, the Supreme Court considered the testimony under oath of an attorney-at-law of San Francisco, California, who
quoted verbatim a section of California Civil Code and who stated that the same was in force at the time the obligations
were contracted, as sufficient evidence to establish the existence of said law. Accordingly, in line with this view, the Supreme
Court in the Collector of Internal Revenue v. Fisher et al., 14 upheld the Tax Court in considering the pertinent law of
California as proved by the respondents' witness. In that case, the counsel for respondent "testified that as an active
member of the California Bar since 1951, he is familiar with the revenue and taxation laws of the State of California. When
asked by the lower court to state the pertinent California law as regards exemption of intangible personal properties, the
witness cited Article 4, Sec. 13851 (a) & (b) of the California Internal and Revenue Code as published in Derring's California
Code, a publication of Bancroft-Whitney Co., Inc. And as part of his testimony, a full quotation of the cited section was
offered in evidence by respondents." Likewise, in several naturalization cases, it was held by the Court that evidence of the
law of a foreign country on reciprocity regarding the acquisition of citizenship, although not meeting the prescribed rule of
practice, may be allowed and used as basis for favorable action, if, in the light of all the circumstances, the Court is "satisfied
of the authenticity of the written proof offered." 15 Thus, in a number of decisions, mere authentication of the Chinese
Naturalization Law by the Chinese Consulate General of Manila was held to be competent proof of that law. 16

There is, however, nothing in the testimony of Mr. Lousich that touched on the specific law of Hong Kong in respect of
service of summons either in actions in rem or in personam, and where the defendant is either a resident or nonresident of
Hong Kong. In view of the absence of proof of the Hong Kong law on this particular issue, the presumption of identity or
similarity or the so-called processual presumption shall come into play. It will thus be presumed that the Hong Kong law on
the matter is similar to the Philippine law. 17

As stated in Valmonte vs. Court of Appeals, 18 it will be helpful to determine first whether the action is in personam, in rem,
or quasi in rem because the rules on service of summons under Rule 14 of the Rules of Court of the Philippines apply
according to the nature of the action. prcd

An action in personam is an action against a person on the basis of his personal liability. An action in rem is an action
against the thing itself instead of against the person. 19 An action quasi in rem is one wherein an individual is named as
defendant and the purpose of the proceeding is to subject his interest therein to the obligation or lien burdening the property.
20

In an action in personam, jurisdiction over the person of the defendant is necessary for the court to validly try and decide
the case. Jurisdiction over the person of a resident defendant who does not voluntarily appear in court can be acquired by
personal service of summons as provided under Section 7, Rule 14 of the Rules of Court. If he cannot be personally served
with summons within a reasonable time, substituted service may be made in accordance with Section 8 of said Rule. If he
is temporarily out of the country, any of the following modes of service may be resorted to: (1) substituted service set forth
in Section 8; 21 (2) personal service outside the country, with leave of court; (3) service by publication also with leave of
court; 22 or (4) any other manner the court may deem sufficient. 23

However, in an action in personam wherein the defendant is a non-resident who does not voluntarily submit himself to the
authority of the court, personal service of summons within the state is essential to the acquisition of jurisdiction over her
person. 24 This method of service is possible if such defendant is physically present in the country. If he is not found therein,
the court cannot acquire jurisdiction over his person and therefore cannot validly try and decide the case against him. 25
An exception was laid down in Gemperle v. Schenker 26 wherein a non-resident was served with summons through his
wife, who was a resident of the Philippines and who was his representative and attorney-in-fact in a prior civil case filed by
him; moreover, the second case was a mere offshoot of the first case.

On the other hand, in a proceeding in rem or quasi in rem, jurisdiction over the person of the defendant is not a prerequisite
to confer jurisdiction on the court provided that the court acquires jurisdiction over the res. Nonetheless, summons must be
served upon the defendant not for the purpose of vesting the court with jurisdiction but merely for satisfying the due process
requirements. 27 Thus, where the defendant is a non-resident who is not found in the Philippines and (1) the action affects
the personal status of the plaintiff; (2) the action relates to, or the subject matter of which is property in the Philippines in
which the defendant has or claims a lien or interest; (3) the action seeks the exclusion of the defendant from any interest
in the property located in the Philippines; or (4) the property of the defendant has been attached in the Philippines — service
of summons may be effected by (a) personal service out of the country, with leave of court; (b) publication, also with leave
of court; or (c) any other manner the court may deem sufficient. 28

149
In the case at bar, the action filed in Hong Kong against HERAS was in personam, since it was based on his personal
guarantee of the obligation of the principal debtor. Before we can apply the foregoing rules, we must determine first whether
HERAS was a resident of Hong Kong.

Fortunata de la Vega, HERAS's personal secretary in Hong Kong since 1972 until 1985, 29 testified that HERAS was the
President and part owner of a shipping company in Hong Kong during all those times that she served as his secretary. He
had in his employ a staff of twelve. 30 He had "business commitments, undertakings, conferences, and appointments until
October 1984 when [he] left Hong Kong for good." 31 HERAS's other witness, Russel Warren Lousich, testified that he had
acted as counsel for HERAS "for a number of commercial matters." 32 ASIAVEST then infers that HERAS was a resident
of Hong Kong because he maintained a business there.

It must be noted that in his Motion to Dismiss, 33 as well as in his Answer 34 to ASIAVEST's complaint for the enforcement
of the Hong Kong court judgment, HERAS maintained that the Hong Kong court did not have jurisdiction over him because
the fundamental rule is that jurisdiction in personam over non-resident defendants, so as to sustain a money judgment,
must be based upon personal service of summons within the state which renders the judgment. 35

For its part, ASIAVEST, in its Opposition to the Motion to Dismiss 36 contended: "The question of Hong Kong court's 'want
of jurisdiction' is therefore a triable issue if it is to be pleaded by the defendant to 'repel' the foreign judgment. Facts showing
jurisdictional lack (e.g. that the Hong Kong suit was in personam, that defendant was not a resident of Hong Kong when
the suit was filed or that he did not voluntarily submit to the Hong Kong court's jurisdiction) should be alleged and proved
by the defendant." 37

In his Reply (to the Opposition to Motion to Dismiss), 38 HERAS argued that the lack of jurisdiction over his person was
corroborated by ASIAVEST's allegation in the complaint that he "has his residence at No. 6, 1st St., New Manila, Quezon
City, Philippines." He then concluded that such Judicial admission amounted to evidence that he was and is not a resident
of Hong Kong.

Significantly, in the pre-trial conference, the parties came up with stipulations of facts, among which was that "the residence
of defendant, Antonio Heras, is New Manila, Quezon City." 39

We note that the residence of HERAS insofar as the action for the enforcement of the Hong Kong court judgment is
concerned, was never in issue. He never challenged the service of summons on him through a security guard in his Quezon
City residence and through a lawyer in his office in that city. In his Motion to Dismiss, he did not question the jurisdiction of
the Philippine court over his person on the ground of invalid service of summons. What was in issue was his residence as
far as the Hong Kong suit was concerned. We therefore conclude that the stipulated fact that HERAS "is a resident of New
Manila, Quezon City, Philippines" refers to his residence at the time jurisdiction over his person was being sought by the
Hong Kong court. With that stipulation of fact, ASIAVEST cannot now claim that HERAS was a resident of Hong Kong at
the time.

Accordingly, since HERAS was not a resident of Hong Kong and the action against him was, indisputably, one in personam,
summons should have been personally served on him in Hong Kong. The extraterritorial service in the Philippines was
therefore invalid and did not confer on the Hong Kong court jurisdiction over his person. It follows that the Hong Kong court
judgment cannot be given force and effect here in the Philippines for having been rendered without jurisdiction.

Even assuming that HERAS was formerly a resident of Hong Kong, he was no longer so in November 1984 when the
extraterritorial service of summons was attempted to be made on him. As declared by his secretary, which statement was
not disputed by ASIAVEST, HERAS left Hong Kong in October 1984 "for good." 40 His absence in Hong Kong must have
been the reason why summons was not served on him therein; thus, ASIAVEST was constrained to apply for leave to effect
service in the Philippines, and upon obtaining a favorable action on the matter, it commissioned the Sycip Salazar
Hernandez & Gatmaitan law firm to serve the summons here in the Philippines.

In Brown v. Brown, 41 the defendant was previously a resident of the Philippines. Several days after a criminal action for
concubinage was filed against him, he abandoned the Philippines. Later, a proceeding quasi in rem was instituted against
him. Summons in the latter case was served on the defendant's attorney-in-fact at the latter's address. The Court held that
under the facts of the case, it could not be said that the defendant was "still a resident of the Philippines because he ha[d]
escaped to his country and [was] therefore an absentee in the Philippines." As such, he should have been "summoned in
the same manner as one who does not reside and is not found in the Philippines."

Similarly, HERAS, who was also an absentee, should have been served with summons in the same manner as a non-
resident not found in Hong Kong. Section 17, Rule 14 of the Rules of Court providing for extraterritorial service will not
apply because the suit against him was in personam. Neither can we apply Section 18, which allows extraterritorial service

150
on a resident defendant who is temporarily absent from the country, because even if HERAS be considered as a resident
of Hong Kong, the undisputed fact remains that he left Hong Kong not only "temporarily" but "for good." cdll

IN VIEW OF ALL THE FOREGOING, judgment is hereby rendered DENYING the petition in this case and AFFIRMING the
assailed Judgment of the Court of Appeals in CA-G.R. CV No. 29513.

No costs.

SO ORDERED.

Bellosillo, Vitug and Panganiban, JJ ., concur.

Quisumbing, J ., took no part.

25. Alba v. Court of Appeals, G.R. No. 164041. July 29, 2005

FIRST DIVISION

[G.R. No. 164041. July 29, 2005.]

ROSENDO ALBA, minor, represented by his mother and natural guardian, Armi A. Alba, and ARMI A. ALBA, in her personal
capacity, petitioners, vs. COURT OF APPEALS and ROSENDO C. HERRERA, respondents.

Ricardo C. Juan, Jr. for petitioner.

Virgilio C. Manguera & Associates for private respondent.

SYLLABUS

1. REMEDIAL LAW; CIVIL PROCEDURE; ANNULMENT OF JUDGMENT; GROUNDS. — Under Section 2, Rule 47
of the 1997 Revised Rules of Civil Procedure, judgments may be annulled on the grounds of lack of jurisdiction and extrinsic
fraud.

2. ID.; ID.; ACTIONS; ACTIONS IN PERSONAM, IN REM AND QUASI IN REM; DISTINGUISHED. — Whether or
not the trial court acquired jurisdiction over the person of petitioner and her minor child depends on the nature of private
respondent's action, that is, in personam, in rem or quasi in rem. An action in personam is lodged against a person based
on personal liability; an action in rem is directed against the thing itself instead of the person; while an action quasi in rem
names a person as defendant, but its object is to subject that person's interest in a property to a corresponding lien or
obligation. Hence, petitions directed against the "thing" itself or the res, which concerns the status of a person, like a petition
for adoption, annulment of marriage, or correction of entries in the birth certificate, as in the instant case, are actions in
rem. cTCEIS

3. ID.; COURTS; JURISDICTION; JURISDICTION OF THE COURT, HOW ACQUIRED. — In an action in personam,
jurisdiction over the person of the defendant is necessary for the court to validly try and decide the case. In a proceeding
in rem or quasi in rem, jurisdiction over the person of the defendant is not a prerequisite to confer jurisdiction on the court,
provided that the latter has jurisdiction over the res. Jurisdiction over the res is acquired either (a) by the seizure of the
property under legal process, whereby it is brought into actual custody of the law; or (b) as a result of the institution of legal
proceedings, in which the power of the court is recognized and made effective. The service of summons or notice to the
defendant is not for the purpose of vesting the court with jurisdiction but merely for satisfying the due process requirements.

4. ID.; ACTIONS; ACTION IN REM; A PETITION FOR SUBSTANTIAL CORRECTIONS OR CANCELLATIONS OF


ENTRIES IN CIVIL REGISTRY RECORDS AFFECTING THE STATUS OR LEGITIMACY OF A PERSON IS A
PROCEEDING IN REM; CASE AT BAR. — In the case at bar, the filing with the trial court of the petition for cancellation
vested the latter jurisdiction over the res. Substantial corrections or cancellations of entries in civil registry records affecting
the status or legitimacy of a person may be effected through the institution of a petition under Rule 108 of the Revised
Rules of Court, with the proper Regional Trial Court. Being a proceeding in rem, acquisition of jurisdiction over the person
of petitioner is therefore not required in the present case. It is enough that the trial court is vested with jurisdiction over the
subject matter.

5. ID.; CIVIL PROCEDURE; ANNULMENT OF JUDGMENT; EXTRINSIC FRAUD, WHEN PRESENT. — Extrinsic
fraud exists when there is a fraudulent act committed by the prevailing party outside of the trial of the case, whereby the
defeated party was prevented from presenting fully his side of the case by fraud or deception practiced on him by the
prevailing party.
151
6. ID.; EVIDENCE; PRESENTATION OF EVIDENCE; EXAMINATION OF WITNESSES; DECLARANTS OF
WRITTEN STATEMENTS PERTAINING TO DISPUTED FACTS MUST BE PRESENTED AT THE TRIAL FOR CROSS-
EXAMINATION. — The basic rule of evidence is that unless the affiants themselves are placed on the witness stand to
testify on their affidavits, such affidavits must be rejected for being hearsay. Stated differently, the declarants of written
statements pertaining to disputed facts must be presented at the trial for cross-examination.

7. ID.; CIVIL PROCEDURE; APPEALS; PETITION FOR REVIEW ON CERTIORARI UNDER RULE 45; MAY BE
RESORTED TO BY THE AGGRIEVED PARTY BY A DECISION OF THE COURT OF APPEALS IN AN ACTION TO
ANNUL A JUDGMENT OF A REGIONAL TRIAL COURT; CASE AT BAR. — The proper remedy of a party aggrieved by a
decision of the Court of Appeals in an action to annul a judgment of a Regional Trial Court is a petition for review on
certiorari under Rule 45 of the Revised Rules of Civil Procedure, where only questions of law may be raised. The resort of
petitioner to the instant civil action for certiorari under Rule 65 is therefore erroneous. The special civil action of certiorari
will not be allowed as a substitute for failure to timely file a petition for review under Rule 45, which should be instituted
within 15 days from receipt of the assailed decision or resolution. The wrong choice of remedy thus provides another reason
to dismiss this petition.

8. CIVIL LAW; FAMILY CODE; PATERNITY AND FILIATION; ILLEGITIMATE CHILDREN; SHALL USE THE
SURNAME OF THEIR MOTHER, UNLESS THEIR FATHER RECOGNIZES THEIR FILIATION, IN WHICH CASE THEY
MAY BEAR THE FATHER'S SURNAME. — Under Article 176 of the Family Code as amended by Republic Act (RA) No.
9255, which took effect on March 19, 2004, illegitimate children shall use the surname of their mother, unless their father
recognizes their filiation, in which case they may bear the father's surname. In Wang v. Cebu Civil Registrar, it was held
that an illegitimate child whose filiation is not recognized by the father, bears only a given name and his mother's surname.
The name of the unrecognized illegitimate child identifies him as such. It is only when said child is recognized that he may
use his father's surname, reflecting his status as an acknowledged illegitimate child. In the present case, it is clear from the
allegations of Armi that petitioner minor is an illegitimate child because she was never married to private respondent.
Considering that the latter strongly asserts that he is not the father of petitioner minor, the latter is therefore an unrecognized
illegitimate child. As such, he must bear the surname of his mother. ISDCHA

DECISION

YNARES-SANTIAGO, J p:

Assailed in this petition for certiorari 1 are the February 27, 2004 decision 2 and the May 14, 2004 resolution 3 of the Court
of Appeals in CA-G.R. SP No. 61883, which dismissed petitioner's original action for annulment of judgment 4 of the
Regional Trial Court of Manila, Branch 37, and denied the motion for reconsideration, respectively. AIDTSE

The antecedent facts show that on October 21, 1996, private respondent Rosendo C. Herrera filed a petition 5 for
cancellation of the following entries in the birth certificate of "Rosendo Alba Herrera, Jr.", to wit: (1) the surname "Herrera"
as appended to the name of said child; (2) the reference to private respondent as the father of Rosendo Alba Herrera, Jr.;
and (3) the alleged marriage of private respondent to the child's mother, Armi A. Alba (Armi) on August 4, 1982 in
Mandaluyong City. He claimed that the challenged entries are false and that it was only sometime in September 1996 that
he learned of the existence of said birth certificate.

Private respondent alleged that he married only once, i.e., on June 28, 1965 with Ezperanza C. Santos and never
contracted marriage with Armi nor fathered Rosendo Alba Herrera, Jr. In support thereof, he presented certifications from
the Civil Registrar of Mandaluyong City 6 and the National Statistics Office, 7 both stating that they have no record of
marriage between private respondent and Armi.

On November 12, 1996, private respondent filed an amended petition, 8 impleading Armi and "all the persons who have or
claim any interest in th[e] petition." 9

On November 27, 1996, the trial court issued an Order setting the petition for hearing on January 24, 1997, and directed
the publication and service of said order to Armi at her address appearing in the birth certificate which is No. 418 Arquiza
St., Ermita, Manila, and to the Civil Registrar of the City of Manila and the Solicitor General. The full text of the order, reads:

In a verified Amended Petition for Correction of Entry, the Petitioner prays, inter alia, that the following entries appearing in
the subject Certificate of Live Birth be deleted:

1. All informations having reference to him as the father of the child mentioned therein;

2. The surname "Herrera" appended to the child's name;

3. His alleged marriage with the natural mother of the child.

152
Finding the Petition to be sufficient in form and substance, let the Petition be set for hearing on January 24, 1997 at nine
o'clock in the morning before this Branch at Rooms 447-449, Fourth Floor, Manila City Hall. All interested parties are hereby
notified of the said hearing and are ordered to show cause why the Petition should not be granted.

Let a copy of this Order be published at the expense of the Petitioner, once a week for three (3) consecutive weeks, in a
newspaper of general circulation in the City of Manila, and raffled pursuant to P.D. 1079. aETADI

Furnish the Office of the Solicitor General and the Office of the Local Civil Registrar of the City of Manila with copies of the
Petition and of this Order.

Let the same be likewise furnished the Private Respondent Armi Alba Herrera at the address indicated in the subject
Certificate of Live Birth.

SO ORDERED. 10

On January 13, 1997, before the scheduled January 24, 1997 hearing, the trial court issued an Amended Order 11 with
substantially the same contents, except that the hearing was re-scheduled to February 26, 1997. A copy of said Amended
Order was published in "Today", a newspaper of general circulation in Manila in its January 20, 27, and February 3, 1997
issues. Copies thereof were also sent to Armi at No. 418 Arquiza St., Ermita, Manila, on January 17, 1997, the Local Civil
Registrar of Manila and the Solicitor General.

At the scheduled hearing on February 26, 1997, the counsel from the Office of the Solicitor General appeared but filed no
opposition to the petition. Armi, on the other hand was not present. The return of the notice sent to her had the following
notation:

This is to certify that on January 17, 1997, the undersigned [process server] personally served a copy of the Amended
Order in Sp. Proc. No. 96-80512 dated January 13, 1997 to the private respondent, Armi Alba Herrera at . . . 418 Arquiza
St., Ermita, Manila, but failed and unavailing for reason that (sic), private respondent is no longer residing at said given
address. 12

On April 1, 1997, the court a quo rendered a decision which became final and executory on June 2, 1997. 13 The dispositive
portion thereof, states:

ACCORDINGLY, and pursuant to Rule 108 of the Revised Rules of Court, judgment is hereby rendered ordering the
correction of the entries in the Certificate of Live Birth of Rosendo Alba Herrera, Jr., in such a way that the entry under the
name of the child, the surname Herrera, Jr.[,] is ordered deleted, and the child shall be known as ROSENDO ALBA; and
that the entry under the date and place of marriage, the date August 4, 1982, Mandaluyong, MM is likewise ordered deleted
or cancelled.

Let a copy of this Decision be furnished the Local Civil Registrar of Manila for proper correction and entry.

SO ORDERED. 14

Private respondent filed a motion 15 for amendment of the decretal portion of the decision to include the cancellation of all
entries having reference to him as the father of petitioner minor. This was granted in the August 11, 1997 order of the trial
court as follows:

ACCORDINGLY, and pursuant to Rule 108 of the Revised Rules of Court, judgment is hereby rendered ordering the
correction of the entries in the Certificate of Live Birth of Rosendo Alba Herrera, Jr., in such a way that the entries under
the name of the child, the surname Herrera, Jr., and the name of the father Rosendo Caparas Herrera are ordered deleted,
and the child shall be known as ROSENDO ALBA; and the entry under the date and place of marriage, the date August 4,
1982, Mandaluyong, MM is likewise ordered deleted or cancelled.

SO ORDERED. 16

On November 24, 2000, Armi and petitioner minor filed a petition for annulment of judgment before the Court of Appeals
on the grounds of extrinsic fraud and lack of jurisdiction over their person. She allegedly came to know of the decision of
the trial court only on February 26, 1998, when San Beda College, where her son was enrolled as a high school student,
was furnished by private respondent with a copy of a court order directing the change of petitioner minor's surname from
Herrera to Alba. EDcICT

Armi averred that private respondent was aware that her address is at Unit 302 Plaza Towers Condominium, 1175 Lorenzo
Guerrero St., Ermita, Manila, because such was her residence when she and private respondent cohabited as husband
and wife from 1982 to 1988; and her abode when petitioner minor was born on March 8, 1985. Even after their separation,
153
private respondent continued to give support to their son until 1998; and that Unit 302 was conveyed to her by private
respondent on June 14, 1991 as part of his support to petitioner minor. According to Armi, her address i.e., No. 418 Arquiza
St., Ermita, Manila, as appearing in the birth certificate of their son, was entered in said certificate through the erroneous
information given by her sister, Corazon Espiritu. She stressed that private respondent knew all along that No. 418 Arquiza
St., is the residence of her sister and that he deliberately caused the service of notice therein to prevent her from opposing
the petition.

In his answer, private respondent denied paternity of petitioner minor and his purported cohabitation with Armi. He branded
the allegations of the latter as "false statements coming from a polluted source." 17

On February 27, 2004, the Court of Appeals dismissed the petition holding, among others, that petitioner failed to prove
that private respondent employed fraud and purposely deprived them of their day in court. It further held that as an
illegitimate child, petitioner minor should bear the surname of his mother. 18 Petitioners filed a motion for reconsideration
but was denied.

Hence, the instant petition.

Under Section 2, Rule 47 of the 1997 Revised Rules of Civil Procedure, judgments may be annulled on the grounds of lack
of jurisdiction and extrinsic fraud. 19

Whether or not the trial court acquired jurisdiction over the person of petitioner and her minor child depends on the nature
of private respondent's action, that is, in personam, in rem or quasi in rem. An action in personam is lodged against a
person based on personal liability; an action in rem is directed against the thing itself instead of the person; while an action
quasi in rem names a person as defendant, but its object is to subject that person's interest in a property to a corresponding
lien or obligation. 20

Hence, petitions directed against the "thing" itself or the res, 21 which concerns the status of a person, 22 like a petition for
adoption, 23 annulment of marriage, 24 or correction of entries in the birth certificate, 25 as in the instant case, are actions
in rem.

In an action in personam, jurisdiction over the person of the defendant is necessary for the court to validly try and decide
the case. In a proceeding in rem or quasi in rem, jurisdiction over the person of the defendant is not a prerequisite to confer
jurisdiction on the court, provided that the latter has jurisdiction over the res. Jurisdiction over the res is acquired either (a)
by the seizure of the property under legal process, whereby it is brought into actual custody of the law; or (b) as a result of
the institution of legal proceedings, in which the power of the court is recognized and made effective. 26 The service of
summons or notice to the defendant is not for the purpose of vesting the court with jurisdiction but merely for satisfying the
due process requirements. 27

In the case at bar, the filing with the trial court of the petition for cancellation vested the latter jurisdiction over the res.
Substantial corrections or cancellations of entries in civil registry records affecting the status or legitimacy of a person may
be effected through the institution of a petition under Rule 108 of the Revised Rules of Court, with the proper Regional Trial
Court. 28 Being a proceeding in rem, acquisition of jurisdiction over the person of petitioner is therefore not required in the
present case. It is enough that the trial court is vested with jurisdiction over the subject matter. aSTHDc

The service of the order at No. 418 Arquiza St., Ermita, Manila and the publication thereof in a newspaper of general
circulation in Manila, sufficiently complied with the requirement of due process, the essence of which is an opportunity to
be heard. Said address appeared in the birth certificate of petitioner minor as the residence of Armi. Considering that the
Certificate of Birth bears her signature, the entries appearing therein are presumed to have been entered with her approval.
Moreover, the publication of the order is a notice to all indispensable parties, including Armi and petitioner minor, which
binds the whole world to the judgment that may be rendered in the petition. An in rem proceeding is validated essentially
through publication. 29 The absence of personal service of the order to Armi was therefore cured by the trial court's
compliance with Section 4, Rule 108, which requires notice by publication, thus:

SEC. 4. Notice and publication. — Upon the filing of the petition, the court shall, by an order, fix the time and place for the
hearing of the same, and cause reasonable notice thereof to be given to the persons named in the petition. The court shall
also cause the order to be published once a week for three (3) consecutive weeks in a newspaper of general circulation in
the province.

In Barco v. Court of Appeals, the trial court granted a petition for correction/change of entries in a minor's birth certificate
to reflect the name of the minor's real father as well as to effect the corresponding change of her surname. In seeking to
annul said decision, the other children of the alleged father claimed that they are indispensable parties to the petition for
correction, hence, the failure to implead them is a ground to annul the decision of the trial court. The Court of Appeals

154
denied the petition which was sustained by this Court on the ground, inter alia, that while petitioner is indeed an
indispensable party, the failure to implead her was cured by the publication of the order of hearing. Thus —

Undoubtedly, Barco is among the parties referred to in Section 3 of Rule 108. Her interest was affected by the petition for
correction, as any judicial determination that June was the daughter of Armando would affect her ward's share in the estate
of her father. It cannot be established whether Nadina knew of Mary Joy's existence at the time she filed the petition for
correction. Indeed, doubt may always be cast as to whether a petitioner under Rule 108 would know of all the parties whose
interests may be affected by the granting of a petition. For example, a petitioner cannot be presumed to be aware of all the
legitimate or illegitimate offsprings of his/her spouse or paramour. The fact that Nadina amended her petition to implead
Francisco and Gustilo indicates earnest effort on her part to comply with Section 3 as quoted above. ADaSET

Yet, even though Barco was not impleaded in the petition, the Court of Appeals correctly pointed out that the defect was
cured by compliance with Section 4, Rule 108, which requires notice by publication, thus:

Section 4. Upon the filing of the petition, the court shall, by order, fix the time and place for the hearing of the same,
and cause reasonable notice thereof to be given to the persons named in the petition. The court shall also cause the order
to be published once a week for three (3) consecutive weeks in a newspaper of general circulation in the province.

The purpose precisely of Section 4, Rule 108 is to bind the whole world to the subsequent judgment on the petition. The
sweep of the decision would cover even parties who should have been impleaded under Section 3, Rule 108, but were
inadvertently left out. The Court of Appeals correctly noted:

The publication being ordered was in compliance with, and borne out by the Order of January 7, 1985. The actual publication
of the September 22, 1983 Order, conferred jurisdiction upon the respondent court to try and decide the case. While
"nobody appeared to oppose the instant petition" during the December 6, 1984 hearing, that did not divest the court from
its jurisdiction over the case and of its authority to continue trying the case. For, the rule is well-settled, that jurisdiction,
once acquired continues until termination of the case.

Verily, a petition for correction is an action in rem, an action against a thing and not against a person. The decision on the
petition binds not only the parties thereto but the whole world. An in rem proceeding is validated essentially through
publication. Publication is notice to the whole world that the proceeding has for its object to bar indefinitely all who might
be minded to make an objection of any sort against the right sought to be established. It is the publication of such notice
that brings in the whole world as a party in the case and vests the court with jurisdiction to hear and decide it. 30

Furthermore, extrinsic fraud, which was private respondent's alleged concealment of Armi's present address, was not
proven. Extrinsic fraud exists when there is a fraudulent act committed by the prevailing party outside of the trial of the
case, whereby the defeated party was prevented from presenting fully his side of the case by fraud or deception practiced
on him by the prevailing party. Here, Armi contended that private respondent is aware of her present address because they
lived together as husband and wife in the condominium unit from 1982 to 1988 and because private respondent continued
to give support to their son until 1998. To prove her claim, she presented (1) private respondent's title over the condominium
unit; (2) receipts allegedly issued to private respondent for payment of homeowner's or association dues; (2) a photocopy
of a January 14, 1991 deed of sale of the subject unit in favor of Armi; and (3) the subsequent title issued to the latter.
However, these documents only tend to prove private respondent's previous ownership of the unit and the subsequent
transfer thereof to Armi, but not the claimed live-in relationship of the parties. Neither does the sale prove that the
conveyance of the unit was part of private respondent's support to petitioner minor. Indeed, intimate relationships and family
relations cannot be inferred from what appears to be an ordinary business transaction. HTacDS

Although the January 14, 1991 deed of sale 31 stated that Armi resides at 1175 L. Guerrero St., Ermita, Manila, the same
is not sufficient to prove that private respondent has knowledge of Armi's address because the former objected to the offer
of the deed for being a mere photocopy. 32 The counsel for petitioners even admitted that they do not have the original of
the deed and that per certification of the Clerk of Court, the Notary Public who notarized the deed of sale did not submit a
copy of the notarized document as required by the rules. 33 The deed cannot thus be the basis of ascribing knowledge of
Armi's address to private respondent inasmuch as the authenticity thereof was neither admitted by private respondent nor
proven by petitioners.

While Armi presented the alleged love letters/notes from private respondent, they were only attached as annexes to the
petition and not formally offered as evidence before the Court of Appeals. More importantly, said letters/notes do not have
probative value because they were mere photocopies and never proven to be an authentic writing of private respondent.
In the same vein, the affidavits 34 of Armi and her sister, Corazon Espiritu, are of no evidentiary weight. The basic rule of
evidence is that unless the affiants themselves are placed on the witness stand to testify on their affidavits, such affidavits
must be rejected for being hearsay. Stated differently, the declarants of written statements pertaining to disputed facts must

155
be presented at the trial for cross-examination. 35 Inasmuch as Armi and her sister were not presented before the Court of
Appeals to affirm the veracity of their affidavits, the same are considered hearsay and without probative value. aTADCE

Ei incumbit probotio qui dicit, non qui negat. He who asserts, not he who denies, must prove. 36 Armi's claim that private
respondent is aware of her present address is anchored on the assertion of a live-in relationship and support to her son.
Since the evidence presented by Armi is not sufficient to prove the purported cohabitation and support, it follows that private
respondent's knowledge of Armi's address was likewise not proven. Thus, private respondent could not have deliberately
concealed from the court that which was not shown to be known to him. The Court of Appeals therefore correctly dismissed
the petition for annulment of judgment on the ground of failure to establish extrinsic fraud.

The proper remedy of a party aggrieved by a decision of the Court of Appeals in an action to annul a judgment of a Regional
Trial Court is a petition for review on certiorari under Rule 45 of the Revised Rules of Civil Procedure, where only questions
of law may be raised. The resort of petitioner to the instant civil action for certiorari under Rule 65 is therefore erroneous.
The special civil action of certiorari will not be allowed as a substitute for failure to timely file a petition for review under Rule
45, which should be instituted within 15 days 37 from receipt of the assailed decision or resolution. The wrong choice of
remedy thus provides another reason to dismiss this petition. 38

Finally, petitioner failed to establish the merits of her petition to annul the trial court's decision. In an action for annulment
of judgment, the petitioner must convince the court that something may indeed be achieved should the assailed decision
be annulled. 39 Under Article 176 40 of the Family Code as amended by Republic Act (RA) No. 9255, which took effect on
March 19, 2004, illegitimate children shall use the surname of their mother, unless their father recognizes their filiation, in
which case they may bear the father's surname. In Wang v. Cebu Civil Registrar, 41 it was held that an illegitimate child
whose filiation is not recognized by the father, bears only a given name and his mother's surname. The name of the
unrecognized illegitimate child identifies him as such. It is only when said child is recognized that he may use his father's
surname, reflecting his status as an acknowledged illegitimate child.

In the present case, it is clear from the allegations of Armi that petitioner minor is an illegitimate child because she was
never married to private respondent. Considering that the latter strongly asserts that he is not the father of petitioner minor,
the latter is therefore an unrecognized illegitimate child. As such, he must bear the surname of his mother. cADEHI

In sum, the substantive and procedural aspects of the instant controversy do not warrant the annulment of the trial court's
decision.

WHEREFORE, the petition is DISMISSED. The February 27, 2004 decision and the May 14, 2004 resolution of the Court
of Appeals in CA-G.R. SP No. 61883 are AFFIRMED.

SO ORDERED.

Davide, Jr., C.J., Quisumbing, Carpio and Azcuna, JJ., concur.

26. Boston Equity Resources, Inc. v. Court of Appeals, G.R. No. 173946, June 19, 2013

SECOND DIVISION

[G.R. No. 173946. June 19, 2013.]

BOSTON EQUITY RESOURCES, INC., petitioner, vs. COURT OF APPEALS AND LOLITA G. TOLEDO, respondents.

DECISION

PEREZ, J p:

Before the Court is a Petition for Review on Certiorari seeking to reverse and set aside: (1) the Decision, 1 dated 28
February 2006 and (2) the Resolution, 2 dated 1 August 2006 of the Court of Appeals in CA-G.R. SP No. 88586. The
challenged decision granted herein respondent's petition for certiorari upon a finding that the trial court committed grave
abuse of discretion in denying respondent's motion to dismiss the complaint against her. 3 Based on this finding, the Court
of Appeals reversed and set aside the Orders, dated 8 November 2004 4 and 22 December 2004, 5 respectively, of the
Regional Trial Court (RTC) of Manila, Branch 24. EIAHcC

The Facts

On 24 December 1997, petitioner filed a complaint for sum of money with a prayer for the issuance of a writ of preliminary
attachment against the spouses Manuel and Lolita Toledo. 6 Herein respondent filed an Answer dated 19 March 1998 but
156
on 7 May 1998, she filed a Motion for Leave to Admit Amended Answer 7 in which she alleged, among others, that her
husband and co-defendant, Manuel Toledo (Manuel), is already dead. 8 The death certificate 9 of Manuel states "13 July
1995" as the date of death. As a result, petitioner filed a motion, dated 5 August 1999, to require respondent to disclose
the heirs of Manuel. 10 In compliance with the verbal order of the court during the 11 October 1999 hearing of the case,
respondent submitted the required names and addresses of the heirs. 11 Petitioner then filed a Motion for Substitution, 12
dated 18 January 2000, praying that Manuel be substituted by his children as party-defendants. It appears that this motion
was granted by the trial court in an Order dated 9 October 2000. 13

Pre-trial thereafter ensued and on 18 July 2001, the trial court issued its pre-trial order containing, among others, the dates
of hearing of the case. 14

The trial of the case then proceeded. Herein petitioner, as plaintiff, presented its evidence and its exhibits were thereafter
admitted.

On 26 May 2004, the reception of evidence for herein respondent was cancelled upon agreement of the parties. On 24
September 2004, counsel for herein respondent was given a period of fifteen days within which to file a demurrer to
evidence. 15 However, on 7 October 2004, respondent instead filed a motion to dismiss the complaint, citing the following
as grounds: (1) that the complaint failed to implead an indispensable party or a real party in interest; hence, the case must
be dismissed for failure to state a cause of action; (2) that the trial court did not acquire jurisdiction over the person of
Manuel pursuant to Section 5, Rule 86 of the Revised Rules of Court; (3) that the trial court erred in ordering the substitution
of the deceased Manuel by his heirs; and (4) that the court must also dismiss the case against Lolita Toledo in accordance
with Section 6, Rule 86 of the Rules of Court. 16 aHADTC

The trial court, in an Order dated 8 November 2004, denied the motion to dismiss for having been filed out of time, citing
Section 1, Rule 16 of the 1997 Rules of Court which states that: "[W]ithin the time for but before filing the answer to the
complaint or pleading asserting a claim, a motion to dismiss may be made . . . ." 17 Respondent's motion for reconsideration
of the order of denial was likewise denied on the ground that "defendants' attack on the jurisdiction of this Court is now
barred by estoppel by laches" since respondent failed to raise the issue despite several chances to do so. 18

Aggrieved, respondent filed a petition for certiorari with the Court of Appeals alleging that the trial court seriously erred and
gravely abused its discretion in denying her motion to dismiss despite discovery, during the trial of the case, of evidence
that would constitute a ground for dismissal of the case. 19 ECcDAH

The Court of Appeals granted the petition based on the following grounds:

It is elementary that courts acquire jurisdiction over the person of the defendant . . . only when the latter voluntarily appeared
or submitted to the court or by coercive process issued by the court to him, . . . . In this case, it is undisputed that when
[petitioner] Boston filed the complaint on December 24, 1997, defendant Manuel S. Toledo was already dead, . . . . Such
being the case, the court a quo could not have acquired jurisdiction over the person of defendant Manuel S. Toledo.

. . . the court a quo's denial of [respondent's] motion to dismiss was based on its finding that [respondent's] attack on the
jurisdiction of the court was already barred by laches as [respondent] failed to raise the said ground in its [sic] amended
answer and during the pre-trial, despite her active participation in the proceedings.

However, . . . it is well-settled that issue on jurisdiction may be raised at any stage of the proceeding, even for the first time
on appeal. By timely raising the issue on jurisdiction in her motion to dismiss . . . [respondent] is not estopped [from] raising
the question on jurisdiction. Moreover, when issue on jurisdiction was raised by [respondent], the court a quo had not yet
decided the case, hence, there is no basis for the court a quo to invoke estoppel to justify its denial of the motion for
reconsideration; HcaDIA

It should be stressed that when the complaint was filed, defendant Manuel S. Toledo was already dead. The complaint
should have impleaded the estate of Manuel S. Toledo as defendant, not only the wife, considering that the estate of Manuel
S. Toledo is an indispensable party, which stands to be benefited or be injured in the outcome of the case. . . .

xxx xxx xxx

[Respondent's] motion to dismiss the complaint should have been granted by public respondent judge as the same was in
order. Considering that the obligation of Manuel S. Toledo is solidary with another debtor, . . ., the claim . . . should be filed
against the estate of Manuel S. Toledo, in conformity with the provision of Section 6, Rule 86 of the Rules of Court, . . . .
20

The Court of Appeals denied petitioner's motion for reconsideration. Hence, this petition.

The Issues
157
Petitioner claims that the Court of Appeals erred in not holding that:

1. Respondent is already estopped from questioning the trial court's jurisdiction; EcAISC

2. Petitioner never failed to implead an indispensable party as the estate of Manuel is not an indispensable party;

3. The inclusion of Manuel as party-defendant is a mere misjoinder of party not warranting the dismissal of the case
before the lower court; and

4. Since the estate of Manuel is not an indispensable party, it is not necessary that petitioner file its claim against the
estate of Manuel.

In essence, what is at issue here is the correctness of the trial court's orders denying respondent's motion to dismiss.

The Ruling of the Court

We find merit in the petition.

Motion to dismiss filed out of time

To begin with, the Court of Appeals erred in granting the writ of certiorari in favor of respondent. Well settled is the rule that
the special civil action for certiorari is not the proper remedy to assail the denial by the trial court of a motion to dismiss.
The order of the trial court denying a motion to dismiss is merely interlocutory, as it neither terminates nor finally disposes
of a case and still leaves something to be done by the court before a case is finally decided on the merits. 21 Therefore,
"the proper remedy in such a case is to appeal after a decision has been rendered." 22 aSTHDc

As the Supreme Court held in Indiana Aerospace University v. Comm. on Higher Education: 23

A writ of certiorari is not intended to correct every controversial interlocutory ruling; it is resorted only to correct a grave
abuse of discretion or a whimsical exercise of judgment equivalent to lack of jurisdiction. Its function is limited to keeping
an inferior court within its jurisdiction and to relieve persons from arbitrary acts — acts which courts or judges have no
power or authority in law to perform. It is not designed to correct erroneous findings and conclusions made by the courts.
(Emphasis supplied)

Even assuming that certiorari is the proper remedy, the trial court did not commit grave abuse of discretion in denying
respondent's motion to dismiss. It, in fact, acted correctly when it issued the questioned orders as respondent's motion to
dismiss was filed SIX YEARS AND FIVE MONTHS AFTER SHE FILED HER AMENDED ANSWER. This circumstance
alone already warranted the outright dismissal of the motion for having been filed in clear contravention of the express
mandate of Section 1, Rule 16, of the Revised Rules of Court. Under this provision, a motion to dismiss shall be filed within
the time for but before the filing of an answer to the complaint or pleading asserting a claim. 24 IDASHa

More importantly, respondent's motion to dismiss was filed after petitioner has completed the presentation of its evidence
in the trial court, 25 giving credence to petitioner's and the trial court's conclusion that the filing of the motion to dismiss was
a mere ploy on the part of respondent to delay the prompt resolution of the case against her.

Also worth mentioning is the fact that respondent's motion to dismiss under consideration herein is not the first motion to
dismiss she filed in the trial court. It appears that she had filed an earlier motion to dismiss 26 on the sole ground of the
unenforceability of petitioner's claim under the Statute of Frauds, which motion was denied by the trial court. More telling is
the following narration of the trial court in its Order denying respondent's motion for reconsideration of the denial of her
motion to dismiss:

As can be gleaned from the records, with the admission of plaintiff's exhibits, reception of defendants' evidence was set on
March 31, and April 23, 2004 . . . . On motion of the defendant[s], the hearing on March 31, 2004 was cancelled.

On April 14, 2004, defendants sought the issuance of subpoena ad testificandum and duces tecum to one Gina M. Madulid,
to appear and testify for the defendants on April 23, 2004. Reception of defendants' evidence was again deferred to May
26, June 2 and June 30, 2004, . . . .

On May 13, 2004, defendants sought again the issuance of a subpoena duces tecum and ad testificandum to the said Gina
Madulid. On May 26, 2004, reception of defendants [sic] evidence was cancelled upon the agreement of the parties. On
July 28, 2004, in the absence of defendants' witness, hearing was reset to September 24 and October 8, 2004 . . . .

On September 24, 2004, counsel for defendants was given a period of fifteen (15) days to file a demurrer to evidence. On
October 7, 2004, defendants filed instead a Motion to Dismiss . . . . 27

158
Respondent's act of filing multiple motions, such as the first and earlier motion to dismiss and then the motion to dismiss
at issue here, as well as several motions for postponement, lends credibility to the position taken by petitioner, which is
shared by the trial court, that respondent is deliberately impeding the early disposition of this case. The filing of the second
motion to dismiss was, therefore, "not only improper but also dilatory." 28 Thus, the trial court, "far from deviating or straying
off course from established jurisprudence on [the] matter, . . . had in fact faithfully observed the law and legal precedents
in this case." 29 The Court of Appeals, therefore, erred not only in entertaining respondent's petition for certiorari, it likewise
erred in ruling that the trial court committed grave abuse of discretion when it denied respondent's motion to dismiss.
IECcAT

On whether or not respondent is estopped from

questioning the jurisdiction of the trial court

At the outset, it must be here stated that, as the succeeding discussions will demonstrate, jurisdiction over the person of
Manuel should not be an issue in this case. A protracted discourse on jurisdiction is, nevertheless, demanded by the fact
that jurisdiction has been raised as an issue from the lower court, to the Court of Appeals and, finally, before this Court. For
the sake of clarity, and in order to finally settle the controversy and fully dispose of all the issues in this case, it was deemed
imperative to resolve the issue of jurisdiction.

1. Aspects of Jurisdiction

Petitioner calls attention to the fact that respondent's motion to dismiss questioning the trial court's jurisdiction was filed
more than six years after her amended answer was filed. According to petitioner, respondent had several opportunities, at
various stages of the proceedings, to assail the trial court's jurisdiction but never did so for six straight years. Citing the
doctrine laid down in the case of Tijam, et al. v. Sibonghanoy, et al. 30 petitioner claimed that respondent's failure to raise
the question of jurisdiction at an earlier stage bars her from later questioning it, especially since she actively participated in
the proceedings conducted by the trial court. CTAIDE

Petitioner's argument is misplaced, in that, it failed to consider that the concept of jurisdiction has several aspects, namely:
(1) jurisdiction over the subject matter; (2) jurisdiction over the parties; (3) jurisdiction over the issues of the case; and (4)
in cases involving property, jurisdiction over the res or the thing which is the subject of the litigation. 31

The aspect of jurisdiction which may be barred from being assailed as a result of estoppel by laches is jurisdiction over the
subject matter. Thus, in Tijam, the case relied upon by petitioner, the issue involved was the authority of the then Court of
First Instance to hear a case for the collection of a sum of money in the amount of P1,908.00 which amount was, at that
time, within the exclusive original jurisdiction of the municipal courts.

In subsequent cases citing the ruling of the Court in Tijam, what was likewise at issue was the jurisdiction of the trial court
over the subject matter of the case. Accordingly, in Spouses Gonzaga v. Court of Appeals, 32 the issue for consideration
was the authority of the regional trial court to hear and decide an action for reformation of contract and damages involving
a subdivision lot, it being argued therein that jurisdiction is vested in the Housing and Land Use Regulatory Board pursuant
to PD 957 (The Subdivision and Condominium Buyers Protective Decree). In Lee v. Presiding Judge, MTC, Legaspi City,
33 petitioners argued that the respondent municipal trial court had no jurisdiction over the complaint for ejectment because
the issue of ownership was raised in the pleadings. Finally, in People v. Casuga, 34 accused-appellant claimed that the
crime of grave slander, of which she was charged, falls within the concurrent jurisdiction of municipal courts or city courts
and the then courts of first instance, and that the judgment of the court of first instance, to which she had appealed the
municipal court's conviction, should be deemed null and void for want of jurisdiction as her appeal should have been filed
with the Court of Appeals or the Supreme Court. ADEacC

In all of these cases, the Supreme Court barred the attack on the jurisdiction of the respective courts concerned over the
subject matter of the case based on estoppel by laches, declaring that parties cannot be allowed to belatedly adopt an
inconsistent posture by attacking the jurisdiction of a court to which they submitted their cause voluntarily. 35

Here, what respondent was questioning in her motion to dismiss before the trial court was that court's jurisdiction over the
person of defendant Manuel. Thus, the principle of estoppel by laches finds no application in this case. Instead, the
principles relating to jurisdiction over the person of the parties are pertinent herein. AICEDc

The Rules of Court provide:

RULE 9

EFFECT OF FAILURE TO PLEAD

159
Section 1. Defenses and objections not pleaded. — Defenses and objections not pleaded either in a motion to
dismiss or in the answer are deemed waived. However, when it appears from the pleadings or the evidence on record that
the court has no jurisdiction over the subject matter, that there is another action pending between the same parties for the
same cause, or that the action is barred by a prior judgment or by statute of limitations, the court shall dismiss the claim.

RULE 15

MOTIONS

Sec. 8. Omnibus motion. — Subject to the provisions of Section 1 of Rule 9, a motion attacking a pleading, order, judgment,
or proceeding shall include all objections then available, and all objections not so included shall be deemed waived.

Based on the foregoing provisions, the "objection on jurisdictional grounds which is not waived even if not alleged in a
motion to dismiss or the answer is lack of jurisdiction over the subject matter. . . . Lack of jurisdiction over the subject matter
can always be raised anytime, even for the first time on appeal, since jurisdictional issues cannot be waived . . . subject,
however, to the principle of estoppel by laches." 36 cHDAIS

Since the defense of lack of jurisdiction over the person of a party to a case is not one of those defenses which are not
deemed waived under Section 1 of Rule 9, such defense must be invoked when an answer or a motion to dismiss is filed
in order to prevent a waiver of the defense. 37 If the objection is not raised either in a motion to dismiss or in the answer,
the objection to the jurisdiction over the person of the plaintiff or the defendant is deemed waived by virtue of the first
sentence of the above-quoted Section 1 of Rule 9 of the Rules of Court. 38

The Court of Appeals, therefore, erred when it made a sweeping pronouncement in its questioned decision, stating that
"issue on jurisdiction may be raised at any stage of the proceeding, even for the first time on appeal" and that, therefore,
respondent timely raised the issue in her motion to dismiss and is, consequently, not estopped from raising the question of
jurisdiction. As the question of jurisdiction involved here is that over the person of the defendant Manuel, the same is
deemed waived if not raised in the answer or a motion to dismiss. In any case, respondent cannot claim the defense since
"lack of jurisdiction over the person, being subject to waiver, is a personal defense which can only be asserted by the party
who can thereby waive it by silence." 39

2. Jurisdiction over the person of a defendant is acquired through a valid service of summons; trial court did not
acquire jurisdiction over the person of Manuel Toledo

In the first place, jurisdiction over the person of Manuel was never acquired by the trial court. A defendant is informed of a
case against him when he receives summons. "Summons is a writ by which the defendant is notified of the action brought
against him. Service of such writ is the means by which the court acquires jurisdiction over his person." 40

In the case at bar, the trial court did not acquire jurisdiction over the person of Manuel since there was no valid service of
summons upon him, precisely because he was already dead even before the complaint against him and his wife was filed
in the trial court. The issues presented in this case are similar to those in the case of Sarsaba v. Vda. de Te. 41

In Sarsaba, the NLRC rendered a decision declaring that Patricio Sereno was illegally dismissed from employment and
ordering the payment of his monetary claims. To satisfy the claim, a truck in the possession of Sereno's employer was
levied upon by a sheriff of the NLRC, accompanied by Sereno and his lawyer, Rogelio Sarsaba, the petitioner in that case.
A complaint for recovery of motor vehicle and damages, with prayer for the delivery of the truck pendente lite was eventually
filed against Sarsaba, Sereno, the NLRC sheriff and the NLRC by the registered owner of the truck. After his motion to
dismiss was denied by the trial court, petitioner Sarsaba filed his answer. Later on, however, he filed an omnibus motion to
dismiss citing, as one of the grounds, lack of jurisdiction over one of the principal defendants, in view of the fact that Sereno
was already dead when the complaint for recovery of possession was filed. aEHASI

Although the factual milieu of the present case is not exactly similar to that of Sarsaba, one of the issues submitted for
resolution in both cases is similar: whether or not a case, where one of the named defendants was already dead at the time
of its filing, should be dismissed so that the claim may be pursued instead in the proceedings for the settlement of the
estate of the deceased defendant. The petitioner in the Sarsaba Case claimed, as did respondent herein, that since one of
the defendants died before summons was served on him, the trial court should have dismissed the complaint against all
the defendants and the claim should be filed against the estate of the deceased defendant. The petitioner in Sarsaba,
therefore, prayed that the complaint be dismissed, not only against Sereno, but as to all the defendants, considering that
the RTC did not acquire jurisdiction over the person of Sereno. 42 This is exactly the same prayer made by respondent
herein in her motion to dismiss.

The Court, in the Sarsaba Case, resolved the issue in this wise:

160
. . . We cannot countenance petitioner's argument that the complaint against the other defendants should have been
dismissed, considering that the RTC never acquired jurisdiction over the person of Sereno. The court's failure to acquire
jurisdiction over one's person is a defense which is personal to the person claiming it. Obviously, it is now impossible for
Sereno to invoke the same in view of his death. Neither can petitioner invoke such ground, on behalf of Sereno, so as to
reap the benefit of having the case dismissed against all of the defendants. Failure to serve summons on Sereno's person
will not be a cause for the dismissal of the complaint against the other defendants, considering that they have been served
with copies of the summons and complaints and have long submitted their respective responsive pleadings. In fact, the
other defendants in the complaint were given the chance to raise all possible defenses and objections personal to them in
their respective motions to dismiss and their subsequent answers. 43 (Emphasis supplied.) TCHcAE

Hence, the Supreme Court affirmed the dismissal by the trial court of the complaint against Sereno only.

Based on the foregoing pronouncements, there is no basis for dismissing the complaint against respondent herein. Thus,
as already emphasized above, the trial court correctly denied her motion to dismiss. EacHSA

On whether or not the estate of Manuel

Toledo is an indispensable party

Rule 3, Section 7 of the 1997 Rules of Court states:

SEC. 7. Compulsory joinder of indispensable parties. — Parties-in-interest without whom no final determination can be had
of an action shall be joined either as plaintiffs or defendants.

An indispensable party is one who has such an interest in the controversy or subject matter of a case that a final adjudication
cannot be made in his or her absence, without injuring or affecting that interest. He or she is a party who has not only an
interest in the subject matter of the controversy, but "an interest of such nature that a final decree cannot be made without
affecting [that] interest or leaving the controversy in such a condition that its final determination may be wholly inconsistent
with equity and good conscience. It has also been considered that an indispensable party is a person in whose absence
there cannot be a determination between the parties already before the court which is effective, complete or equitable."
Further, an indispensable party is one who must be included in an action before it may properly proceed. 44

On the other hand, a "person is not an indispensable party if his interest in the controversy or subject matter is separable
from the interest of the other parties, so that it will not necessarily be directly or injuriously affected by a decree which does
complete justice between them. Also, a person is not an indispensable party if his presence would merely permit complete
relief between him or her and those already parties to the action, or if he or she has no interest in the subject matter of the
action." It is not a sufficient reason to declare a person to be an indispensable party simply because his or her presence
will avoid multiple litigations. 45 cEAaIS

Applying the foregoing pronouncements to the case at bar, it is clear that the estate of Manuel is not an indispensable party
to the collection case, for the simple reason that the obligation of Manuel and his wife, respondent herein, is solidary.

The contract between petitioner, on the one hand and respondent and respondent's husband, on the other, states:

FOR VALUE RECEIVED, I/We jointly and severally 46 (in solemn) promise to pay BOSTON EQUITY RESOURCES, INC.
. . . the sum of PESOS: [ONE MILLION FOUR HUNDRED (P1,400,000.00)] . . . . 47

The provisions and stipulations of the contract were then followed by the respective signatures of respondent as "MAKER"
and her husband as "CO-MAKER." 48 Thus, pursuant to Article 1216 of the Civil Code, petitioner may collect the entire
amount of the obligation from respondent only. The aforementioned provision states: "The creditor may proceed against
any one of the solidary debtors or some or all of them simultaneously. The demand made against one of them shall not be
an obstacle to those which may subsequently be directed against the others, so long as the debt has not been fully
collected." EaISTD

In other words, the collection case can proceed and the demands of petitioner can be satisfied by respondent only, even
without impleading the estate of Manuel. Consequently, the estate of Manuel is not an indispensable party to petitioner's
complaint for sum of money.

However, the Court of Appeals, agreeing with the contention of respondent, held that the claim of petitioner should have
been filed against the estate of Manuel in accordance with Sections 5 and 6 of Rule 86 of the Rules of Court. The
aforementioned provisions provide:

SEC. 5. Claims which must be filed under the notice. If not filed, barred; exceptions. — All claims for money against the
decedent, arising from contract, express or implied, whether the same be due, not due, or contingent, all claims for funeral
161
expenses and judgment for money against the decedent, must be filed within the time limited in the notice; otherwise, they
are barred forever, except that they may be set forth as counterclaims in any action that the executor or administrator may
bring against the claimants. . . . .

SEC. 6. Solidary obligation of decedent. — Where the obligation of the decedent is solidary with another debtor, the claim
shall be filed against the decedent as if he were the only debtor, without prejudice to the right of the estate to recover
contribution from the other debtor. . . . .

The Court of Appeals erred in its interpretation of the above-quoted provisions.

In construing Section 6, Rule 87 of the old Rules of Court, the precursor of Section 6, Rule 86 of the Revised Rules of
Court, which latter provision has been retained in the present Rules of Court without any revisions, the Supreme Court, in
the case of Manila Surety & Fidelity Co., Inc. v. Villarama, et al., 49 held: 50 aAHTDS

Construing Section 698 of the Code of Civil Procedure from whence [Section 6, Rule 87] was taken, this Court held that
where two persons are bound in solidum for the same debt and one of them dies, the whole indebtedness can be proved
against the estate of the latter, the decedent's liability being absolute and primary; . . . . It is evident from the foregoing that
Section 6 of Rule 87 provides the procedure should the creditor desire to go against the deceased debtor, but there is
certainly nothing in the said provision making compliance with such procedure a condition precedent before an ordinary
action against the surviving solidary debtors, should the creditor choose to demand payment from the latter, could be
entertained to the extent that failure to observe the same would deprive the court jurisdiction to take cognizance of the
action against the surviving debtors. Upon the other hand, the Civil Code expressly allows the creditor to proceed against
any one of the solidary debtors or some or all of them simultaneously. There is, therefore, nothing improper in the creditor's
filing of an action against the surviving solidary debtors alone, instead of instituting a proceeding for the settlement of the
estate of the deceased debtor wherein his claim could be filed. SaIHDA

The foregoing ruling was reiterated and expounded in the later case of Philippine National Bank v. Asuncion 51 where the
Supreme Court pronounced:

A cursory perusal of Section 6, Rule 86 of the Revised Rules of Court reveals that nothing therein prevents a creditor from
proceeding against the surviving solidary debtors. Said provision merely sets up the procedure in enforcing collection in
case a creditor chooses to pursue his claim against the estate of the deceased solidary debtor. The rule has been set forth
that a creditor (in a solidary obligation) has the option whether to file or not to file a claim against the estate of the solidary
debtor. . . .

xxx xxx xxx

It is crystal clear that Article 1216 of the New Civil Code is the applicable provision in this matter. Said provision gives the
creditor the right to "proceed against anyone of the solidary debtors or some or all of them simultaneously." The choice is
undoubtedly left to the solidary creditor to determine against whom he will enforce collection. In case of the death of one of
the solidary debtors, he (the creditor) may, if he so chooses, proceed against the surviving solidary debtors without
necessity of filing a claim in the estate of the deceased debtors. It is not mandatory for him to have the case dismissed as
against the surviving debtors and file its claim against the estate of the deceased solidary debtor, . . . . For to require the
creditor to proceed against the estate, making it a condition precedent for any collection action against the surviving debtors
to prosper, would deprive him of his substantive rights provided by Article 1216 of the New Civil Code. (Emphasis supplied.)
EHDCAI

As correctly argued by petitioner, if Section 6, Rule 86 of the Revised Rules of Court were applied literally, Article 1216 of
the New Civil Code would, in effect, be repealed since under the Rules of Court, petitioner has no choice but to proceed
against the estate of [the deceased debtor] only. Obviously, this provision diminishes the [creditor's] right under the New
Civil Code to proceed against any one, some or all of the solidary debtors. Such a construction is not sanctioned by principle,
which is too well settled to require citation, that a substantive law cannot be amended by a procedural rule. Otherwise
stated, Section 6, Rule 86 of the Revised Rules of Court cannot be made to prevail over Article 1216 of the New Civil Code,
the former being merely procedural, while the latter, substantive.

Based on the foregoing, the estate of Manuel is not an indispensable party and the case can proceed as against respondent
only. That petitioner opted to collect from respondent and not from the estate of Manuel is evidenced by its opposition to
respondent's motion to dismiss asserting that the case, as against her, should be dismissed so that petitioner can proceed
against the estate of Manuel. IHTASa

On whether or not the inclusion of Manuel as

party defendant is a misjoinder of party


162
Section 11 of Rule 3 of the Rules of Court states that "[n]either misjoinder nor non-joinder of parties is ground for dismissal
of an action. Parties may be dropped or added by order of the court on motion of any party or on its own initiative at any
stage of the action and on such terms as are just. Any claim against a misjoined party may be severed and proceeded with
separately."

Based on the last sentence of the afore-quoted provision of law, a misjoined party must have the capacity to sue or be sued
in the event that the claim by or against the misjoined party is pursued in a separate case. In this case, therefore, the
inclusion of Manuel in the complaint cannot be considered a misjoinder, as in fact, the action would have proceeded against
him had he been alive at the time the collection case was filed by petitioner. This being the case, the remedy provided by
Section 11 of Rule 3 does not obtain here. The name of Manuel as party-defendant cannot simply be dropped from the
case. Instead, the procedure taken by the Court in Sarsaba v. Vda. de Te, 52 whose facts, as mentioned earlier, resemble
those of this case, should be followed herein. There, the Supreme Court agreed with the trial court when it resolved the
issue of jurisdiction over the person of the deceased Sereno in this wise: ICHcaD

As correctly pointed by defendants, the Honorable Court has not acquired jurisdiction over the person of Patricio Sereno
since there was indeed no valid service of summons insofar as Patricio Sereno is concerned. Patricio Sereno died before
the summons, together with a copy of the complaint and its annexes, could be served upon him.

However, the failure to effect service of summons unto Patricio Sereno, one of the defendants herein, does not render the
action DISMISSIBLE, considering that the three (3) other defendants, . . ., were validly served with summons and the case
with respect to the answering defendants may still proceed independently. Be it recalled that the three (3) answering
defendants have previously filed a Motion to Dismiss the Complaint which was denied by the Court.

Hence, only the case against Patricio Sereno will be DISMISSED and the same may be filed as a claim against the estate
of Patricio Sereno, but the case with respect to the three (3) other accused [sic] will proceed. (Emphasis supplied.) 53

As a result, the case, as against Manuel, must be dismissed.

In addition, the dismissal of the case against Manuel is further warranted by Section 1 of Rule 3 of the Rules of Court, which
states that: [o]nly natural or juridical persons, or entities authorized by law may be parties in a civil action." Applying this
provision of law, the Court, in the case of Ventura v. Militante, 54 held: TaCEHA

Parties may be either plaintiffs or defendants. . . . . In order to maintain an action in a court of justice, the plaintiff must have
an actual legal existence, that is, he, she or it must be a person in law and possessed of a legal entity as either a natural
or an artificial person, and no suit can be lawfully prosecuted save in the name of such a person.

The rule is no different as regards party defendants. It is incumbent upon a plaintiff, when he institutes a judicial proceeding,
to name the proper party defendant to his cause of action. In a suit or proceeding in personam of an adversary character,
the court can acquire no jurisdiction for the purpose of trial or judgment until a party defendant who actually or legally exists
and is legally capable of being sued, is brought before it. It has even been held that the question of the legal personality of
a party defendant is a question of substance going to the jurisdiction of the court and not one of procedure.

The original complaint of petitioner named the "estate of Carlos Ngo as represented by surviving spouse Ms. Sulpicia
Ventura" as the defendant. Petitioner moved to dismiss the same on the ground that the defendant as named in the
complaint had no legal personality. We agree. cETCID

. . . . Considering that capacity to be sued is a correlative of the capacity to sue, to the same extent, a decedent does not
have the capacity to be sued and may not be named a party defendant in a court action. (Emphases supplied.)

Indeed, where the defendant is neither a natural nor a juridical person or an entity authorized by law, the complaint may be
dismissed on the ground that the pleading asserting the claim states no cause of action or for failure to state a cause of
action pursuant to Section 1 (g) of Rule 16 of the Rules of Court, because a complaint cannot possibly state a cause of
action against one who cannot be a party to a civil action. 55

Since the proper course of action against the wrongful inclusion of Manuel as party-defendant is the dismissal of the case
as against him, thus did the trial court err when it ordered the substitution of Manuel by his heirs. Substitution is proper only
where the party to be substituted died during the pendency of the case, as expressly provided for by Section 16, Rule 3 of
the Rules of Court, which states:

Death of party; duty of counsel. — Whenever a party to a pending action dies, and the claim is not thereby extinguished, it
shall be the duty of his counsel to inform the court within thirty (30) days after such death of the fact thereof, and to give
the name and address of his legal representative or representatives. . . .

163
The heirs of the deceased may be allowed to be substituted for the deceased, without requiring the appointment of an
executor or administrator . . . .

The court shall forthwith order said legal representative or representatives to appear and be substituted within a period of
thirty (30) days from notice. (Emphasis supplied.)

Here, since Manuel was already dead at the time of the filing of the complaint, the court never acquired jurisdiction over his
person and, in effect, there was no party to be substituted.

WHEREFORE, the petition is GRANTED. The Decision dated 28 February 2006 and the Resolution dated 1 August 2006
of the Court of Appeals in CA-G.R. SP No. 88586 are REVERSED and SET ASIDE. The Orders of the Regional Trial Court
dated 8 November 2004 and 22 December 2004, respectively, in Civil Case No. 97-86672, are REINSTATED. The Regional
Trial Court, Branch 24, Manila is hereby DIRECTED to proceed with the trial of Civil Case No. 97-86672 against respondent
Lolita G. Toledo only, in accordance with the above pronouncements of the Court, and to decide the case with dispatch.
HScAEC

SO ORDERED.

Carpio, Brion, Del Castillo and Villarama, Jr.,* JJ., concur.

164

You might also like